You are on page 1of 237

Environment and Sustainable Development

Environment development is a subject of sensitivity in the context of natural


resources from one to another generation. *The main objective
*Environment is defined as ‘the sum total of biotic and abiotic
of sustainable development is the conservation of natural
components that surrounds and influences an organism’.
Everything that surrounds an organism is collectively known resources, proper management of natural resources and use of
as its environment. *The biotic and abiotic components of the natural resources in a just manner.
environment interact with each other.
*‘Silent Spring’ is an environmental science book published
in 1962. It was written by an American biologist and ecologist
‘Rachel Carson’.
*According to Environment (Protection) Act 1986,
‘Environment includes physical / abiotic and biotic components
and their interaction with each other’.
*Environment includes three components-

*The Brundtland Report, published in 1987 by United Nations


World Commission on Environment and Development, coined
the term "Sustainable Development". This word was coined by
WCED report titled "Our Common Future".
*It captured full attention by Agenda 21 (Rio Conference) in
Earth Summit of 1992. It was the main point of discussion in
the Johannesburg Conference of 2002.
Events Year
Publication of ‘The Limit to Growth’ 1972
Report

(1) Physical / abiotic components - Soil, air and water. Publication of Brundtland Commission April, 1987
(2) Biotic component - plant, human, animal and Report
microorganisms. Adoption of Montreal Protocol September, 1987
(3) Cultural Element - cultural environment include man- Rio Earth Summit June, 1992
made features such as economical, social and political
*Sustainable development includes - poverty mitigation,
elements.
conservation of biodiversity and control of environmental
Sustainable Development pollution.
*Sustainable development is the concept of development that *Nature has its own beauty and boundaries, so at first, we should
meets the need of the present without compromising the ability understand the nature and then the road map of development
of future generations to meet their own needs. *Sustainable should be prepared.

Environment and Ecology General Studies F–9


*Sustainable development is a balanced approach between
Economic development and Environmental conservation.
*Compelling terms for sustainable development are based on CO2 in Earth’s
atmosphere
the availability of information.
*These information are obtained by Remote sensing technology,
Statistical method, Geographical Information System (GIS),
Environment information system etc. On the basis of these
(Plants)

information, road map of development is prepared.

Carbonates

(Shells)

*Solar radiation plays the most important role in the water


Population Size cycle. Solar radiation initiates water cycle by evaporating
water. Humidity is defined as some measure of the water
vapour content of air (or other gas). The term "humidity" is
a general term to quantity the amount of water vapour in the
gas. Significantly humidity is measured using a hygrometer or
psychrometer.
*The water cycle is driven primarily by the energy from the
sun. The solar radiation drives the cycle by evaporating water
from the oceans, lakes, rivers and even the soil. It forms water
droplets and clouds, where water droplets eventually gain
enough mass to fall back to earth. The water cycle is essentially
Water & Carbon Cycle in Nature a closed system which means that the volume of water that is
*Environment includes biotic and abiotic factors. These in the hydrosphere today is the same amount of water that has
factors interact with each other. In general, this interaction always been present in the earth system.
occurs in a cyclic manner. The flow of energy and matter
Low temperature
takes place between these factors (abiotic & biotic) in a
cyclic process.
*These cycles exist in nature in the form of Carbon Cycle,
Water Cycle etc. *CO2 is released in the atmosphere by a
number of processes, some of them are-respiration, volcanic
eruptions, combustion of fossil fuels etc. *Autotrophic plants
perform photosynthesis wherein they use CO2 and water in the
presence of sunlight and chlorophyll to produce carbohydrates.
*It is noteworthy that carbon is found in various forms on
the earth. Elemental carbon is generally found in nature as
three crystaline allotropes forms: graphite, fullerene and
diamond. In compound form, it is found in the form of
CO2, Carbonates and Hydrogen carbonates. Carbon is also
recycled by different physical and biological processes.

F–10 General Studies Environment and Ecology


*Many transplanted seedlings do not grow because most of *The share of various gases in the composition of air is as
the root hairs are lost during transplantation. The function of follows:

root hairs is to collect water and mineral nutrients present in Gas Percentage
the soil and take this solution up through the roots to the rest Nitrogen (N2) 78.08%
of the plant. Oxygen (O2) 20.95%
Argon (Ar) 0.93%
Carbon dioxide (Co2) 0.036%
Neon (Ne) 0.002%
Helium (He) 0.0005%
Krypton 0.001%
Xenon 0.00009%
Hydrogen 0.00005%

*Nitrogen is the most abundant gas available in the


environment.
*We need oxygen to survive and therefore we depend on the
plants and trees for our survival. The process by which plants
*These are made up of cellulose and nominal Pectin which
make their food is called photosynthesis. In this process, they
helps it as a gelling agent with the soil. Another reason is consume CO2 present in the atmosphere and release oxygen
the use of fertilizers which increases the concentrations of in the atmosphere.
the soil and make it difficult for plants to absorb water. This *The United Nations has identified 17 Sustainable Development

situation causes desiccation of plant cells causing the leaves Goals which have to be realised by the year 2030. These goals
are -
to turn brown.
(1) No Poverty : End poverty in all its forms everywhere.
Atmosphere (2) Zero Hunger : End hunger, achieve food security and
*The thin layer of gases surrounding the earth is called the improved nutrition and promote sustainable agriculture.
(3) Good Health and Well Being: Ensure healthy lives &
atmosphere. The atmosphere is majorly consists of Nitrogen,
promote well-being for all at all ages.
Oxygen, Carbon dioxide etc. The air around us is a mixture
(4) Quality Education: Ensure inclusive and equitable
of gases.
education and promote lifelong learning opportunities
for all.
(5) Gender Equality: Achieve gender equality and empower
all women and girls.
(6) Clean Water and Sanitation: Ensure availability and
sustainable management of water and sanitation for all.
(7) Affordable and Clean Energy: Ensure access to
affordable, reliable, sustainable and modern energy for
all.
(8) Decent Work and Economic Growth: Promote
sustained, inclusive and sustainable economic growth,
full and productive employment and decent work for all.

Environment and Ecology General Studies F–11


(9) Industry, Innovation and Infrastructure: Build *Sustainable Farming is also an important aspect of
resilient infrastructure, promote inclusive and sustainable Sustainable development. *UN declared 2022 as "International
industrialization and foster innovation. year of Basic sciences for Sustainable Development".
(10) Reduced Inequalities: Reduce inequality within and *Sustainable agriculture is the production of food, fibre or other
among countries. plants or other animal products using farming techniques that
(11) Sustainable Cities and Communities: Make Cities protect the environment, public health, human communities
and human settlements inclusive, safe resilient and and animal welfare. In short, it is proper utilization of the land
sustainable. without affecting the environment so that it's quality remains
(12) Responsible Consumption and Production: Ensure intact.
sustainable consumption and production pattern. *Use of chemical fertilizers and pesticides has adversely
(13) Climate Action: Take urgent action to combat climate affected soil health. To cope up with this, National Mission
change and its impacts. for Sustainable Agriculture (NMSA) has been formulated
(14) Life Below Water: Conserve and sustainably use for enhancing agricultural productivity, water use efficiency,
the oceans, seas and marine resources for sustainable promoting nutrient management and Livelihood diversification
development. through the adoption of sustainable development pathways.
(15) Life on Land: Combat desertification, sustainably This mission is a part of the National Action Plan of Climate
manage forests, half and reverse land degradation and Change.
halt biodiversity loss. *Club of Rome is an association of economists, civil servants
(16) Peace and Justice Strong Institutions: Promote peaceful and scientists with a mission to promote understanding of
and inclusive societies and provide access to justice. the global challenges facing humanity. The organisation has
(17) Partnerships to achieve the Goal: Global Partnerships prioritised five key areas of impact: emerging New Civilisations;
for Sustainable Development. Planetary Emergency; Reframing Economics; Rethinking
*A Sustainable Development Goal - Index had also been Finance; and Youth Leadership and Intergenerational Dialogues.
initiated to measure the progress made by various countries Earth Summit (RIO Summit)
in the direction of achieving these goals. In 2021, India was *The United Nations Conference on Environment and
placed at 120th position among 165 Countries. Development (UNCED), also known as the 'Earth Summit' was
Position of Countries in Sustainable Development Goals held in Rio de Janeiro, Brazil from 3 - 14 June 1992. This global
(SDG) Index 2021 conference, held on the occasion of the 20th anniversary of the
Sr. No. Position Country first Human Environment Conference in Stockholm, Sweden
in 1972. The primary objective of the Rio 'Earth Summit' was
1. 1 Finland
to produce a broad agenda and a new blueprint for international
2. 2 Sweden
action on environmental action and development issue.
3. 3 Denmark
*The Commission on Sustainable Development (CSD) was
4. 4 Germany created to monitor and report on the implementation of the
5. 5 Belgium Earth Summit agreement. It was agreed that a five-year review
6. 6 Austria of Earth Summit progress would be made in 1997 at the United
7. 7 Norway Nations General Assembly meeting in special session. And so,
8. 8 France a special session of UN General Assembly was held from 23-27
June, 1997 in New York to take stock of how well countries,
9. 9 Slovenia
international organizations and sectors of civil society have
10. 10 Estonia
responded to the challenges of the Earth Summit. This meeting
11. 120 India was known as Earth Summit +5.
12. 129 Pakistan *Rio + 10 summit was held in Johannesburg in 2002 and Rio

F–12 General Studies Environment and Ecology


+20 was held in Rio de Janeiro in 2012. Agenda - 21
*GEF provides funds to the least developed countries "Least
*Agenda 21 is legally non-binding, voluntarily implemented
Developed Countries Fund" (LDCF). It helps in provision
action plan of the United Nations on Sustainable Development.
& co-ordination for National Adaptation Programmes of
Action (NAPAs) for these countries.
*It is notable that meeting of COP7 was held at Marrakesh
from 29 October to 10 November 2001. *The Special
Climate Change Fund (SCCF) is established by the COP7.
*This fund helps developing countries in the adaptation as well
as transmission of technologies.
*UNDP and UNEP and World Bank are associated with
the GEF.
*The Global Environment Facility (GEF) was established on
the eve of the 1992 Rio Earth Summit to tackle our planet's most
pressing environmental problems. Carlos Manuel Rodriguez is *It is a product of the Earth Summit (UN Conference on
presently the Chief Executive Officer and Chairperson of the Environment and Development) held in Rio de Janeiro,
Global Environment Facility. Brazil in 1992.

RIO + 20 Conference
*Rio+20 is the short name for the United Nations Conference
on Sustainable Development. This conference was held in
Brazil in June 2012 to mark the 20th anniversary of the 1992
United Nations Conference on Environment and Development
(UNCED), in Rio de Janeiro. The conference's outcome
document entitled 'The Future We Want' was a call to action
for governments, businesses and the UN alike to support
countries interested in the transition to a green economy. PAGE
*It is a financial mechanism for five major international
(Partnership for Action on Green Economy) was created as the
conventions: the Minamata Convention on Mercury, the
Stockholm Convention on Persistent Organic Pollutants, the UN's direct response to this call of action.
United Nations Convention on Biological Diversity, the
United Nations Convention to Combat Desertification and
Organizations & Institutions
the United Nations Framework Convention on Climate *National and International organizations and institutions are
Change. established for environmental conservation and protection.

Environment and Ecology General Studies F–13


*National Environmental Engineering Research Institute Sustainable Development Goals in India.
(NEERI) is situated in Nagpur, Maharashtra. It was established Acts - Related to Environment
as Central Public Health Engineering Research Institute
Conservation
(CPHERI) in 1958 to focus on water supply, sewage disposal
*Some Acts are passed for promoting conservation of
and related diseases and to some extent on industrial pollution.
Environment. The Environment (Protection) Act was passed
*NEERI is a pioneering laboratory in the field of environmental
in India in 1986. In India, The Environment (Protection) Act,
science and engineering and part of Council of Scientific and
1986 is also known as Umbrella Legislation.
Industrial Research (CSIR).
*The 1986 Act was passed for the purpose of implementation
*NEERI has five zonal laboratories at Chennai, Delhi,
of decisions that were taken by United Nations Conference
Hyderabad, Kolkata & Mumbai. NEERI works under the
on the Human Environment held in Stockholm in 1973.
Ministry of Science & Technology of the Government of India.
*The Environment (Protection) Act, 1986 was enacted after
*The National Environment Appellate Authority (NEAA)
Bhopal Gas Tragedy under Article 253 of the Constitution.
better known as NEA was set up by the Ministry of Environment
This Act provides a framework to the Central Government
and Forest to address cases in which environment clearances
for coordination of the activities of various central and state
were required in certain restricted areas. It was established by
the National Environment Appellate Authority Act 1997 to authorities under various previous laws.

hear appeals with respect to restriction of areas in which any *The Genetic Engineering Approval Committee is constituted
industries, operations or processes shall or shall not be carried under the Environment (Protection) Act, 1986.
out, subject to certain safeguards under the Environment *On 22nd July, 2010 Genetic Engineering Approval
(Protection) Act, 1986. Committee was renamed as Genetic Engineering Appraisal
*The World Meteorological Organization (WMO) provides Committee.
a framework for international cooperation in the development *The National Green Tribunal was established on 18 October,
of meteorology and operational hydrology and their practical 2010 under the National Green Tribunal Act, 2010 for effective
application. WMO is an intergovernmental organization with a and expeditious disposal of cases relating to environmental
membership of 193 Member States and Territories WMO was protection and conservation of forest to provide the right to
originated from the International Meteorological Organization healthy environment, construed as a part of Right to a Life
(IMO) which was founded in 1873 to facilitate the exchange of under Article 21.
weather information across national borders. WMO became a *Justice Adarsh Kumar Goel is the present Chairperson of
specialized agency of the United Nations on 17 March, 1951. the National Green Tribunal.
*World Meteorological Convention was implemented *It was established to solve the problems relating to
on 23 March, 1950. WMO has its headquarters in Geneva, environmental safety, forest conservation, enforcement of
Switzerland. Its current Secretary-General is Petteri Taalas legal rights and to provide compensation to the victim for their
and President is Gerhard Adrian of Germany. property damage.
*UNEP (United Nations Environment Programme) was *This tribunal will help to reduce the number of trials
established in 1972. It is headquartered in Nairobi (Kenya). which are pending in high courts. Tribunal is bound to solve
Head of UNEP is Inger Andersen of Denmark. applications and petition within one month from the time of
*EPA (Environment Protection Agency) is an agency of the their filing.
U.S. It was established on 2 Dec. 1970 for enforcing standards
and laws that promote the health of human/individuals and the Environment - Some Important
environment. Facts For Exams
*Green Peace International is a non-governmental *Excess urbanization and industrialization is a challenge
organization. *Its headquarter is in Amsterdam, Netherlands. for sustainable development, ecology & environment and
*NITI (National Institution for Transforming India) Aayog biodiversity conservation.
is a Nodal Institution for coordination and implementation of *Green Development is a book written by W.M. Adams. *The

F–14 General Studies Environment and Ecology


full name of this Book is 'Green Development: Environment for well being of human society.
and Sustainability in a Developing World'. First edition of Reason (R) : Sustainable development is a kind of
this book was published in the year1990. development that meets the needs of
*Masanobu Fukuoka, the discoverer and innovator of Natural the present without compromising the
Farming, was a Japanese farmer and philosopher. ability of future generations to meet
*Australia launched Green Army for environmental their own needs.
Conservation in which 15000 youngsters of age between 17-24 Codes :
years old will be recruited. (a) Both (A) and (R) are true and (R) is correct explanation
*Under an agreement between Markfed of Punjab and Sayag of (A)
Group of Israel on 21 Apr 2001 the vegetables are grown by (b) Both (A) and (R) are true but (R) is not the correct
Green House farming in Punjab. explanation of (A)
*Mixed cropping, organic manures, nitrogen-fixing plants (c) (A) is true but (R) is false
and pest resistant crop varieties are the best strategy for (d) (A) is false but (R) is true
environment-friendly sustainable development in agriculture. U.P.P.C.S. (Pre) 2019
*The underlying dimension of SDG targets are closely related to Ans. (a)
education and health aspect as evident in the positive correlation
Sustainable development is important for well being of
between HDI (Human Development Index) and SDG rankings human society. Sustainable development has been defined
of Indian State. in many ways. The most frequently quoted definition is from
*World Environment Day is celebrated on 5th June. The first our common future, also known as the Brundtland Report :
World Environment Day was celebrated in 1974. "Sustainable development is development that meets the
*Stockholm Summit was held in 1972. It was the 1st World needs of the present without compromising the ability of
Summit on Environment. It was decided in this summit that future generation to meet their own needs".
World Environment Day will be celebrated on 5th June every 2. Arrange the following events in chronological order and
year. select the correct answer from the codes given below:
*In 2018 theme of World Environment Day was 'Beat I. Rio Earth Summit
Plastic Pollution'. India was the global host of 2018 World II. Publication of Brundtland Commission Report
Environment Day. III. Enforcement of Montreal Protocol
*World Environment Day 2019 was hosted by China with a IV. Publication of ‘The Limit to Growth’ Report
theme of "Air Pollution". Codes:
*In 2020, theme of World Environment Day is 'Biodiversity'. (a) I, IV, III, II (b) IV, II, III, I
This year's host is Colombia in partnership with Germany. (c) IV, III, II,I (d) IV, I, III, II
*Pakistan hosted World Environment Day 2021. This year's U.P. P.C.S. (Pre) 2020
observance of World Environment Day's theme was 'Ecosystem Ans. (b)
Restoration'.
Events Year
*World Environmental Day 2022 theme only one Earth Focus
Publication of ‘The Limit to 1972
on living sustainably in harmony with nature. Growth’ Report
*2022 marks 50 years science stockholm conference which Publication of Brundtland August, 1987
led to the designation of 5 June as World Environment Day. Commission Report
Enforcement of Montreal January, 1989
Basic Questions Protocol
Rio Earth Summit June, 1992
1. Given below are two statements, one labelled as
Assertion (A) and other as the Reason (R). 3. The discussion on ‘Sustainable Development’ was
Assertion (A) : Sustainable development is important started after submitting a report on environment to

Environment and Ecology General Studies F–15


the United Nations in 1987. That report was: U.P.U.D.A./L.D.A. (Pre) 2013
(a) First Report on Climate Change Ans. (b)
(b) Our Common Future Sustainable development and use of paper bags have a
(c) Second Report on Climate Change positive impact on the environment while air conditioning
(d) Fifth Assessment Report has a negative impact on the environment as it affects
U.P.P.C.S. (Pre) 2020 the ozone layer. Poverty alleviation has both positive and
Ans. (b) negative impact on the environment. Alternatively, poverty
After the submission of the report Our Common Future, is indirectly linked to the environment whereas sustainable
also known as the Brundtland Report, which was published development, use of paper bags and air conditioning directly
by World Commission on Environment and Development affect the environment. So, if we consider the positive impact
(WCED) in 1987, the discussion on 'Sustainable Development' on the environment from poverty alleviation, option (c) too
was started. may be the correct answer.

4. Environment refers to: 6. The Environment (Protection) Act of India was enacted
(a) The natural world of land, water, air, plants and in–
animals that exist around it. (a) 1981 (b) 1986
(b) The sum total of conditions which surround human (c) 1995 (d) 2000
beings at a given point of time. U.P.U.D.A./L.D.A. (Pre) 2013
(c) The interacting system of physical, biological and Ans. (b)
cultural elements which are interlinked. The Environment (Protection) Act of India was enacted in
(d) All the above the year 1986. It extends to the whole of India.
U.P.P.C.S. (Pre) 2006
7. When the Environment (Protection) Act for
Ans. (d) preservation and protection of the environment was
The word environment is derived from the French word passed by the Government of India?
‘Environ’ which means to 'encircle or surround'. The (a) 1986 (b) 1981
environment is a very broad concept and involves everything (c) 1987 (d) 1978
that affects an organism during its lifetime. It is the totality U.P. P.C.S. (Mains) 2017
of all social, biological, physical, and chemical elements Ans. (a)
individually as well as collectively that compose nature
See the explanation of the above question.
and human-made surroundings. It refers to the total of
conditions which surround man at a given point in space 8. In which of the following years the Environment
and time. The environment is defined as the social, cultural Protection Act was passed?
and physical conditions that surround, affect and influence (a) 1982 (b) 1986
the survival, growth and development of people, animals (c) 1990 (d) 1994
or plants. As defined in the Environment (Protection) Act, U.P.P.C.S. (Pre) 2014
1986, "environment includes water, air and land and the Ans. (b)
inter-relationship which exists among and between water, air and See the explanation of the above question.
land, and human beings, other living creatures, plants, micro-
9. The ‘Environmental Protection Act’ in India was
organisms and property. Hence (d) is the correct answer.
enacted in which of the following years?
5. Which of the following is not related to environmental (a) 1980 A.D. (b) 1986 A.D.
protection? (c) 1992 A.D. (d) 1994 A.D.
(a) Sustainable Development U.P.B.E.O. (Pre) 2019
(b) Poverty Alleviation Ans. (b)
(c) Air Conditioning
See the explanation of the above question.
(d) Use of Paper Bags

F–16 General Studies Environment and Ecology


10. Environment is a composite state of – Development) report of 1987 titled "Our common future".
(a) Biotic factors (b) Physiographic factors Sustainable development was fully supported in the
(c) Abiotic factors (d) All of the above. Agenda-21 (Rio Declaration) declared in Earth Summit held
53rd to 55th B.P.S.C. (Pre) 2011 in 1992. Sustainable development was a prominent issue
Ans. (d) of the Johannesburg Summit 2002. Notably, the United
The environment may be divided into biotic and abiotic Nations has identified 17 Sustainable Development Goals
environments. The biotic environment is made up of biotic (SDGs) which have to be realized by 2030. A Sustainable
factors, that is all living things in the environment. The abiotic Development Index has also been initiated to measure
or physical environment is made up of all the physical factors the progress made by different nations in the direction of
achieving these goals. Importantly India was ranked 120
that affect organisms, e.g. temperature, light intensity, the
out of 165 nations in Sustainable Development Index 2021.
amount of available water etc.
Sustainable development Index- 2021
11. Generally, environment can be classified as following
Position/Rank Country
categories. Which among the following is NOT a part
of such category? 1 Finland
(a) Operational Environment 2 Sweden
(b) Physical Environment 3 Denmark
(c) Cultural Environment
4 Germany
(d) Biological Environment
5 Belgium
U.P.P.C.S. (Pre) 2020
Ans. (a) 120 India
129 Pakistan
Generally, environment can be classified as physical, cultural
and biological environment. Physical environment includes 13. What is blue carbon?
landforms, water bodies, climate, soils, rocks, minerals, (a) Carbon captured by oceans and coastal ecosystems
etc. Biological environment includes plants, animals,
(b) Carbon sequestered in forest biomass and agricultural
microorganisms, etc. whereas cultural environment include
soils
man-made features such as economical, social and political
(c) Carbon contained in petroleum and natural gas
elements.
(d) Carbon present in atmosphere
12. Sustainable development is a matter of inter-
I.A.S (Pre) 2021
generational sensibility in respect of the use of?
Ans. (a)
(a) Natural resources (b) Material resources
(c) Industrial resources (d) Social resources Blue carbon is simply the term for carbon captured by the
U.P.P.C.S.(Pre) 2012 world's ocean and coastal ecosystems.
Ans. (a) Sea grasses, mangroves, and salt marshes along our coast
"capture and hold" carbon, acting as something called a
The concept of sustainable development was emphasized
by the United Nations Conference on Environment and carbon sink.
Development (UNCED), which defined it as ‘Development These coastal systems, though much smaller in size than the
that meets the need of the present generation without planet's forests, sequester this carbon at a much faster rate,
compromising the ability of the future generation to meet and can continue to do so for millions of years.
their needs. Sustainable development is a matter of inter-
14. 'Saving energy and other resources for the future
generational sensibility in respect of the use of natural
without sacrificing people's comfort in the present' is
resources. The main objective of sustainable development is
the just and prudent use of natural resources, it's conservation the definition of which of the following concepts?
and proper management. This term was explained in (a) Economic growth
the WCED (World Commission on Environment and (b) Economic development

Environment and Ecology General Studies F–17


(c) Sustainable development Ans. (c)
(d) Human development NITI (National Institution for Transforming India) Aayog is
U.P. P.C.S. (Pre) 2018 a Nodal Institution for coordination and implementation of
Ans. (c) Sustainable Development Goals in India.

See the explanation of the above question. 18. World Environment Day is celebrated on:
(a) 21 March (b) 23 September
15. Which of the following Sustainable Development Goal (c) 5 June (d) 25 May
(SDG) will target the water availability for all and its Jharkhand P.C.S. (Pre) 2013
permanent management upto 2030 in India? Uttarakhand Lower (Sub.) (Pre) 2010
(a) SDG - 6 (b) SDG - 7 Ans. (c)
(c) SDG - 8 (d) SDG - 9 World Environment Day is celebrated on 5 June every year.
U.P.P.C.S. (Pre) 2019 China was the global host of 2019 World Environment Day
Ans. (a) with "Air Pollution" as the theme of that year's edition. In
The United Nations has identified 17 SDGs (Sustainable 2021, theme of World Environment Day was 'Ecosystem
Development Goals-SDGs) which have to be realised by Restoration'.
the year 2030. Among these goals SDG-6 "Clean Water and 19. The World Environment Day is celebrated on –
Sanitation" ensures availability and sustainable management (a) December 1
of water and sanitation for all. Today, 2.2 billion people lack (b) June 5
access to safely managed drinking water, and more than 4.2 (c) November 14
billion people lack safely managed sanitation.
(d) August 15
16. The Decade of Sustainable Energy for all is the U.P.P.C.S. (Mains) 2014
initiative of: U.P.P.C.S.(Pre) 2012
(a) United Nations (b) India U.P.P.C.S. (Mains) 2011
(c) Germany (d) World Bank Uttarakhand U.D.A./L.D.A (Pre) 2007
U.P.P.C.S. (Pre) 2017 Uttarakhand P.C.S. (Pre) 2006
Ans. (a) U.P.P.C.S. (Mains) 2004
Renewable energy is a form of energy that is not the cause Ans. (b)
of pollution nor has its adverse effect on natural resources. See the explanation of the above question.
Sustainable energy is the seventh goal of the 17 UN Sus-
tainable Development Goals (SDGs), with a call to ensure 20. The book ‘Silent Spring’ published in 1962 which set
access to affordable, reliable, sustainable and modern energy the tone for an environmental movement in the world,
for all by 2030. The Sustainable Energy for all initiative, acts was written by:
in support of the 2014-2024 decade of Sustainable Energy (a) Carolyn Merchant
for All as declared by the UN General Assembly. (b) Karl Marx
17. Which of the following is a Nodal Institution for (c) Rachel Carson
implementation of Sustainable Development Goals (d) Rajgopalan
in India? U.P.P.C.S. (Pre) 2020
(a) Planning Commission Ans. (c)
(b) Disinvestment Commission ‘Silent Spring’ is an environmental science book published in
(c) NITI Aayog 1962. It was written by an American biologist and ecologist
(d) Finance Commission ‘Rachel Carson’.
U.P.P.C.S. (Pre) 2019 21. A country’s natural capital included all of the following

F–18 General Studies Environment and Ecology


except: 23. The main role of solar radiation is in –
(a) Forest (b) Water (a) Carbon cycle (b) Hydrogen cycle
(c) Roads (d) Minerals (c) Water cycle (d) Nitrogen cycle
U.P.P.C.S.(Pre) 2012 U.P. Lower Sub. (Spl.) (Pre) 2004
Ans. (c) U.P.P.C.S. (Pre) 2000
‘Natural capital includes all renewable and non-renewable Ans. (c)
environmental resources. It includes air, water, land, minerals The water cycle is driven primarily by the energy from the
and forests. In other words only those resources, which are sun. This solar radiation drives the cycle by evaporating water
created by nature, without any human interference, can be from the oceans, lakes, rivers and even the soil. It forms
deemed as natural capital. Hence, roads cannot be included water vapour and clouds, where water droplets eventually
in the country’s natural capital. gain enough mass to fall back to earth. The water cycle is
essentially a closed system, meaning that the volume of water
that is in the hydrosphere today is the same amount of water
that has always been present in the earth system.
24. National Environmental Engineering Research
Institute is situated at:
(a) Bangaluru (b) Hyderabad
(c) Nagpur (d) New Delhi
U.P.P.C.S. (Pre) 2014
U.P.P.C.S. (Pre) 2012
U.P.P.C.S (Mains) 2011
Uttarakhand P.C.S. (Pre) 2010
U.P.P.C.S (Mains) 2005
U.P. Lower Sub. (Pre) 2004
Ans. (c)
The National Environmental Engineering Research
Institute (NEERI) is a research institute created and funded
by the Government of India. It was established in Nagpur in
1958 as Central Public Health Engineering Research
22. What is the theme for 'World Environment Day 2018'? Institute (CPHERI) with focus on water supply, sewage
(a) Connecting People to Nature disposal and related diseases and to some extent on control
(b) One World, Our Environment of industrial pollution. NEERI is a pioneering laboratory in
the field of environmental science and engineering and part
(c) Beat Plastic Pollution
of the Council of Scientific and Industrial Research (CSIR).
(d) Save Water for Future
NEERI has five zonal laboratories at Chennai, Delhi,
U.P. R.O./A.R.O. (Pre) 2017
Hyderabad, Kolkata and Mumbai. NEERI falls under
Ans. (c)
the Ministry of Science and Technology, Government of India.
India was the global host of 2018 World Environment
Day (5 June). The theme of this year's edition was 'Beat 25. When and where the National Environment Engineering
Plastic Pollution'. China was the global host of 2019 'World Research Institute (NEERI) was established?
Environment Day'. The theme for 2019 was 'Air Pollution'. In (a) 1970, Dhanbad (b) 1958, Nagpur
2020, theme of World Environment Day was 'Biodiversity'. (c) 1956 ,New Delhi (d) 1960, Chennai
The theme of 2021 World Environment Day was 'Ecosystem U.P. P.C.S. (Mains) 2017
Restoration', Pakistan acted as global host of the day. Ans. (b)

Environment and Ecology General Studies F–19


28. Which one of the following is not the objective of smart
See the explanation of the above question.
city development?
26. What is important for Sustainable Development? (a) Good governance
(a) Conservation of biological diversity
(b) Clean green city
(b) Prevention and Control of Pollution
(c) Stabilizing quality of life
(c) Decreasing poverty
(d) Smart mobility
(d) All of these.
U.P. P.C.S. (Mains) 2017
U.P. Lower Sub. (Pre) 2004
Ans. (c)
Ans. (d)
The Smart City Mission is an innovative and new initiative
Conservation of biological diversity, prevention and control by the Government of India to drive economic growth and
of pollution, decreasing poverty, all are important for improve the quality of life of people by enabling local
Sustainable Development. It is imperative for achieving development and harnessing technology as a means to
the goal of sustainable development to make policies on create smart outcomes for citizens.
information about our environment and ecology. Presently The core infrastructure elements of the mission are-
sustainable development has become an alternative approach * Adequate water supply.
to the theories of development. Certain information are * Assured electricity supply.
required for policy formulation on sustainable development. * Sanitation including solid waste management.
27. The 'Common Carbon Metric' supported by UNEP, * Efficient urban mobility and public transport.
has been developed for * Affordable housing, especially for the poor.
* Robust IT connectivity and digitalization.
(a) assessing the carbon footprint of building operations
* Good Governance, especially e-Governance and people's
around the world
participation.
(b) enabling commercial farming entities around the world
* Sustainable environment.
to enter carbon emission trading
* Safety and security of citizens, particularly women,
(c) enabling governments to assess the overall carbon
children and the elderly.
footprint caused by their countries * Health and Education.
(d) assessing the overall carbon footprint caused by the
29. ‘Earth Summit’ was held at:
use of fossil fuels by the world in a unit time
(a) Cairo (b) Rio-de-Janeiro
I.A.S (Pre) 2021
Ans. (a) (c) Durban (d) Kyoto
Uttarakhand U.D.A./L.D.A. (Pre) 2003
Common carbon metric for measuring energy use and
reporting greenhouse gas emissions from building operations. Ans. (b)
Leading experts from around the world have, through The first Earth Summit by United Nations Conference on
extensive international cooperation, developed a universal Sustainable Development was held in Rio-de-Janeiro in
method of measuring a building's carbon footprint. 1992. In 2012, the United Nations Conference on Sustainable
The Common Carbon Metric is the calculation used to define Development was also held in Rio and is also commonly
measurement, reporting, and verification for GHG emissions called Rio+20 or Rio Earth Summit 2012.
associated with the operation of buildings types of particular
climate regions. 30. What was the title of the Rio -20 declaration?
While it is not a building rating tool, it is consistent with (a) The future we want (b) The future we seek
methods for assessing the environmental performance of (c) The future we have (d) The future we see
buildings used globally such as the World Business Council (e) None of these
for Sustainable Development (WBCSD), World Resources Chhattisgarh P.C.S. (Pre) 2015
Institute (WRI) GHG Protocol, etc.
Ans. (a)

F–20 General Studies Environment and Ecology


The United Nations Conference on Sustainable Development (c) 50% (d) 78%
(UNCSD) also known as Rio 2012, Rio+20 or Earth Summit M.P.P.C.S. (Pre) 1990
2012 was the third international conference on sustainable Ans. (d)
development hosted by Brazil in Rio-de-Janeiro. The primary
The air around us is a mixture of gases, mainly Nitrogen
result of the conference was the non-binding document “ The
and Oxygen, but containing much smaller amounts of water
future we want” a 49-page action plan.
vapour, Argon, and Carbon Dioxide and very small amounts
31. The first ‘Earth Summit’ was held at- of other gases.
(a) Washington (b) Geneva Gas Percentage
(c) Rio-de-Janeiro (d) Buenos Aires
Nitrogen (N2) 78.08%
M.P.P.C.S. (Pre), 1998
Ans. (c) Oxygen (O2) 20.95%

See the explanation of the above question. Argon (Ar) 0.93%

32. International Conference held in Rio-de-Janeiro was: Carbon Dioxide (CO2) 0.036%
(a) United Nations Conference on Sustainable Neon (Ne) 0.002%
Development Helium (He) 0.0005%
(b) Environment and Pollution Summit
Krypton 0.001%
(c) International new Economics
(d) None of these Xenon 0.00009%
M.P.P.C.S. (Pre) 1992 Hydrogen 0.00005%
Ans. (a)
36. Which of the following is considered to be the optimum
International conference held in Rio-de-Janeiro in 1992 was
concentration of Carbon-Dioxide for natural balance
the United Nations Conference on Sustainable Development. in atmosphere?
33. What is the thin layer of gases around the earth called? (a) 0.02 percent (b) 0.03 percent
(a) Biosphere (b) Water particle (c) 0.04 percent (d) 0.05 percent
(c) Atmosphere (d) Hydrosphere U.P.P.C.S. (Pre) 2017
44th B.P.S.C. (Pre) 2000 Ans. (b)
Ans. (c)
See the explanation of the above question.
The thin layer of gases surrounding the earth is called the
atmosphere. The majority of the Earth’s atmosphere consists 37. Which one of the following Noble Gases is not present
of a high amount of Nitrogen, along with smaller amounts in the air?
of Oxygen and Carbon Dioxide. (a) Helium (b) Argon
34. Air is a/an: (c) Radon (d) Neon
(a) Compound (b) Element U.P.P.C.S. (Pre) 2005
(c) Mixture (d) Electrolyte Ans. (c)
39th B.P.S.C. (Pre) 1994
See the explanation of the above question.
Ans. (c)
38. In the atmosphere, the highest percentage is of:
Air is a mixture of gases. By volume, about 78% of this
(a) Oxygen (b) Carbon dioxide
mixture is nitrogen and about 21% is oxygen. Carbon dioxide,
argon, methane, ozone and water vapour are also present in (c) Nitrogen (d) Hydrogen
very small quantities. Uttrakhand U.D.A./LDA (Mains) 2007
Ans. (c)
35. Percentage of Nitrogen in dry air –
(a) 21% (b) 27% See the explanation of the above question.

Environment and Ecology General Studies F–21


39. The highest percentage of gas in the air is – The decay of organic matter, volcanic action, respiration
(a) Hydrogen (b) Carbon Dioxide and burning of fossil fuel etc contribute carbon dioxide to
(c) Oxygen (d) Nitrogen the carbon cycle, whereas plants use carbon dioxide in the
41st B.P.S.C. (Pre) 1996 process of photosynthesis. Thus plants do not add carbon
Ans. (d) dioxide to the carbon cycle rather they release oxygen
See the explanation of the above question. and consume carbon dioxide. Three natural allotropes of
elemental carbon are – fullerene, graphite and diamond.
40. If all the plants and trees vanish from the earth, the Besides, as a compound, it is found as carbon dioxide,
gas which will decrease is – carbonate and Hydrogen carbonate.
(a) Carbon dioxide (b) Nitrogen
43. Which of the following does not add the carbon dioxide
(c) Water vapour (d) Oxygen to carbon cycle on the planet Earth?
R.A.S./R.T.S.(Pre) 2012 (a) Respiration
Ans. (d) (b) Photosynthesis
We need oxygen to survive and we depend on trees to (c) Decay of organic matter
provide this oxygen. To make food for themselves, trees (d) Volcanic action
go through a process known as photosynthesis where they U.P.P.C.S. (Mains) 2012
take in carbon dioxide from the air and release oxygen back Ans. (b)
into the atmosphere. Hence option (d) is the correct answer. See the explanation of the above question.

41. Which of the following is not a function of the tree? 44. The concept of weathering relates to –
(a) Absorption of carbon dioxide (a) Deposition of decomposed material
(b) Absorption of noise (b) Change in weather conditions day to day
(c) Air pollution (c) Natural process leading to the decomposition of rocks
(d) Releasing oxygen (d) None of the above
R.A.S./R.T.S. (Pre) 1993 Uttarakhand P.C.S. (Pre) 2006
Ans. (c) Ans. (c)
Trees contribute to the environment by providing oxygen, Weathering is defined as mechanical disintegration and
improving air quality, climate amelioration, conserving water, chemical decomposition of rocks through the actions of
preserving soil and supporting wildlife. Hence they clean various elements of weather and climate. Weathering
the environment and are not the reason behind air pollution. processes are conditioned by many complex geological,
In addition to this, trees on both sides of roads reduce the climatic, topographic and vegetative factors.
sound pollution by reducing the intensity of sound by 10 to
45. The Headquarter of World Meteorological Organization
15 decibels.
is located at –
42. Consider the following : (a) Geneva (b) Rome
1. Photosynthesis 2. Respiration (c) Sidney (d) Tokyo
3. Decay of organic matter 4. Volcanic action U.P.P.C.S. (Mains) 2010
Which of the above add carbon dioxide to the carbon Ans. (a)
cycle on Earth? The World Meteorological Organization (WMO) provides a
(a) 1 and 4 (b) 2 and 3 framework for international cooperation in the development
(c) 2, 3 and 4 (d) 1, 2, 3 and 4 of meteorology and operational hydrology and their practical
I.A.S (Pre) 2014 application. WMO originated from the International
I.A.S. (Pre) 2011 Meteorological Organization (IMO) which was founded in
Ans. (c) 1873 to facilitate the exchange of weather information across

F–22 General Studies Environment and Ecology


national borders. WMO became a specialized agency of Uttarakhand U.D.A./L.D.A. (Pre) 2003
the United Nations on 17 March, 1951. Notably, World Ans. (c)
Meteorological Convention was implemented on 23 March, Environment Protection Agency (EPA) is an agency of
1950. It has its headquarter in Geneva, Switzerland. the Federal Government of the United States which was
46. The headquarter of U.N.E.P. is located at – created for the purpose of protecting human health and the
(a) Geneva (b) Kathmandu environment by writing and enforcing regulations based
(c) Nairobi (d) New Delhi on laws passed by the Congress. It was established on 2
U.P.P.C.S. (Mains) 2010 December 1970.
U.P.P.S.C. (GIC) 2010 50. In which of the following institutes, the Environmental
Ans. (c) Information System (ENVIS)'s 'Centre on Population
UNEP (United Nations Environment Programme ) was and Environment' is located?
established in 1972. Its headquarter is in Nairobi (Kenya). (a) I.I.T. Kharagpur
Inger Andersen of Denmark is currently the Executive (b) I.I.T. New Delhi
Director of the United Nations Environment Programme. (c) I.I.P.S. Mumbai
(d) N.I.R.S. Dehradun
47. Where is the headquarter of the United Nations
U.P. P.C.S. (Mains) 2017
Environment Programme (UNEP)?
(a) The Hague (b) Nairobi Ans. (c)
(c) New York (d) Washington D.C. The Environmental Information System (ENVIS)'s 'Centre
Jharkhand P.C.S. (Pre) 2016 on Population and Environment' is located in International
Ans. (b) Institute for Population Sciences (IIPS), Mumbai.

See the explanation of the above question. 51. N.E.A. Stands for:
(a) National Enquiry Act
48. Where was the 4th session of the UN Environment
Assembly (UNEA-4) held recently? (b) National Environment Authority
(a) Paris-France (b) Ulaanbaatar-Mongolia (c) National Examination Agency
(c) Beijing - China (d) Nairobi-Kenya (d) National Event Association
(e) None of the above/More than one of the above Chhattisgarh P.C.S. (Pre) 2003
65th B.P.S.C. (Pre) 2019 Ans. (b)
Ans. (d) The National Environment Appellate Authority (NEAA)
The fourth session of the United Nations Environment better known as NEA was set up by the Ministry of
Assembly (UNEA-4) took place from 11-15 March 2019 Environment and Forest to address cases in which
in Nairobi, Kenya. The theme of UNEA-4 was "Innovative environment clearances were required in certain restricted
solutions for Environmental Challenges and Sustainable areas. It was established by the National Environment
Consumption and Production". It is noteworthy that the Appellate Authority Act 1997 to hear appeals with respect
fifth session of UNEA took place (Virtually) in Nairobi in to restriction of areas in which any industries, operations or
February 2021, under the theme, " Strengthening Actions for processes shall or shall not be carried out, subject to certain
Nature to Achieve the Sustainable Development Goals". A safeguards under the Environment (Protection) Act, 1986.
resumed in-person session of UNEA-5 is scheduled to take On the establishment of National Green Tribunal on 18th
place from 28th February-2 March, 2022. October, 2010, under National Green Tribunal Act, 2010,
the NEAA stands dissolved.
49. What does EPA stand for –
(a) Environment Pollution Agency 52. The headquarters of the Green Peace International
(b) Environmental Prohibition Agency is located at –
(c) Environment Protection Agency (a) Amsterdam (b) Canberra
(d) None of these (c) Ottawa (d) Nagasaki

Environment and Ecology General Studies F–23


U.P.P.C.S. (Mains) 2005 SDG (Sustainable Development Goal) India Index 2019-20,
Ans. (a) released by NITI Aayog, was launched on December 30th,
Greenpeace is a non-governmental environmental 2019. Kerala achieved the first rank in the Index with a score
organization with offices in over 55 countries. It is of 70 followed by Himachal Pradesh (69). Andhra Pradesh,
headquartered in Amsterdam, Netherlands. Greenpeace states Telangana and Tamil Nadu were ranked at the third position
its goal is to “ensure the ability of the Earth to nurture life in with the score of 67. In July 2021, the third edition of SDG
all its diversity”. India Index 2020-21 was released, with a composite score
of 75, Kerala topped the NITI Aayog's Index. Second and
53. Eco-Mark is given to an Indian product which is – third position was obtained by Tamil Nadu and Himachal
(a) Pure and unadulterated (b) Rich in protein Pradesh respectively.
(c) Environment-friendly (d) Economically viable
56. Which of the following states is NOT in the list of
U.P.P.C.S. (Pre) 2012 top five states on the Sustainable Development Goals
Ans. (c) Index, 2019-20 India?
Eco-mark is a certification mark issued by the Bureau of (a) Gujarat (b) Himachal Pradesh
Indian Standards (The National Standards Organization of (c) Andhra Pradesh (d) Tamil Nadu
India) to products conforming to a set of standards aimed U.P.B.E.O. (Pre) 2019
at ensuring the least impact on the ecosystem. The marking Ans. (a)
scheme was started in 1991. According to NITI Aayog's Index-2019 top-5 states and
54. To label environment-friendly consumer products, the respective scores are following.

Government has introduced:


State Score
(a) Agmark (b) Eco mark
Kerala 70
(c) ISI Mark (d) Watermark
Himachal Pradesh 69
U.P.P.C.S. (Spl) (Mains) 2004
Andhra Pradesh 67
Ans. (b)
Tamil Nadu 67
Eco Mark scheme was instituted by the Government of Telangana 67
India for labeling of environment friendly products. Notably Karnataka 66
this scheme is being administered by the Bureau of Indian
Standards. It is important to mention here that the scheme 57. Sustainable agriculture means:
covers products categories like Soaps and detergents, Food (a) Self-sufficiency
items, Lubricating oils, Packing/packaging materials, (b) To be able to export and import under WTO norms
Architectural paints and powder coatings, Batteries, Electric (c) To utilize land so that its quality remains intact
and Electronic Goods, Food additives, Wood substitutes, (d) To utilize Wasteland for agricultural purposes
Cosmetics, Aerosols and propellants, Plastic products, U.P.P.C.S.(Pre) 2001
Textiles, Fire-extinguisher Leather and coir & coir products.
Ans. (c)
55. According to ‘Sustainable Development Goal (SDG)
Sustainable agriculture is the production of food, fibre or other
India Index, 2019’ recently released by NITI Aayog,
plants or other animal products using farming techniques that
which State holds the first position?
protect the environment, public health, human communities
(a) Uttar Pradesh (b) Bihar and animal welfare. In short, it is proper utilization of the
(c) Jharkhand (d) Kerala land without affecting the environment so that it's quality
U.P.B.E.O. (Pre) 2019 remain intact. Use of chemical fertilizers and pesticides has
Ans. (d) adversely affected soil health. To cope up with

F–24 General Studies Environment and Ecology


this, National Mission for Sustainable Agriculture (NMSA) (a) For balanced development
has been formulated for enhancing agricultural productivity, (b) Environment and ecology
water use efficiency, promoting nutrient management, and (c) For biodiversity conservation
livelihood diversification through the adoption of sustainable (d) For all the above.
development pathways. U.P.P.C.S. (Mains) 2002

58. Which one of the following States in India has recently Ans. (d)
introduced the ‘Greenhouse Farming’: Extreme urbanization and industrialization are harmful
(a) Haryana (b) Punjab to balanced development, environment and ecology and
(c) Maharashtra (d) Uttar Pradesh biodiversity conservation. Thus option (d) is the correct
U.P.P.C.S.(Pre) 2003
answer.
U.P.P.C.S.(Pre) 2001
Ans. (b)
High-Level Questions
Markfed of Punjab and Sayag Group of Israel signed an
agreement to grow vegetables in Greenhouses Farming on 21 1. The National Green Tribunal Act, 2010 was enacted
April, 2001. Greenhouse farming is the unique farm practice in consonance with which of the following provisions
of growing crops within sheltered structures covered by a of the Constitution of India?
transparent or partially transparent material. 1. Right to a healthy environment, construed as a
59. Given below are two statements – part of Right to Life under Article 21.
Assertion (A) : Natural vegetation is the true index of 2. Provision of grants for raising the level of
climate. administration in the Scheduled Areas for the
Reason (R) : Water-loving plants are found in a welfare of Scheduled Tribes under Article 275(1).
moist climate. 3. Powers and functions of Gram Sabha as mentioned
In the context of the above, which one of the following under Article 243(A).
is correct? Select the correct answer using the codes given below:
Codes : (a) Only 1 (b) 2 and 3
(a) Both statements are true and (R) is the correct (c) 1 and 3 (d) 1, 2 and 3
explanation of (A). I.A.S. (Pre) 2012
(b) Both statements are true but (R) is not the correct Ans. (a)
explanation of (A). The National Green Tribunal was established on 18
(c) (A) is true, but (R) is false. October, 2010 under the National Green Tribunal Act, 2010
(d) (A) is false, but (R) is true. for effective and expeditious disposal of cases relating to
U.P.P.C.S. (Pre) 2009 environmental protection and conservation of forest to
Ans. (a) provide the right to a healthy environment, construed as a
Natural vegetation is the true index of climate and an increase part of Right to Life under Article 21. It draws inspiration

in the quantity of carbon dioxide in the atmosphere also from the constitution of India (Article 21) which assures the
citizens of India the right to a halthy environment. Notably
affects the natural vegetation. Water-loving or Aquatic plants
on 18th October 2010, Justice Lokeshwar Singh Panta became
are those which are found mostly in muddy land and near
its first chairman. Present chairman of NGT is Justic Adarsh
the ponds. They are found in the moist climate. Reduction
Kumar Goel.
of moisture in the atmosphere results in the erosion of such
plants. Hence both Assertion (A) and Reason (R) are correct 2. The National Green Tribunal (NGT) came into force in:
(a) October, 2010 (b) November, 2008
and Reason (R) is the correct explanation of Assertion (A).
(c) January, 2011 (d) April, 2012
60. Urbanization and industrialization are harmful- U.P. P.C.S. (Mains) 2017

Environment and Ecology General Studies F–25


Ans. (a) Uttarakhand P.C.S. (Pre) 2016
See the explanation of the above question. Ans. (c)

3. National Green Tribunal (NGT) was established by the In the year 1997, the World Environment Conference was
Government of India in: organised in Kyoto, Japan. The Kyoto Protocol to the United
(a) 2008 (b) 2009 Nations Framework Convention on Climate Change was
(c) 2010 (d) 2011 adopted in this conference which came into existence on 16
February, 2005.
Jharkhand P.C.S. (Pre) 2016
Ans. (c) 7. The Earth Summit + 5 was held in?
See the explanation of the above question. (a) 2005 (b) 2000
(c) 1999 (d) 1997
4. The author of ‘Green Development’ is –
U.P.P.C.S. (Mains) 2016
(a) M.J. Bradshaw (b) M. Nicolson
Ans. (d)
(c) R.H. Whittaker (d) W.M. Adams
In 1992, more than 100 heads of states met in Rio-de-Janeiro,
U.P.P.C.S (Pre) 2011
Brazil for the first International Earth Summit. This summit
Ans. (d)
was convened to address urgent problems of environmental
William M. Adams is the author of the book Green protection and socio-economic development. The assembled
Development: Environment and Sustainability in a leaders signed the convention on climate change and con-
Developing World. Its first edition was published in 1990. vention of Biodiversity, endorsed the Rio-Declaration and
the Forest Principles and adopted Agenda 21 for achieving
5. What is Rio+20 Conference, often mentioned in the
sustainable development in the 21st century. The Commission
news?
on Sustainable Development (CSD) was created to monitor
(a) It is the United Nations Conference on Sustainable
and report on the implementation of the Earth Summit Agree-
Development.
ment. It was agreed that a five-year review of Earth Summit
(b) It is a Ministerial Meeting of the World Trade progress would be made in 1997 by the United Nations
Organization. General Assembly meeting in special session. Accordingly, a
(c) It is a Conference of the Inter governmental Panel on special session of UN General Assembly was held from 23-27
Climate Change. June, 1997 in New York to take stock of how well countries,
(d) It is a Conference of the Member Countries of the international organizations and sectors of civil society have
Convention on Biological Diversity. responded to the challenge of the Earth Summit. This meeting
I.A.S. (Pre) 2015 was known as Earth Summit +5. Notably, Rio + 10 summit
was held in Johannesburg in 2002 and Rio +20 was held in
Ans. (a)
Rio-de-Janeiro in 2012.
Rio+20 is the short name for the United Nations Conference 8. Consider the following statements:
on Sustainable Development. This conference took place
1. The Sustainable Development Goals were first
in Brazil in June 2012 to mark the 20th anniversary of the
proposed in 1972 by a global think tank called the
1992 United Nations Conference on Environment and
‘Club of Rome’.
Development (UNCED), in Rio-de-Janeiro, and the 10th
2. The Sustainable Development Goals have to be
anniversary of the 2002 World Summit on Sustainable
achieved by 2030.
Development (WSSD) in Johannesburg.
Which of the statements given above is/are correct?
6. In the year 1997, the World Environment Conference (a) 1 only (b) 2 only
was organised in? (c) Both 1 and 2 (d) Neither 1 nor 2
(a) Reo-de-Janeiro (b) Nairobi I.A.S. (Pre) 2016
(c) Kyoto (d) New York Ans. (b)

F–26 General Studies Environment and Ecology


Sustainable Development Goals were proposed in 2012 at 11. With reference to the 'Global Alliance for Climate-
Smart Agriculture (GACSA)', which of the following
Rio+20. Hence statement 1 is not correct. The Sustainable
statement is/are correct?
Development Goals are officially known as 'Transforming
1. GACSA is an outcome of the Climate Summit held
our World: the 2030 Agenda for Sustainable Development'.
in Paris in 2015.
These are a set of seventeen aspirational “Global Goals”
2. Membership of GACSA does not create any
with 169 targets among them. Hence, statement 2 is correct.
binding obligations.
9. Who has propounded the concept of 'Limits to 3. India was instrumental in the creation of GACSA.
Growth'? Select the correct answer using the code given below:
(a) Club of Rome (a) 1 and 3 only (b) 2 only
(b) UNESCO (c) 2 and 3 only (d) 1, 2 and 3
(c) Brundtland Commission I.A.S. (Pre) 2018
(d) Agenda 21 Ans. (b)
U.P.P.C.S. (Pre) 2019 Statement (1) is incorrect because the concept of Climate
Ans. (a) Smart Agriculture (CSA) was originally developed by FAO in
In April of 1968, 30 people including scientists, educators, 2010 through the paper "Climate Smart Agriculture: Policies,
economists, humanists, industrialists and government Practices and Financing for food security, Adaptation and
officials met at the Academia dei Lincei in Rome. "The Mitigation." In 2014 an alliance was set up with this issue
Club of Rome" was born from this meeting as an informal as its focal point: the GASCA (Global Alliance for Climate
organization. The concept of 'Limits to Growth' was Smart Agriculture).
propounded by the Club of Rome. Statement (2) is correct because membership in the Alliance
does not create any binding obligations.
10. Given below are two statements, one labelled as
Statement (3) is incorrect because India is just a signatory.
Assertion (A) and other as the Reason (R).
Assertion (A) : There is a positive relation between 12. The Partnership for Action on Green Economy
Human Development Index (HDI) (PAGE), a UN mechanism to assist countries transition
and Sustainable Development Goals towards a greener and more inclusive economies,
(SDG) ranking of Indian State. emerged at?
Reason (R) : The underlying dimension of SDG (a) The Earth Summit on Sustainable Development 2002,
targets are closely related to education Johannesburg.
and health aspect. (b) The United Nations Conference on Sustainable
Select the correct answer using the codes below: Development 2012, Rio de Janeiro.
(a) Both (A) and (R) are true and (R) is correct explanation (c) The United Nations Framework Convention on
of (A) Climate Change 2015, Paris.
(b) Both (A) and (R) are true but (R) is not the correct (d) The World Sustainable Development Summit 2016,
explanation of (A) New Delhi.
(c) (A) is true but (R) is false I.A.S. (Pre) 2018
(d) (A) is false but (R) is true Ans. (b)
U.P.P.C.S. (Pre) 2019
In 2012, Rio +20 (the United Nations Conference on
Ans. (a)
Sustainable Development) was held in Brazil. The conference's
The underlying dimension of SDG targets are closely related outcome document entitled 'The Future We Want' was a call to
to education and health aspect as evident in the positive action for governments, businesses and the UN alike to support
correlation between HDI (Human Development Index) and countries interested in the transition to a green economy.
SDG rankings of Indian State. Hence, both (A) and (R) are PAGE (Partnership for Action on Green Economy) was
true and (R) is the correct explanation of (A). created as the UN's direct response to this call of action.

Environment and Ecology General Studies F–27


13. With reference to ‘Agenda 21’, sometimes seen in the 16. Which of the following is related to Protection of
news, consider the following statements: Ecological Balance?
1. It is a global action plan for sustainable development. 1. Forest Policy
2. It originated in the World Summit on Sustainable 2. Environment (Protection) Act
Development held in Johannesburg in 2002. 3. Industrial Policy
Which of the following statements given above is/are 4. Educational Policy
correct? Select the correct answer using the codes given below:
(a) 1 only (b) 2 only (a) 1 & 2 (b) 2 & 3
(c) Both 1 and 2 (d) Neither 1 nor 2 (c) 1,2 & 3 (d) All the above
I.A.S. (Pre) 2016 U.P.P.C.S. (Spl) (Mains) 2004
Ans. (a) Ans. (d)
Protection of the ecological balance is directly related
Agenda 21 is a non-binding, voluntarily implemented action
to the Forest Policy and the Environmental (Protection)
plan of the United Nations on Sustainable Development.
Act, 1986. While Industrial Policy and Education Policy
It is a product of the Earth Summit (UN Conference on
also include the environmental approach. Thus the four
Environment and Development) held in Rio-de-Janeiro,
policies mentioned in question could be considered as
Brazil in 1992. Hence only statement 1 is correct.
being related to the protection of ecological balance.
14. Many transplanted seedlings do not grow because:
17. Concerning ‘Global Environment Facility’, which of
(a) The new soil does not contain favourable minerals
the following statement/s is/are correct?
(b) Most of the root hairs grip the new soil too hard
(a) It serves as the financial mechanism for ‘Convention
(c) Most of the root hairs are lost during transplantation
on Biological Diversity’ and ‘United Nations
(d) Leaves get damaged during transplantation
Framework Convention on Climate Change’
I.A.S. (Pre) 2013
(b) It undertakes scientific research on environmental
Ans. (c)
issues at the global level
Many transplanted seedlings do not grow because most of the (c) It is an agency under OECD to facilitate the transfer
root hairs are lost during transplantation. The function of root of technology and funds to underdeveloped countries
hairs is to collect water and mineral nutrients present in the with a specific aim to protect their environment
soil and take this solution up through the roots to the rest of (d) Both (a) and (b)
the plant. These are made up of cellulose and nominal Pectin I.A.S. (Pre) 2014
which helps it as a gelling agent with the soil. Another reason Ans. (a)
is the use of fertilizers which increases the concentrations of The Global Environment Facility (GEF) was established
the soil and make it difficult for plants to absorb water. This on the eve of the 1992 Rio Earth Summit to help tackle
situation causes plant cells causing the leaves turn brown. our planet's most pressing environmental problems. GEF
is an international partnership of 184 countries and 18
15. Environmental degradation means –
implementing agencies that address global environmental
(a) Overall degradation of environmental attributes.
issues. It is a financial mechanism for five major international
(b) Adverse changes brought in by human activities.
conventions: the Minamata Convention on Mercury, the
(c) Ecological imbalance because of ecological diversity.
Stockholm Convention on Persistent Organic Pollutants,
(d) All the above.
the United Nations Convention on Biological Diversity,
U.P.P.C.S. (Pre) 2006
the United Nations Convention to Combat Desertification
Ans. (d) and the United Nations Framework Convention on Climate
Environmental degradation is the deterioration of the Change. GEF supports multi-stakeholder alliance to preserve
environment through depletion of resources such as air, water threatened ecosystems on land and in the oceans, build green
and soil. Environmental degradation is either natural or by cities, boost food security for a more prosperous, climate
human beings. Hence (d) is the correct answer. resilient world. Carlos Manuel Rodriguez is presently the

F–28 General Studies Environment and Ecology


Chief Executive Officer and Chairperson of the Global The Genetic Engineering Appraisal Committee is constituted
Environment Facility. under the Environment (Protection) Act, 1986. On 22 July,
2010 Genetic Engineering Approval Committee was renamed
18. Plachimada which has been in the news for immense
as Genetic Engineering Appraisal Committee. It is the apex
damage to its environment lies in –
body constituted in the Ministry of Environment and Forest
(a) Karnataka (b) Konkan
under 'Rule for manufacture, use, import, export and storage
(c) Kerala (d) Tamil Nadu of hazardous microorganism/genetically engineered organism
U.P. Lower Sub. (Spl.) (Pre) 2009 or cells 1989' under the Environment (Protection) Act 1986.
Ans. (c)
21. With reference to the circumstances in Indian
Plachimada is a small village in Palakkad district of Kerala. agriculture, the concept of "Conservation Agriculture"
It is situated in Chittur block’s Perumatty Panchayat of the assumes significance. Which of the following fall under
district. In 2000, this Panchayat gave license to the Hindustan the Conservation Agriculture?
Coca-Cola Beverages Private Limited (HCCBPL) to 1. Avoiding monoculture practices.
manufacture Coca-Cola (Coke) and other products viz. Fanta, 2. Adopting minimum tillage.
Limca, Thums Up, Sprite, Kinley, etc. But very soon due to the 3. Avoiding the cultivation of plantation crops.
contamination of groundwater in that locality by the chemical 4. Using crop residues to cover the soil surface.
waste discharged by the company, local people started a 5. Adopting spatial and temporal crop sequencing / crop
small-scale protest against the company. Later, this protest rotations.
was supported by the Panchayat authorities. In the long run, Select the correct answer using the code given below:
with national as well as international support to this protest (a) 1, 3 and 4 (b) 2, 3, 4 and 5
against a multinational company (MNC), this movement (c) 2, 4 and 5 (d) 1, 2, 3 and 5
became an icon of other such movements worldwide. I.A.S. (Pre) 2018
19. Environment (Protection) Act (EPA) is also known as: Ans. (c)
(a) Umbrella Legislation (b) Chhadi Mubarak Conservation Agriculture (CA) can be defined by a statement
(c) Environment Legislation (d) Eco Safety Law given by the Food and Agricultural Organisation of the United
M.P.P.C.S. (Pre) 2013 Nations as "a concept for resource - saving agricultural crop
Ans. (a) production that strives to achieve acceptable profits together
Environment (Protection) Act, 1986 is also know as with high and sustained production level while concurrently
Umbrella Legislation. In the wake of the Bhopal Tragedy, the conserving the environment."
Government of India enacted the Environment (Protection) Minimum tillage and soil disturbance.
Act of 1986 under Article 253 of the Constitution. The Act Permanent soil cover with crop residues and live mulches.
is an “umbrella” legislation designed to provide a framework Crop rotation and intercropping are the principles of
for Central Government coordination of the activities of Conservation Agriculture.
various Central and State authorities established under 22. As a result of their annual survey, the National
previous laws, such as the Water Act and the Air Act. Geographic Society and an international polling firm
20. The Genetic Engineering Appraisal Committee is Globe Scan gave India top rank in Greendex 2009
constituted under the: score. What is this score?
(a) Food Safety and Standards Act, 2006 (a) It is a measure of efforts made by different countries in
(b) Geographical Indications of Goods (Registration and adopting technologies for reducing the carbon footprint
Protection) Act, 1999 (b) It is a measure of environmentally sustainable
(c) Environment (Protection) Act, 1986 consumer behaviour in different countries
(d) Wildlife (Protection) Act, 1972 (c) It is an assessment of programs/schemes undertaken
I.A.S. (Pre) 2015 by different countries for improving the conservation
Ans. (c) of natural resources

Environment and Ecology General Studies F–29


(d) It is an index showing the volume of carbon credits Ans. (d)
sold by different countries According to the census of 2011 approximately 121 crore
I.A.S. (Pre) 2010 people (68.9%) still live in rural areas as compared to 37.7
Ans. (b) crores (31.1%) people living in the urban areas. Although
National Geographic conducted its inaugural Greendex survey the rate of urbanization in India is high yet the process
in January 2008. That first Greendex survey of 14 countries lacks solutions to provide basic infrastructural amenities
around the world ranked average consumers in those countries like Drinking Water, Transport, Sanitation, Electricity, etc.
according to the environmental sustainability of their behaviour. 25. The investigator of Natural Farming was-
Thus, option (b) will be the right answer. In 2009, India was at (a) Masanobu Fukuoka (b) M.S. Randhawa
first place and in 2014 too India retained its position.
(c) M.S. Swaminathan (d) Norman Borlaug
23. Which one of the following is the best strategy for Uttarakhand P.C.S. (Pre) 2002
environment-friendly sustainable development in Ans. (a)
Indian agriculture?
Masanobu Fukuoka was a Japanese farmer and philosopher,
(a) Expansion of cultivable land, increased use of famous for his natural farming and revegetation of desertified
superphosphate, urea and effective biocides lands.
(b) Wider popularization of high-yielding crop varieties,
26. Which country introduced the Green Army for
better aid, more frequent irrigation and increased
Environment conservation?
frequency of aerial span of inorganic fertilizers and
(a) Japan (b) China
pesticides
(c) Australia (d) Egypt
(c) Mixed cropping, organic manures, nitrogen-fixing
plants and pest-resistant crop varieties M.P.C.S.(Pre) 2014
(d) Improved farm implements and machinery, use of Ans. (c)
potent insecticides to minimize post-harvest grain The Green Army is a six-month programme for 17-24-year-
losses and monoculture cropping practices old youths in Australia to train and work for the environment.
I.A.S. (Pre) 1993 Green Army projects include restoring native vegetation,
Ans. (c) heritage restoration, protecting animal habitats and
regenerating wetlands in urban, rural and remote areas. The
Mixed cropping, organic manures, nitrogen-fixing plants Green Army Programme closed on 30 June 2018.
and pest resistant crop varieties are the best strategy for
environment-friendly sustainable development in Indian 27. During the mid 80’s decade of twentieth century, the
agriculture. negative aspects of development were discussed at

24. Consider the following statements and select the large. Which one among the following was NOT such
correct answer from the codes given below: important?
Assertion (A) : India is a case of an over-urbanised (a) Degradation of natural resources
country. (b) Environment pollution
Reason (R) : Most of the large cities in India do not (c) Politics and Development
have adequate infrastructure. (d) Displacement and Rehabilitation of masses
Codes: U.P.P.C.S.(Pre) 2020
(a) Both (A) and (R) are correct and (R) explains (A). Ans. (c)
(b) Both (A) and (R) are correct but (R) does not explain During the mid 80’s decade of twentieth century, Degradation
(A). of natural resources, Environment pollution and Displacement
(c) (A) is true but (R) is false. and Rehabilitation of Masses were the negative aspects
(d) (A) is false but (R) is true. of development, discussed at large, but Politics and
U.P. P.C.S. (Mains) 2017 Development was not such important.

F–30 General Studies Environment and Ecology


Ecology
*Ecology is the branch of biology that explains the relationship *Forbes called ecosystem as Microcosm.
between organism and their environments. *Ecosystem is known as the functional unit of nature. *An
*The term 'Ecology' was coined by the German zoologist, Ernst ecosystem can be as small as water droplets. These kinds
Hackle in 1866 to describe the 'economics' of living forms. of ecosystem are known as Micro ecosystem. On the other
*Ernst Haeckel used the term Oekologie instead of Ecology hand, it can be as large as an ocean. Our earth is itself an huge
and explained it briefly. The word Oekologie is formed by ecosystem.
combining two Greek words Oikos (House or living space) *Biosphere is a thin layer of soil, rock, water and air. It is a
and logos (Science or Study). basic global system having biotic and abiotic components.
*According to Haeckel ‘Ecology is the study of the relation *Ecosystem is a functional unit of nature in which biotic factors
of the animal both to its organic as well as its inorganic interact with abiotic factors and forms a living unit.
environment’. Characteristics of Ecosystem - *It covers/occupies a definite
*According to Eugene Odum, ‘Ecology is the study of area in the Biosphere. *The ecosystem is a functional unit of
structure and function of nature’. nature.
*Ecosystem has its own productivity. Ecosystem is an open
system where there are continuous input and output of energy
and substances.

Components of Ecosystem
*An ecosystem is a community of living organisms in
conjunction with the non-living components of their
environment (things like air, water and mineral soil), interacting
as a system.
Important features of the ecosystem are:
*It is normally an open system with a continuous, but variable
influx and loss of materials and energy.
*An ecosystem is an overall integration of the whole mosaic
of the interacting organisms and their environment.
*It is a basic functional unit.
*It consists of biotic and abiotic components interacting with
*Now, ecology has been presented with a broad dimension. each other.
Now it also includes the study of humans, society and the *It’s functional unit is capable of energy transformation,
activities of its physical environment. It is a fundamental natural circulation and accumulation.
system having biotic and abiotic components. *Different types of ecosystem are present in different areas.
*In 1935, A.G Tansley proposed the concept of Ecosystem. *All components that take part in mutual interaction in the
According to him, Ecosystem is type of a physical system biosphere form an ecosystem. Living organisms form biosphere
formed by biotic and abiotic components. According to him, by different biological associations.
ecosystem is an open system. These systems may differ in *All the organisms and the abiotic components of the
their size. environment form the ecosystem of the place. Thus, ecology
*Karl Mobius called the idea of the ecosystem as Biocoenosis. deals with the organism and its environment.

Environment and Ecology General Studies F–31


some of the examples of artificial ecosystems.
*Ecosystems can broadly be categorized as Aquatic and
Terrestrial ecosystems. Grasslands, forests and deserts are the
examples of terrestrial ecosystems while lakes, rivers and
oceans fall under aquatic ecosystem.
*Ecosystems can be classified as:

*The correct order of biological organization ranging from


organism to biosphere is –
*Under man- made ecosystem Agriculture has been instrumental
to the existence of society and development.
*Aquatic ecosystem refers to the living and non-living parts of
a water body and the interactions that take place among them.
*Aquatic ecosystems can be categorized in the marine
ecosystem and freshwater ecosystem.
*Marine ecosystem covers approximately 70% of the earth's
surface and contain approximately 97% of the planet's water.
*Approximately 85 % of the dissolved materials in seawater
are sodium and chlorine.
* Another feature stratification is observed in the marine
*The number of living beings of the same species is known as ecosystem.
their population. The population of two or more than two ethnic
groups creates a community. Several communities together
form an ecosystem. In a fixed terrain one or more ecosystems
(Aphotic Zone)
are found. These ecosystems together form a Landscape of a
particular region.
(Benthic Zone)
Components of Ecology
(Abyssal Zone)

*Biotic components are the living beings things that shape


an ecosystem.
*Biotic components usually include:
Producers: e.g. plants.
Consumers: e.g. animals, humans
Decomposers: e.g. fungi and bacteria *The oceans are the cradle of life on Earth which harbor the
*An artificial ecosystem generally fulfill all the criteria of highest level of biodiversity. No terrestrial ecosystem can match
a natural eco-system but is made and controlled by humans. the biological richness of a coral reef, not only in some species
Orchards, farmlands, gardens and human-made reservoirs are but also in variety and originality of survival strategies.

F–32 General Studies Environment and Ecology


Ecological Imbalance The biotic potential is the maximum reproductive capacity
of an organism under optimum environmental conditions. It
*The disruption of the natural balance of an ecosystem by
is denoted with the letter 'r'. However, there is a limit beyond
natural or human-caused disturbances is known as ecological
which the environment can not sustain organismal growth, this
imbalance. However Wildlife conservation and prevention of
limit is called carrying capacity.
environmental pollution is helpful in maintaining ecological
balance. Food Web or Food Chain
Deforestation is one of the main causes of ecological imbalance *The most obvious aspect of nature is that energy must pass
in India. While desertification, floods and famines and rainfall from one living organism to another.
variations are secondary causes of deforestation. *When herbivorous animals feed on plants, energy is transferred
from plants to animals. In an ecosystem, some of the animals
feed on other living organisms while some feed on dead organic
matter. It creates the food chain. Each chain usually has only
four to five such links.
*Any substance consumed to provide nutritional support for
an organism is called food.
*Food chain depicts the flow of energy through different
organisms in an ecosystem. Notably, this flow is unidirectional.
The correct order of different components in one of the food
chain of grassland is as follows:
Carrying Capacity
*The carrying capacity of a biological species in an Grass o Grasshopper o Frog o Snake
environment is the maximum population size of the species pppp
that the environment can sustain indefinitely, given the (Producer) (Primary Consumer) (Secondary Consumer) (Tertiary Consumer)

food, habitat, water and other necessities available in the *Grass, goat and human also form a food chain –
environment.
Grass Goat Humans
*Carrying capacity is the number of individuals an environment
ppp
can support without significant negative impacts on the given
(Autotroph) (Herbivorous consumer) (Omnivorous consumer)
organism and its environment. Whenever the population
increases more than the carrying capacity, it puts pressure on *In food chain there are trophic levels i.e., Autotroph, Primary
the ecosystem. consumer, Secondary consumer and Tertiary consumer.
*Excessive population growth can lead to the failure of the *The primary consumer is the herbivores (that feed on the
ecosystem. plant), the secondary consumer is small carnivores (feeding
on the primary consumer) and the tertiary consumer is the
large carnivores.
*Man is the primary as well as a secondary consumer who feeds
on both plant products and meat.

*Phytoplankton, also known as microalgae, are similar to


terrestrial plants in that they contain chlorophyll and require
sunlight to live and grow. Most phytoplanktons are buoyant and
float in the upper part of the ocean, where sunlight penetrates
the water. Phytoplankton also requires inorganic nutrients such
as nitrates, phosphates, and sulfur which they convert into
proteins, fats, and carbohydrates.

Environment and Ecology General Studies F–33


*In a food chain mainly herbivores, are considered as primary
consumers.
*Deer is herbivorous hence it is a primary consumer.
*Ants fall under both decomposer, primary consumers and
scavengers. It eats plants and receives energy from cellulose.
*Tiger is carnivorous and fox is omnivorous.
*Decomposers are nature’s recyclers as they break down the
organic matter found in the dead bodies of plants and animals.
Biosphere is the sum total of all ecosystems present on
*The term ‘organic matter’ refers to the matter that comes
the Earth.
from living organisms.
According to the 1st law of thermodynamics ‘Energy
*Decomposers are mainly the micro-organisms such as bacteria
can neither be created nor be destroyed. Energy can
and fungi in the form of yeasts and moulds that break down be changed from one form into another form’.
bodies of dead organisms and release compounds that can be According to the 2nd law of thermodynamics, during the
used by producers. process of energy conversion, a part of the energy is al-
*A common marine food chain is as follows – ways lost in the form of heat so the product will always
have lesser energy than their constituent matters from
Diatom which product is formed.
p Magnitude of energy always decreases when it passes
from one trophic level to another.
Crustaceans
It is notable that Herbivores can only assimilate 10% of
p the gross productivity of products.
Herring *The Ten percent law of transfer of energy from one trophic
level to the next was introduced by Raymond Lindeman
Flow of Energy in 1942.
*The flow of energy in the ecosystem is unidirectional (or *According to this law, during the transfer of energy from

one-directional). The energy enters the plants (from the sun) organic food from one trophic level to the next, only about
ten percent of the energy from organic matter is stored as
through photosynthesis during the making of food. This
flesh. The remaining is lost during transfer, broken down
energy is then passed on from one organism to another in a
in respiration or lost to incomplete digestion by higher trophic
food chain. The energy given out by the organisms as heat is
level.
lost to the environment. It does not return to be used by the
*Thus, at every level of the food chain (first, second, third and
plants again. This makes the flow of energy in the ecosystem top level), the consumer converts only 10% of the accumulated
‘unidirectional’. energy into his body weight.

F–34 General Studies Environment and Ecology


Trophic Levels DDT is a chemical substance which cannot be degraded
biologically. Its concentration increases in the organism
*There are four types of trophic levels in the food chain of the
from lower trophic level to higher trophic level.
ecosystem. They are-
Snake is a Tertiary consumer. So, the concentration of
First trophic level- Producer (autotrophs)
DDT will be more in snake. It is deposited in fatty tissues
Second trophic level- Primary consumers (herbivorous)
of an organism.
Third trophic level- Secondary consumers (carnivorous) DDT is an insecticide.
Fourth trophic level- Tertiary consumers ( omnivorous)
Decomposers feed on all the above trophic levels. Bionomics & Biomagnification
*In ecology, bionomics (Greek: bio = life; nomos = law) is
Pyramids the comprehensive study of an organism and its relation to its
*The Pyramid of Biomass is a graphical representation that environment. It is synonymous to Ecology. It stresses the value
depicts the extent of biomass per unit area within different of natural systems which influence human systems and is not
trophic levels in an ecological system. related to the management of life.
*In grasslands and forests, there is a gradual decrease in biomass
of organisms at successive levels from the producers to the top Biogeochemical Cycle
*In Earth science, a biogeochemical cycle is a pathway by
carnivores. Thus pyramids are upright.
which a chemical substance moves through both biotic and
*However, in a pond as the producers are small organisms,
abiotic components of Earth. Biogeochemical cycles of
their biomass is very less and this value gradually shows an
nitrogen, phosphorus and carbon etc. are found in nature. These
increase towards the apex of the pyramid, thus making the
are mainly of two kinds:
pyramid inverted in shape.

*Ecological succession is the gradual process by which


ecosystems change and develop over time. In grassland, trees
have already been cleared away due to fires in the past, which
also changes the ground composition. Without the trees to hold
them, water washes away most of the soil and the underlying
Biological Magnification ground becomes the pioneer population, which is then replaced
by grass. The grass, however, absorbs most of the water before
(Biomagnification) it reaches the roots of the shrubs.
Biomagnification refers to an increase in the concentration *F. Clements studied different species of plants and gave a
of the toxic materials at successive trophic levels. This generally agreed upon definition in 1916. According to him,
happens because a toxic substance is accumulated by an biotic succession is a natural process of displacement of one
organism which cannot be metabolized or excreted, and is particular community by another in a particular area. A scientist
thus passed on to the next trophic level. This phenomenon named E. Odum described it as a sequenced process. The
is well known for Mercury (Hg) and DDT. correct order of biotic succession is as follows:

Environment and Ecology General Studies F–35


Agency) and other federal agencies use estimates of the social
Nudation
cost of carbon to evaluate the climate impacts of rulemaking.
p
Migration
p
Ecesis
p
Reaction
p
Stabilization
Figure: Stages of Ecological Succession

Environmental Movements
*The Chipko movement was a forest conservation movement
in India started in 1973. On 26 March, 1974 Gaura Devi
gathered other women around her village in the Garhwal *According to the World Development Report 2010
Himalayas and prevented loggers from felling trees by sticking ‘Millennium Ecosystem Assessment’ describes five
to the trees. This act by illiterate tribal and village women to groups of Ecosystem Services.
reclaim their traditional forest rights was a dramatic moment *Nutrient recycling and crop pollination are examples of
Ecosystem services.
known as ‘Chipko Andolan’, a non-violent struggle that gave
birth to the modern Indian environmental movement.
*“Ecology is permanent economy” was the slogan of Chipko
Movement.
*'Narmada Bachao Andolan' was against the construction of
a multi-modal dam on the Narmada river.
*'Appiko Movement' was started in Karnataka for
environmental protection.

Ecological Footprint and


Millennium Ecosystem Assessment
*The ecological footprint is a measure of human demand on
the Earth’s ecosystems. It’s a standardized measure of demand
for natural resource that may be contrasted with the planet’s
ecological capacity to regenerate.
*It can be defined as the biologically productive area required to
provide everything people need to sustain their life. It includes
fruits, vegetables, wood, fibres, fossil fuel use and also the
release of carbon dioxide. Its unit is Global Hectares.
*It is notable that our Ecosystem provides different services
to human and nature.
*Social Cost of Carbon or SC-CO2 is a measure, in monetary
value, of the long-term damage done by a tonne of carbon dioxide
(CO2) emission in a given year. EPA (Environmental Protection

F–36 General Studies Environment and Ecology


Ecosystem: Some facts for Exam *Oceanic outflow is that process in which warm & nutrient less
water at the surface is replaced by cold & nutrients rich water
*Eucalyptus is known as Environmental evil.
by air flow (wind).
*It absorbs too much inland water and decreases water level at
*Eco-sensitive zones are mentioned under the Environment
the place of its plantation.
(Protection) Act, 1986. The vehicles used for commercial
*It is mostly found in Australia.
activities in eco-sensitive zones have been regulated but
*Lentic Ecosystem refers to the static water habitats like
are not prohibited hence except agriculture, rest of the human
ponds, lakes, swamps and marshes. Lotic Ecosystem refers
activities are not prohibited but some of these are restricted and
to the moving water habitats like rivers.
some of them have been regulated.
*Ecotone is a region of transition between two biological
communities.
Basic Questions
1. ‘Ecology’ is the study of the relationship between–
(a) Organism and environment
(b) Man and forest
(c) Soil and water
(d) Husband and wife
U.P.R.O./A.R.O. (Pre) 2014
Ans. (a)
The word 'ecology' originated from Greek 'Oikos' meaning
house or environment. Ecology is the study of interactions
among organisms and their environment. It also includes
*Oceanic ecosystem is the most stable ecosystem.
the study of organismal interaction with abiotic components
Term 'Ecosystem' was coined by A.G. Tansley.
of their environment. The word 'ecology' ('Oekologie') was
Order of Ecosystem productivity in decreasing order-
coined by German scientist, Ernst Haeckel (1834-1919).
Mangrove, Grasslands, Lake, Ocean. An ecosystem with more
Haeckel defined Ecology as the study of the relationship of
diversity has more productivity.
an organism with their environment. Ecology in current age
*The entire sequence of communities that successively change
refers to a broader spectrum encompassing not only plants,
in a particular area are called ‘Sere’. animal species and climate but also to human beings and it's
*Pycnocline - It shows density gradient in an aquatic system. physical surroundings.
*Halocline - It shows a salt gradient in an aquatic system. 2. The term Ecosystem was proposed by –
*Thermocline- Represents the change in temperature with a (a) G. Tailor (b) E. Hartigan
depth of an aquatic system. (c) D.R. Stoddart (d) Tansley
*Arne Naess in 1973 used the term 'Deep ecology' for the Uttarakhand P.C.S. (Pre) 2005
first time. Ans. (d)
*Ecological Niche term was coined by Joseph Grinnell in The term ecosystem was first used by A. G. Tansley in 1935
1917, who called it Micro-Habitats. who defined an ecosystem as ‘a particular category of physical
*The concept of ‘Ecological Transition’ was first used by systems, consisting of organisms and inorganic components in
John W. Bennett. He proposed the concept in his book ‘‘The a relatively stable equilibrium, open and of various sizes and
Ecological Transition : Cultural Anthropology and Human kinds’. An Ecosystem is a community of living organism (plants,
Adaptation’’. animals and microbes) along with the nonliving components of
*Indian Wild Life Protection Act was passed in 1972. their environment (like air, water, mineral, soil, etc.) interacting
*Environment (Protection) Act was passed in 1986. as a system. These biotic and abiotic components are considered
*Forest Conservation Act was passed in 1980. as linked together through nutrient cycles and energy flows. As

Environment and Ecology General Studies F–37


ecosystems are defined by the network of interactions among There are four basic components of an ecosystem which
organisms and between organisms and their environment, include Abiotic materials, Producers, Consumers and
these can vary in size from as small as a drop in the pond to Decomposers or Reducers. Whereas Producers, Consumers
as large as an ocean. The earth itself is a large ecosystem. and Decomposers are included in a broad component, 'Biotic
materials or 'Biotic Factors'.
3. Which of the following is not true about the eco-
system? 6. Which one of the following is the best description of
(a) It represents all living organisms and the physical the term ‘ecosystem’?
environment in any given space-time unit. (a) A community of organisms interacting with one
(b) It is a functional unit. another.
(c) It has own productivity. (b) That part of the Earth which is inhabited by living
(d) It is a closed-system. organisms.
U.P.P.C.S. (Mains) 2014
(c) A community of organisms together with the
Ans. (d)
environment in which they live
An ecosystem is a community of living organisms in (d) Flora and fauna of a geographical area.
conjunction with the non-living components of their I.A.S. (Pre) 2015
environment (things like air, water and mineral soil), Ans. (c)
interacting as a system.
All the organisms and the abiotic components of the
Important features of ecology are:
*It is normally an open system with a continuous, but variable environment form the ecosystem of the place. Thus ecology
influx and loss of materials and energy. deals with the organism and its environment.
*An ecosystem is an overall integration of the whole mosaic 7. Which of the following is an artificial ecosystem?
of the interacting organism and their environment. (a) Ponds (b) Field
*It is a basic functional unit with no limits of boundaries. (c) Forest (d) None of these
*It consists of biotic and abiotic components interacting Jharkhand P.C.S. (Pre) 2013
with each other. Ans. (b)
*Its functional unit is capable of energy transformation,
circulation and accumulation. Field, garden are artificial ecosystem while forests, ponds
Different types of the ecosystem are present in different areas. and lakes are natural ecosystems.

4. Which of the following is not correct about the 8. Which of the following is an artificial ecosystem?
ecosystem? (a) Rice field (b) Forest
(a) It comprises both, abiotic and biotic components (c) Grassland (d) Lake
(b) It has its productivity. U.P.P.C.S. (Pre) 2016
(c) It is a closed system. Ans. (a)
(d) It is a natural resource system.
An artificial ecosystem meets all the criteria of a natural
U.P.P.C.S. (Pre) (Re. Exam) 2015
eco-system but is made and controlled by humans. Orchards,
Ans. (c)
farmlands, gardens and human-made reservoirs are some
See the explanation of the above question. examples of artificial ecosystems. Ecosystems can broadly
be categorized as Aquatic and Terrestrial ecosystems.
5. Which four components constitute an ecosystem?
(a) Water, Carbon, Oxygen, Sulphur Grasslands, forests and deserts are examples of terrestrial
(b) Soil, Temperature, Humidity, Rainfall ecosystems while lakes, rivers and oceans fall under aquatic
(c) Abiotic materials, Producers, Consumers, Decomposers ecosystem.
(d) Herbivores, Carnivores, Bacteria, Parasites Notably, agriculture under human ecosystem has been
Jharkhand P.C.S. (Mains) 2016 instrumental to the existence of society and development.
Ans. (c) Hence (a) is the correct answer.

F–38 General Studies Environment and Ecology


9. The number of organisms that can be supported by the There are mainly two kinds of ecosystems- Aquatic and
environment in a given area is known as: Terrestrial. Marine ecosystems are the biggest ecosystems,
(a) Population which cover around 71% of Earth’s surface and contain 97%
(b) Carrying Capacity of our planet’s water. Thus option (d) is the correct answer.
(c) Pyramid of number or biomass
13. Which one of the following is the largest ecosystem of
(d) None of the above
Uttarakhand P.C.S. (Pre) 2012 the world?
Ans. (b) (a) Ocean (b) Grassland
(c) Forest (d) Mountains
The maximum population size that can ecosystem can
U.P.P.C.S. (Pre) 2014
support is called capacity carrying capacity is the number of
Ans. (a)
organism that an ecosystem can sustainably support. Hence
option (b) is correct. Aquatic ecosystem refers to living and non-living parts of
a water body and the interactions that take place among
10. Which one of the following terms describes not only them. Aquatic ecosystems can be categorized in the marine
the physical space occupied by an organism but also ecosystem and freshwater ecosystem. Marine ecosystem
its functional role in the community of organisms? covers approximately 71% of the earth's surface and contains
(a) Ecotone (b) Ecological niche approximately 97% of the planet's water.
(c) Habitat (d) Home range
14. Which of the following does not help to maintain
I.A.S. (Pre) 2013
ecological balance?
Ans. (b)
(a) Cutting forest (b) Afforestation
Ecological niche characterizes the position of a species within (c) Rainwater Management (d) Biosphere Reserves
an ecosystem, comprising species habitat requirements as U.P.U.D.A./L.D.A. (Pre) 2013
well as its functional role. Ans. (a)
11. The actual location or place where an organism lives Wildlife conservation and prevention of environmental
is called: pollution is helpful in maintaining ecological balance.
(a) Habitat (b) Ecosystem Deforestation adds to the environmental imbalance.
(c) Niche (d) Biome
15. Which one of the following is one of the main causes
(e) None of the above/More than one of the above
of ecological imbalance in India?
U.P.P.C.S. (Pre) 2019
(a) Deforestation (b) Desertification
Ans. (a)
(c) Floods and Famines (d) Rainfall variations
The location or place where an organism lives is called a R.A.S./R.T.S.(Pre) 2012
habitat. The organisms depend for their food, water, air, Ans. (a)
shelter and other needs on their habitat. A biome is a plant
and animal community that covers a large geographical area. Deforestation is one of the main causes of ecological
Some of the major biomes of the world are: forest, grassland, imbalance in India. While, desertification, floods and famines
desert and tundra biomes. and rainfall variations are secondary causes.

12. Which one of the following ecosystems covers the 16. Which of the following is an act that disturbs the
ecological balance?
largest area of the earth’s surface:
(a) Lumbering (b) Social forestry
(a) Desert Ecosystem (b) Grassland Ecosystem
(c) Vanamahotsav (d) Afforestation
(c) Mountain Ecosystem (d) Marine Ecosystem
U.P.R.O./A.R.O. (Pre) 2014
U.P. Lower Sub. (Spl) (Pre) 2003
Ans. (a)
Ans. (d) It is lumbering which disturbs the ecological balance.

Environment and Ecology General Studies F–39


17. The highest trophic level in an ecosystem is obtained the following kinds of an organism is/are known as
by: decomposer organism/organisms?
(a) Herbivores (b) Carnivores 1. Virus 2. Fungi
(c) Omnivores (d) Decomposers 3. Bacteria
U.P.U.D.A./L.D.A. (Pre) 2002 Select the correct answer using the code given below:
Ans. (c) (a) Only 1 (b) 2 and 3
There are four types of trophic levels in the food chain of the (c) 1 and 3 (d) All of these
ecosystem. They are- I.A.S. (Pre) 2013
First trophic level- Producer Ans. (b)
Second trophic level- Primary consumers (herbivorous) Decomposers are nature’s recyclers as they break down
Third trophic level- Secondary consumers (carnivorous) the organic matter found in the dead bodies of plants and
Fourth trophic level- Tertiary consumers ( omnivorous)
animals. The term ‘organic matter’ refers to the matter that
Decomposers feed on all the above trophic levels. Hence
comes from living organisms. Decomposers are mainly the
option (c) is the correct answer.
microorganisms such as bacteria and fungi in the form of
18. Which of the following is not a biotic component of yeasts and moulds that break down bodies of dead organisms
an ecosystem? and release compounds that can be used by producers.
(a) Air (b) Plant
22. Which of the following are the primary consumers
(c) Bacteria (d) Animals
in an ecosystem? Select the correct answer from the
R.A.S./R.T.S. (Pre) 2013
codes given below:
Ans. (a)
1. Ants 2. Deer
Biotic components are the living things that shape 3. Fox 4. Tiger
an ecosystem. Code :
Biotic components usually include: (a) 1 and 2 (b) 2 and 3
Producers: e.g. plants. (c) 1, 2 and 3 (d) 2, 3 and 4
Consumers: e.g. animals, humans U.P.P.C.S. (Mains) 2006
Decomposers: e.g. fungi and bacteria Ans. (a)
19. The primary source of energy in Eco-body is – Mainly herbivores, in a food chain, are considered as primary
(a) Energy emitted by fermentation consumers. In the given options, Deer is herbivorous hence
(b) Preserved vegetable sugar it is a primary consumer. Ants fall under decomposer and
(c) Solar energy primary consumers both. It eats plants and receives energy
(d) None of the above. from cellulose. Tiger is carnivorous and the fox is omnivorous.
U.P.P.C.S. (Mains) 2015 Significantly, primary consumers feed on primary producers
Ans. (c) (plants). Hence option (a) is the correct answer.
Solar energy is the primary source of energy in the 23. In a food chain, man is:
ecological body. It is the driving force of an ecosystem. (a) producer
(b) only primary consumer
20. Driving force of an ecosystem is:
(a) Carbohydrate (b) Biomass (c) only secondary consumer
(c) Carbon (d) Solar energy (d) primary as well as a secondary consumer
U.P.R.O./A.R.O. (Re-Exam) (Pre) 2016 U.P.P.C.S. (Pre) 2016
Ans. (d) Ans. (d)
In food chain there are trophic levels i.e., Autotroph, Primary
See the explanation of the above question.
consumer, Secondary consumer and Tertiary consumer.
21. Concerning the food chains in ecosystems, which of The primary consumer is the herbivores (that feed on the

F–40 General Studies Environment and Ecology


plant), the secondary consumer is small carnivores (feeding (a) Cow (b) Peacock
on primary consumer) and the tertiary consumer is larger (c) Tiger (d) Green Plants
carnivores. So, in food chain, man is the primary as well as U.P.P.C.S. (Pre) 2013
a secondary consumer who feeds on both plant, products Ans. (d)
and meat. Plants are self-feeding organisms that produce complex
24. 10 percent law is related with: organic compounds such as carbohydrates, fats and proteins
(a) Transfer of energy as food from one trophic level to generally using energy from light (photosynthesis). These
other are known as autotrophs and are fundamental to the food
(b) Transfer of heat from one matter to another chain of all ecosystems. These are called primary producers.
(c) Transfer of birds from one zone to another
(d) Transfer of water from one zone to another 27. Trophic level-I includes –
(e) None of these (a) Herbivorous animals (b) Carnivorous animals
Chhattisgarh P.C.S. (Pre) 2016 (c) Omnivorous animals (d) Green plants
Ans. (a) M.P.P.C.S. (Pre) 2016
The Ten Percent Law of transfer of energy from one trophic Ans. (d)
level to the next was introduced by Raymond Lindeman in Each step or level of the food chain forms a trophic level. Tropic
1942. According to this law, during the transfer of energy level-I includes autotrophs or the producers (i.e. green plants)
from organic food from one trophic level to the next, only which fix up solar energy and make it available for heterotrophs
about ten percent of the energy from organic matter is stored or the consumers. The herbivores (primary consumer) come at
as flesh. The remaining is lost during transfer, broken down in second, small carnivores at third and tertiary consumer (large
respiration or lost to incomplete digestion by higher trophic carnivores) form the fourth trophic level.
level. When organisms are consumed, 10% of the energy in
the food is fixed into flesh and is available for next trophic 28. What is true about ecosystem?
level (carnivores or omnivores). When a carnivore or an (a) Primary consumers are least dependent upon producer
omnivore consume that animal, only about 10% of energy (b) Primary consumers are out-number producers
is fixed in its flesh for the higher level. Thus, at every level (c) Producers are more than primary consumers
of the food chain (first, second, third and top level), the (d) Secondary consumers are the largest and most powerful
consumer converts only 10% of the accumulated energy into U.P.P.C.S. (Pre) 2019
his body weight. Ans. (c)
25. In the marine environment, the main primary In a given ecosystem, there are generally more producers
producers are: than primary consumers, and more primary consumers than
(a) Phytoplanktons (b) Seaweeds secondary consumers, and so on.
(c) Marine angiosperms (d) Aquatic bryophytes 29. Which one of the following pairs is correctly matched?
U.P.P.C.S. (Mains) 2005
(a) Mountain - Most stable ecosystem
Ans. (a)
(b) Abiotic Component - Bacteria
Phytoplankton, also known as microalgae, are similar to (c) Green plants - Ecosystem
terrestrial plants in that they contain chlorophyll and require (d) Rainfall - Global Warming
sunlight to live and grow. Most phytoplankton are buoyant U.P.P.C.S. (Pre) 2015
and float in the upper part of the ocean, where sunlight Ans. (c)
penetrates the water. Phytoplankton also requires inorganic
All the pairs mentioned in the question are related to
nutrients such as nitrates, phosphates, and sulfur which they
ecosystems but correctly matched is only (c). An ecosystem
convert into proteins, fats, and carbohydrates.
is a community of living organisms in conjunction with the
26. Among the biotic components of the ecosystem, the non-living components of their environment. Since plants
producer system is – are living organism hence they are related to the ecosystem.

Environment and Ecology General Studies F–41


30. The correct order in the food chain of the various ecosystem?
components of a terrestrial grass ecosystem is – (a) Forest (b) Pond
(a) Grass, Grasshopper, Frog, Snake (c) Grassland (d) Drylands
(b) Grass, Grasshopper, Snake, Frog U.P.P.S.C. (GIC) 2010
(c) Grasshopper, Frog, Grass, Snake Ans. (b)
(d) Grasshopper, Snake, Frog, Grass
The Pyramid of Biomass is a graphical representation that
U.P.P.S.C. (GIC) 2010 depicts the extent of biomass per unit area within different
Ans. (a) trophic levels in an ecological system. In grasslands and
Any substance consumed to provide nutritional support forests, there is a gradual decrease in the biomass of
for an organism is called food. Plants are self-feeding and organisms at successive levels from the producers to the
make for their own food whereas animals depend on plants top carnivores. Thus, pyramids are upright. However, in a
for food. Food chain depicts the flow of energy through pond, as the producers are small organisms, their biomass
different organisms in an ecosystem. Notably, this flow is is least and this value gradually shows an increase towards
unidirectional. The correct order of different components in the apex of the pyramid, thus making the pyramid inverted
the above food chain is as follows: in shape.
Grass o Grasshopper o Frog o Snake
34. An inverted pyramid of biomass can be found in which
pppp
(Producer) (Primary Consumer) (Secondary Consumer) (Tertiary Consumer) ecosystem?
(a) Marine (b) Grassland
31. Which of the following constitute a food chain? (c) Forest (d) Tundra
(a) Grass, wheat and mango (b) Grass, goat and human R.A.S./R.T.S. (Pre) 2021
(c) Goat, cow and elephant (d) Grass, fish and goat Ans. (a)
(e) None of the above
An inverted pyramid of biomass can be found in marine
Chhattisgarh P.C.S. (Pre) 2016
ecosystem. The pyramid of biomass in sea is generally
Chhattisgarh P.C.S. (Pre) 2015
inverted because the biomass of fishes far exceeds that of
Ans. (b)
phytoplankton. Hence option (1) is correct.
Grass, goat and humans form a food chain –
35. Which one of the following organisms is likely to show
Grass Goat Humans
the highest concentration of DDT, once it has been
ppp introduced into the ecosystem?
(Autotroph) (Herbivorous consumer) (Omnivorous consumer)
(a) Grasshopper (b) Toad
32. The pyramid of energy in any ecosystem is
(c) Snake (d) Cattle
(a) Always upright I.A.S. (Pre) 1997
(b) May be upright and inverted Ans. (c)
(c) Always inverted
DDT is a non-biodegradable, biomagnifying pollutant
(d) None of the above
which increases in concentration from producers, primary
M.P.P.C.S. (Pre) 2020
consumers, secondary consumers to tertiary consumers. In
Ans. (a)
the question, only the snake represents the tertiary consumers,
Pyramid of energy is always upright, can never be inverted, so the concentration of DDT will be highest in the snake.
because when energy flows from a particular trophic level
36. The amount of energy during the transfer from one
to the next trophic level, some energy is always lost as heat
trophic level to another in an eco-system–
at each step. Each bar in the energy pyramid indicates the
(a) Increases (b) Decreases
amount of energy present at each trophic level in a given
(c) Remains constant (d) May increase or decrease
time or annually per unit area. Hence option (a) is correct.
U.P. R.O./A.R.O. (Pre), 2017
33. Biomass Pyramid is reversed in which type of Ans. (b)

F–42 General Studies Environment and Ecology


Energy is the basic force responsible for all metabolic activities. 41. The transitional zone between two distinct communities
The flow of energy from producer to top consumers is called is known as:
energy flow, which is unidirectional. Energy flows through the (a) Ecotype (b) Arcade
trophic levels i.e. from producers to subsequent trophic levels. (c) Ecosphere (d) Ecotone
U.P.P.C.S.(Pre) 2012
This energy always flows from lower (producer) to higher
Ans. (d)
(herbivore, carnivore, etc.) trophic level. There is a loss of
some energy in the form of unusable heat at each trophic level Ecotone is a region of transition between two biological
so the amount of energy during the transfer from one trophic communities.
level to another decreases in an ecosystem. 42. Which of the following is the most stable ecosystem?
37. The amount of energy during transfer from one trophic (a) Desert (b) Mountain
level to another in an ecosystem– (c) Ocean (d) Forest
(a) Increases (b) Decreases U.P.P.C.S. (Pre) 2013
(c) Remains constant (d) May increase or decrease Ans. (c)
U.P.P.C.S. (Pre) 2019 Oceans are the cradle of life on Earth which harbor the highest
Ans. (b) level of biodiversity. No terrestrial ecosystem can match the
See the explanation of above question. biological richness of a coral reef, not only in some species
but also in variety and originality of survival strategies.
38. Which one of the following trees is not eco-friendly?
43. Most stable Ecosystem is –
(a) Babul (b) Eucalyptus
(a) Forest (b) Grassland
(c) Neem (d) Pipal
(c) Desert (d) Ocean
U.P. U.D.A./L.D.A. (Mains) 2010
R.A.S./R.T.S. (Pre) 2008
U.P.P.C.S. (Mains) 2011
Ans. (d)
Ans. (b)
See the explanation of the above question.
Eucalyptus consumes more ground water than other trees.
That is why it often called ‘enemy of the environment’. 44. Which is the most stable ecosystem?
The water level goes down across the area of Eucalyptus (a) Marine (ocean) (b) Forest
plantation. These plants are mostly found in Australia. (c) Mountain (d) Desert
U.P. P.C.S. (Pre) 2018
39. Which one of the following trees is considered to be an
Ans. (a)
environmental hazard?
See the explanation of the above question.
(a) Babul (b) Amaltas
(c) Neem (d) Eucalyptus 45. Which of the following is an example of anthropogenic
biome?
U.P.P.C.S. (Pre) 2005
(a) Fresh water (b) Grassland
Ans. (d)
(c) Rain forest (d) Cropland
See the explanation of the above question. U.P. P.C.S. (Pre) 2018
Ans. (d)
40. The example of “Lentic Habitat” in the freshwater
Freshwater, Grassland and Rain forest are examples of natural
community is:
biome whereas cropland is an example of the anthropogenic
(a) Ponds and swamps (b) Waterfalls and rivers
biome.
(c) Ponds and rivers (d) All the above.
46. Consider the following statements:
MP.P.C.S. (Pre) 2014
1. Most of the world's coral reefs are in tropical
Ans. (a) waters.
Lentic Ecosystem refers to the static water habitats like ponds, 2. More than one-third of the world's coral reefs are
lakes, swamps and marshes. Lotic Ecosystem refers to the located in the territories of Australia, Indonesia
dynamic water habitats like rivers. and the Philippines.

Environment and Ecology General Studies F–43


3. Coral reefs host far more number of animal phyla (c) Geological cycles (d) Geochemical cycles
than those hosted by tropical rainforests. Which U.P.P.C.S.(Pre) 2012
of the statements given above is/are correct? Ans. (b)
(a) 1 and 2 only In Earth science, a biogeochemical cycle is a pathway by
(b) 3 only which a chemical substance moves through both biotic and
(c) 1 and 3 only abiotic components of Earth. Biogeochemical cycles of
(d) 1, 2 and 3 nitrogen, phosphorus and carbon, etc. are found in nature.
I.A.S. (Pre) 2018
Ans. (d) 49. In case of which one of the following biogeochemial
cycles, the weathering of rocks is the main source of
Corals are invertebrate animals. Reefs are created by hard release of nutrient to entry the cycle?
corals which secrete a skeleton that can turn into giant (a) Carbon cycle (b) Nitrogen cycle
structures like Australia's 'Great Barrier Reef'. Most of the (c) Phosphorus cycle (d) Sulphur cycle
world's coral reefs are located in tropical oceans near the equator. I.A.S (Pre) 2021
More than one-third of the world's coral reef is located in the Ans. (c)
territories of Australia. Indonesia and Philippines.
Coral reefs are one of the most diverse ecosystems on the Phosphorus is a chemical element found on Earth in numerous
planet. These tropical marine communities occupy less than compound forms, such as the phosphate ion (PO43-), located
1% of the ocean floor but are inhabited by more than 25% in water, soil and sediments.
of all known marine fish species. 32 of the 34 recognized Much of the phosphorus on Earth is tied up in rock
animal phyla are formed on coral reefs compared to 9 phyla and sedimentary deposits, from which it is released by
in tropical rainforests. weathering, leaching, and mining.
Some of it passes through freshwater and terrestrial
47. Which one of the following is the largest ecosystem of ecosystems via plants, grazers, predators, and parasites, to
the earth? be returned to those ecosystems by death and decay. Much
(a) Hydrosphere (b) Biome of it, however, is deposited in the sea, in shallow sediments,
(c) Lithosphere (d) Biosphere where it circulates readily, or in ocean deeps, whence it wells
U.P. R.O./A.R.O. (Pre) 2017 up only occasionally.
Ans. (a) Phosphorus is brought back to the land through fish harvests
and through collection of guano deposited by seabirds.
The largest ecosystem of the earth is Biosphere. The 50. The movement of nutrient elements through various
biosphere is a part of the earth where life can exist. It
components of an ecosystem is called:
is a highly integrated and interacting zone comprising
(a) Bio-geo-chemical cycle (b) Bio-geological cycle
of atmosphere (air) hydrosphere (water) and lithosphere
(c) Ecological succession (d) Biological cycle
(land). It is a narrow layer around the surface of the earth.
U.P.P.C.S. (Pre) 2020
The biosphere is absent at extremes of the North and South
Ans. (a)
Poles, the highest mountains and the deepest oceans.
See the explanation of the above question.

51. Which one of the following is not related to ecological


balance?
(a) Water management (b) Afforestation
(c) Industrial Management (d) Wildlife protection
U.P.P.C.S. (Pre) 2005
Ans. (c)
Water management, afforestation and wildlife protection
is associated with Ecological balance while industrial
management is not associated with ecological balance.

48. The cycling of elements in an ecosystem is called: 52. “Ecology is permanent economy” is the slogan of which
(a) Chemical cycles (b) Biogeochemical cycles movement?

F–44 General Studies Environment and Ecology


(a) Appiko Movement (b) Narmada BachaoAndolan ecosystem by decomposers. Decomposers are organisms such
(c) Chipko Movement (d) None of the above
as bacteria and fungi that breake down the organic matter of
U.P.P.C.S. (Mains) 2007
the dead bodies of plants and animals into inorganic material.
Ans. (c)
Hence statement (4) is also wrong.
“Ecology is permanent economy” is the slogan of Chipko
55. Which one of the following is the correct sequence of
Movement. 'Narmada Bachao Andolan' was against
ecosystems in the order of decreasing productivity?
the construction of a multi-modal dam on the Narmada (a) Oceans, lakes, grasslands, mangroves
river. 'Appiko Movement' was started in Karnataka for (b) Mangroves, oceans, grasslands, lakes
environmental protection. (c) Mangroves, grasslands, lakes, oceans
53. Chipko movement is related to: (d) Oceans, mangroves, lakes, grasslands
(a) Plant conservation (b) Tiger project I.A.S. (Pre) 2013
(c) Crocodile conservation (d) Plant reproduction Ans. (c)
U.P.P.C.S. (Pre) 2015 The correct sequence of the ecosystem in the order of
Ans. (a)
decreasing productivity is- Mangrove, grasslands, lakes,
The Chipko movement was a forest conservation movement oceans.
in India started in 1973. Gaura Devi gathered other women 56. Concerning food chains in ecosystems, consider the
around her village in the Garhwal Himalayas and prevented following statements
loggers from felling trees by sticking to the trees. This act by 1. A food chain illustrates the order in which a chain
illiterate tribal and village women to reclaim their traditional of organisms feeds upon each other.
forest rights was a dramatic moment known as ‘Chipko 2. Food chains are found within the population of a
species.
Andolan’, a non-violent struggle that gave birth to the modern
3. A food chain illustrates the number of organism,
Indian environmental movement.
which is eaten by others.
54. Consider the following statements relating to the Which of the statements given above is /are correct?
ecosystem: (a) Only 1 (b) 1 and 2
1. The term ecosystem was first used by A.G. Tansley. (c) 1, 2 and 3 (d) None of these
2. Those organisms which produce their food I.A.S. (Pre) 2013
themselves are termed as autotrophs. Ans. (a)
3. Consumers consume their food through the process
Now most obvious aspect of nature is that energy must pass
of photosynthesis.
from one living organism to another. When herbivorous
4. Decomposers convert inorganic matter into organic
animals feed on plants, energy is transferred from plants
matter.
to animals. In an ecosystem, some of the animals feed
Which of the above statements are correct:
on other living organisms while some feed on dead
Code:
organic matter. The latter form the detritus food chain.
(a) 1 and 2 (b) 1 and 3
Each chain usually has only four to five such links.
(c) 2 and 4 (d) 3 and 4
Food chains are not found within the population of a
U.P.U.D.A./L.D.A. (Pre) 2002
species. Because this would mean a human being eating
Ans. (a)
another human being (human beings are single species)
The term ecosystem was first used by A.G. Tansley. Hence
57. Which of the following statement is correct?
statement (1) is correct. Biotic components are the living (a) The energy flow in the biospheric ecosystem is
things that shape an ecosystem. Autotrophic components unidirectional.
are self-feeding organisms of the ecosystem. These produce (b) The energy flows in the biospheric is cyclical.
their food themselves. Hence statement (2) is also correct. (c) Relative loss of energy in natural ecosystem decrease
Consumers are organisms of an ecological food chain with increasing trophic levels.
which receive energy by consuming other organisms. These (d) Species at progressively higher trophic levels appear
to be less efficient in using available energy
organisms are formally referred to as heterotrophs. Hence,
U.P.P.C.S. (Pre) (Re. Exam) 2015
statement (3) is wrong. Organic matter is recycled in an Ans. (a)

Environment and Ecology General Studies F–45


The flow of energy in the ecosystem is unidirectional (or (a) Biota (b) Ecological footprint
(c) Biome (d) Niche
one-directional). The energy enters the plants (from the
U.P.P.C.S. (Pre) 2012
sun) through photosynthesis during the making of food.
Ans. (b)
This energy is then passed on from one organism to another
in a food chain. The energy given out by the organisms See the explanation of the above question.
as heat is lost to the environment. It does not return to be 5. Which one of the following statements best describes
used by the plants again. This makes the flow of energy in the term ‘Social Cost of Carbon’?
the ecosystem ‘unidirectional’. According to the first law It is a measure, in monetary value, of the:
of thermodynamics, energy can neither be created nor be (a) long-term damage done by a tonne of CO2 emissions
destroyed in an isolated system. According to the second law in a given year.
whenever energy transforms into a new form, there is evident (b) requirement of fossil fuels for a country to provide
decay in its intensity. Similarly, with increasing trophic levels goods and services to its citizens, based on the burning
availability of energy decreases in the natural ecosystem. of those fuels.
(c) efforts put in by a climate refugee to adapt to live in
a new place.
High-Level Questions (d) contribution of an individual person to the carbon
footprint on the planet Earth.
1. Who of the following used the term ‘deep ecology’ for
I.A.S. (Pre.) 2020
the first time?
Ans. (a)
(a) E.P. Odum (b) C. Raunkiaer
(c) F.E. Clements (d) Arne Naess Social Cost of Carbon or SC-CO2 is a measure, in monetary
U.P.P.C.S. (Mains) 2014 value, of the long-term damage done by a tonne of carbon
Ans. (d) dioxide (CO2) emission in a given year. EPA (Environmental
Protection Agency) and other federal agencies use estimates
Arne Naess was a Norwegian philosopher who coined the of the social cost of carbon to evaluate the climate impacts
term “deep ecology” in 1972. of rulemaking.
2. The concept of the ecological niche was enunciated by – 6. Read the list of Ecological concerns and the year in
(a) Grinnell (b) Darwin which Acts were executed in India:
(c) E.P. Odum (d) C.C. Park Ecological concerns Year,Act was passed
U.P.P.C.S. (Pre) 2005 A. Wildlife Protection (i) 1986
Ans. (a) B. Environment Protection (ii) 2013
Joseph Grinnell referred to the “ecological or environmental C. The Scheduled Tribes and other (iii) 1972
niche” as the ultimate distributional unit of “species and Traditional Forest Dweller’s
subspecies”. (Recognition of Forest Rights) Act
D. The Forest Conservation Act (iv) 1988
3. The measurement unit of the ecological footprint is: Which of the following is not correctly matched?
(a) Global hectares (b) Nanometer Code:
(c) Hoppus cubic foot (d) Cubic ton A B C D
U.P.R.O./A.R.O. (Pre) 2016 (a) iii i ii iv
Ans. (a) (b) iv ii iii i
The ecological footprint is a measure of human demand (c) i ii iii iii
on the Earth’s ecosystems. It’s a standardized measure of (d) ii iii iv i
demand for natural resource that may be compared with the R.A.S./R.T.S.(Pre) 2013
planet’s ecological capacity to regenerate. It can be defined Ans. (*)
as the biologically productive area required to provide for
The correctly matched list is as follows:
everything people need to sustain their life. It includes
Wildlife Protection - 1972
fruits, vegetables, wood, fibres, fossil fuel use and also
Environment (Protection) Act - 1986
the release of carbon dioxide. Its unit is Global hectares.
The Scheduled Tribes and - 2006
4. The minimum area of the land required to completely other Traditional Forest Dwellers
sustain the life of the person is called his – (Recognition of Forest Rights) Act

F–46 General Studies Environment and Ecology


The Forest Conservation Act - 1980 influence human systems.
Hence, none of the option is correctly matched. Code :
(a) Only 1 (b) 1 and 2
7. The Millennium Ecosystem Assessment describes (c) 2 and 3 (d) 1, 2 and 3
the following major categories of ecosystem services- U.P. U.D.A./L.D.A. (Pre) 2001
provisioning, supporting, regulating, preserving and Ans. (c)
cultural. Which one of the following is supporting
service? In ecology, bionomics (Greek: bio = life; nomos = law) is
(a) Production of food and water the comprehensive study of an organism and its relation to
(b) Control of climate and disease its environment. Another way of expressing this word is the
term currently referred to as “ecology”. It stresses the value
(c) Nutrient cycling and crop pollination
of natural systems which influence human systems and is not
(d) Maintenance of diversity
related to the management of life.
I.A.S. (Pre) 2012
Ans. (c) 10. Which of the following shows a density gradient of the
water system?
According to the World Development Report, 2010, the (a) Ecocline (b) Halocline
Millennium Ecosystem Assessment describes the following
(c) Pycnocline (d) Thermocline
five major categories of ecosystems services:
U.P.P.C.S. (Mains) 2016
Provisioning Services
Ans. (c)
Regulating Services
Supporting Services Pycnocline indicates a density gradient of any water system.
Cultural Services Halocline shows salinity gradient of the water system.
Preserving Services Thermocline shows a change in temperature with respect
Supporting services are those that are necessary for the to the depth of a water system.
production of all other ecosystem services. They differ from
11. In the context of ecosystem productivity, marine
provisioning, regulating, and cultural services in that their
upwelling zones are important as they increase the
impacts on people are either indirect or occur over a very
marine productivity by bringing the:
long time. For example: Soil formation and retention, nutrient
cycling, crop pollination, water cycling etc. 1. Decomposer micro-organisms to the surface.
‘Production of food and water’ is considered under Provisioning 2. Nutrients to the surface.
Services, while ‘Control of climate and disease’ is a Regulating 3 Bottom-dwelling organisms to the surface.
Service. ‘Maintenance of diversity is a Preserving service. Which of the statements given above is/are correct?
(a) 1 and 2 (b) 2 only
8. Which one of the following is the correct sequence of
(c) 2 and 3 (d) 3 only
a food chain?
I.A.S. (Pre) 2011
(a) Diatoms-Crustaceans-Herrings
Ans. (b)
(b) Crustaceans-Diatoms-Herrings
(c) Diatoms-Herrings-Crustaceans Winds blowing across the ocean surface push water away.
(d) Crustaceans-Herrings-Diatoms Water then rises up from beneath the surface to replace
I.A.S. (Pre) 2014 the water that was pushed away. This process is called
Ans. (a) "upwelling. Water that rises to the surface as a result of
upwelling is typically colder & is rich in nutrients. These
The correct order of a common marine food chain is – nutrients "fertilize" surface waters, meaning that these surface
Diatom waters often have high biological productivity. Therefore,
p good fishing grounds are found where upwelling is common.
Crustaceans
p 12. Concerning ‘Eco-Sensitive Zones’, which of the
Herrings following statements is/are correct?
1. Eco-Sensitive Zones are the areas that are declared
9. Which of the following statements is true about under the Wildlife (Protection) Act, 1972.
Bionomics? 2. The purpose of the declaration of Eco-Sensitive
1. It means “management of life’. Zones is to prohibit all kinds of human activities
2. It is synonymous with ecology. in those zones except agriculture.
3. It stresses the value of natural systems which Select the correct answer using the code given below:

Environment and Ecology General Studies F–47


(a) 1 only (b) 2 only (a) Insects and fungi
(c) Both 1 and 2 (d) Neither 1 nor 2 (b) Limited sunlight and paucity of nutrients
I.A.S. (Pre) 2014 (c) Water limits and fire
Ans. (d) (d) None of the above
I.A.S. (Pre) 2013
Eco - sensitive zones are mentioned under the Environment
Ans. (c)
(Protection) Act, 1986. Hence statement 1 is incorrect.
Prohibited activities: Ecological succession is the gradual process by which
Tourism activities like flying over protected areas in an ecosystems change and develop over time. In grassland,
aircraft or hot air balloon, and discharge of effluents and solid trees have already been cleared away due to fires in the
waste in natural water bodies or terrestrial areas. past, which also changes the ground composition. Without
Regulated activities: the trees to hold them, water washes away most of the soil
Felling of trees, establishment of hotels and resorts, and the underlying ground becomes the pioneer population,
commercial use of natural water, erection of electrical cables, which is then replaced by grass. The grass, however, absorbs
drastic change of agriculture system, e.g. adoption of heavy most of the water before it reaches the roots of the shrubs.
technology, pesticides etc. widening of roads. Thus, the ecological succession does not occur beyond
Permitted activities: the grass.
Ongoing agriculture and horticulture practices by local
communities, rainwater harvesting, organic farming, 15. The concept of ‘Ecological Transition’ was first used by:
adoption of green technology and use of renewable energy (a) Elton (b) Bennett
sources. (c) Berkeley (d) Ratzel
Hence, statement 2 is also incorrect. Hence, option (d) is the U.P.P.C.S. (Pre) 2020
correct answer. Ans. (b)
13. Given below are two statements, one is labelled as
The concept of ‘Ecological Transition’ was first used by
Assertion (A) and other as Reason (R).
John W. Bennett. He proposed the concept in his book ‘‘The
Assertion (A) : Various components of an ecosystem
Ecological Transition : Cultural Anthropology and Human
are not interdependent.
Adaptation’’. "Ecological transition “ mean, the development
Reason (R) : Human activities have the impacts on the
of an anthropocentric orientation toward the natural world
environment.
that emerged in the Western Renaissance but has since
Choose the correct answer from the code given below.
characterized every civilization and nation.
Code:
Ecologically speaking, the transition is expressed in the
(a) Both (A) and (R) are true, and (R) is the correct
growing incorporation of Nature into Culture and by the
explanation of (A).
breakdown of local self sufficiency—the ability of the
(b) Both (A) and (R) are true, and (R) is not the correct
local group to satisfy its needs with existing resources in a
explanation of (A).
particular geographical range.
(c) (A) is true, but (R) is false.
(d) (A) is false, but (R) is true. 16. The entire sequence of communities that successively
U.P.R.O./A.R.O. (Mains) 2016 change in a particular area are called:
U.P.P.C.S. (Pre) 2021 (a) Ecological succession (b) Sere
Ans. (d) (c) Community dynamics (d) Pyramid of biomass
U.P.P.C.S. (Pre) 2020
All organisms in an ecosystem depend upon each other. If the Ans. (b)
population of one organism rises or falls, then this can affect
the rest of the ecosystem. If the foxes in the food chain above The entire sequence of communities that successively change
were killed, the population of rabbits would increase because in a particular area are called ‘Sere’.
they are no longer prey to the foxes So option A is incorrect. 17. Which one of the following is the correct sequence of
Humans impact the physical environment in many ways: the phases of biotic succession?
overpopulation, pollution, burning fossil fuels, and (a) Nudation, Migration, Ecesis, Reaction, Stabilization
deforestation. Changes like these have triggered climate (b) Migration, Nudation, Ecesis, Reaction, Stabilization
change, soil erosion, poor air quality, and undrinkable water. (c) Ecesis, Migration, Nudation, Reaction, Stabilization
So R is correct. (d) Stabilization, Reaction, Nudation, Migration, Ecesis
14. In the grassland, trees do not replace the grasses as a U.P.P.C.S. (Mains) 2016
part of an ecological succession because of: Ans. (a)

F–48 General Studies Environment and Ecology


F. Element studied different species of plants and gave a (c) Population - Community- Ecosystem- Landscape
generally agreed upon definition in 1916. According to (d) Population- Landscape- Community- Ecosystem
him, biotic succession is a natural process of displacement U.P.P.C.S. (Pre) 2017
of one particular community by another in a particular Ans. (c)
area. A scientist named Odum described it as a sequenced
process. The correct order of biotic succession is as follows: The correct order of biological organization ranging from
Nudation organism to biosphere is –
p A population is a group of individuals within the same spe-
cies, within the same area, and that are capable of reproducing
Migration
with one another.
p
A community is a defined as the populations of two or more
Ecesis
species within the same geographical area and within the
p
same time period.
Reaction
An ecosystem is a community along with its abiotic (non-liv-
p
ing) components, such as the water and air.
Stabilization
A biome is a large ecological area with similar climatic
1. Nudation: It is the development of a bare site uninhabited
conditions. A biome can have multiple ecosystems within it.
by any organisms. The process is usually caused by
The biosphere includes all living organisms along with the
disturbances. These factors can be either topographic (soil
spheres they interact with (lithosphere, hydrosphere, etc.).
erosion, wind action etc); climatic (hails, storm, glaciations,
Several communities together form an ecosystem. One or
fire etc.); or biotic (human activities).
more ecosystems are found in a fixed terrain.
2. Invasion or migration: The process of invasion or
migration helps the arrival of seeds, spores or other 19. Consider the following statements about eco-system:
reproductive propagules for establishment of species. 1. The production at the autotroph level is said to be
Invasive species are non native organisms which can spread primary productivity.
widely in a community.
2. The secondary productivity refers to the production
high reproductive rates and better dispersal mechanisms.
3. Ecesis: This is the initial establishment of plant community. at the heterotroph level.
This is dependent on the soil structure. The stage is also called Of the above, the correct statement is/are:
as 'colonization'. In this stage, the early colonizing species (a) Only 1 (b) Only 2
proliferate abundantly through germination, growth, and (c) Both 1 and 2 (d) Neither 1 and 2
reproduction. Ecesis is due to allogenic mechanisms alone.
U.P. R.O./A.R.O. (Pre) 2017
This is the stage at which the pioneer species survive the
Ans. (c)
dispersal mechanisms.
4. Reaction: The environmental conditions get modified by Autotrophs like Green plants are the primary producers.
the action of species occupying the habitat. These changes Heterotrophs like Herbivores are the secondary producers.
subsequently trigger the displacement and replacement of
20. Which of the following is correct sequence of ecosytem
one species by another.
5. Stabilization is the process by which the climax community in order of decreasing productivity?
gets established. A climax community is mature, self (a) Ocean, Lakes, Grasslands and Mangroves
sustaining, stable and is the final stage of succession. The (b) Mangroves, Oceans, Grasslands and Lakes
physical and chemical conditions are altered and stabilized to (c) Mangroves, Grasslands, Lakes and Oceans
such levels that it supports the entire community. The climax (d) Oceans, Mangroves, Lakes and Grasslands
communities are best adapted to the regions of succession and
U.P.R.O./A.R.O. (Pre) 2021
the community structure is likely to continue until another
disturbance steps in. Ans. (c)

18. Which one of the following is the correct order of The correct sequence of ecosystem in order of decreasing
biological organization ranging from organism to productivity is- Mangroves, Grasslands, Lakes and Oceans.
biosphere? Despite occupying about 70 percent of the surface, the
(a) Population- Ecosystem- Community - Landscape productivity of the oceans are only 55 billion tons. Hence
(b) Landscape- Community-Ecosystem-Population option (c) is correct.

Environment and Ecology General Studies F–49


Biodiversity
*The diverse range of organisms (plants and animals) found in *Biodiversity is the diversity of living beings (plant & animals)
an ecosystem, is known as bio-diversity. Species are organized present in an Ecosystem.
in different communities which are found in a particular *Term Biological diversity was coined by Raymond F.
ecosystem. Dasmann in his book ‘A Different kind of Country’ in 1968.
*Bio-diversity is denoted with ( ) alfa, ( ) beta and ( ) gamma *The word biodiversity is a contraction of the phrase “biological
indicating different categories. This classification was done by diversity” and was first coined in 1985 by Walter Rosen of
Whittaker in 1960. the National Research Council as a title word in a seminar,
Alpha diversity refers to the diversity within a particular area
organized to discuss biological diversity.
or ecosystem, and is usually expressed by the number of species
*The most significant aspect of biodiversity is the maintenance
(i.e., species richness) in that ecosystem.
of the ecosystem. Bio-diversity functions at different levels of
For example, if we are monitoring the effect that British farming
species, communities and ecosystems.
practices have on the diversity of native birds in a particular
*The United Nations has proclaimed May 22 as the
region of the country, then we might want to compare species
International Day for Biological Diversity, while many other
diversity within different ecosystems, such as an undisturbed
deciduous wood, a well-established hedgerow bordering a small countries celebrate 29 December as Biodiversity Day.
pasture, and a large arable field. We can walk a transect in each *It is notable that on 29 December,1993 the Convention of
of these three ecosystems and count the number of species Biological Diversity came into force.
we see; this gives us the alpha diversity for each ecosystem.
If we examine the change in species diversity between these
ecosystems then we are measuring the beta diversity.
Gamma diversity is a measure of the overall diversity for the
different ecosystems within a region. Scholars defines gamma
diversity as "geographic-scale species diversity".

*Government of India in collaboration with the Norwegian


Government has established a "Centre for Biodiversity
Policy and Law (CEBPOL)" in the National Biodiversity
Authority (NBA), Chennai, to develop professional expertise
in biodiversity policies and laws and develop capacity
building.

F–50 General Studies Environment and Ecology


Biodiversity Gradient *For instance, Silent valley (Kerala) has very rich Biodiversity.
*It is a part of Western Ghats.
*Biodiversity is not uniform on the Earth. It depends on the
*Silent valley is now included in the list of the biosphere
factors like climate, soil, temperature, rainfall, humidity, etc.
reserve.
of the region.
*Silent valley was declared a National Park is 1984.
*The maximum biodiversity is usually found in the tropical rain
*‘Silent Valley Project’ is related to Kerala state, it is located
forest. They extend from 23.5°N to 23.5°S latitudes. This area
in silent valley.
has a suitable condition for the growth and development of flora
*Valley of Flowers is present in Chamoli district of Uttrakhand.
and fauna as it has heavy rains and high temperature throughout
*Himalayan mountain region also has very high bio-diversity.
the year. That is why it is also called as optimum Biome.
*Biodiversity increases towards the equator. Biodiversity is Biodiversity - Hotspots
not equal on earth. It varies greatly across the globe as well as *A biodiversity hotspot is a biogeographical region of the
within regions. Earth which is extremely biologically diverse and also under
*Among other factors, the diversity of all living things (biota) severe threat due to habitat loss, climate change or extensive
depends on temperature, precipitation, altitude, soil, geography species loss.
and the presence of other species. *As a general rule, to be considered a biodiversity hotspot, a
region must be biologically diverse, with a high proportion of
endemic species which are not found anywhere else on Earth,
and the security of the region must be threatened. So, hotspots
are the richest and most threatened reservoirs of plant and
animal life on Earth.
*India has four bio-diversity hotspots – Western Ghat,
Eastern Himalaya, Indo-Myanmar border and Sundaland.
Nicobar group of Islands is regarded as a part of the Sundaland
biodiversity area.
*According to Conservation International (https://www.
conservation.org), at present, there are 36 biodiversity rich
areas in the world that have been qualified as hotspots, which
represent just 2.5% of earth’s land surface, but support over
50% of the world’s endemic plant species, and nearly 43%
*Biodiversity is normally greater in the lower latitudes as
of bird, mammal, reptile and amphibian species as endemics.
compared to the higher latitudes.
*In February, 2016, CEPF (Critical Ecosystem Partnership
*Tropical rain forests with great biodiversity are found in lower
Fund) declared the North American Coastal Plains as 36th
latitudes. Similarly, valleys found in lower altitudes harness
bio-diversity hot spot.
greater bio-diversity than mountains of high altitude.
*India is a land of remarkable biodiversity. India inhabits 7.6
precent of all mammals, 12.6 percent of birds, 6.2 percent of
reptiles and 4.4 percent of amphibians found in 18 mega-diverse
countries.
*By 36th Constitutional Amendment Act, 1975 Sikkim was
made full-fledged State of the Union of India. It is considered
as a botanist paradise because it lies under hotspot of eastern
Himalaya which is rich in biodiversity. Population over there
mainly comprises of Lepchas, Bhotias and Nepalis.

Causes of Biodiversity loss


*Some of the main threats to biodiversity are:
1. Human Activities and Loss of Habitat,
Environment and Ecology General Studies F–51
2. Deforestation, *Rest other factors like the productivity of the ecosystem,
3. Desertification, intermediate-disturbance and age of the ecosystem contribute
4. Marine Pollution, to the richness of bio-diversity of the ecosystem.

5. Increasing Wildlife Trade and Conservation of Biodiversity


6. Climate Change. Types of conservation
*The current practice of shifting cultivation or Jhum
*Ex-situ Conservation - Conserving biodiversity outside
cultivation in eastern and northeastern regions of India is
the areas where they naturally occur is known as ex-situ
an extravagant and unscientific form of land use. The effects
conservation. Examples - Seed banks, Botanical gardens,
of shifting cultivation are devastating and far-reaching in
Zoological parks etc.
degrading the environment and ecology of these regions.
*In-situ Conservation - Conserving the animals and plants in
*Global warming, fragmentation of habitat and invasion of
their natural habitats is known as in-situ conservation. Examples
alien species can be threats to the biodiversity of a geographical
- National Parks, Sanctuaries, Biosphere reserves, Reserved
area.
forest, Protected forest and Nature Reserves.
*Ex-situ (‘off site’) conservation is a set of conservation
techniques involving the transfer of a target species away from
its native habitat.
*There are two methods for Ex- situ conservation-
*The primary facilities that hold ex-situ conservation and
collections of plants and animals are aquaria, botanical gardens,
and zoos.
*The second method is the storage of seeds, pollen, tissue,
or embryos in liquid nitrogen. This method can be used for
virtually indefinite storage of materials without deterioration
over a much longer period.

*Natural habitat destruction is the main reason for the


loss of biodiversity as the transformation of the natural areas
determines not only the loss of the plant species but also a
decrease in the animal species associated with them.
*It is notable that India’s population is increasing continuously
but birds population is decreasing because of loss in their Sacred groves,
Zoological parks
forests and
habitats, use of insecticides, chemical fertilizers and mosquito lakes
repelling medicines etc. Insecticide like D.D.T. reach up to
coastal

birds through food chain so that premature shell cracks before


time and their population decreases.
*Ecosystem is a dynamic entity. The interdependence of
organisms on other organisms for food contribute to the creation
of different trophic levels. The decrease in trophic level
indicates an abundance of a particular type of organism,
thus resulting in a decrease in biodiversity.

F–52 General Studies Environment and Ecology


*Establishment of the Biosphere reserve is an important reefs are common. They occur around the Gulf of Mannar,
strategy for the conservation of Biodiversity. Gulf of Kachchh, Andaman and Nicobar Islands. Atoll reefs
*The most important strategy for the conservation of are found in Lakshadweep. Sundarbans is known for mangrove
biodiversity alongwith cultural diversity is the establishment forest.
of biosphere reserves. *Coral Bleaching occurs due to increase in temperature and
*The main objective of biosphere reserves are- acidity of the sea, global warming and atmospheric pressure.
- to contribute to the conservation of landscapes, ecosystems,
species and genetic variations. Uses of Biodiversity
- to foster economic and human development which is socio-
*Biodiversity forms the basis for human existence in the
culturally and ecologically sustainable.
form of soil formation, prevention of soil erosion, recycling
- to provide support for research, monitoring, education and
of waste and pollination of crops. Other important uses of
information exchange related to local, national and global issues
biodiversity are:
of conservation and development.

*The Indian Government has established 18 Biosphere


Reserves in India, which protects larger areas of natural habitat.
*Twelve of the eighteen biosphere reserves are part of the World
Network of Biosphere Reserves, based on the UNESCO Man
and the Biosphere Programme List (MAB).
1. Nilgiri Biosphere Reserve
2. Gulf of Mannar Biosphere Reserve
3. Sundarbans Biosphere Reserve
4. Nanda Devi Biosphere Reserve
5. Nokrek Biosphere Reserve
IUCN
6. Pachmarhi Biosphere Reserve
*IUCN is an international organization, which works for nature
7. Simlipal Biosphere Reserve conservation & sustainable use of natural resources. IUCN is
8. Great Nicobar Biosphere Reserve the only international observer organization in the UN General
9. Achanakmar-Amarkantak Biosphere Reserve Assembly with expertise in issues concerning the environment,
10. Agasthyamala Biosphere Reserve specifically biodiversity, nature conservation and sustainable
11. Khangchendzonga natural resource use.
12. Panna *The ‘Red Data Books’ published by the International Union
*The total coral reef area in India is 5,790 km2, distributed for Conservation of Nature and Natural Resources (IUCN)
between 4 major regions: Lakshadweep, Gulf of Mannar, Gulf contain lists of threatened plant and animal species.
of Kachchh, and Andaman and the Nicobar Islands. Fringing *Endangered species are listed in Red Data Book.

Environment and Ecology General Studies F–53


China, India, Maldives, Nepal, Pakistan and Sri Lanka. It is
also known as Great Indian Fruit Bat.
*The Dugong is a large marine mammal which are vulnerable.
The Dugong is a herbivorous animal, who eats grass hence also
known as Sea Cow.
Dugong is found in Eastern Africa, South & Southeast Asia
and Australia.
*A large number of Dugong are inhabiting Gulf of Mannar and
Palk Strait. These species are also found in Saurashtra coast of
Gulf of Kachchh and Andaman and the Nicobar Islands.
*It has been given legal protection under Schedule I of the
Wildlife (Protection) Act, 1972.
*Rann of Kachchh is the natural habitat of Indian wild ass.
This animal has no predators in that area but its existence is
threatened due to the destruction of its habitat.
*Lion-tailed macaque is found in Western Ghats of India. It
is an endangered species which is mostly found in Kerala,
*In June, 2012, IUCN released Red Data list of animals facing Karnataka and the border region of Tamil Nadu.
the danger of extinction. 15 species of India were listed in *Cheetah was declared extinct in India in 1952.
critically endangered category. *Snow leopard's habitat is found 3000-4500 meters above the
*In this list, Great Indian Bustard, Siberian crane and sea level. Surveys held in Uttarakhand and Himachal Pradesh
Sociable Lapwing were listed in the critically endangered have confirmed the presence of Snow leopards in India.
category, Musk Deer in the vulnerable category while Wild *Black-necked crane is the state bird of Jammu and Kashmir.
Ass was listed in the near threatened category. Flying squirrels are found in India, Russia, Japan, China,
*Red panda was listed in the endangered category. Europe, Africa and North America.
*South Asia is known for its high diversity of flying squirrels.
Biodiversity & There are about 17 species belonging to 7 genera.In India,
Endangered Species most of these species are found in the eastern Himalayas and
*In May 2003, a scientist working with the Peregrine Fund the north-east, at the confluence of two biogeographically
presented a research paper that confirmed that the vulture deaths significant regions, the Himalayan and the Indo-Malayan
were due to the anti-inflammatory painkilling drug Diclofenac regions. A study that was conducted in Arunachal Pradesh
Sodium. A vulture that had died had high levels of diclofenac reported occurrences of a whopping number, 14 species of
in their kidneys. flying squirrels from the state, with the red giant flying squirrel
*In January 2004, the results of a joint study conducted by being the most common one, followed by the parti-coloured
the Peregrine Fund and the Ornithological Society of Pakistan flying squirrel (Hylopetes alboniger).
confirmed that diclofenac sodium was indeed the primary *Marine animals like Dugongs, Dolphins, Whale, Salt Water
reason for vulture deaths. Crocodiles, Turtle, Sea Snake etc are found abundantly in
*The devastating effect of this medicine can be understood from Andaman & Nicobar Island.
the fact that it caused the death of 8.5 crore vultures in India, *Shrew and Tapir are found in Great Himalayan ranges. These
Nepal and Pakistan in 10 years. are not found in Malabar regions.
*Tambalacoque (Dodo plant) of Mauritius have been extinct *The red panda is a territorial mammal. Its zoological name
because of the extinction of Dodo bird who was helping this is Ailurus Fulgens. Red Panda lives in temperate climates, in
plant in its pollination. Plants failed to reproduce in the absence deciduous and coniferous forests, usually with an understory
of Dodo. Seed of Dodo plants was the food of these birds and of bamboo and hollow trees. This makes them a key species
medium of seed dispersal of these plants were by their faecal of these forests and indicators of forest health.
matter. *They are found in the Himalayan region, in parts of Nepal,
*Indian flying Fox is a species of bats found in Bangladesh Bhutan, Myanmar and in the Indian States of Sikkim, West

F–54 General Studies Environment and Ecology


Bengal, Meghalaya and Arunachal Pradesh. Pong Dam Lake Himachal 19.8.2002 156.62
*Most of the red pandas of the world are found in China whereas Pradesh
the majority of the Indian red panda is found in Arunachal Renuka Wetland Himachal 8.11.2005 0.2
Pradesh. Pradesh
*According to ENVIS Center for Faunal Diversity hosted Wular Lake Jammu & 23.3.1990 189
by Zoological Survey of India sponsored by Ministry of Kashmir
Environment, Forest and Climate Change, below is the risk Hokera Wetland Jammu and 8.11.2005 13.75
status of the animals: Kashmir
Golden Langur : Endangered Surinsar-Mansar Jammu and 8.11.2005 3.5
Hoolock Gibbon : Endangered Lakes Kashmir
Asiatic Wild Dog : Endangered Tsomoriri Lake Jammu and 19.8.2002 120
Desert Cat : Least Concern Kashmir
*There is no animal named Desert Cat listed in Red List of Asthamudi Wet- Kerala 19.8.2002 614
IUCN. Chinese Desert Cat is listed as a vulnerable category land
in Red List. Sasthamkotta Kerala 19.8.2002 3.73
Ramsar Convention Lake
Vembanad Kol Kerala 19.8.2002 1512.5
*The Ramsar Convention is associated with the conservation
Wetland
and sustainable use of wetlands. It is also known as the
Tso Kar Wetland Ladakh 17.11.2020 95.77
Convention on Wetlands. Complex
*India is a party to Ramsar Convention and has declared many
Bhoj Wetlands Madhya 19.8.2002 32.01
areas as Ramsar Sites to conserve all the site through ecosystem Pradesh
approach and allow their simultaneous sustainable use.
Lonar Lake Maharashtra 22.7.2020 4.27
*The list of Ramsar Sites comprises wetland of international
Nandur Mad- Maharashtra 21.6.2019 14.37
importance. India currently has 49 sites designated as wetlands hameshwar
of international importance (Ramsar Sites).
Loktak Lake Manipur 23.3.1990 266
Ramsar Sites in India Bhitarkanika Orissa 19.8.2002 650
Date of Area Mangroves
Name of Site State Location Declaration (in Sq. Chilka Lake Orissa 1.10.1981 1165
km.) Beas Conservation Punjab 26.9.2019 64.289
Kolleru Lake Andhra 19.8.2002 901 Reserve
Pradesh Harike Lake Punjab 23.3.1990 41
Deepor Beel Assam 19.8.2002 40 Kanjli Lake Punjab 22.1.2002 1.83
Kabartal Wetland Bihar 21.07.2020 26.2 Keshopur-Mi- Punjab 26.9.2019 3.439
Khijadia Wildlife Gujarat 13.04.2021 5.12 ani Community
Sanctuary Reserve
Nalsarovar Bird Gujarat 24.09.2012 120 Nangal Wildlife Punjab 26.9.2019 1.16
Sanctuary Sanctuary
Thol Lake Wildlife Gujarat 05.04.2021 6.99 Ropar Lake Punjab 22.1.2002 13.65
Sanctuary Keoladeo Ghana Rajasthan 1.10.1981 28.73
Wadhvana Wet- Gujarat 05.04.2021 6.3 NP
land Sambhar Lake Rajasthan 23.3.1990 240
Bhindawas Wild- Haryana 25.05.2021 4.12 Point Calimere Tamil Nadu 19.8.2002 385
life Sanctuary Wildlife and Bird
Sultanpur Nation- Haryana 25.05.2021 1.425 Sanctuary
al Park Rudrasagar Lake Tripura 8.11.2005 2.4
Chandertal Wet- Himachal 8.11.2005 0.49 Bakhira Wildlife Uttar Pradesh 29.06.2021 28.94
land Pradesh Sanctuary

Environment and Ecology General Studies F–55


Haiderpur Wet- Uttar Pradesh 8.12.2021 69.08 results and application for Intellectual Property Right (IPR)
land relating to Indian biological resources.
Nawabganj Bird Uttar Pradesh 19.9.2019 2.246 *The International Treaty on Plant Genetic Resources for
Sanctuary Food and Agriculture (IT PGRFA) is popularly known as the
Parvati Agra Bird Uttar Pradesh 2.12.2019 7.22 International Seed Treaty.
Sanctuary *It is a comprehensive international agreement in harmony
Saman Bird Sanc- Uttar Pradesh 2.12.2019 52.63 with the Convention on Biological Diversity. It aims at
tuary guaranteeing food security through the conservation, exchange
Samaspur Bird Uttar Pradesh 3.10.2019 79.94 and sustainable use of the world’s plant genetic resources for
Sanctuary food and agriculture.
Sandi Bird Sanc- Uttar Pradesh 26.9.2019 30.85 *Bird Life International is a global partnership
tuary of conservation organizations that strives to conserve birds,
Sarsai Nawar Jheel Uttar Pradesh 19.9.2019 16.13 their habitats and global biodiversity.
Sur Sarovar Uttar Pradesh 21.8.2020 4.31 *It is the world’s largest partnership of conservation
Upper Ganga Uttar Pradesh 8.11.2005 265.9 organizations with 119 Bird Life Partners Worldwide.
River *This organization identifies important bird and biodiversity
(Brijghat to Naro- areas. It has so far identified more than 12000 such areas.
ra Stretch) *Concept of bio-diversity hot spots was propounded by British
Asan Conserva- Uttarakhand 21.7.2020 4.444 environmentalist Norman Myers.
tion Reserve *The Cartagena Protocol on Biosafety to the Convention on
East Kolkata Wet- West Bengal 19.8.2002 125 Biological Diversity is an international agreement which aims
lands to ensure the safe handling, transport and use of living modified
Sunderbans Wet- West Bengal 30.1.2019 4230 organism (LMO) resulting from modern biotechnology that
land
may have adverse effects on biological diversity, also taking
*It was signed on 2 February, 1971 at Ramsar in Iran. This into account risks to human health.
date is celebrated as World Wetland Day now. Theme of World *It was adopted on 29 January, 2000 and entered into force on
Wetland Day, 2020 was 'Wetlands and Biodiversity', whereas 11 September, 2003.
in 2021, the theme of the Day is 'Wetlands and water'. *India signed the Biosafety Protocol on 23rd January, 2001.
*The Montreux Record is a register of wetland sites on the In India, the Ministry of Environment and Forest implements
list of Wetlands of International Importance where changes in this protocol.
ecological character have occurred, are occurring, or are likely *TRAFFIC - Trade-Related Analysis of Fauna & Flora in
to occur as a result of technological developments, pollution commerce is a non-governmental organization. It supervises
or other human interference. It is maintained as part of the trade of plants and animals in the context of sustainable
Ramsar list. development and biodiversity conservation.
Various Acts & Agreements *TRAFFIC was established in 1976. It is an alliance of WWF
*The 'Bio-Diversity Act' was passed in the year 2002. It and IUCN. It is not a bureau under the UNEP.
provides for the preservation of Biological Diversity in India, Biodiversity United Nation &
a mechanism for equitable sharing of benefits arising out of the
use of traditional biological resources and knowledge.
Other International Organizations
*The National Biodiversity Authority (NBA) was established *The tenth meeting of the Conference of the Parties
in 2003 to implement India’s Biodiversity Act (2002). The (COP 10) of CBD was held in Nagoya, Japan, from 18 to
headquarter of NBA is in Chennai, Tamil Nadu. 29 October, 2010. Nagoya Biodiversity Summit concluded
*India’s Biological Diversity Act (2002) covers conservation, with the adoption of historical decisions that would permit
use of biological resources and associated knowledge. It the community of nations to meet the unprecedented challenges
provides a framework for access to biological resources and of the continued loss of biodiversity compounded by climate
sharing the benefits arising out of such access and use. change.
*The Act also includes in its ambit the transfer of research *The eleventh meeting of the Conference of the Parties

F–56 General Studies Environment and Ecology


(COP-11) was held in Hyderabad, India, from 8 to 19 *It is the leading scientific expert on peatlands and climate
October, 2012. change as well as wetlands and waterbird migration based on
*The Twelfth meeting of the Conference of the Parties extensive research as well as field projects.
(COP-12) was held in Pyeongchang, the Republic of Korea *Wetlands International’s work ranges from research and
from 6 - 17 October, 2014. community-based field projects to advocacy and engagement
*Thirteenth meeting of the Conference of the Parties with governments, corporate and international policy fora and
(COP-13) was held in Cancun, Mexico, from 4 - 17 conventions.
December, 2016. *Wetlands International works through partnerships and is
CoP-14 to the United Nations convention on Biological supported by contributions from an extensive specialist expert
Diversity (CBD) was held in Sharm El-Sheikh, Egypt, from network and tens of thousands of volunteers.
17 to 29 November 2018. COP-15 will be held in Kunming, Biodiversity: Miscellaneous
China in 2021. *Seabuckthorn is a medicinal plant found in the Himalayan
*The United Nations Collaborative Programme on Reducing region.
Emissions from Deforestation and Forest Degradation *Seabuckthorn fruits grow in the cold deserts of Ladakh region
in Developing Countries – or UN-REDD Programme of Jammu and Kashmir, Lahaul-Spiti in Himachal Pradesh and
is a collaborative initiative of the United Nations Food some parts of Arunachal Pradesh.
and Agriculture Organization (FAO), the United Nations *The bushy character of the plant with a high density of branches
Development Programme (UNDP) and the United Nations help in checking soil erosion and helps in soil stabilization. The
Environment Programme (UNEP), and harnesses the technical fruit of this plant is a source of nutritious juice.
expertise of these UN agencies. This programme was started *It is generally accepted that climate factors regulate most insect
in 2008. species' life cycle, including butterflies. It is well established
*The objective of this programme is to mitigate climate change that temperature is a key factor for increasing daily activities
through reducing net emission of greenhouse gases through (flight, foraging, movements) of butterflies. Butterflies cannot
enhanced forest management in developing countries. UN- tolerate low temperatures. So maximum number of the species
REDD + Programme focuses on conservation and sustainable of butterflies are found in tropical countries.
management of forests. *With reference to India’s biodiversity, Ceylon frogmouth
*Proper design and effective implementation of UN-REDD (Batrachostomus moniliger), Coppersmith barbet (Megalaima
+ Programme can significantly contribute to the protection of haemacephala), Grey-chinned minivet (Pericrocotus solaris)
biodiversity and maintaining resilience of forest ecosystem. and White-throated redstart (Phoenicurus schisticeps) are the
It also provides technical and financial support to developing species of birds.
countries thus contributing to poverty alleviation. *Hibernation or winter sleep refers to a state of inactivity
*The Economics of Ecosystems and Biodiversity (TEEB) is a among animals by spending winter in a dormant state. It
global initiative focused on “making values visible”. is a survival strategy designed to conserve energy when
*Its principal objective is to mainstream the values of weather conditions are not favourable. This condition is
biodiversity and ecosystem services into decision-making at observed among both types of animal cold blooded and
all levels. It aims to achieve this goal by following a structured warm blooded.
approach to valuation that helps decision makers recognize the *Squirrels, bears and some bats resort to hibernation in winter
wide range of benefits provided by ecosystems and biodiversity, when they do not find means to feed themselves. In this state,
demonstrates their values in economic terms and where animal's metabolism slows, its temperature plunges and
appropriate, captures these values in decisions making. breathing slows.
*Wetlands International is a global organization that works *The tallest known tree is Redwood (Sequoia). It is found in
to sustain and restore wetlands and their resources for people California (U.S.A.). This species includes the tallest living trees
and biodiversity. on Earth, reaching up to 379 feet (115.5 m) in height.
*It is an independent, not-for-profit, global organization *Brahmani and Vaitarni rivers merge at Bhitarkanika before
supported by Government and NGO membership from around emptying into the sea. Bhitarkanika is a protected area for
the world. wildlife and biodiversity. Brahmani river is known as South
*Its head office is in the Netherlands. Koel in Jharkhand.

Environment and Ecology General Studies F–57


*The Brahmani river is formed by the confluence of South Koel (a) Food
and Sankh rivers near Rourkela. Vaitarni river originates from (b) Drug
Guptaganga hills in Keonjhar district of Orissa. (c) Industrial Use
*Biomass is a term for all organic materials that arise from (d) Maintenance of ecosystem
plants (including algae, tree and crops). Biomass is produced U.P.P.C.S. (Pre) 2015
by green plants converting sunlight into plant material through Ans. (d)
photosynthesis and includes all land and water based vegetation
The most significant aspect of biodiversity is the maintenance
as well as all organic wastes. Hence deep sea has minimum
of the ecosystem. Biodiversity functions on different levels
biomass production.
of species, communities and ecosystems.
*Highest biomass production is found among tropical rain
forest. It is approximately 2200 gram/square meter/per year. 4. The destruction of biodiversity is due to:
Biomass production for biomes: (a) The lack of natural habitats of the organism
Deciduous forest - 1200 gram (dry weight)/square (b) Environmental pollution
(c) Destruction of forests
meter/every year
(d) All the above
Taiga - 800 gram (dry weight)/square
U.P.P.C.S. (Mains) 2002
meter/every year
Ans. (d)
Prairie - 600 gram (dry weight)/square
meter/every year The main cause of the loss of biodiversity is the influence
Deep Sea - Negligible gram (dry weight)/ of human beings on the world’s ecosystem. The threats to
square meter/every year biodiversity can be summarized in the following main points:
Alteration and loss of the habitats
Basic Questions Introduction of exotic species and genetically modified
organisms
1. Biodiversity is described as –
Pollution
(a) The range of different species in an environment
Climate change
(b) The seasonal and daily changes in an environment
(c) The way species differ from one another Overexploitation of resources
(d) The influence of physical factors on an environment 5. Which of the following is NOT an example of in-situ
U.P.R.O./A.R.O. (Mains) 2014 conservation strategy?
Ans. (a) (a) Biosphere reserve
The diverse range of organisms (plants and animals) found (b) Botanical garden
in an ecosystem, is defined as biodiversity. The species are (c) National Park
organized in different communities which are found in a (d) Sacred groves
particular ecosystem. Bio-diversity is denoted with (α) alfa, U.P. P.C.S. (Pre) 2018
(E) beta and (γ) gamma indicating different categories. This Ans. (b)
classification was advanced by Whittaker in 1960. Examples of Ex-situ conservation - Seed banks, Zoological
2. Biodiversity means – parks, Botanical gardens etc.
(a) Diverse kinds of plants and vegetation. Examples of in-situ conservation - National Parks, Sanctuaries,
(b) Diverse kinds of animals. Biosphere reserves, Reserved Forest, Protected Forest and
(c) Diverse kinds of plants and animals in a particular area. Nature Reserves.
(d) Diverse kinds of exotic plants and animals.
U.P.P.C.S. (Pre) 2014 6. The main reason for the decrease in biodiversity is:
Ans. (c) (a) Habitat pollution
(b) Introduction of exotic species
The diversity of organisms (Plants and animals) present in
(c) Over exploitation
an ecosystem is defined as bio-diversity.
(d) Natural habitat destruction
3. The most significant aspect of biodiversity is: U.P.P.C.S. (Pre) 2016

F–58 General Studies Environment and Ecology


Ans. (d) U.P. R.O./A.R.O. (Mains) 2017
Ans. (b)
Natural habitat destruction is the main reason for the decrease
in biodiversity as the transformation of the natural areas Government of India in collaboration with the Norwegian
determines not only the loss of the plant species but also a Government has established a "Centre for Biodiversity
decrease in the animal species associated with them. Policy and Law (CEBPOL)" in the National Biodiversity
Authority (NBA), Chennai, to develop professional expertise
7. Which one of the following is the most important factor
in biodiversity policies and laws and develop the capacity
responsible for the decline of biodiversity?
building.
(a) Genetic assimilation
(b) Controlling assimilation 11. Biodiversity day is celebrated on –
(c) Destruction of habitat (a) 29 December
(d) Controlling pests (b) 27 June
U.P.P.C.S (Pre) 2010 (c) 28 February
Ans. (c) (d) 30 January
R.A.S./R.T.S.(Pre) 2012
Habitat loss is one of the biggest threats to biodiversity.
Ans. (*)
Cutting of forests to create fields, filling in wetlands to
build houses, and creating dams that change river flow, The United Nations has proclaimed May 22 The International
are all examples of habitat destruction. The process of Day for Biological Diversity (IDB) to increase understanding
genetic assimilation is one by which a phenotypic character, and awareness of biodiversity issues. When first created
which initially is produced only in response to some by the Second Committee of the UN General Assembly in
environmental influence, becomes, through a process of late 1993, 29 December (the date of entry into force of the
selection, taken over by the genotype, so that it is formed Convention of Biological Diversity), was designated The
even in the absence of the environmental influence. International Day for Biological Diversity. In December
8. The main reason for the decrease in biodiversity is – 2000, the UN General Assembly adopted 22 May as IDB,
(a) Habitat pollution to commemorate the adoption of the text of the Convention
(b) Introduction of exotic species on 22 May 1992 by the Nairobi Final Act of the Conference
(c) Over-exploitation for the Adoption of the Agreed Text of the Convention on
(d) Habitat destruction Biological Diversity. This was partly done because it was
U.P.P.C.S. (Pre) 2015 difficult for many countries to plan and carry out suitable
Ans. (d) celebrations for the date of 29 December, given the number
of holidays that coincide around that time of year.
See the explanation of the above question.
12. When the World Biodiversity Day is observed?
9. Biodiversity has maximum danger from: (a) March, 22
(a) Destruction of natural habitats and vegetation (b) May, 22
(b) Improper agricultural operations (c) June, 23
(c) Climate change (d) April, 16
(d) Water pollution U.P. P.C.S. (Pre) 2018
U.P.P.C.S. (Pre) 2017 Ans. (b)
Ans. (a)
See the explanation of the above question.
See the explanation of the above question.
13. Which of the following can be threats to the biodiversity
10. Government of India in collaboration with the of a geographical area?
Norwegian Government has established a centre for 1. Global warming
Biodiversity Policy and Law at- 2. Fragmentation of habitat
(a) Dehradun (b) Chennai 3. Invasion of alien species
(c) Shillong (d) New Delhi 4. Promotion of vegetarianism

Environment and Ecology General Studies F–59


Select the correct answer using the code given below: 16. Consider the following regions:
(a) 1, 2 and 3 (b) 2 and 3 1. Eastern Himalayas
(c) 1 and 4 (d) 1, 2, 3 and 4 2. Eastern Mediterranean region
I.A.S. (Pre) 2012 3. North-western Australia
Ans. (a) Which of the above is/are Biodiversity Hotspot(s)?
(a) 1 only (b) 1 and 2
Global warming, fragmentation of habitat and invasion
(c) 2 and 3 (d) 1, 2 and 3
of alien species can be threats to the biodiversity of a
I.A.S. (Pre) 2009
geographical area while promotion of vegetarianism is not
Ans. (a)
responsible for the same. Hence option (a) is the correct
answer. Eastern Himalayas is a biodiversity hotspot and it is
14. Which of the following can be threats to the biodiversity Mediterranean basin and Southwest Australia which are
of a region? biodiversity hotspots. Hence only 1 is correct.
1. Global Warming
17. The ‘Hotspot’ of Biodiversity in India are:
2. Fragmentation of habitat
(a) Western Himalayas and Eastern Ghat
3. Invasion of alien species
(b) Western Himalayas and Sunderban
4. Promotion of Vegetarianism
(c) Eastern Himalayas and the Western Ghats
Select the correct answer using the code given below:
(d) Eastern Himalayas and Silent Valley
Codes :
R.A.S./R.T.S.(Pre) 2008
(a) (2) and (3) only
Ans. (c)
(b) (2), (3) and (4)
(c) (1), (2) and (3) According to Conservation International at present, there
(d) (1) and (4) are 36 biodiversity rich areas in the world that have been
R.A.S./R.T.S. (Pre) 2016 qualified as hotspots, which represent just 2.5% of earth’s
Ans. (c)
land surface, but support over 50% of the world’s endemic
Five main threats to biodiversity are commonly recognized, plant species, and nearly 43% of bird, mammal, reptile and
these are invasive alien species, climate change, nutrient amphibian species as endemics. India has four biodiversity
loading and pollution, habitat change and overexploitation.
hotspots namely Himalaya, Indo-Burma (Northeastern India
Hence, option (c) is the correct answer.
15. Which one of the following strongly threatens and Andaman Islands), Sundalands (Nicobar Islands) and
biodiversity? Western Ghats (and Sri Lanka).
(a) Fragile ecosystem such as mangroves and wetlands. 18. Which of the following is considered a 'hot-spot' of
(b) Inaccessible habitats in the Himalayas. biodiversity in India?
(c) Destruction of natural habitats and vegetation, and (a) Aravalli hills (b) Indo-Gangetic plain
Jhum cultivation (c) Eastern Ghats (d) Western Ghats
(d) Creation of biosphere reserves Uttarakhand P.C.S. (Pre) 2016
Jharkhand P.C.S. (Pre) 2010 Ans. (d)
Ans. (c)
See the explanation of the above question.
Some of the main threats to biodiversity are: 1. Human
19. Consider the following statements:
Activities and Loss of Habitat, 2. Deforestation, 3.
1. Biodiversity hotspots are located only in tropical
Desertification, 4. Marine Environment, 5. Increasing Wildlife
regions.
Trade and 6. Climate Change. The current practice of shifting
cultivation or Jhum cultivation in eastern and northeastern 2. India has four biodiversity hotspots ie, Eastern
regions of India is an extravagant and unscientific form of Himalayas, Western Himalayas, Western Ghats
land use. The effects of shifting cultivation are devastating and Andaman and the Nicobar Islands.
and far-reaching in degrading the environment and ecology Which of the statements given above is/are correct?
of these regions. (a) 1 only (b) 2 only

F–60 General Studies Environment and Ecology


(c) Both 1 and 2 (d) Neither 1 nor 2 R.A.S./R.T.S. (Re. Exam) (Pre) 2013
I.A.S. (Pre) 2010 Ans. (c)
Ans. (d)
See the explanation of the above question.
Hotspots are not only located in tropical regions but also
23. The most biodiversity rich area in India is–
in temperate regions (California Floristic Province, Japan).
India has four bio-diversity hot spots- Eastern Himalayas, (a) Gangetic plain (b) Trans Himalayas
Western Ghats, Indo-Myanmar border and Sundaland. The (c) Western Ghats (d) Central India
Sundaland includes the Nicobar group of islands but does U.P.P.C.S. (Pre) 2019
not includes the Andaman group of islands. Ans. (c)

20. Which of the following is a biodiversity saturated area See the explanation of the above question.
in India?
24. Which of the following has been identified as a
(a) Eastern Ghat (b) Western Ghat
‘biodiversity hotspot’ in India?
(c) Thar Desert (d) The Bay of Bengal
(a) Sundarban (b) Western Ghats
U.P.P.C.S. (Pre) 2015
(c) Manas (d) Cherapunji
Ans. (b)
U.P.P.C.S. (Pre) (Re. Exam) 2015
India is a land of remarkable biodiversity. India inhabits Ans. (b)
7.6 percent of all mammals, 12.6 percent of birds, 6.2 per
cent of reptiles and 4.4 per cent of amphibians found in See the explanation of the above question.
18 megadiverse countries. Western Ghat is a biodiversity 25. Biodiversity hotspots in India are the-
saturated area in the given options. (i) Eastern Ghats (ii) Western Ghats
21. With reference to biodiversity, which of the following (iii) Eastern Himalayas (iv) Western India
regions in India is regarded as ‘HOTSPOT’? Codes :
(a) Andaman and Nicobar Island (a) (i) and (ii) (b) (ii) and (iii)
(b) Gangetic Plain (c) (i), (iii) and (iv) (d) (ii), (iii) and (iv)
(c) Central India R.A.S./R.T.S. (Pre) 2013
(d) None of the above Ans. (b)
U.P.P.C.S. (Mains) 2009
See the explanation of the above question.
Ans. (a)
26. Which of the following is considered ‘Hot Spot’ of
A biodiversity hotspot is a biogeographical region of the Earth biodiversity?
which is extremely biologically diverse and also under severe
(a) Gangetic Plains (b) Eastern Himalaya
threat due to habitat loss, climate change or extensive species
(c) Gujarat (d) Central India
loss. As a general rule, to be considered a biodiversity hotspot,
U.P.U.D.A./L.D.A. (Spl) (Mains) 2010
a region must be biologically diverse, with a high proportion
of endemic species which are not found anywhere else on Ans. (b)
Earth, and the security of the region must be threatened. So, See the explanation of the above question.
hotspots are the richest and most threatened reservoirs of
plant and animal life on Earth. 27. The ‘Hotspots’ (in the context of the environment)
India has four bio-diversity hot spots – found in India are –
Western Ghat, Eastern Himalaya, Indo-Myanmar border (a) Eastern Ghat, Western Ghat
and Sundaland. Nicobar island is regarded as a part of the (b) Vindhya Range, Eastern Ghat
Sundaland bio-diversity area and Andaman group of Islands (c) Eastern Himalayan Range, Western Ghat
come under the Indo-Myanmar hotspot. Hence, the correct (d) Shivalik Range, Eastern Ghat
answer is option (a). U.P.P.C.S. (Mains) 2012
22. Which one of the following areas in India, is a hotspot Ans. (c)
of biodiversity?
See the explanation of the above question.
(a) Gangetic Plains (b) Sundarbans
(c) Western Ghats (d) Eastern Ghats 28. Which one of the following is not a ‘biodiversity

Environment and Ecology General Studies F–61


hotspot’ of India? (a) 15 years (b) 25 years
(a) Himalayas (b) Vindhyas (c) 40 years (d) 50 years
(c) North-East India (d) Western Ghats U.P.P.C.S. (Pre) 2005
U.P.P.C.S. (Mains) 2016 Ans. (d)
Ans. (b)
According to the definition provided by the International
See the explanation of the above question. Union for Conservation of Nature, a species not seen during
the last 50 years is considered to be extinct.
29. Which of the following regions of India has been
designated as biodiversity hot spots? 34. Which one of the following is not responsible for the
Select the correct answer from the code given below: extinction of a species?
1. Eastern Himalaya 2. Eastern Ghat (a) Broad niche
3. Western Ghat 4. Western Himalaya (b) Large body size
Codes : (c) Narrow niche
(a) 1 and 2 only (b) 1 and 3 only (d) Lack of genetic variability
(c) 2 and 4 only (d) 3 and 4 only U.P.P.C.S. (Pre) 2014
U.P.P.C.S.(Pre) 2012 Ans. (a)
Ans. (b)
In ecology, the term “niche” describes the role an organism
See the explanation of the above question. plays in a community. A species’ niche encompasses both
30. Which one of the following regions of India is regarded the physical and environmental conditions it requires (like
as an ‘ecological hot spot’? temperature or terrain) and the interactions it has with other
(a) Western Himalayas species (like predation or competition). In general, species
(b) Eastern Himalayas that have narrow or limited niches are considered to be
(c) Western Ghats specialist species. Koalas which feed only on leaves from
(d) Eastern Ghats eucalyptus trees in Australia, are an example of a specialist
I.A.S. (Pre) 1996 species. Species with broader niches, like coyotes or raccoons
Ans. (b & c) are considered generalists.
See the explanation of the above question. 35. India aims to achieve land degradation neutrality by
the year:
31. Which of the following is a bio-diversity rich place in
(a) 2025 (b) 2030
India?
(c) 2035 (d) 2040
(a) Western Ghat (b) Eastern Ghat
U.P.P.C.S. (Pre) 2019
(c) Thar Desert (d) The Bay of Bengal
Ans. (b)
U.P.P.C.S. (Pre) 2014
Ans. (a) India aims to achieve Land Degradation Neutrality (LDN)
by the year 2030. LDN represents a paradigm shift in land
See the explanation of the above question.
management policies and practices. It is a unique approach
32. The tallest known living tree: that counter balances the expected loss of productive land
(a) Eucalyptus (b) Sequoia with the recovery of degraded areas.
(c) Deodar (d) Parnang
40th B.P.S.C. (Pre) 1995 36. The ‘Red Data Books’ published by the International
Ans. (b) Union for Conservation of Nature and Natural
Resources (IUCN) contain lists of:
The tallest known tree is Redwood (Sequoia). It is found in 1. Endemic plant and animal species present in the
California (U.S.A.). This species includes the tallest living
biodiversity hotspots.
trees on Earth, reaching up to 379 feet (115.5 m) in height.
2. Threatened plant and animal species.
33. A species is considered to be extinct if it has not been 3. Protected sites for conservation of nature and
seen in its natural habitat for – natural resources in various countries.

F–62 General Studies Environment and Ecology


Select the correct answer using the code given below?
See the explanation of the above question.
(a) 1 and 3 (b) 2 only
(c) 2 and 3 (d) 3 only 42. ‘Red Data Book’ is concerned with –
I.A.S. (Pre) 2011 (a) Facts about biodiversity
Ans. (b) (b) Organisms & animals facing the danger of extinction
The ‘Red Data Books’ published by the International Union (c) Plantation
(d) Illegal hunting of forest animals by smugglers
for Conservation of Nature and Natural Resources (IUCN)
U.P.P.C.S. (Mains) 2002
contain lists of threatened plants and animal species.
Ans. (b)
37. Organization related to “Red Data Book” or “Red List
See the explanation of the above question.
is”
(a) U.T.E.S (b) I.U.C.N. 43. Which one of the following groups of animals belongs
(c) I.B.W.C. (d) W.W.F to the category of endangered species?
M.P.P.C.S. (Pre) 2014 (a) Great Indian Bustard, Musk Deer, Red Panda and
Ans. (b) Asiatic Wild Ass
(b) Kashmir Stag, Cheetal, Blue Bull and Great Indian
See the explanation of the above question.
Bustard
38. "Red Data Book" is published by – (c) Snow Leopard, Swamp Deer, Rhesus Monkey and
(a) USEPA (b) IUCN Saras (Crane)
(c) WWF (d) IG-BP (d) Lion-tailed Macaque, Blue Bull, Hanuman Langur
Uttarakhand P.C.S. (Pre) 2017 and Cheetal
Ans. (b) I.A.S. (Pre) 2012
Ans. (a)
See the explanation of the above question.
In June, 2012, IUCN released Red Data list of animals facing
39. Endangered species are listed in –
the danger of extinction. In which 15 species of India were
(a) Dead Stock Book (b) Red Data Book
listed in the most endangered category. In this list, Great
(c) Live Stock Book (d) None of the above
Indian Bustard, Siberian Crane and Sociable Lapwing
U.P.P.C.S.(Pre) 2013
Ans. (b) were listed in the most endangered category, Musk Deer
in the vulnerable category while Wild Ass was listed in the
Endangered species are listed in Red Data Book. near threatened category. The Red Panda was listed in the
40. 'Red Data Book' contains data of – endangered category. Thus option (a) is the correct answer.
(a) Exotic species 44. Which one of the following Indian birds is a highly
(b) Endangered species endangered species?
(c) Economically important species (a) Golden Oriole
(d) All plant and animal species (b) Great Indian Bustard
Chhattisgarh P.C.S. (Pre) 2019 (c) Indian Fantail Pigeon
Ans. (b) (d) Indian Sunbird
U.P.P.C.S. (Mains) 2005
See the explanation of the above question.
Ans. (b)
41. Red Data Book provides data on –
Great Indian Bustard is a highly endangered species.
(a) Red flower plants
(b) Red coloured fishes 45. Which one of the following is not correctly matched?
(c) Red coloured insects (a) Golden Langur : Endangered
(d) Endangered plants and animals (b) Desert Cat : Endangered
Jharkhand P.C.S. (Mains) 2016 (c) Hoolock Gibbon : Endangered
Ans. (d) (d) Asiatic Wild Dog : Threatened

Environment and Ecology General Studies F–63


Uttarakhand P.C.S. (Pre) 2012 (a) Expansion of Roads
U.P.P.C.S. (Pre) 2014 (b) Urbanization
Ans. (*) (c) Afforestation of barren land
According to ENVIS Center for Faunal Diversity hosted (d) Extension of Agriculture
by Zoological Survey of India sponsored by Ministry of Uttarakhand P.C.S. (Pre) 2006
Environment, Forest and Climate Change, below is the risk Ans. (c)
status of the animals given in the options.
Afforestation of barren land is not a reason for the loss of
Golden Langur : Endangered
biodiversity while the expansion of roads, urbanization
Hoolock Gibbon : Endangered
and extension of agriculture are responsible for the loss of
Asiatic Wild Dog : Endangered
biodiversity.
Desert Cat : Least Concern
There is no animal named Desert Cat listed in Red List 48. Which of the following statements about Sikkim are true?
of IUCN. Chinese Desert Cat are listed as a vulnerable Select the correct answer from the code given below –
category in Red List. Hence option (b) and (d) are not I. It became an integral part of India in 1975.
correctly matched. Uttar Pradesh Public Service Commission II. It is considered as a botanist paradise.
considered (a) as the correct answer in its initially released
III. Its population is mainly made up of the Lepchas.
answer key but in revised answer key this question was
Codes :
removed from assessment category.
(a) I (b) I and II
46. While India’s Human population is growing at an (c) II and III (d) I, II and III
outstanding pace, the bird population is shrinking U.P.Lower Sub. (Spl) (Pre) 2004
fastly mainly because: Ans. (d)
1. There has been an abnormal increase in the
By 36th Constitutional Amendment Act, 1975 Sikkim was
number of hunters.
2. Bio-pesticides and organic manure are being used made full-fledged State of the Union of India. It is considered
on a large scale. as a botanist paradise because it lies under hotspots of
3. There has been a large-scale reduction in the northern Himalaya which is rich in biodiversity. Population
habitats of the birds. over here mainly comprises of Lepchas, Bhotias and Nepalis.
4. There has been large-scale use of pesticides, 49. The most important strategy for the conservation of
chemical fertilizers and mosquito repellents. biodiversity together with traditional human life is the
Select your answer correctly using the code given establishment of –
below: (a) Biosphere reserves
Codes : (b) Botanical Gardens
(a) 1 and 2 are correct (c) National parks
(b) 2 and 3 are correct (d) Wildlife Sanctuaries
(c) 3 and 4 are correct I.A.S. (Pre) 2014
(d) 1 and 4 are correct Ans. (a)
U.P.P.C.S.(Pre) 2001
The most important strategy for the conservation of
Ans. (c)
biodiversity together with traditional human life is the
India’s human population is growing at an outstanding pace
establishment of biosphere reserves. A biosphere reserve is an
but the bird population is shrinking fast mainly because there
has been a large-scale reduction in the habitats of the birds integral part of the human system. In these areas, multifaceted
and there has been an excessive use of pesticides, chemical development of ecosystem is given prominence.
fertilizers and mosquito repellents. Pesticides like D.D.T. The Indian Government has established 18 Biosphere
enters into the foodchain so that premature shells crack before Reserves in India, which protects larger areas of natural
time & their population decreases. habitats. Twelve of the eighteen biosphere reserves are part
47. Which of the following is not a reason for the loss of of the World Network of Biosphere Reserves, based on the
bio-diversity in Uttarakhand? UNESCO Man and the Biosphere (MAB) programme list.

F–64 General Studies Environment and Ecology


50. Which one of the following is an important strategy 53. The maximum biodiversity is found in –
for the conservation of biodiversity? (a) Tropical rain forest
(a) Biosphere Reserves (b) Temperate forest
(b) Botanical Gardens (c) Coniferous forest
(c) National Parks (d) Arctic forest
(d) Wild Life Sanctuaries U.P.P.C.S. (Pre) 2012
U.P.P.C.S. (Pre) 2013 Ans. (a)
Ans. (a)
The maximum biodiversity is found in the tropical rain forest.
Establishment of the Biosphere Reserve is an important They extend from 23°5'N to 23°5'S. This area has a suitable
strategy for the conservation of Biodiversity. Biosphere condition for the growth and development of flora and fauna
Reserves are areas of terrestrial and coastal ecosystems as it receives heavy rains and high temperature throughout
promoting solutions to reconcile the conservation of
the year. That is why it is also called as optimum Biome.
biodiversity with its sustainable use. Notably, In-Situ and
Ex-Situ techniques are adopted for the conservation of 54. In which of the following ecosystems the species
biodiversity. diversity is relatively higher?
(a) Deep sea
51. Which one of the following is not a site for an in-situ
(b) Tropical rain forest
method of conservation of flora?
(c) Coral reefs
(a) Biosphere Reserve
(d) Desert
(b) Botanical Garden
U.P. P.C.S. (Pre) 2018
(c) National Park
Ans. (b & c)
(d) Wildlife Sanctuary
I.A.S. (Pre) 2011 Tropical rain forest on land and coral reefs in marine systems
Ans. (b) are among the most biologically diverse ecosystems on Earth.
For conservation of flora, Botanical Gardens do not come Hence, Uttar Pradesh Service Commission has considered
under in-situ method while Biosphere Reserves, National both option (b) and option (c) as the correct answer.
Parks and Wild Life Sanctuaries fall under the in-situ method. 55. The maximum biodiversity is found in:
52. Which of the following gas is commonly used in Cryo- (a) Tundra
Bank for ex-situ conservation? (b) Coniferous forests
(a) Nitrogen (b) Oxygen (c) Tropical rain forests
(c) Carbon dioxide (d) Methane (d) Temperate forests
U.P.P.C.S. (Mains) 2009 U.P.P.C.S. (Pre) 2016
Ans. (a) Ans. (c)

Ex-situ is the process of conserving components of biological Bio-diversity refers to a variety of organism found in a
diversity by restricting them to natural habitat and then particular geographical area. Utmost biodiversity is found
managing them in a controlled or modified environment. in tropical rain forests among all ecosystems on the Earth.
There are two methods for Ex- situ conservation- Tropical rain forest extends near the equator.
The primary facilities that hold ex-situ conservation 56. Which of the following has the largest biodiversity?
collections of plants and animals are aquaria, botanical (a) Temperate deciduous forest biome
gardens, and zoos. (b) Tropical rain forest biome
The second method is the storage of seeds, pollen, tissue, (c) Temperate grassland biome
or embryos in liquid nitrogen. This method can be used for (d) Savana biome
U.P.P.C.S. (Mains) 2014
virtually indefinite storage of materials without deterioration
Ans. (b)
over a much greater period about all other methods of ex-situ
conservation. See the explanation of the above question.

Environment and Ecology General Studies F–65


57. Which of the following is considered a "Hotsopt" of Ans. (b)
biodiversity in India?
See the explanation of the above question.
(a) Aravalli Hills (b) Western Ghats
(c) Eastern Ghats (d) Indo-Gangetic Plains 61. Biodiversity changes occur:
U.P.R.O./A.R.O. (Pre) 2021 (a) Increases towards the equator
Ans. (b) (b) Decreases towards the equator
(c) Remains unchanged throughout the Earth
Biodiversity is a contraction of the term "biological diversity"
(d) Increases towards the poles
that refers to variety among and between living organisms.
Uttarakhand P.C.S. (Pre) 2012
Areas that have extremely rich and diverse flora and fauna and
Ans. (a)
are under threat of getting endangered are called biodiversity
'Hotspot'. Officially, four out of the 36 biodiversity Hotspots Biodiversity increases towards the equator. Biodiversity is
not equal on earth. It varies greatly across the globe as well
in the world are present in India : the Himalayas, the Western
as within regions. Among other factors, the diversity of all
Ghats, the Indo-Burma region and the Sundaland. Hence
living things (biota) depends on temperature, precipitation,
option (b) is correct.
altitude, soil, geography and the presence of other species.
58. In which of the following types of forests maximum
62. The richest biodiversity is found in the:
plant diversity is found?
(a) Silent Valley (b) Kashmir Valley
(a) Tropical moist deciduous forests
(c) Valley of Flowers (d) Surma Valley
(b) Sub-tropical mountain forests
Chhattisgarh P.C.S. (Pre) 2011
(c) Temperate moist forests
Uttarakhand P.C.S. (Pre) 2005
(d) Tropical evergreen forests
U.P. Lower Sub. (Spl) (Pre) 2003
U.P. R.O./A.R.O. (Pre) 2017
Ans. (a)
Ans. (d)
The richest biodiversity is found in Silent Valley in Kerala. It
Maximum plant diversity is found in tropical evergreen
is located in Palakkad District of Kerala, India. Silent Valley
forests. The tropical evergreen forests usually occur in
areas receiving more than 200 cm of rainfall and having a is rectangular, twelve kilometres from north to south and
temperature of 15 to 30 - degree Celsius. They occupy about seven km. from east to west. It is located between 11°03' to
seven percent of the earth's land surface and harbors more 11°13' N (latitude) and 76°21' to 76°35' E (longitude). It is
than half of the world's plants and animals. a part of the Western Ghats.

59. The greatest diversity of animal and plant species 63. ‘Silent Valley’ is located in –
occurs in: (a) Uttarakhand
(a) Temperate deciduous forests (b) Kerala
(b) Tropical moist forests (c) Arunachal Pradesh
(c) Heavily polluted rivers (d) Jammu & Kashmir
(d) Deserts and Savanna U.P. Lower Sub. (Pre) 2008
I.A.S. (Pre) 1994 Ans. (b)
Ans. (b) See the explanation of the above question.
See the explanation of the above question. 64. Which one of the following States is related to ‘Silent
60. Which one of the following ecosystems has maximum Valley Project’?
plant biomass? (a) Uttarakhand
(a) Tropical deciduous forest (b) Himachal Pradesh
(b) Tropical rain forest (c) Kerala
(c) Temperate deciduous forest (d) Tamil Nadu
(d) Desert shrubs M.P.P.C.S. (Pre) 2016
U.P.P.C.S. (Pre) 2017 Ans. (c)

F–66 General Studies Environment and Ecology


Long before the internet era, a remarkable people’s movement Keoladeo Ghana 1.10.1981 Rajasthan 2,873
saved a pristine moist evergreen forest in Kerala’s Palakkad NP
district from being destroyed by a hydroelectric project. In Kolleru Lake 19.8.2002 Andhra 90,100
1986, Silent Valley was declared a national park. Pradesh
Nalsarovar Bird 24.9.2012 Gujarat 12,000
65. ‘Valley of Flowers’ is situated in –
Sanctuary
(a) Kerala
Point Calimere 19.8.2002 Tamil Nadu 38,500
(b) Jammu & Kashmir
Loktak Lake 23.3.1990 Manipur 26,600
(c) Uttarakhand
Pong Dam Lake 19.8.2002 Himachal 15,662
(d) Himachal Pradesh
Pradesh
U.P. Lower Sub. (Pre) 2002
Renuka Wetland 8.11.2005 Himachal 20
Ans. (c)
Pradesh
‘Valley of Flowers’ is situated in the Chamoli District of Ropar Lake 22.1.2002 Punjab 1,365
Uttarakhand Rudrasagar Lake 8.11.2005 Tripura 240
66. Which of the following wetlands are designated as Sambhar Lake 23.3.1990 Rajasthan 24,000
Ramsar sites? Sasthamkotta 19.8.2002 Kerala 373
1. Chilka Lake 3. Wuller Lake Lake
2. Keoladeo 4. Loktak Surinsar-Mansar 8.11.2005 Jammu and 350
Select the correct answer form the code given below: Lakes Kashmir
(a) 1 and 2 only (b) 2 and 3 only Tsomoriri Lake 19.8.2002 Jammu and 12,000
Kashmir
(c) 1, 2 and 3 (d) All
Vembanad Kol 19.8.2002 Kerala 151,250
U.P.P.C.S.(Pre) 2013
Wetland
Ans. (d)
Upper Ganga 8.11.2005 Uttar 26,590
The list of Ramsar Sites comprises wetland of international River (Brijghat Pradesh
importance. India currently has 49 sites designated as to Narora
wetlands of international importance (Ramsar Sites). Stretch)
Name of the Site Designation State Area Wular Lake 23.3.1990 Jammu and 18,900
date (hectare) Kashmir
Asthamudi 19.8.2002 Kerala 61,400 Sundarban 30.1.2019 West Bengal 423,000
Wetland Wetland
Bhitarkanika 19.8.2002 Odisha 65,000 Keshopur-Miani 26.9.2019 Punjab 344
Mangroves
Bhoj Wetlands 19.8.2002 Madhya 3,201 Beas Conservation 26.9.2019 Punjab 6,429
Pradesh Reserve
Chandertal 8.11.2005 Himachal 49 Nangal Wildlife 26.9.2019 Punjab 116
Wetland Pradesh Sanctuary
Chilika Lake 1.10.1981 Odisha 116,500 Nandur 21.6.2019 Maharashtra 1,437
Deepor Beel 19.8.2002 Assam 4,000 Madhameshwar
East Calcutta 19.8.2002 West Bengal 12,500 Nawabganj Bird 19.9.2019 Uttar 225
Wetlands Sanctuary Pradesh
Harike Lake 23.3.1990 Punjab 4,100 Parvati Arga Bird 2.12.2019 Uttar 722
Hokera Wetland 8.11.2005 Jammu and 1,375 Sanctuary Pradesh
Kashmir Samaspur Bird 3.10.2019 Uttar 799
Kanjli Lake 22.1.2002 Punjab 183 Sanctuary Pradesh

Environment and Ecology General Studies F–67


Sarsai Nawar 19.9.2019 Uttar 161 (a) Climate change
Jheel Pradesh (b) Pesticide pollution
Sandi Bird 26.9.2019 Uttar 309 (c) Ozone layer depletion
Sanctuary Pradesh (d) Wetland conservation
U.P.R.O./A.R.O. (Re-Exam) (Pre) 2016
Saman Bird 2.12.2019 Uttar 526
Ans. (d)
Sanctuary Pradesh
Sur Sarovar 21.08.2020 Uttar 431 See the explanation of the above question.
Pradesh 71. Wetland day is observed on –
Lonar Lake 22.07.2020 Maharashtra 427 (a) 2nd February (b) 2nd April
Kabartal Wetland 21.07.2020 Bihar 2,620 nd
(c) 2 May (d) 2nd March
Asan Conservation 21.07.2020 Uttarakhand 444 U.P.P.C.S. (Mains) 2008
Reserve Ans. (a)
Tso kar Wetland 17.11.2020 Ladakh 9,577
World Wetlands Day is celebrated every year on 2
Complex
February. This day marks the date of the adoption of
67. Which of the following is a Ramsar site under Ramsar
the Convention on Wetlands on 2 February, 1971, in the
Convention?
Iranian city of Ramsar. Theme of World Wetland Day,
(a) Godavari delta (b) Krishna delta
2021 is - 'Wetlands and Water'.
(c) Sunderbans (d) Bhoj wetland
U.P.P.C.S. (Mains) 2015 72. In India, which one of the following states has the
Ans. (c & d) largest inland saline wetland?
See the explanation of the above question. (a) Gujarat (b) Haryana
(c) Madhya Pradesh (d) Rajasthan
68. The Ramsar Convention was associated with the I.A.S. (Pre) 2009
conservation of – Ans. (d)
(a) Bio-fuels (b) Forests
(c) Wetlands (d) Drylands Sambhar Lake (Rajasthan) is the largest inland Saline wetland
U.P.P.C.S. (Mains) 2008 of India.
Ans. (c)
73. Biosphere reserves are areas for conservation of–
The Ramsar Convention is associated with the conservation (a) Grasslands (b) Agriculture production
and sustainable use of wetlands. It is also known as the (c) Atmospheric balance (d) Genetic Variation
Convention on Wetlands. It was signed on 2 February, 1971 U.P. Lower (Spl.) (Pre) 2004
at Ramsar in Iran. That date is celebrated as World Wetland Ans. (d)
Day now. A biosphere reserve is an ecosystem with plants and animals
69. Bihar's first Ramsar Site is located in: of unusual scientific and natural interest. It helps to protect
(a) Begusarai (b) Banka genetic variations of different organisms and plants.
(c) Bhagalpur (d) Bhojpur 74. Cartagena protocol is related to:
(e) None of the above/More than one of the above (a) Bio-safety agreement (b) Pollution
66th BPSC Re-Exam 2020
(c) Ozone depletion (d) Climate Change
Ans. (a)
Jharkhand P.C.S. (Pre) 2016
Bihar's first Ramsar Site Kabartal Wetland is located in Ans. (a)
Begusarai District of the State. Kabartal Wetlands covers
The Cartagena Protocol is an international agreement on
an area of 26.20 square kilometers and declared as Ramsar bio-safety. It was adopted on 29 January 2000 and
Sites on 21st July, 2020. implemented on 11 September 2003. India confirmed
70. ‘‘Ramsar Convention’’ is related to: bio-safety on Cartagena Protocol on 23 January 2003.

F–68 General Studies Environment and Ecology


75. Which one of the following is the most effective factor U.P.P.C.S. (Mains) 2013
of coral bleaching? Ans. (c)
(a) Marine pollution The word biodiversity is a contraction of the phrase
(b) Increase of salinity of seas
“biological diversity” and was first coined in 1985 by Walter
(c) Rise in normal temperature of sea-water
G. Rosen of the National Research Council as a title word in
(d) the outbreak of diseases and epidemics
a seminar, organized to discuss biological diversity.
U.P.P.C.S.(Pre) 2012
Ans. (c) 2. Who coined the term 'Bio-diversity'?
(a) B.P. Singh (b) Karl Mobius
Warmer water temperatures can result in coral bleaching.
(c) Sir A.G. Tansley (d) Walter G. Rosen
When water is too hot, corals will expel the algae
Chhattisgarh P.C.S. (Pre) 2019
(zooxanthellae) living in their tissues causing the coral to
Ans. (d)
turn completely white. This is called coral bleaching.
76. Which of the following is the most significant factor in See the explanation of the above question.
coral bleaching? 3. Biodiversity forms the basis for human existence in
(a) Mining of coral rocks the following ways:
(b) Outbreak of coral diseases 1. Soil formation
(c) Siltation of seawater 2. Prevention of soil erosion
(d) Global warming 3. Recycling of waste
U.P.P.C.S. (Pre) (Re-Exam) 2015 4. Pollination of crops
Ans. (d) Select the correct answer using the code given below:
See the explanation of the above question. Code :
(a) 1, 2 and 3 (b) 2, 3 and 4
77. Which of the following have coral reefs?
(c) 1 and 4 (d) 1, 2, 3 and 4
1. Andaman and Nicobar Islands
I.A.S. (Pre) 2011
2. Gulf of Kachchh
Ans. (d)
3. Gulf of Mannar
4. Sunderbans Biodiversity forms the basis for human existence in the form
Select the correct answer using the code given below: of soil formation, prevention of soil erosion, recycling of
(a) 1, 2 and 3 only (b) 2 and 4 only waste and pollination of crops. Other important uses of
(c) 1 and 3 only (d) 1, 2, 3 and 4 biodiversity are listed for medicinal purpose also.
I.A.S. (Pre) 2014 4. The decade 2011-20 has been designated by the U.N.O.
Ans. (a) as the decade of –
The total coral reef area in India is 5,790 km2, distributed (a) Natural disaster decade
between 4 major regions: Lakshadweep, Gulf of Mannar, (b) Bio-diversity decade
Gulf of Kutch, and Andaman and Nicobar Islands. Fringing (c) Climate-change decade
(d) Environment decade
reefs are common. They occur around the Gulf of Mannar,
U.P.P.C.S. (Mains) 2012
Gulf of Kutch, Andaman and the Nicobar Islands. Atoll
Ans. (b)
reefs are found in Lakshadweep. Sundarban is known for
mangrove forest. The United Nations General Assembly declared 2011–20
the United Nations Decade on Biodiversity (Resolution
High-Level Questions 65/161) on 22, December 2010.

1. Who among the following had used the term 5. United Nations Organization has declared decade
‘Biodiversity’ for the first time? 2011-2020 as:
(a) C.J. Barrow (b) De Carter (a) Decade on Biodiversity
(c) Walter G. Rosen (d) D.R. Batish (b) Decade on Pollution

Environment and Ecology General Studies F–69


(c) Decade on Space Technology U.P. U.D.A./L.D.A. (Mains) 2010
(d) Decade on Animal Health Ans. (b)
Chhattisgarh P.C.S. (Pre) 2019
Biodiversity Act was passed by the Indian Parliament on 11,
Ans. (a)
December, 2002.
United Nations Organization has declared the period 2011-
9. In which year 'Biodiversity Act' was passed in India?
2020 as 'Decade on Biodiversity'. The main goal is to
(a) 2000 (b) 2002
mainstream biodiversity at different levels. Throughout the
(c) 2010 (d) 2017
decade, governments are encouraged to develop, implement
Chhattisgarh P.C.S. (Pre) 2018
and communicate the results of national strategies for
Ans. (b)
implementation of the strategic Plan for Biodiversity.
The 'Bio-Diversity Act' was passed in the year 2002. It
6. Which of the following is not responsible for the provides for the preservation of Biological Diversity in
increase in the biodiversity of an ecosystem? India and mechanism for equitable sharing of benefits
(a) Productivity of the ecosystem arising out of the use of traditional biological resources
(b) Intermediate-disturbance and knowledge.
(c) Age of the ecosystem
10. National Biodiversity Authority of India was
(d) Less number of trophic levels
established in–
U.P.P.C.S. (Pre) (Re Exam ) 2015
(a) 2003, in Chennai
Ans. (d)
(b) 2003, in Bengaluru
The ecosystem is a dynamic entity. The interdependence (c) 2003, in Hyderabad
of organisms on other organisms for food contribute to (d) 2003, in Kerala
the creation of different trophic levels. The decrease in U.P.R.O./A.R.O. (Mains) 2014
trophic level indicates an abundance of a particular type Ans. (a)
of organism, thus resulting in a decrease in bio-diversity. The National Biodiversity Authority (NBA) was established
Rest other factors like the productivity of the ecosystem, in 2003 to implement India’s Bio-Diversity Act (2002).
intermediate-disturbance and age of the ecosystem contribute The headquarter of NBA is in Chennai, Tamil Nadu. It is a
to the richness of bio-diversity of the ecosystem. statutory autonomous body under Ministry of Environment.
7. The Himalayan Range is very rich in species diversity. 11. How does the National Biodiversity Authority (NBA)
Which one among the following is the most appropriate help in protecting Indian agriculture?
reason for this phenomenon? 1. NBA checks the biopiracy and protects indigenous
(a) It has a high rainfall that supports luxuriant vegetative and traditional genetic resources.
growth 2. NBA directly monitors and supervises the scientific
(b) It is a confluence of different biogeographical zones research on genetic modification of crop plants.
(c) Exotic and invasive species have not been introduced 3. Application for Intellectual Property Rights related
in this region to genetic/biological resources cannot be made
(d) It has less human interference without the approval of the NBA.
I.A.S. (Pre) 2011 Which of the statement(s) given above is/are correct?
Ans. (b) (a) Only 1 (b) 2 and 3
The Himalayan Range is very rich in species diversity. The (c) 1 and 3 (d) 1, 2 and 3
most appropriate reason for this phenomenon is a confluence I.A.S. (Pre) 2012
of different bio-geographical zones in this range. Ans. (c)
The National Biodiversity Authority (NBA) with its
8. The Biodiversity Act was passed by the Indian
Parliament in – headquarters in Chennai, was established in 2003 to
(a) May, 2000 (b) December, 2002 implement India’s Biological Diversity Act (2002). The Act
(c) January, 2004 (d) October, 2008 covers conservation, use of biological resources and

F–70 General Studies Environment and Ecology


associated knowledge. It provides a framework for access to 14. Given below are two statements, one labelled as
biological resources and sharing the benefits arising out of Assertion (A) and other as the Reason (R).
such access and use. The Act also includes in its ambit the Assertion (A) : The biosphere is an example of an
transfer of research results and application for Intellectual open system.
Reason (R) : There cannot be reinvestment of new
Property Right (IPR) relating to Indian biological resources.
substances in the biosphere but there
The Genetic Engineering Appraisal committee (GEAC)
is no restriction on the investment and
functioning under Ministry of Environment, Forest and
exit of energy.
Climate Change is responsible for monitoring and supervising
Select the correct answer using the codes below:
scientific research on genetic modification of crop plants and
(a) Both (A) and (R) are true and (R) is correct explanation
other organisms.
of (A)
12. Sea buckthorn has a huge global market potential. The (b) Both (A) and (R) are true but (R) is not the correct
berries of this plant are rich in vitamins and nutrients. explanation of (A)
Genghis Khan used them to improve the stamina of (c) (A) is true but (R) is false
his troops. Russian cosmonauts used its oil to combat (d) (A) is false but (R) is true
radiation. Where is this plant found in India? U.P. R.O./A.R.O. (Mains) 2017
(a) Nagaland (b) Ladakh Ans. (c)
(c) Kodaikanal (d) Pondicherry
U.P.P.C.S. (Mains) 2004 The biosphere is an example of an open system. The input
Ans. (b) and output of substances take place in their system. There
can be reinvestment of new substances in the biosphere.
Sea buckthorn is a medicinal plant found in the Himalayan
region. Our country holds tremendous potential in respect 15. Recently, which of the following drug has been reported
of Sea buckthorn fruit production and diverse varieties as responsible for vulture death?
which have health-promoting properties and can play a (a) Aspirin (b) Chloroquine
crucial role in preventing soil erosion and help nitrogen (c) Diclofenac Sodium (d) Penicillin
fixation in cold and desert areas. Sea buckthorn fruits U.P.P.C.S. (Spl.) (Mains) 2008
grow in the cold deserts of Ladakh, Lahaul-Spiti in Ans. (c)
Himachal Pradesh and some parts of Arunachal Pradesh. In May, 2003, a scientist working with the Peregrine Fund
13. The Government of India encourages the cultivation of presented a research paper that confirmed that the vulture
‘sea buckthorn’. What is the importance of this plant? deaths were due to the anti-inflammatory painkilling drug
1. It helps in controlling soil erosion and preventing Diclofenac Sodium. A vulture that had got had high levels
desertification. of Diclofenac in their kidneys. In January, 2004, the results
2. It is a rich source of biodiesel. of a joint study conducted by the Peregrine Fund and the
3. It has nutritional value and is well-adapted to grow Ornithological Society of Pakistan confirmed that Diclofenac
in cold areas of high altitudes.
Sodium was indeed the primary reason for vulture deaths.
4. Its timber is of great commercial value.
That was an important study that resulted in a ban on
Which of the statements given above is/are correct?
the manufacturing of veterinary Diclofenac by the Drug
(c) Only 1 (d) 2, 3, and 4
Controller General of India. The devastating effect of this
(c) 1 and 3 (d) 1, 2, 3 and 4
I.A.S. (Pre) 2012 medicine can be understood from the fact that it caused the
Ans. (c) death of 8.5 crore vultures in India, Nepal and Pakistan in 10
years. This ban was soon introduced in Nepal and Pakistan.
Sea buckthorn plant grows in the atmosphere of cold dry
16. Which pain killer given to cattle is responsible for the
climate like parts of Ladakh. The bushy character of the
plant with a high density of branches helps in checking soil near extinction of vultures in India?
erosion and helps in soil stabilization. The fruit of this plant (a) Ibuprofen (b) Acetaminophen
is a source of nutritious juice. (c) Aspirin (d) Diclofenac

Environment and Ecology General Studies F–71


U.P. P.C.S. (Pre) 2018 of the extinction of a fruit-eating bird.
Ans. (d) Which one of the following was that bird ?
(a) Dove (b) Dodo
See the explanation of the above question.
(c) Condor (d) Skua
17. The rapid rate of decline in the vulture population in I.A.S. (Pre) 1998
India is mainly due to: Ans. (b)
(a) Overuse of Diclofenac drug
Dodo was a giant bird with short stumpy legs and a large
(b) Low death rate of animals
hooked beak. It used to live on the Indian Ocean islands of
(c) High death rate of animals
Mauritius but had completely vanished by the end of the 17th
(d) Cleanliness
century. Dodo was helpful in pollinating and propagating
Chhattisgarh P.C.S. (Pre) 2019 seed of the species called Tambalacoque.
Ans. (a)
21. Consider the following pairs:
See the explanation of the above question. Wildlife Naturally found in
18. The most important factor contributing to the loss of 1. Blue-finned Mahseer : Cauvery River
2. Irrawaddy Dolphin : Chambal River
the vulture population in India is:
3. Rusty-spotted Cat : Eastern Ghats
(a) Viral infection
Which of the pairs given above are correctly matched?
(b) Bacterial infection
(a) 1 and 2 only (b) 2 and 3 only
(c) Administrating of painkillers to cattle
(c) 1 and 3 only (d) 1, 2 and 3
(d) Administrating of estrogen injection to cattle
I.A.S. (Pre) 2019
U.P.P.C.S. (Pre) (Re Exam) 2015 Ans. (c)
Ans. (c)
Blue-finned Mahseer is found naturally in the Cauvery River.
Administrating of painkillers to cattle is the main cause of Irrawaddy Dolphine is found near sea coasts in estuaries
loss of vulture population in India. Diclofenac is the drug and rivers in parts of Bay of Bengal and South East Asia. In
that acted as a major cause of death of vultures. Diclofenac Chilika Lake also Irrawaddy Dolphine is found. In Chambal
is an anti-inflammatory, painkiller drug that was used widely River Gangetic Dolphine is found. Rusty Spotted Cat is found
in Eastern Ghats. Hence option (c) is the correct answer.
in India in the 90s. It proved fatal to vultures.
22. Due to some reasons, if there is a huge fall in the
19. Vultures which used to be very common in the Indian
population of species of butterflies, what could be its
countryside some years ago are rarely seen now a days.
likely consequence/consequences?
This is attributed to:
1. Pollination of some plants could be adversely
(a) The destruction of their nesting sites by new invasive
affected.
species 2. There could be a drastic increase in the fungal
(b) A drug used by cattle owners for treating their diseased infections of some cultivated plants.
cattle 3. It could lead to a fall in the population of some
(c) Scarcity of food available to them species of wasps, spiders and birds.
(d) A widespread, persistent and fatal disease among them Select the correct answer using the code given below:
I.A.S. (Pre) 2012 (a) 1 only (b) 2 and 3 only
Ans. (b) (c) 1 and 3 only (d) 1, 2 and 3
I.A.S. (Pre) 2017
The reason behind the heavy decline in the population of
Ans. (c)
vultures is Diclofenac taken by vultures from the carcasses of
dead cattle lying in open fields. Cattle owners used this drug Butterflies play a vital role in the pollination of many
for treating their diseased animals. Diclofenac is highly toxic flowering plants, so their declining number may have an
adverse effect on the pollination of plants. Butterflies act as
and has killed vultures in very large numbers in this way. It
the lower member in the food chain. They are the meal of
has drastically brought down their population. spider, bird, snake etc. Therefore, the drop in a number of
20. A tree species in Mauritius failed to reproduce because butterflies can adversely affect the food chain.

F–72 General Studies Environment and Ecology


23. In the context of Indian wildlife, the flying fox is a: Which three of the above are correct criteria in this
(a) Bat (b) Kite context?
(c) Stork (d) Vulture (a) 1, 2 and 6 (b) 2, 4 and 6
I.A.S. (Pre) 2009 (c) 1, 3 and 5 (d) 3, 4 and 6
Ans. (a) I.A.S. (Pre) 2011
Ans. (c)
The Indian flying fox (Pteropus Gigantes), also known
as the greater Indian fruit bat, is a species of bats in The criteria for the recognition of regions as hotspot are
the family Pteropodidae. It is found in Bangladesh, species richness, endemism and threat perception that led to
China, India, the Maldives, Nepal, Pakistan, and Sri Lanka. the recognition of Western Ghats, Sri Lanka and Indo-Burma
24. The marine animal called dugong which is vulnerable regions as hotspots of biodiversity.
to extinction is a/an: 27. Given below are two statements, one labelled as
(a) Amphibian (b) Bony fish Assertion (A) and other as the Reason (R).
(c) Shark (d) Mammal Assertion (A) : Maximum number of the species
I.A.S. (Pre) 2009 of butterflies are found in tropical
Ans. (d) countries.
Reason (R) : Butterflies cannot tolerate low
Dugong is a medium-sized marine mammal, which is
temperatures.
vulnerable to extinction.
Codes :
25. Concerning ‘dugong’, a mammal found in India, which (a) Both (A) and (R) are true and (R) is correct explanation
of the following statements is/are correct? of (A)
1. It is a herbivorous marine animal. (b) Both (A) and (R) are true but (R) is not the correct
2. It is found along the entire coast of India. explanation of (A)
3. It is given legal protection under Schedule I of the (c) (A) is true but (R) is false
Wildlife (Protection) Act, 1972. (d) (A) is false but (R) is true
Select the correct answer using the code given below: U.P.P.C.S. (Pre) 2019
(a) 1 and 2 (b) 2 only Ans. (a)
(c) 1 and 3 (d) 3 only It is generally accepted that climate factors regulate most
I.A.S. (Pre) 2015
insect species' life cycle, including butterflies. It is well
Ans. (c)
established that temperature is a key factor for increasing
The dugong known as ‘Sea Cow’ is mostly found in Eastern daily activities (flight, foraging, movements) of butterflies.
Africa, South Asia and Australia. In India, it is found in the Butterflies cannot tolerate low temperatures. So maximum
Gulf of Mannar and Palk Bay. Feeding on seagrass dugongs number of the species of butterflies are found in tropical
are found in seagrass beds, sheltered waters, lagoons and countries.
Bays. It has been given legal protection under Schedule I of
28. Concerning an organization known as ‘Bird Life
the Wildlife (Protection) Act, 1972.
International’ which of the following statements is/ are
26. Three of the following criteria have contributed to the correct?
recognition of Western Ghats, Sri Lanka and Indo- 1. It is a Global Partnership of Conservation
Burma regions as hotspots of biodiversity: Organizations.
1. Species richness 2. The concept of ‘biodiversity hotspots’ originated
2. Vegetation density from this organization.
3. Endemism 3. It identifies the sites referred to as ‘Important Bird
4. Ethno-botanical importance and Biodiversity Areas’.
5. Threat perception Select the correct answer using the code given below:
6. Adaptation of flora and fauna to warm and humid (a) 1 only (b) 2 and 3 only
conditions (c) 1 and 3 only (d) 1, 2 and 3

Environment and Ecology General Studies F–73


I.A.S. (Pre) 2015 The Cartagena Protocol on Biosafety or the Convention on
Ans. (c) Biological Diversity is an international agreement which
Bird Life International is a global partnership of non- aims to ensure the safe handling, transport and use of
governmental conservation organizations that strives to Living Modified Organism (LMO) resulting from modern
conserve birds, their habitats and global biodiversity. It is biotechnology that may have adverse effects on biological
diversity, also taking into account risks to human health. It
the world’s largest partnership of conservation organizations,
was adopted on 29 January, 2000 and entered into force on
with 115 partner organizations. This organization identifies
11 September, 2003. India signed the Biosafety Protocol on
important bird and bio-diversity areas. It has so far identified
23rd January, 2001. In India, the Ministry of Environment,
more than 13000 such areas. Concept of biodiversity hot
Forest and Climate Change implements this protocol.
spots was propounded by British environmentalist Norman
31. A sandy and saline area is the natural habitat of an
Myres. Hence, statement 2 is incorrect while statements 1
Indian animal species. The animal has no predators
and 3 are correct. in that area but its existence is threatened due to the
29. Which one of the following statements is correct? destruction of its habitat. Which one of the following
(a) The First Meeting of the Parties (MOP 1) to the could be that animal?
Cartagena Protocol on Biosafety was held in the (a) Indian wild buffalo
Philippines in the year 2004. (b) Indian wild ass
(b) India is not a signatory to the Biosafety Protocol/ (c) Indian wild boar
Convention on Biological Diversity. (d) Indian gazelle
(c) The Biosafety Protocol deals with genetically modified I.A.S. (Pre) 2011
organisms. Ans. (b)
(d) The United States of America is a member of the Rann of Kachchh is the natural habitat of Indian Wild Ass.
Biosafety Protocol/ Convention on Biological This animal has no predators in that area but its existence is
Diversity threatened due to the destruction of its habitat.
I.A.S. (Pre) 2005
32. The 10th Conference of the Parties to the United Nations
Ans. (c)
Conference on ‘Biological Diversity’ was held at:
The First Meeting of the Parties (MOP 1) to the Cartagena (a) Beijing (b) Copenhagen
Protocol on Biosafety was held on 23-27 February, 2004 in (c) Nagoya (d) New York
Kuala Lumpur, Malaysia. Hence, statement (a) is incorrect. U.P. U.D.A./L.D.A. (Mains) 2010
India on 23 January, 2001, signed Cartagena Protocol on Ans. (c)
Biosafety or the Convention on Biological Diversity. Hence The tenth meeting of the Conference of the Parties (COP
statement (b) is also incorrect. It is designed to protect both 10) was held in Nagoya, Japan, from 18 to 29 October, 2010.
biological diversity and human life from any adverse effects Nagoya Biodiversity Summit concluded with the adoption
of organisms modified by technology. Hence, statement (c) of historical decisions that would permit the community
is correct. The United States of America is not a member of of nations to meet the unprecedented challenges of the
the Biosafety Protocol/Convention on Biological Diversity. continued loss of biodiversity compounded by climate
Hence statement (d) is also incorrect. change. The eleventh meeting of the Conference of the
30. Which one of the following Union Ministries Parties (COP-11) was held in Hyderabad, India, from 8 to 19
implements the Cartagena Protocol on Biosafety? October, 2012. The Twelfth meeting of the Conference of the
(a) Ministry of Science and Technology Parties (COP-12) was held in Pyeongchang, the Republic of
(b) Ministry of Health and Family Welfare Korea from 6 - 17 October, 2014. Fifteenth meeting of the
(c) Ministry of Environment and Forests Conference of the Parties (COP-15) was held in Kunming,
(d) Ministry of Chemical and Fertilizers China from 11-15 October 2021 and 2nd phase will be held
I.A.S. (Pre) 2009 from 25 April - 8 May 2022.
Ans. (c) 33. Which of the following statements is/are correct?

F–74 General Studies Environment and Ecology


Proper design and effective implementation of UN- in a habitat at a local scale?
REDD+ Programme can significantly contribute to: (a) Alpha index
1. Protection of biodiversity (b) Beta index
2. Resilience of forest ecosystems (c) Gamma index
3. Poverty reduction (d) None of these
Select the correct answer using the code given below: R.A.S./R.T.S. (Pre) 2018
(a) 1 and 2 only (b) 3 only Ans. (a)
(c) 2 and 3 only (d) 1, 2 and 3
Species Richness - It is the measure of species found in a
I.A.S. (Pre) 2016
community.
Ans. (d)
Alpha Diversity - In ecology, Alpha Diversity (D-diversity)
UN-REDD (Reducing Emission from Deforestation and is the mean species diversity in sites or habitats at a local
Forest Degradation) is an inter-institutional progamme scale.
under the aegis of UNFCCC member countries. This Beta Diversity - It is the comparison of diversity between
programme was started in 2008. Institutions like UNDP, ecosystems, usually measured as the change in the amount
FAO and UNEP are associated with this programme. The of species between the ecosystems.
objective of this programme is to mitigate climate change Gamma Diversity - It is a measure of the overall diversity
by reducing net emission of greenhouse gases through for the different ecosystems within a region.
enhanced forest management in developing countries.
UN-REDD + Programme focuses on conservation and 36. Concerning the International Union for Conservation
sustainable management of forests. It plays a vital role in of Nature and Natural Resources (IUCN) and the
mitigating adverse effects of climate change. Proper design Convention on International Trade in Endangered
and effective implementation of UN-REDD + programme Species of Wild Fauna and Flora (CITES), which of
can significantly contribute to the protection of biodiversity the following statements is/are correct?
and maintaining the resilience of forest ecosystem. It also 1. IUCN is an organ of the United Nations and
provides technical and financial support to developing CITES is an international agreement between
countries thus contributing to poverty alleviation. Governments.
34. Two important rivers–one with its source in Jharkhand 2. IUCN runs thousands of field projects around the
(and known by a different name in Odisha) and another world to better manage the natural environment.
with its source in Odisha–merge at a place only a short 3. CITES is legally binding on the States that have
distance from the coast of Bay of Bengal before flowing joined it, but this Convention does not take the
into the sea. This is an important site of wildlife and place of national laws.
biodiversity and a protected area. Which one of the Select the correct answer using the codes given below:
following could be this? (a) 1 only (b) 2 and 3 only
(a) Bhitarkanika (b) Chandipur-on-sea (c) 1 and 3 only (d) 1, 2 and 3
(c) Gopalpur-on-sea (d) Simlipal I.A.S. (Pre) 2015
I.A.S. (Pre) 2011 Ans. (b)
Ans. (a) Created in 1948, the International Union for Conservation
Brahmani and Vaitarni rivers merge at Bhitarkanika before of Nature (IUCN) is a membership union which provides
emptying into the sea. Bhitarkanika is a protected area for public, private and non-governmental organizations
wildlife and biodiversity. Brahmani river is known as South with the knowledge and tools to enable human progress,
Koel in Jharkhand. The Brahmani river is formed by the economic development and nature's conservation to take
confluence of South Koel and Sankh rivers near Rourkela. place together. It is not a part of the United Nations.
Vaitarni river originates from Guptaganga hills in Keonjhar IUCN’s expertise and extensive network provide a
district of Orissa. solid foundation for a large and diverse portfolio of
35. Among the mathematical indices for measuring bio- conservation projects around the world. Hence, statement
diversity, which one shows the mean species diversity (1) is incorrect while statement (2) and (3) are correct.

Environment and Ecology General Studies F–75


37. With reference to an initiative called ‘The Economics Cheetah was declared extinct in India in 1952. Recently, the
of Ecosystems and Biodiversity (TEEB)’, which of the Supreme Court of India withheld the government's plan to
following statements is/are correct? rehabilitate Cheetah in India. Significantly the government
1. It is an initiative hosted by UNEP, IMF and World had planned to introduce Cheetah in a wildlife sanctuary in
Economic Forum. Madhya Pradesh from Namibia. Snow Leopard's habitat is
2. It is a global initiative that focuses on drawing found 3000-4500 meters above the sea level. Surveys held
attention to the economic benefits of biodiversity. in Uttarakhand and Himachal Pradesh have confirmed the
3. It presents an approach that can help decision- presence of Snow Leopards in India. Black-necked Crane is
makers recognize, demonstrate and capture the the state bird of Jammu and Kashmir. Flying Squirrels are
value of ecosystems and biodiversity. found in India, Russia, Japan, China, Europe, Africa and
Select the correct answer using the codes given below: North America. India is habitat to seven species of flying
(a) 1 and 2 only (b) 3 only Squirrel, six of which are found in the Himalayan region.
(c) 2 and 3 only (d) 1, 2 and 3 40. With reference to India’s biodiversity, Ceylon
I.A.S. (Pre) 2016 frogmouth, Coppersmith barbet, Grey-chinned
Ans. (c) minivet and White-throated redstart are:
The Economics of Ecosystems and Biodiversity (TEEB) is (a) Birds (b) Primates
a global initiative focused on “making values visible”. Its (c) Reptiles (d) Amphibians
principal objective is to mainstream the values of biodiversity I.A.S. (Pre.) 2020
and ecosystem services into decision-making at all levels. It Ans. (a)
aims to achieve this goal by following a structured approach With reference to India’s biodiversity, Ceylon frogmouth
to valuation that helps decision makers recognize the wide (Batrachostomus moniliger), Coppersmith barbet (Megalaima
range of benefits provided by ecosystems and biodiversity, haemacephala), Grey-chinned minivet (Pericrocotus solaris)
demonstrates their values in economic terms and where and White-throated redstart (Phoenicurus schisticeps) are the
appropriate, captures these values in decisions making. It was species of birds.
launched by Germany and European Union in March 2007
41. Consider the following:
and the study was led by Pavan Sukhdev from 2007 to 2011.
1. Bats 2. Bears 3. Rodents
38. In which of the following States is lion-tailed macaque The phenomenon of hibernation can be observed in
found in its natural habitat? which of the above kinds of animals?
1. Tamil Nadu 2. Kerala (a) 1 and 2 only
3. Karnataka 4. Andhra Pradesh (b) 2 only
Select the correct answer using the codes given below: (c) 1, 2 and 3
(a) 1, 2 and 3 (b) Only 2 (d) Hibernation cannot be observed in any of the above
(c) 1, 3 and 4 (d) All of these I.A.S. (Pre) 2014
I.A.S. (Pre) 2013 Ans. (c)
Ans. (a)
Hibernation or winter sleep refers to a state of inactivity
Lion- tailed macaque is found in Western Ghats of India. It among animals spending winter in a dormant state. It is
is an endangered species which is mostly found in Kerala, a survival strategy designed to conserve energy when
Karnataka and the border region of Tamil Nadu.
weather conditions are not favourable. This condition
39. Consider the following: is observed among both types of animals, cold blooded
1. Black-necked Crane 2. Cheetah and warm blooded. Squirrels, bears and some bats resort
3. Flying Squirrel 4. Snow Leopard to hibernation in winter when they do not find means to
Which of the above are naturally found in India? feed themselves. In this state, animal's metabolism slows,
(a) 1, 2 and 3 (b) 1, 3 and 4 it's temperature plunges and breathing slows. To face this
(c) 2 and 4 (d) 1, 2, 3 and 4 situation they eat heavily in summer and autumn, storing
I.A.S. (Pre) 2012 fat to survive in winter. Mammals like squirrels, muskrats,
Ans. (b) mice and other rodents fall under this category of animals.

F–76 General Studies Environment and Ecology


42. Consider the following statements: The Red Panda is a territorial mammal. Its zoological name
1. Biodiversity is normally greater in the lower is Ailurus fulgens. Red Panda lives in temperate climates, in
latitudes as compared to the higher latitudes. deciduous and coniferous forests, usually with an understory
2. Along the mountain gradients, biodiversity of bamboo and hollow trees. This makes them a key species
is normally greater in the lower altitudes as of these forests and indicators of forest health. They are found
compared to the higher altitudes. in the Himalayan region, in parts of Nepal, Bhutan, Myanmar
Which of the statements given above is/are correct?
and in the Indian States of Sikkim, West Bengal, Meghalaya
(a) 1 only (b) 2 only
and Arunachal Pradesh. Most of the red pandas of the world are
(c) Both 1 and 2 (d) Neither 1 nor 2
found in China whereas the majority of the Indian red panda
I.A.S. (Pre) 2011
is found in Arunachal Pradesh. Thus, statement 1 is incorrect.
Ans. (c)
Slow Loris species are found in Bangladesh, Cambodia,
Biodiversity is normally greater in the lower latitudes as China, north-eastern India (Arunachal Pradesh, Assam,
compared to the higher altitudes. Tropical rain forests which Manipur, Meghalaya, Mizoram, Nagaland, and Tripura),
have the largest biodiversity are found in lower latitudes. Myanmar (including the Mergui Archipelago), Thailand,
Similarly, Valleys found in lower altitudes harness greater and Vietnam (except the south). Thus, statement 2 is correct.
biodiversity than mountains of high altitude. 45. Concerning a conservation organization called
43. Consider the following statements: 'Wetlands International’, which of the following
1. Salt-water crocodile is found in the Andaman and statements is/are correct?
Nicobar Islands. 1. It is an inter-governmental organization formed
2. Shrew and tapir are found in the Western Ghats by the countries which are signatories to Ramsar
of the Malabar region. Convention.
Codes : 2. It works at the ground level to develop and mobilize
(a) 1 only (b) 2 only knowledge, and use the practical experience to
(c) Both 1 and 2 (d) Neither 1 nor 2 advocate for better policies.
I.A.S. (Pre) 2008 Select the correct answer using the codes given below:
Ans. (a) (a) 1 only (b) 2 only
(c) Both 1 and 2 (d) Neither 1 nor 2
The Andaman and Nicobar Islands are very rich in
I.A.S. (Pre) 2014
biodiversity harbouring unique endemic life forms. The
Ans. (b)
marine biodiversity includes marine mammals such as
whales, dolphins, dugong, marine turtles, estuarine or Wetlands International is a global organization that works
saltwater crocodile, fish, prawns and lobsters, corals, to sustain and restore wetlands and their resources for
seashells including rare and endangered Trochus species. people and biodiversity. It is an independent, not-for-profit,
global organization supported by Government and NGO
Shrew and tapir are found in large Himalayan Ranges, not
membership from around the world. Its head office is in
in the Western Ghats of the Malabar region.
the Netherlands. Wetlands International works in many
44. Consider the following statements: thematic areas throughout the world. It is the leading
1. In India, Red Panda is naturally found in the scientific expert on peatlands and climate change as well
western Himalayas only. as wetlands and waterbird migration based on extensive
2. In India, Slow Loris lives in the dense forests of the research as well as field projects. Wetlands International’s
North East. work ranges from research and community-based field
Which of the statements given above is/are correct? projects to advocacy and engagement with Governments,
(a) 1 only (b) 2 only corporate and international policy fora and conventions.
(c) Both 1 and 2 (d) Neither 1 nor 2 Wetlands International works through partnerships and is
I.A.S. (Pre) 2007 supported by contributions from an extensive specialist
Ans. (b) expert network and tens of thousands of volunteers.

Environment and Ecology General Studies F–77


46. India is a party to the Ramsar Convention and has (b) For releasing heavy metals through absorption by
declared many areas as Ramsar Sites. Which of the plants.
following statements best describes as to how we should (c) In reducing siltation of rivers by retaining sediments
maintain these sites in the context of this Convention? (d) All the above
(a) Keep all the sites completely inaccessible to man so U.P.P.C.S. (Pre) 2012
that they can not be exploited. Ans. (d)
(b) Conserve all the sites through ecosystem approach
Wetlands prevent flooding by holding water much like a
and permit tourism and recreation only.
sponge. By doing so, wetlands help keep river levels normal
(c) Conserve all the sites through ecosystem approach
and filter and purify the surface water. Wetlands accept water
for a specific period without any exploitation, with
during a flood and whenever water levels are high. When
specific criteria and specific period for each site,
and then allow sustainable use of them by future water levels are low, wetlands slowly release water. Wetlands
generations. also release vegetative matters into rivers, which help feed
(d) Conserve all the sites through ecosystem approach fish in the rivers. Wetlands help to counterbalance the human
and allow their simultaneous sustainable use. effect on rivers by rejuvenating them and surrounding
I.A.S. (Pre) 2010 ecosystems.
Ans. (d) 49. Concerning the wetlands of India, consider the
India is a party to Ramsar Convention and has declared many following statements
areas as Ramsar Sites to conserve all the site through an 1. The country’s total geographical area under the
category of wetlands is recorded more in Gujarat
ecosystem approach and allow their simultaneous sustainable
as compared to other States.
use.
2. In India, the total geographical area of coastal
47. If a wetland of international importance is brought wetlands larger than that of the total is inland area
under the ‘Montreux Record’, what does it imply? of wetlands
(a) Changes in ecological character have occurred, are Which of the statements given above is/are correct?
occurring or are likely to occur in the wetland as a (a) Only 1
result of human interference. (b) Only 2
(b) The country in which the wetland is located should (c) 1 and 2
enact a law to prohibit any human activity within five (d) Neither 1 nor 2
kilometres from the edge of the wetland. I.A.S. (Pre) 2012
(c) The survival of the wetland depends on the cultural Ans. (a)
practices and traditions of certain communities living
The share of the inland wetland of the country’s total
in its vicinity and therefore the cultural diversity
geographical area under the category of the wetland is 69.23
therein should not be destroyed.
% and 27.13% area is under Coastal wetland category while
(d) It is given the status of ‘World Heritage Site’.
I.A.S. (Pre) 2014 3.64% is under low wetland (<2.25 ha) category. Gujarat
Ans. (a) (3474950 ha), has the greatest share of wetlands. Andhra
Pradesh is at second place. Hence, statement 1 is correct
The Montreux Record is a register of wetland sites on the
while statement 2 is incorrect.
list of Wetlands of International Importance where changes
50. Consider the following statements:
in ecological character have occurred, are occurring, or are
1. Under Ramsar Convention, it is mandatory on the
likely to occur as a result of technological developments,
part of the Government of India to protect and
pollution or other human interference. It is maintained as
conserve all the wetlands in the territory of India.
part of the Ramsar list. 2. The Wetlands (Conservation and Management)
48. As an ecosystem, wetlands are useful for which of the Rules, 2010 were framed by the Government of
following? India based on the recommendations of Ramsar
(a) For nutrient-recovery and cycling Convention.

F–78 General Studies Environment and Ecology


3. The Wetlands (Conservation and Management) Deciduous forest - 1200 gram (dry weight)/square
Rules, 2010 also encompass the drainage area or meter/every year
catchment regions of the wetlands as determined Taiga - 800 gram (dry weight)/square meter/
by the authority. Which of the statements given
every year
above is/are correct?
Prairie - 600 gram (dry weight)/square meter/
(a) 1 and 2 only
every year
(b) 2 and 3 only
(c) 3 only Deep Sea - Negligible gram (dry weight)/square
(d) 1, 2 and 3 meter/ every year
I.A.S. (Pre) 2019 52. The author of ‘Tomorrow’s Biodiversity’ is –
Ans. (c) (a) Vandana Shiva
Under Ramsar Convention it is not mandatory on the part (b) V.K. Sharma
of the Government of India to protect and conserve all the (c) M.S. Swaminathan
wetlands in the territory of India. (d) Medha Patekar
Hence statement 1 is not correct. U.P.P.C.S. (Mains) 2004
The Wetland (Conservation and Management) Rules, 2010
U.P.P.C.S. (Mains) 2005
were framed by the Government of India is not based on the
Ans. (a)
recommendation of Ramsar Convention. It was framed by
the recommendation of a Expert Panel constituted according The author of “Tomorrow Biodiversity” is Vandana Shiva.
to provisions of National Environment Policy 2006. Hence
53. Consider the following international agreements:
statement 2 is incorrect.
1. The International Treaty on Plant Genetic
Moreover the Wetlands (Conservation and Management)
Rules, 2010 also encompass the drainage area or catchment Resources for Food and Agriculture
regions of the wetlands as determined by the authority. Hence 2. The United Nations Convention to Combat
statement 3 is correct. So (c) will be the correct answer. Desertification
Further government has notified wetlands (Conservation 3. The World Heritage Convention
and Management) Rules, 2017 which has enhanced the Which of the above has/had a bearing on the
focus of management of wetlands from a central authority biodiversity?
to state bodies.
(a) 1 and 2 only
51. The annual production of biomass is minimal in which (b) 3 only
of the following environments. (c) 1 and 3 only
(a) Deep sea (b) Prairie (d) 1, 2 and 3
(c) Deciduous forest (d) Taiga I.A.S. (Pre) 2014
U.P.P.C.S. (Pre) 2002 Ans. (d)
Ans. (a)
The International Treaty on Plant Genetic Resources for
Biomass is a term for all organic materials that stem from Food and Agriculture (IT PGRFA) is popularly known as the
plants (including algae, tree and crops). Biomass is produced International Seed Treaty. It is a comprehensive international
by green plants converting sunlight into plant material agreement in harmony with the Convention on Biological
through photosynthesis and includes all land and water based
Diversity. It aims at guaranteeing food security through the
vegetation as well as all organic wastes. Hence deep sea has
conservation, exchange and sustainable use of the world’s
minimum biomass production. Highest biomass production
is found among tropical rain forest. It is approximately 2200 plant genetic resources for food and agriculture. The World
gram/square meter/per year. Biomass production for biomes Heritage Convention and the United Nations Convention to
given in the options are as follows: Combat Desertification also has a bearing on the biodiversity.

Environment and Ecology General Studies F–79


Greenhouse Effect & Climate Change
Green House Effect *The increase in the average atmospheric temperature due to
increase in the Greenhouse gases in the atmosphere is called
*Solar energy reaches the Earth in the form of electromagnetic
Global warming.
waves. As the sun is very hot, the energy is emitted in high
*It is notable that the concept of Greenhouse was given by
energy short wavelengths that penetrate the Earth's atmosphere.
Joseph Fourier (1824).
*Solar energy or solar radiation includes, UV Rays, Visible *Green House effect is a natural phenomenon by which some
radiation and Infrared radiations. Of these, harmful Ultraviolet gases present in the atmosphere of a planet helps in increasing
rays are absorbed by the ozone layer while visible and infrared the temperature relatively.
radiations reaches the earth as thermal radiation. *Main cause of Global Warming is CO2 gas which is also
known as greenhouse gas. Methane (CH4) and Nitrous Oxide
(N2O) increases global warming. These gases absorb thermal
energy like Glass of Greenhouse which does not release warm
air present inside. This effect is known as ‘greenhouse effect’.
*Green House Gases present in the atmosphere of the earth are
- CO2, CH4, N2O and CFCs and water vapour. *It is notable
that Argon (Ar) is not a greenhouse gas so it is not responsible
for global warming.
Note :- *Carbon tetrafluoride (CF4) and Nitrous Oxides (N2O)
are relatively less responsible for global warming. The relative
contribution of main greenhouse gases towards global warming
are as follows:

Water Vapour about 60%


Carbon Dioxide (CO2) about 26%
Methane (CH4) about 4-9%
Ozone (O3) about 3-7%
*Thermal radiation heats up the Earth by means of conduction, Nitrous oxide (N2O) about 1.5%
convection and radiation. *Once earth gets heated it re-emits Chlorofluorocarbons (CFCs) and others about 0.1-0.4%
energy back into the atmosphere in the longwave radiation
*Hydrogen (H2) is not a greenhouse gas it is found in the
because Earth is cooler than the Sun. But Green House gases atmosphere as diatomic gas which is the main component of
present in atmosphere like CFCs (cloroflurocarbon), Nitrous Outer layer of atmosphere. *IPCC (Intergovernmental Panel
Oxide, water-vapour, methane and others absorb much of the on Climate Change) identified 6 Major greenhouse gases. CO2,
Longwave energy (CO2 absorb > 90%) emitted from the Earth's CH4. N2O, CFCs, SF6, NF6 are directly related to greenhouse
Surface preventing it from escaping from the Earth's surface. gases while Nox (Nitrogen Oxide), Carbon Monoxide (CO),
The greenhouse gases then re-emit this energy in all directions, Non-methane volatile organic compounds (NM VOC) and SO2
warming the Earth's surface and lower atmosphere. (sulphur dioxide) are indirectly related to greenhouse gases.

F–80 General Studies Environment and Ecology


Cause of Greenhouse Effect *CH4 (Methane) gas is also known as Marsh Gas. Its sources
include biogas, organic decomposition and remnants. *It is a
greenhouse gas and an air pollutant. Paddy fields, coal mines
and reared cattles that emits CH4 in the atmosphere are manmade
sources of CH4 gas. While natural sources of CH4 gas emission
are wetlands, Ocean and Hydrates. Wetlands emit the highest
proportion of CH4 gas (about 76%).
*Anaerobic conditions related to paddy fields are responsible
for CH4 gas emission.
*When nitrogen containing fertilizer is used by farmers then
N2O (Nitrous Oxide) is emitted from the soil. *Anthropogenic
environmental pollution may lead to climate change.
*Factors that increase carbon emission are excess burning of
fossil fuels, the rampant increase in the number of automobiles
*CO2 is must for life on the earth. Plants take CO2 from the along with the use of petrol and diesel as a fuel.
atmosphere for the synthesis of their food (photosynthesis) and *NASA and other researchers reported that increase in the Solar
release Oxygen in the atmosphere which is important for our Flare is not responsible (not major cause) for Climate Change.
life. But the excess of CO2 is leading to Global warming which Halocarbons, refrigerants, foam blowing agents CFCs, HCFCs
is detrimental for the life present on the Earth. halons etc. are factors responsible for ozone layer depletion.
*CO2 is released due to anthropogenic activities. It is absorbed These gases are constituents of greenhouse gases. *Greenhouse
by soil on the land and by Marine plants which use CO2 to gases are directly responsible for climate change and some
make food during photosynthesis. During photosynthesis, greenhouse gases are responsible for ozone layer depletion.
Phytoplanktons take CO2 and convert into plant food material.
*Major greenhouse gases present in the Earth’s atmosphere
Effects of Greenhouse Gases
*Effect of Global warming may be seen as melting of polar ice
are water, CO2, CH4, N2O and CFCs. Among these CO2 gas is
caps. Increase in temperature over so many years is resulting in
the main cause of global warming because its concentration is
a rise of the sea level and submerging of many coastal regions.
more than other greenhouse gases in the atmosphere.
Many European countries have lost many coastal regions.
*CO2 concentration in the atmosphere is increasing continuously.
According to research, the concentration of CO 2 in the *According to a report assessing key vulnerabilities and

atmosphere was 315 ppm (Parts Per Million) in 1950 but now the risk from climate change published by IPCC (Inter-

it is more than 415 ppm in the atmosphere. *If the level of Governmental Panel on Climate Change) if the present level

CO2 concentration increases at this rate then by the end of the of global temperature is increased by 2oC from pre-industrial

21st century the CO2 concentration would reach up to 650-950 era temperature level then Coral Mortality will occur in all
PPM and the temperature of the earth will be increased by 6oC. over world also 1/6 part of the ecosystem will transform and
*According to WMO (World Meteorological Organization) 1/4 known species will extinct which are present on the Earth.
CO2 Concentration in the atmosphere was 278 ppm (0.03%) *If global temperature level increases by 3oC from pre industrial
before pre-industrial era maintaining the balance between era temperature level then terrestrial biosphere will shift
Atmosphere, Ocean and Biosphere. But now this balance is towards Net Carbon Source, while more than 1/5 part of the
disturbed due to anthropogenic activities like the combustion ecosystem will be transformed and 30% known species may
of fossil fuels, Industries etc. become extinct in near future.

Environment and Ecology General Studies F–81


*According to an estimate given by IPCC on climate change Climate Change
the global temperature will be raised by 1.4-5.8 C during
o
*The change in the position of the earth with respect to the
1990-2100. Due to this increase in temperature 142 out of 144 sun is also responsible for the variation in the amount of solar
polar ice caps in the polar regions may melt and the sea level
radiation reaching to the earth. This change is according to the
will increase.
astronomical principle of climate change. Scientists called this
*According to another estimate given by IPCC, sea level will
principle as Milankovitch Hypothesis.
be raised by 0.33-0.45 m during 1990-2100. *According to an
*Milutin Milankovitch was a Serbian astronomer, climatologist,
estimation, sea level has increased by 390 feet from last ice
geophysicist etc.
age till now. Sea level is increasing at a rate of 3 mm per year.
*Milutin Milankovich has given the principle related to climate
For instance, sea level is increasing in Fiji at the rate of 0.15
change, which is related to the long term orbital position of
m (6 inches) per year. So there is a risk that Fiji will submerge
the earth. These principles are recognised as the astronomical
in the near future.
principle of the earth. The eccentricity of the Earth's orbit, the
*During the past century, the temperature of Earth has increased
obliquity of the earth's rotation axis, the equinoxes are the part
by 0.8oC. *It is remarkable that plankton is important for sea
ecosystem as grasses are important for terrestrial ecosystem. of this principle whereas the solar energy is not related to this

Increased CO2 uptake by plankton will negatively influence principle.

the nutritional quality of plankton. This phenomenon can have *The Intergovernmental Panel on Climate Change (IPCC) is
consequences for entire food webs in the ocean. an intergovernmental body of the United Nations. According
to IPCC, greenhouse gas in an atmosphere is a gas that absorbs
and emits radiation within the thermal infrared range. *The
primary greenhouse gases in the earth's atmosphere are Carbon
dioxide (CO2), Methane (CH4), Water vapour (H2O), Nitrous
Oxide (N2O) and Ozone (O3).
*Climate change is measured by the Ice core which is a
cylindrical sample obtained by piercing in the glacier or in the
ice sheet. An ice core is the most direct and detailed record of
the past to examine the climate and atmosphere of that time.
*Cryogenic indicators of the climate are obtained from the Ice
core. An ice core is extracted from the large Ice sheet found in
Greenland and Antarctica. It is used to gather the information
of the gases in the Atmosphere in the past. *It is noteworthy
that climate change has the potential to affect agriculture. The
moisture and the efficiency of the soil are impacted by the
rise in temperature. This will increase salinity in the soil. The
productivity of the crops will also be affected by climate change.
*Climate change is increasing the pressure on the resources.
There are many evidence of conflict for the resources like
forest, grassland, agricultural land, river water, land water etc.
which prove climate change as a major factor in increasing
social conflict stress.

F–82 General Studies Environment and Ecology


*Many studies confirm the serious consequences of climate in the year 2016. The 25th United Nations Climate Change
change on both production and productivity of agriculture and Conference (COP 25) was held in Madrid, Spain, from 2 to 13
forestry. The possibility of having a more severe situation in the December 2019. In 2021, COP 26 was held in Glasgow, United
future cannot be denied. The agricultural sector is affected the Kingdom from 31 October 2021 - 12 November 2021. The COP
most by the frequency of extreme weather conditions. 27 will be held in El-sheikh, Egypt from 7 - 18 November 2022.
*In these cases, the question of food security arises. In the *Several important Goals were set at the UNFCCC meeting in
case of food insecurity, there is a tendency of unnecessary food Paris 2015. This Agreement aims at limiting of the emission
storage and food processing, which eventually leads to social of greenhouse gases. It's goal is to control the increase in the
tension. So, food insecurity is also a major cause of Social average global temperature by 2oC or 1.5oC related to the Pre-
stress. Carbon overlay refers to the long-term storage of carbon industrial level by the end of the century. *The important aspect
in plants, geologic structure and in oceans. of the conference was that the developed countries accepted
*Currently (according to the latest available data in 2020), the their responsibility in reducing Global warming, as well as
country contributing most to emissions of carbon dioxide (CO2) with the help of developed countries it committed to deposit
is China (28% of the world's total CO2 emission). United States 100 billion dollars by the year 2020 for Climate Fund. This
of America is placed second with 15%, while India is third with agreement is effective from 1st November 2016. This agreement
7% of world's CO2 emission and Russia with 5% of world's CO2 has been signed by 195 countries till now.
emission is fourth and with 3% Japan is fifth. *United Nations Climate Change Conference, 2010 (CoP16)
*China is the largest emitter of carbon globally. Bhutan is was called in Cancun, Mexico from 29 November to 10
known as the world's top carbon negative country. The December 2010.
population of Asia Pacific Cooperation (APEC) countries is *A Green Climate Fund (GCF): Provision was made in
41% of the world's population. They consume the world's 48% the Cancun conference under which the developed nation
of energy and contribute nearly 48% of the world's greenhouse will provide 100 billion dollars per year, by 2020 so that the

gases. They intend to support the Kyoto Protocol however some developing nation can overcome to the challenges of climate

of the countries have not ratified it. change. The Green Climate Fund is a fund under the framework
of the UNFCCC and its establishment was aimed at helping
Climate Change Conference developing countries in adaptation and mitigation practices to
*The United Nations Framework Convention on Climate counter climate change. This fund is governed by GCF board.
Change (UNFCCC) is an international environmental treaty There are 24 members on the board.
adopted at the United Nations Conference on Environment
and Development (UNCED) held in Rio-de-Janeiro in 1992.
*The United Nations Convention on Climate Change
(UNFCCC) supported by more than 50 countries was effective
from March 21, 1994. Currently, it is ratified by the world's
197 countries.
*The United Nations has taken several important initiatives
from time to time in the direction of prevention of climate
change. [COP1-The first Conference of the Parties held in
Berlin, Germany.] It is notable that the 21st Climate Change
Conference was held in Paris in 2015. The 22nd Climate Change
Conference was held as COP22 in Marrakech in (Morocco)

Environment and Ecology General Studies F–83


*Earth Hour is a worldwide movement organized by the World *The global warming potential (GWP) of a gas depends on
Wide Fund for Nature (WWF) to create awareness about climate the efficiency of the atoms of that gas and its environmental
change and the need to save the Earth. Participants around the life-cycle. The environmental life cycle of Carbon dioxide is
world of this program every year on the last Saturday of March, changeable while its global warming potential remains constant
turn off the power for 1 hour from 8:30 pm to 9:30 pm. (GWP=1). The global warming potential of Methane was
*In 2021, Earth Hour was celebrated on March 27 and in 2022 observed 72 in the last 20 years. Therefore Methane is more
it is scheduled to be celebrated on 26th March. dangerous and effective as a greenhouse gas than carbon
*The Kyoto Protocol is an international treaty which extends dioxide.
the 1992 United Nations Framework Convention on Climate *CDM (Clean Development Mechanism) is a system for
Change (UNFCCC). Its objective is to reduce global warming controlling the greenhouse gas emissions for the reduction in
by reducing greenhouse gases concentration in the atmosphere. Global Warming. CDM came under the Kyoto Protocol.

The Kyoto Protocol was adopted in Kyoto, Japan on 11 *Aforestation, carbon sink and minimization of carbon types

December 1997 and came into effect on 16 February 2005. of the project came under the CDM. It is helpful in reducing
global warming.
*It was adopted as a binding agreement to reduce Greenhouse
*Developed countries are earning carbon credits by investing in
gas emission. According to the Kyoto Protocol, countries where
the carbon emission reduction projects in developing countries.
the emission of Greenhouse gases is more, would collectively
"Carbon Trading" is being encouraged internationally for
achieve a 5.2% reduction in Greenhouse gases emission
the control of carbon dioxide (CO2) emission and to reduce
during the year 2008 to 2012. *The Clean Development
Global warming.
Mechanism (CDM), defined in Article 12 of UNFCC's Kyoto
*Clean Development Mechanism (CDM) is the name of a
Protocol. It allows a country with an emission reduction or
market-driven device under the UNFCCC United Nations
emission-limitation commitment under the Kyoto Protocol
Framework Convention on Climate Change) that allows
(Annex-B party) to implement an emission-reduction project
developing countries to get funds or incentives from the
in developing countries.
developed countries to adopt better technologies that reduce
*Under this, the developed countries of Annex - I can earn
greenhouse gas emissions. Carbon tax is levied on those energy
'Carbon Credit' by implementing clean development projects
resources which releases carbon dioxide (CO2). It is an example
in Non-Annex-I countries. Carbon credits can be used by the
of Pollution tax.
developed countries to meet the shortage in national emission
*Countries like Sweden, Finland, Netherlands and Norway has
commitments. These projects are implemented by the countries
already levied a carbon tax. New Zealand is imposing carbon
of Annex-I but these projects are implemented in Non-Annex-I tax to help in the reduction of Global warming. In February,
developing countries. 2018 the Singapore government has announced that it will be
*Certified Emissions Reductions (CER's) is a Tradable Unit imposing a carbon tax to reduce its Greenhouse gas emissions
of CDM. By reducing the amount of 1-ton carbon dioxide 1 from the year 2019. All facilities in all business areas producing
CER unit is obtained. It is also obtained by afforestation and 25,000 tons or more greenhouse gas emissions per year will be
so on. Under the Kyoto Protocol, carbon credit system was subjected to a carbon tax.
introduced to reduce carbon emissions in the environment. *The Greenhouse Gas Protocol was developed by "World
*A carbon credit is provided to a country or a group of countries Resource Institute" (WRI) and World Business Council on
by reducing per metric ton of maximum carbon emission which Sustainable Development (W.B.C.S.D). It is an International
is allowed. Carbon credits are traded in the international market institution that guides and provides direction to the governments
according to their current market value. and people related to business to understand Greenhouse gas
*Kyoto Protocol came into force after the ratification of Russia emission and its consequences. It developed guidelines to
on 16 February, 2005. measure, quantity and control greenhouse gas emission.

F–84 General Studies Environment and Ecology


*Forest carbon partnership facility is a programme initiated four broad theme-based communities-
by World Bank in June 2008. It is a global partnership of *Extreme Physics and Chemistry- Dedicated to improving
governments, businesses, civil society and indigenous people. It our understanding of the physical and chemical behaviour of
has been created with the main objective of helping developing carbon at extreme conditions, as found in the deep interiors of
countries to reduce the emissions caused by deforestation. Earth and other planets.
*This facility provides financial and technical assistance to *Reservoirs and Fluxes - Dedicated to identifying deep
developing countries in their efforts. But this facility does carbon reservoirs, determining how carbon moves among these
not provide financial support to universities, individuals and reservoirs, and assessing Earth's total carbon budget.
institutions. *Deep Energy - Dedicated to understanding the volume and
*The Bio Carbon Fund Initiative for Sustainable Forest rates of abiogenic hydrocarbons and other organic species in
Landscapes (ISFL) is a multilateral fund managed by the the crust and mantle through geological time.
World Bank. Funding is maximum from Norway up to 135 *Deep Life - Dedicated to assessing the nature and extent of
million dollars followed by the UK up to 120 million dollars the deep microbial and viral biosphere.
and then America with 25 million dollars. It promotes reducing Action Plan of India in
greenhouse gas emissions from the land sector. perspective of climate change
*The Cartagena Protocol is the complementary Protcol of
*In June 2008, the then Prime Minister, Manmohan Singh
Convention on Biological Diversity (CBD), which aims to ensure
had released India's first National Action Plan on climate
the safe handling, transport and use of living modified organisms
change. India's National Action Plan on climate change was
(LMOs) resulting from modern biotechnology that may have
launched on June 30, 2008. National Action Plan focuses on
adverse effects on biological diversity. It was adopted on 29
eight national mission.
January 2000 and entered into force on 11 September 2003.
*Under the National Action Plan on Climate Change the
*Nagoya Protocol came into effect on 12 October 2014. It
following eight national missions are included:
is the complementary agreement of CBD. It is related to the
1. National solar mission.
proper and fair sharing of genetic resources and benefits
2. National mission for enhanced energy efficiency.
received from them.
3. National mission on sustainable habitat.
*United Nations Climate Action Summit 2019, was held at
4. National water mission.
the headquarters of the United Nations in New York City on
5. National mission for sustaining the Himalayas Ecosystem.
23 September 2019. The summit was convened on the theme,
6. National mission for Green India.
"Climate Action Summit 2019: A Race We can Win. A Race
7. National Mission for sustainable agriculture.
We Must Win."
8. National Mission on Strategic Knowledge for climate
*The Global Climate Change Alliance (GCCA) was
change.
established in 2007 (operational : 2008) by the European Union
*One of the eight missions outlined under the National Action
(EU) in collaboration with developing countries. This treaty is
Plan on Climate Change (NAPCC), National mission for Green
active in 38 countries, 8 regions and sub-regions at global levels.
India was launched in February, 2014 and is aimed at protecting,
*The overall aim of GCCA is to create a new treaty between the
restoring and enhancing India's diminishing forest cover and
European Union and the poor developing countries, who have
responding to climate change by a combination of Adaptation
the least potential to deal with climate change. It is coordinated
& Mitigation Measure.
by the European Commission (EC).
*In the second meeting of the National Executive Council
*The Deep Carbon Observatory (DCO) is a global community
(NEC) of the National Green India Mission in October
of more than 1000 scientists on a ten-year quest to understand
2015, the annual plan for the possible schemes and operations
the quantities, movements, forms and origins of carbon in Earth.
proposed by the four states of Mizoram, Manipur, Kerala and
From 2009-2019, DCO science was categorized into following
Environment and Ecology General Studies F–85
Jharkhand were approved.
Basic Questions
Attain 40 percent non-fossil 1. Climate change is caused by –
based power capacity by
2030 (country has achieved (a) Greenhouse gases
this target in November
2021 itselt) (b) Depletion of the ozone layer
(c) Pollution
(d) All the above
U.P.P.C.S.(Pre) 2013
Ans. (d)

Greenhouse gases and Pollutions are directly responsible


for ozone depletion while depletion of the ozone layer is
indirectly responsible for climate change. Hence option (d)
is the correct answer.
2. The main reason of climate change is –
(a) Greenhouse gases
(b) Depletion of the ozone layer
(c) Pollution
(d) All of the above
*India has made many announcements under Intended
Jharkhand P.C.S. (Pre) 2013
Nationally Determined Contributions (INDCs) which
Ans. (d)
is for the reductions in greenhouse gas emission under the
United Nations Framework Convention on Climate Change The main reason of climate change is the emission of
(UNFCCC). greenhouse gases (a gas that contributes to the greenhouse
*The INDC centre around India's policies and programmes effect by absorbing infrared radiation e.g. carbon dioxide
on promotion of clean energy, especially renewable energy, and chlorofluoro carbons) which are the main causes of the
enhancement of energy efficiency, development of less carbon- depletion of the ozone layer. This ozone layer prevents most
intensive and resilient urban centres, promotion of waste harmful ultraviolet light (UV) from passing through the
earth’s atmosphere. Pollution due to human activity is also
to wealth, safe, smart and sustainable green transportation
a prime reason for climate change.
network, abatement of pollution and India's efforts to enhance
carbon sink through creation of forest and tree cover. 3. Where was the climate change conference held in the
*There are 32 Global Atmospheres Observing Stations. Global year 2015?
Atmospheric Observation Stations were established in Algeria, (a) Paris (b) Qatar
Brazil and Kenya but in India, it has yet to be established. (c) Bonn (d) Colombo
*It is notable that in May 2011, for reducing greenhouse (e) None of these
gas emission, with the help of United Nations Development Chhattisgarh P.C.S. (Pre) 2015
Program (UNDP) the State Climate Change Centre has been Ans. (a)
established in Jharkhand. It provides consultation to the state
government on climate change. The main functions are; The 2015 United Nations Climate Change Conference,
COP 21 was held in Paris, France, from 30 November to 12
(1) Information about climate change for the various
December, 2015. In 2022, COP 27 will be held in El-Shiekh,
stakeholders.
Egypt which was earlier scheduled to be held in 2021.
(2) Making a network with other organizations working on
climate change problems. 4. Which of the following United Nation's Climate
(3) To help the government officials to increase public Change Conferences was held in Paris?
awareness. (a) 25th (b) 23rd
Jharkhand Action Plan on climate change was published in the (c) 22nd (d) 21st
year 2013 and 2014. U.P. Lower Sub. (Pre) 2015

F–86 General Studies Environment and Ecology


Ans. (d) 8. What is 'Green House Effect'?
(a) Increase in global temperature
See the explanation of the above question.
(b) Decrease in global temperature
5. How much has the global average temperature risen (c) Increase in seawater temperature
in the last century? (d) Increase in temperature of rivers and lakes
(a) 3.0ºF (b) 1.8ºF U.P. P.C.S. (Pre) 2018
(c) 3.4ºF (d) 2.4ºF Ans. (a)
R.A.S./R.T.S. (Pre) 2021
See the explanation of the above question.
Ans. (b)
9. Greenhouse effect means:
Earth’s average surface air temperature has increased
(a) Increase in atmospheric temperature due to the
by about 1 °C (1.8 °F) since 1900, with over half of the
concentration of greenhouse gases at the atmosphere.
increase occurring since the mid-1970s. A wide range of
other observations (such as reduced Arctic sea ice extent and (b) Growing vegetables and flowers in increased
increased ocean heat content) and indications from the natural temperature.
world (such as pole ward shifts of temperature-sensitive (c) Growing food crops in glass houses.
species of fish, mammals, insects, etc.) together provide (d) None of the above.
incontrovertible evidence of planetary-scale warming.
Uttarakhand P.C.S. (Pre) 2006
6. United Nations Climate Action Summit, 2019 was held
Ans. (a)
at which of the following cities?
(a) Paris (b) Geneva See the explanation of the above question.
(c) Doha (d) New York 10. What is “Greenhouse effect”–
U.P.P.C.S. (Pre) 2019 (a) Increase in temperature of Earth due to concentration
Ans. (d) of gases in atmosphere
United Nations Climate Action Summit 2019, was held at the (b) Acid rain
headquarters of the United Nations in New York City on 23 (c) Black rain
September 2019. The summit was convened on the theme, (d) None of the above
"Climate Action Summit 2019: A Race We can Win. A Race
M.P.P.C.S. (Pre) 1991
We Must Win."
Ans. (a)
7. Greenhouse effect is the process –
Green House effect is caused by the absorption of infra-red
(a) To obtain suitable temperature for plantation
radiation by environmental gases (mainly carbon dioxide).
(b) Of heating the atmosphere by absorption of infrared This phenomenon raises the temperature of the environment.
radiations by atmospheric carbon dioxide.
11. The concept of Greenhouse Gases was postulated by –
(c) By which barren mountains changes into the green.
(a) C.C. Park (b) J.N.N. Jeffers
(d) In which deep water seems green.
(c) Joseph Fourier (d) L. Zobler
R.A.S./R.T.S. (Pre) 1992
U.P.P.C.S. (Mains) 2011
Ans. (b)
Ans. (c)
The greenhouse effect is a natural process that warms
The concept of Greenhouse Gases was postulated by Joseph
the earth’s surface. When the sun’s energy reaches the
Fourier in 1824.
earth’s atmosphere, some of it is reflected back to space
and the rest is absorbed and re-radiated by greenhouse 12. Which one of the following atmospheric changes take
gases. Greenhouse gases include water vapour, carbon place due to greenhouse effect?
dioxide, methane, nitrous oxide, ozone and some artificial
(a) Increase in carbon dioxide concentration in atmosphere
chemicals such as Chlorofluoro Carbons (CFC). The
absorbed energy warms the atmosphere and the surface (b) Increase in humidity in atmosphere
of the earth, popularly known as greenhouse effect. (c) Increase in a biological population

Environment and Ecology General Studies F–87


(d) Increase in wind speed 2. Kyoto Protocol deals primarily with the depletion
U.P.P.C.S. (Pre) 2017 of the ozone layer.
Ans. (*) 3. Methane as a greenhouse gas is more harmful than
carbon dioxide.
The green house gases present in the earth's environment Which of the statements given above is/are correct ?
behave like a cover from which the short wave radiation
(a) 1 and 2 (b) 1 and 3
coming from the sun can flow but the longwave radiation
(c) 1 only (d) 3 only
returning from the earth can't overpass it. As a result of that
atmospheric temperature of the earth increases. This process is I.A.S. (Pre) 2005
known as greenhouse effect. An increase in the concentration Ans. (b)
of carbon dioxide in the atmosphere has a positive effect on The Kyoto Protocol was adopted in Kyoto on 11 December,
the greenhouse effect, but the evidence of any meaningful 1997, and entered into force on 16 February, 2005.
increase in the concentration of carbon dioxide has not been Hence statement (1) is correct. The Kyoto Protocol is
found due to greenhouse effect. So option (a) is incorrect. an international agreement linked to the United Nations
Water vapour is a major greenhouse gas and it also plays Framework Convention on Climate Change. Hence statement
a vital role in greenhouse effect, but it gets affected by the (2) is incorrect. The global warming potential (GWP) of a
intensity of the greenhouse effect. The process of evaporation gas depends on the efficiency of the atoms of that gas and
increases due to intense atmospheric heat of greenhouse it's environmental life-cycle. The environmental life cycle
effect which increases the amount of humidity in the air. So, of carbon dioxide is changeable while its global warming
option (b) is correct. potential remain constant (GWP = 1). The global warming
Greenhouse effect which increases the heat and humidity in potential of Methane was observed 72 in the last twenty
the atmosphere can also increase the number of the organism years. Therefore, Methane is more dangerous and effective
as heat and moisture have a positive effect on reproduction or as a greenhouse gas than carbon di-oxide.
fertility. Although this statement is correct, it can not be the
answer because it is not an atmospheric change. Greenhouse 16. Which of the following is not correctly matched?
effect has an effect on temperature and pressure. Increasing (a) Cartagena Protocol Biosafety
temperature increases the pressure and increases the mobility (b) Stockholm Convention Persistent organic
of the air. So, it will be the correct answer in context to the pollutant
question. In this way, option (b) and option (d) both answers
(c) Montreal Protocol Ozone Layer
will be correct.
(d) Kyoto Protocol Save water
13. ‘Kyoto Protocol’ is related to- U.P.P.C.S. (Pre) 2019
(a) Air pollution (b) Greenhouse Gas Ans. (d)
(c) Climate change (d) Water pollution
Cartagena Protocol Biosafety
M.P.P.C.S. (Pre) 2014
Stockholm Convention Persistent organic
Ans. (c)
pollutant
The Kyoto Protocol is an international agreement linked Montreal Protocol Ozone Layer
to the United Nations Framework Convention on Climate Kyoto Protocol Climate change
Change which commits its parties by setting internationally 17. Greenhouse effect is enhanced in the environment
binding emission reduction targets.
due to?
14. Kyoto Protocol is related to: (a) Carbon monoxide
(a) Air pollution (b) Climate change (b) Carbon dioxide
(c) Ozone depletion (d) Wetland Conservation (c) Oxygen
U.P. P.C.S. (Mains) 2017 (d) None of the above
Ans. (b) U.P.P.C.S. (Pre) 2015
See the explanation of the above question. Ans. (b)

15. Consider the following statements: Greenhouse effect is the result of the absorption of infrared
1. Kyoto Protocol came into force in the year 2005. radiation by environmental gases. Main greenhouse gases

F–88 General Studies Environment and Ecology


are carbon dioxide, methane, nitrous oxide and clorofluoro Ans. (e)
carbon. The six major gases identified as Green House Gases
18. The main component of greenhouse gas is? by Intergovernmental Panel on climate change include
CH 4, CFC, and N 2O. While sulphur dioxide (SO 2) is
(a) Carbon dioxide (b) Methane
not a greenhouse gas, it is responsible for acid rain. The
(c) Nitrous oxide (d) Ozone
Chhattisgarh Public Service Commission has marked the
(e) None of the above/More than one of the above correct answer as option (e) whereas they have considered
64th B.P. S.C. (Pre) 2018 option (c) as correct in the previous answer sheet. Probably
Ans. (e) this would have been done on the basis of direct and indirectly
involved gases in greenhouse effect. The gases directly
See the explanation of the above question.
responsible for greenhouse effect are – Carbon dioxide (CO2),
19. Which among the following countries has announced Methane (CH4), Nitrous Oxide (N2O), CFC, SF6, NF3, etc.
to impose 'Carbon Tax' from 2019 to cut its greenhouse While the gases indirectly responsible for green house effect
gas emission? include - Nitrogen oxide (NOX), Carbon monoxide (CO),
Non-methane volatile organic compounds (NMVOC) and
(a) Thailand (b) China
sulphur dioxide (SO2).
(c) Singapore (d) Malaysia
U.P. R.O./A.R.O. (Pre) 2017 23. The main component of greenhouse gases is
Ans. (c) (a) Carbon dioxide

Government of Singapore in February 2018 declared that (b) Methane


it will impose a carbon tax to reduce Greenhouse gas (c) Nitrous oxide
emissions as a part of its commitments under the Paris (d) Ozone
Climate agreement. (e) None of the above/More than one of the above
20. Which one of the following gases is not a greenhouse 65th B.P.S.C. (Pre) 2019
gas? Ans. (e)
(a) CO2 (b) CH4
See the explanation of the above question.
(c) NO2 (d) O2
U.P.P.C.S. (Pre) 2016 24. Which of the following group of gases contribute to
Ans. (d) the “Greenhouse Effect”?
(a) Ammonia and Ozone
Greenhouse gas in an atmosphere is a gas that absorbs and
(b) Carbon monoxide and Sulphur dioxide
emits radiation within the thermal infrared range. This
process is the fundamental cause of the greenhouse effect. (c) Carbon tetrafluoride and Nitrous oxide
The primary greenhouse gases in earth’s atmosphere are (d) Carbon dioxide and Methane
carbon dioxide, methane, nitrous oxide and ozone. Hence, U.P.P.C.S.(Pre) 2013
(d) is the correct answer. Ans. (d)
21. Which of the following is not a greenhouse gas? The relative contribution of main Greenhouse Gases are as
(a) Oxygen (b) Carbon dioxide follows–
(c) Chlorofluorocabons (d) Methane Water Vapor about 60%
U.P. R.O./A.R.O. (Mains) 2017 Carbon Dioxide about 26%
Ans. (a) Methane about 4-9%
Ozone about 3-7%
See the explanation of the above question. Nitrous Oxide about 1.5%
22. Which is not Green House Gas? Chloroflurocarbons and others about 0.1-0.4%
Thus, option (d) will be correct answer.
(a) CH4 (b) CFC
(c) SO2 (d) N2O 25. What will be the decreasing order of the following
(e) None of these which contribute to the 'Greenhouse effect'?
Chhattisgarh P.C.S. (Pre) 2016 (a) CO2 > CH4 > CFCs > N2O

Environment and Ecology General Studies F–89


(b) CH4 > CO2 > CFCs > N2O U.P. P.C.S. (Mains) 2017
(c) CO2 > CFCs > CH4 > N2O Ans. (c)
(d) CO2 > CH4 > N2O > CFCs
The primary greenhouse gases are - Water Vapour, Methane,
Chhattisgarh P.C.S. (Pre.) 2020 Nitrous Oxide and Ozone.
Ans. (d) Substituting Propane for other fuels such as gasoline and
See the explanation of the above question. fuel oil is an economical and viable step towards cleaner
air. Using Propane reduces greenhouse gas like Carbon-
26. Which of the following gas has highest contribution in dioxide and air pollutants like carbon monoxide and nitrogen
global warming? oxide.
(a) Carbon-di-oxide (b) Chlorofluorocarbon
30. Which one of the following is not responsible for global
(c) Nitrous oxide (d) Methane
warming?
M.P.P.C.S. (Pre) 2019
(a) Methane (b) Water vapour
Ans. (a)
(c) Argon (d) Carbon dioxide
See the explanation of the above question. U.P.P.C.S. (Mains) 2016
27. Which one of the following naturally occurring Ans. (c)
greenhouse gases causes the highest percentage of the A layer of greenhouse gases namely carbon dioxide,
greenhouse effect? methane and nitrous oxide acts as a thermal blanket for the
(a) Carbon Dioxide (b) Methane earth, absorbing heat and warming the surface known as
(c) Ozone (d) Water Vapour global warming. Hence, Argon is not responsible for global
U.P.P.C.S. (Mains) 2011 warming.
Ans. (d) 31. Which one of the following statements regarding the
See the explanation of the above question. Emission Reduction Purchase Agreement signed with
the World Bank in May, 2011 is not correct?
28. Which set of the following gases is most important for
the greenhouse effect? (a) The agreement has been signed between the World
(a) Carbon dioxide, Methane, Chlorine, Nitric oxide. Bank and the Government of India.
(b) Nitric oxide, Methane, Ethane, Chlorfluoro carbons. (b) The agreement shall be valid for ten years.
(c) Carbon dioxide, Methane, Nitrous oxide, Water (c) The agreement is for securing carbon credits for a
vapour. project of Himachal Pradesh.
(d) Carbon Monoxide, Sulphur dioxide, Methane, Water (d) As per the agreement, one ton of carbon-di-oxide will
vapour be equivalent to one credit unit.
Uttarakhand P.C.S. (Pre) 2016 U.P.P.C.S. (Mains) 2010
Ans. (c) Ans. (a)
When the earth starts frosting after getting heat from the
In May, 2011, ‘The emission reduction purchase agreement’
sun, then the heat is dissipated out of the earth. But gases
for the purchase of certified emission reductions (CER)
like carbon dioxide, nitrous oxide, water vapour and
was signed by Hubert Nove Josserand, deputy country
methane absorb some part of this heat in return of which
director of World Bank, on behalf of International Bank
some additional heat is collected in the lower part of the
for Reconstruction and Development, and Sudripto Roy,
atmosphere. Over the past few years, due to increase in
additional chief secretary on behalf of the Himachal
the amount of these gases in the atmosphere, the average
Government. Rest all the options are correct.
temperature of the atmosphere has increased which is
called global warming or greenhouse effect and the above- 32. Which reference to 'NET-ZERO' GOAL FOR 2050,
mentioned gases are called greenhouse gases. which of the following statements is/are correct?
29. Which one of the following is not a greenhouse gas? 1. It means that a country will bring down its emissions
(a) Carbon dioxide (b) Methane to zero by 2050.
(c) Propane (d) Ozone 2. It means that a country's emissions will be

F–90 General Studies Environment and Ecology


compensated by absorption and removal of greenhouse (a) China (b) Japan
gases from the atmosphere. (c) Russia (d) U.S.A
Select the correct answer using the codes given below. U.P.P.C.S (Pre) 2010
Codes. U.P.P.C.S (Pre) 2018
(a) Only 1 (b) Only 2 Ans. (a)
(c) Both 1 and 2 (d) Neither 1 nor 2
According to the latest data available in the 2020, largest
U.P.P.C.S (Pre) 2021 contributor to carbon dioxide emission is China. It emits
Ans. (b) about 28% of the world's total carbon emission. The United
States is in second place with 15% and India is in third place
In the month of February, 2021 the International Energy
with 7%. Russia is in fourth place with 5% and Japan is in
Agency's (IEA) released its Net Zero Emissions (NZE) Road
fifth place in terms of carbon emission with 3% of the world's
map-named 'Net Zero by 2050'.
total CO2 emission.
It is the World's first comprehensive energy road map which
came ahead of the United Nations November 2021. 35. Which of the following countries is the largest emitter
'Net zero emissions' refers to achieving an overall balance
of carbon?
between greenhouse gas emissions taken out of the
atmosphere. (a) USA (b) China
What is Net Zero? (c) India (d) European Union
Net-zero, which is also referred to as carbon-neutrality, does U.P. Lower Sub. (Pre) 2015
not mean that a country would bring down its emissions to
Ans. (b)
zero.
Rather, net-zero is a state in which as country's emissions China is the largest emitter of carbon globally.
are compensated be absorption and removal of greenhouse
gases from the atmosphere. 36. Which of the following is known as 'carbon negative
Absorption of the emissions can be increased by creating country' in the world?
more carbon sinks such as forests, while the removal of gases (a) Afghanistan (b) Bahrain
from the atmosphere requires futuristic technologies such as
(c) Bhutan (d) Nepal
carbon capture and storage.
This way, it is even possible for a country to have negative U.P. Lower Sub. (Pre) 2015
emissions, if the absorption and removal exceed the actual Ans. (c)
emissions.
Bhutan is known as the world's top 'carbon negative country'.
A good example is Bhutan which is often described as carbon-
negative because it absorbs more than it emits. 37. Which one among the following substances does not
33. Which one of the following gases is both harmful and contribute to global warming directly ?
beneficial to life on earth? (a) Oxides of Sulphur (b) Methane
(a) Oxygen (b) Ozone (c) Carbon dioxide (d) Water vapour
(c) Nitrogen (d) Carbon dioxide U.P.P.C.S.(Pre) 2013
U.P. Lower Sub. (Mains) 2013 Ans. (a)
Ans. (d)
Oxide of Sulphur do not contribute to global warming
Plants use carbon dioxide to produce carbohydrates (sugars directly, while other gases given in option contribute to
and starches) in the process known as photosynthesis. Since global warming.
humans and all other animals depend on plants for their
38. Which one of the following is NOT a green house gas
food, photosynthesis is necessary for the survival of all life
on Earth. It is also a greenhouse gas which is very harmful found naturally in the atmosphere?
to us. Therefore carbon dioxide is both useful as well as (a) Nitrogen oxide (b) Carbon dioxide
harmful to life on earth. (c) Methane (d) Ozone
34. Today the largest contributor to carbon dioxide (CO2) U.P.P.C.S. (Pre) 2020
emissions is – Ans. (a)

Environment and Ecology General Studies F–91


Green house gas has the property of absorbing infrared (a) Methane (b) Nitrous Oxide
radiation emitted from Earth's surface and reradiating it (c) Chlorofluorocarbons (d) Hydrogen
back to Earth's surface. Carbon dioxide, Methane, ozone U.P.P.C.S. (Mains) 2012
and water vapour are the most important green house gases, Ans. (d)
found naturally in atmosphere. Nitrogen Oxide is not a Green
House Gas found naturally in the atmosphere. See the explanation of the above question.

39. Which of the following is not Greenhouse gas? 44. Which of the following is not Greenhouse gas?
(a) Carbon Dioxide (b) Sulphur Dioxide (a) Carbon dioxide (b) Methane
(c) Methane (d) All the above (c) Nitrous oxide (d) Nitrogen
U.P.P.C.S. (Mains) 2007 U.P.P.C.S. (Pre) 2012
Ans. (b) R.A.S./R.T.S (Pre) 1993
Ans. (d)
A greenhouse gas (sometimes abbreviated GHG) is a gas
in an atmosphere that absorbs and emits radiation within See the explanation of the above question.
the thermal infrared range. This process is the fundamental
45. Which among the following gases is most responsible
cause of the greenhouse effect. The primary greenhouse gases
in Earth’s atmosphere are water vapour, carbon dioxide, for global warming?
methane, nitrous oxide and ozone. (a) Nitrogen (b) Methane
(c) Carbon-di-oxide (d) Carbon mono-oxide
40. Which one of the following gas is responsible for global
Uttarakhand P.C.S. (Pre) 2012
warming?
Ans. (c)
(a) Only Oxygen
(b) Oxygen and Carbon dioxide Greenhouse gases in the environment are water vapour,
Carbon Dioxide, Methane, Nitrous oxide and Clorofluoro
(c) Carbon dioxide and Methane
Carbon. Among these, carbon dioxide is most responsible
(d) Only methane for global warming as its concentration in the environment
Uttarakhand P.C.S. (Pre) 2006 is more as compared to other greenhouse gases. Notably,
Ans. (c) the concentration of carbon dioxide is increasing in the
atmosphere.
See the explanation of the above question.
46. The total amount of green-house gases produced, to
41. Which of the following is responsible for global directly and indirectly support human activities, is
warming? referred to as?
(a) Methane (b) Carbon dioxide (a) Carbon dioxide index
(c) Water vapour (d) All of the above (b) Carbon footprint
Jharkhand P.C.S. (Pre) 2013 (c) Carbon sequestration
Ans. (d) (d) Carbon capture
R.A.S./R.T.S. (Pre) 2018
See the explanation of the above question.
Ans. (b)
42. The Gases most responsible for Global Warming are:
The total amount of greenhouse gases produced, directly
(a) Carbon-di-oxide and Methane
or indirectly to support human activities, is referred to as
(b) Ammonia and Butane carbon footprint.
(c) Radon and Nitrogen
47. Which one of the following is not correctly matched?
(d) Ozone and Hydrogen Chloride
Green House Gas Source
U.P.P.C.S. (Spl) (Mains) 2004
(a) Carbon dioxide - Thermal Power Stations
Ans. (a)
(b) Chlorofluorocarbon - Automobile
See the explanation of the above question. (c) Nitrous Oxide - Waterlogged Paddy
fields
43. Which of the following is not Greenhouse gas?

F–92 General Studies Environment and Ecology


(d) Sulphur dioxide - Brick Kilns 4. Bleaching of Coral reefs
U.P.P.C.S. (Pre) 2017 Select the correct answer using the codes given below.
Ans. (b) Codes :
(a) 1, 2 and 3 only (b) 2 and 3 only
Chlorofluorocarbon is a man-made gaseous and liquified
material made from carbon, chlorine, fluorine and oxygen (c) 1, 3 and 4 only (d) 1, 2, 3 and 4
which is used as a refrigerant in refrigerators and air- U.P.P.C.S. (Pre) 2020
conditioning devices. Chlorofluoro Carbon is an important Ans. (d)
factor of ozone depletion in the atmosphere.
Global warming causes climate change which poses a serious
48. Which one of the following is not a Greenhouse Gas? threat to life on the earth. Rise in sea level, melting of glaciers,
(a) Methane spread of diseases and bleaching of coral reefs are the impacts
(b) Nitrous oxide of global warming.
(c) Chlorofluoro Carbon
51. Which of the following conditions indicate the impact
(d) Hydrogen
of global warming? Select the correct answer from
U.P.P.C.S. (Mains) 2012
codes given below:
Ans. (d)
1. Melting of glaciers
Hydrogen is not a greenhouse gas. 2. Rise in sea level
49. As a result of global warming there has been: 3. Changes in weather conditions
1. Better global communication 4. Rise in global temperature
2. Melting of glaciers Codes :
3. Flowering of mango trees before times (a) 1 and 2 (b) 1, 2 and 3
4. Adverse impact on health (c) 2, 3 and 4 (d) All the above
Select your answer from the code given below: I.A.S. (Pre) 2009
Codes : Ans. (d)
(a) 1, 2 and 3 are correct All the four mentioned above indicate the impact of global
(b) 2, 3 and 4 are correct warming. The main gases responsible for global warming
(c) 1, 2 and 4 are correct are water vapour, CO2, CH4 and CFC.
(d) 1, 3 and 4 are correct
52. Global Warming is caused due to the emission of:
U.P.P.C.S. (Pre) 2003
(a) Nitrogen (b) Carbon Dioxide
U.P.U.D.A/L.D.A. (Pre) 2002
(c) Carbon Mono-oxide (d) Hydro Carbon
U.P.P.C.S.(Pre) 2001
U.P.P.C.S. (Mains) 2004
Ans. (b)
Ans. (b)
As a result of global warming, glaciers are melting
worldwide. Species that depend on one another may become Global warming is primarily a problem of too much carbon
out of sync. For example, plants could bloom earlier than dioxide in the atmosphere. This carbon overload is caused
their pollinating insects become active. Global warming mainly when we burn fossil fuels like coal, oil and gas or
would have an adverse impact on human health. The main cut down and burn forests. The presence of carbon dioxide
categories of health risks include (i) Direct-acting effects (ii) in the atmosphere was 315 PPM (Parts per million) in 1950.
Impacts mediated via climate-related changes in ecological Currently, it has crossed 415 PPM. If the increase of carbon
systems and relationship (eg. crop fields, mosquito ecology, dioxide continues in the atmosphere at this speed, it will reach
marine productivity). approximately 650-950 PPM by the end of the 21st century.
This will raise the earth's temperature by 60C.
50. Which of the following are the impacts of Global
Warming? 53. The apprehension of Global Warming is aggravated
1. Rise in sea level by increased concentration, in the atmosphere, of –
2. Melting of glaciers (a) Carbon dioxide (b) Nitrous oxide
3. Spread of diseases (c) Ozone (d) Sulphur dioxide

Environment and Ecology General Studies F–93


U.P.P.C.S. (Spl) (Mains) 2008 burning fossil fuels) have begun to warm earth’s climate at
U.P.P.C.S. (Pre) 2006 a problematic rate.
U.P.P.C.S. (Mains) 2004 57. Consider the following:
Ans. (a) 1. Rice fields 2. Coal mining
See the explanation of the above question. 3. Domestic animals 4. Wetlands
54. “Global Warming” is due to: Which of the above is a source of methane, a major
(a) Heavy rainfall greenhouse gas?

(b) Increase in carbon dioxide (a) 1 and 4 (b) 2 and 3

(c) Decrease in carbon dioxide (c) 1, 2 and 3 (d) 1, 2, 3 and 4

(d) Increase in forestation I.A.S. (Pre) 2008

U.P. Lower Sub. (Mains) 2013 Ans. (d)

Ans. (b) Rice fields, coal mines and domestic animals are human
sources of methane emissions in the atmosphere while
See the explanation of the above question.
wetlands and sea hydrates are natural sources of methane
55. Which one of the following is the most fragile ecosystem emission. Maximum (about 76% of the total methane
emission) from natural sources is emitted through the wetland
that will be first affected by global warming?
only.
(a) The Arctic and Greenland Ice sheet
58. The gas which is produced by Paddy fields and is
(b) Amazon rain forest
involved in global warming is?
(c) Taiga (a) Chlorine (b) Methane
(d) Indian Monsoon (c) Carbon dioxide (d) Hydrogen sulphide
Uttarakhand P.C.S. (Pre) 2010 U.P. P.C.S. (Mains) 2017
Ans. (a) Ans. (b)

The Arctic and Greenland ice sheet are the most fragile See the explanation of the above question.
ecosystem that will be first affected by global warming. The
59. The gas, which is emitted in the paddy fields and
melting of ice in this area will also affect the entire earth.
increases the earth's temperature is:
56. The increasing amount of carbon dioxide in the air (a) Nitrogen (b) Carbon dioxide
is slowly raising the temperature of the atmosphere (c) Carbon monoxide (d) Methane
because it absorbs: U.P.P.C.S. (Pre) 2019
(a) The water vapour of the air and retains its heat Ans. (d)

(b) The ultraviolet part of the solar radiation See the explanation of the above question.
(c) All the solar radiations 60. Methane is emitted from which of the following?
(d) The infrared part of the solar radiation (a) Paddy fields only (b) Termitary only
I.A.S. (Pre) 2012 (c) Both form (a) and (b) (d) None of the above
Ans. (d) U.P. P.C.S. (Pre) 2018
Ans. (c)
The molecules of carbon dioxide in the atmosphere can
Methane (CH4) is emitted by natural resources such as
absorb 90% of the infrared radiation received from Sun in
wetlands as well as human activities.
the form of solar energy. This ability to absorb and re-emit
Natural Resources - Wetlands are the largest source, emitting
infrared energy is what makes carbon dioxide effective heat-
CH4 from bacteria that decompose organic materials in the
trapping greenhouse gas. In recent years, however, excess absence of oxygen. Smaller sources include termites, oceans,
emissions of carbon dioxide from human activities (mostly sediments, volcanoes and wildfires.

F–94 General Studies Environment and Ecology


Human Induced - 1. It is an initiative of UNEP and UNESCO.
a. Agriculture - Domestic live stocks such as cattle, produce 2. It is a movement in which the participants switch
large amounts of CH4. Paddy cultivation and storage of off the lights for one hour on a certain day every
manure in lagoons also releases methane. Globally, the year.
agriculture sector is the primary source of CH4 emission. 3. It is a movement to raise awareness about climate
b. Industry - Methane is the primary component of natural
change and the need to save the planet.
gas.
Which of the statements given above is/are correct?
c. Waste from Home and Businesses - Methane is generated
from the decomposition of waste in landfills. (a) 1 and 3 only (b) 2 only
(c) 2 and 3 only (d) 1, 2 and 3
61. Consider the following :
I.A.S. (Pre) 2014
1. Carbon monoxide 2. Methane
Ans. (c)
3. Ozone 4. Sulphur dioxide
Which of the above are released into atmosphere due Earth Hour is a worldwide movement for the planet organized
by the World Wide Fund for Nature (WWF). The event is held
to the burning of crop/biomass residue?
worldwide annually encouraging individuals, communities,
(a) 1 and 2 only (b) 2, 3 and 4 only
households and businesses to turn off their non-essential
(c) 1 and 4 only (d) 1,2,3 and 4 lights for one hour, from 8.30 to 9.30 p.m. towards the end
I.A.S. (Pre) 2019 of March, as a symbol of their commitment to the planet. It
Ans. (d) was famously started as a lights-off event in Sydney, Australia
in 2007. In 2022, it is scheduled to celebrate Earth Hour on
After harvesting the leftover crop residues are disposed for
March 26.
land clearing and pest control. The cheapest and easiest
method to dispose these crop/biomass residues is to burn 65. Which reference to the 'New York Declaration on
it. However, burning of these crop/biomass residues release Forest', which of the following statements are correct?
Carbon monoxide (CO), Sulphur dioxide (SO2), Methane
1. It was first endorsed at the United Nations Climate
(CH4) and Ozone (O3) like gases in atmosphere. Hence option
Summit in 2014.
(d) is correct answer.
2. It endorses a global timeline to end the loss of
62. Which human activity is most affected by climate?
forests.
(a) Fishing (b) Mining
3. It is a legally binding international declaration.
(c) Manufacturing (d) Farming
4. It is endorsed by governments, big companies and
Jharkhand P.C.S. (Pre) 2013
indigenous communities.
Ans. (d)
5. India was one of the signatories at its inception.
Agriculture is most affected by climate change. Changes in
Select the correct answer using the code given below.
climate may also impact the water availability and water
needs for agriculture. Due to the increase in temperature, the (a) 1, 2 and 4 (b) 1, 3 and 5
humidity of soil and its productivity is affected. It also raises (c) 3 and 4 (d) 2 and 5
the salinity of soil and reduces productivity. I.A.S (Pre) 2021
63. The greenhouse gas liberated by cud-chewing animals is– Ans. (a)
(a) Carbon dioxide (b) Methane The New York Declaration on Forests is a voluntary and
(c) C.F.C. (d) Nitrous oxide non-legally binding political declaration which grew
Uttarakhand P.C.S. (Pre) 2005 out of dialogue among governments, companies and
Ans. (b) civil society, spurred by the United Nations Secretary-
Methane (CH4) is also called Marsh Gas whose source is General’s Climate Summit held in New York in 2014.
biogas, bacterial decomposition and cud-chewing animals. For the first time, world leaders endorse a global timeline to cut
This is a greenhouse gas which pollutes the air. natural forest loss in half by 2020, and strive to end it by 2030.
64. Consider the following statements regarding ‘Earth It also calls for restoring forests and croplands of an
Hour’: area larger than India. Meeting these goals would cut

Environment and Ecology General Studies F–95


between 4.5 and 8.8 billion tons of carbon pollution every Reason (R) : The frequency and intensity of the
year – about as much as the current emissions of the United extreme weather events will have
States. serious repercussions on food security.
The Declaration is currently endorsed by over 190 entities Choose the correct answer from codes given below:
including more than 50 governments, more than 50 of the
Codes :
world’s biggest companies, and more than 50 influential civil
society and indigenous organizations. It is not endorsed by (a) Both (A) and (R) are true and (R) is the correct reason
India. of (A).
66. The United Nations Convention on Climate Change (b) Both (A) and (R) are true but (R) is not the correct
ratified by more than 50 countries became effective on – reason of (A).
(a) March 21, 1994 (b) May 21, 1995 (c) (A) is true but (R) is false.
(c) June 21, 1996 (d) June 21, 1999 (d) (A) is false but (R) is true.
U.P.P.C.S.(Pre) 2012 U.P.P.C.S. (Pre) 2017
Ans. (a) Ans. (a)

The United Nations Framework Convention on Climate Climate change is increasing the pressure on resources.
Change (UNFCCC) was negotiated at the Earth Summit There are many evidences available which prove that
in Rio-de-Janeiro from 3 to 14 June, 1992, and came into climate change has an adverse impact on both production
force on 21 March, 1994. Currently, it has been ratified by and productivity of agriculture and forestry. Therefore, the
197 countries. possibility of a more complex situation in the future cannot
be denied. The agriculture sector is most affected by the
67. The place of India among 58 Countries of the world as
extreme weather conditions which raise the question of food
per 2017 Climate Change Performance Index (CCPI) security. In the case of food insecurity, there is a tendency for
published by the German watch is: control over unnecessary food storage which ultimately leads
(a) 23rd (b) 22nd to social stress. Thus food insecurity is also a major factor of
(c) 21st (d) 20th social stress. In this way assertion and reason both are correct
and the reason is also making the correct interpretation of
U.P. P.C.S. (Mains) 2017
the assertion.
Ans. (d)
In 2017, India was placed at the 20th rank among the 58
countries on the Climate Change Performance Index (CCPI).
High-Level Questions
The index was released by Germanwatch, an independent 1. What is ‘Greenhouse Gas Protocol’?
German NGO. India was ranked at 14th and 11th in the year (a) It is an international accounting tool for government
2018 and 2019 respectively. and business leaders to understand, quantify and
India ranked 10th in CCPI-2021, which covers the year 2020.
manage greenhouse gas emissions
68. When was India's first National Action Plan on Climate (b) It is an initiative of the United Nations to offer
Change released? financial incentives to developing countries to reduce
(a) 2000 (b) 2008 greenhouse gas emissions and to adopt eco-friendly
(c) 2012 (d) 2015 technologies
U.P. P.C.S. (Pre) 2018 (c) It is an inter-governmental agreement ratified by all
Ans. (b) the member countries of the United Nations to reduce
India released its first National Action Plan on Climate greenhouse gas emissions to specified levels by the
Change to mitigate and adapt to climate change on 30 June, year 2022
2008. (d) It is one of the multilateral REDD + initiatives hosted
69. Given below are two statements: by the World Bank
Assertion (A) : Climate change is leading to an increase I.A.S. (Pre) 2016
in social tension in India. Ans. (a)

F–96 General Studies Environment and Ecology


The Greenhouse Gas Protocol (GHG Protocol) is the most (c) 2 and 3 only (d) 1, 2 and 3
widely used international accounting tool for Government I.A.S. (Pre) 2016
and business leaders to understand, quantify and manage Ans. (b)
greenhouse gas emission. A decade-long partnership
between the World Resource Institute (WRI) and the A historic agreement to combat climate change and unleash
World Business Council for sustainable development, the actions and investment towards a low carbon, resilient and
GHG Protocol is working with businesses, governments sustainable future was agreed by 194 nations at UNFCCC
and environmental groups around the world to build a (United Nations Framework Convention on Climate Change)
new generation of credible and effective programs for in Paris on 12 December, 2015. Hence statement 1 is not
tackling climate change. Hence (a) is the correct answer. correct. The universal agreement’s main aim is to keep global
2. Consider the following statements: temperature rise this century well below 2 degrees Celsius
and to drive efforts to limit the temperature increase even
(i) Kyoto Protocol came into force in the year 2005.
further to 1.5 degrees Celsius above pre-industrial levels.
(ii) Kyoto protocol primarily deals with reducing
Hence statement 2 is correct. Governments decided that they
overall greenhouse emissions. will work to define a clear roadmap on raising climate finance
(iii) Carbon dioxide is five times more effective as a to USD 100 billion by 2020. Hence statement 3 is not correct.
greenhouse gas than methane.
4. In the context of India's preparation for Climate-smart
Which of the above statements is/are correct?
Agriculture, consider the following statements :
(a) (i) and (ii) (b) (ii) and (iii)
1. The 'Climate-Smart Village' approach in India
(c) (i) only (d) (iii) only
is a part of a project led by the Climate Change,
R.A.S./R.T.S. (Pre) 2013
Agriculture and Food Security (CCAFS), and
Ans. (a)
international research programme.
The Kyoto Protocol, an international agreement under the
2. The Project of CCAFS is carried out under
United Nations Framework Convention on climate change
was adopted on 11 December, 1997 in Kyoto, Japan. Kyoto Consultative Group on International Agricultural
Protocol came into effect on 16 February, 2005. It was adopted Research (CGIAR) headquartered in France.
as a binding agreement to reduce greenhouse gas emission to 3. The International Crops Research Institute for the
the level of 1990. It had provision to reduce by 5.2%. Methane Semi-Arid Tropics (ICRISAT) in India is one of the
is about 86 times more power as greenhouse than CO2.
CGIAR's research centres.
3. With reference to the Agreement at the UNFCCC Which of the statements given above are correct?
meeting in Paris in 2015, which of the following (a) 1 and 2 only (b) 2 and 3 only
statements is/are correct? (c) 1 and 3 only (d) 1, 2 and 3
1. The Agreement was signed by all the member I.A.S (Pre) 2021
countries of the UN and it will go into effect in 2017. Ans. (d)
2. The Agreement aims to limit the greenhouse
Rajapakar is a Climate-Smart Village (CSV), a part of a
gas emissions so that the rise in average global
project led by the CGIAR Research Program on Climate
temperature by the end of this century does not Change, Agriculture and Food Security (CCAFS). Built on
exceed 2oC or 1.5oC above pre-industrial levels. the principles of CSA, this program provides technological,
3. Developed countries acknowledged their historical organizational and systemic support to farmers in association
responsibility in global warming and committed with institutions to help them cope with climate change, in
to donate $ 1000 billion a year from 2020 to help 36 sites across 20 countries.
The governance and management of the CGIAR Research
developing countries to cope with climate change.
Program on Climate Change, Agriculture and Food
Select the correct answer using codes given below:
Security (CCAFS) are based on lessons learned by CGIAR
(a) 1 and 3 only (b) 2 only headquartered in France.

Environment and Ecology General Studies F–97


The International crop Research Institute for the Semi-Arid (b) Carbon Credit Rating
Tropics (ICRISAT) has been awarded the 2021 Africa food (c) Clean Development Mechanism
Prize. ICRISAT, a CGIAR Research Centre, is a non-profit, (d) Emission Reduction Norm
non-political public international research organization that I.A.S. (Pre) 2009
conducts agricultural research for development in Asia and Ans. (c)
Sub-Saharan Africa with a wide array of partners throughout Carbon trading is an approach used to control carbon
the world. dioxide (CO2) pollution by providing economic incentives
5. Which of the following countries does not have a global for achieving emissions reductions. In the context of
atmosphere watch station to collect data on world CO2 emission and Global warming, Clean Development
temperatures? Mechanism (CDM) is the name of a market-driven device
(a) Algeria (b) Brazil under the UNFCC that allows developing countries to get
(c) Kenya (d) India funds or incentives from the developed countries to adopt
U.P.P.C.S.(Pre) 2012 better technologies that reduce greenhouse gas emissions.
Ans. (d) 8. Consider the following statements:
The Global Atmosphere Watch (GAW) program of the World 1. Clean Development Mechanism (CDM) in respect
Meteorological Organization (WMO) promotes systematic of carbon credits in one of the Kyoto Protocol
and reliable observations of the global atmospheric Mechanisms.
environment. Currently GAW coordinates activities and 2. Under the CDM, the projects handled pertain only
data from 32 global stations, but at present in India, there is to the Annex-I countries.
no GAW station. Which of the statements given above is/are correct?
6. Which of the following statement is not true about (a) 1 only (b) 2 only
C.D.M.? (c) Both 1 and 2 (d) Neither 1 nor 2
(a) It controls Greenhouse emission. I.A.S. (Pre) 2008
(b) It reduces global warming. Ans. (a)
(c) Kyoto Protocol suggests for assessment of the The Clean Development Mechanism (CDM), defined in
sustainable development of it. Article 12 of the Protocol, allows a country with an emission
(d) It prohibits developed countries from investing in reduction or emission limitation commitment under the
projects in developing countries. Kyoto Protocol (Annex B Party) to implement an emission
U.P.U.D.A./L.D.A. (Pre) 2001 reduction project in developing countries. Such projects can
Ans. (d) earn saleable certified emission reduction (CER) credits,
each equivalent to one ton of CO2, which can be counted
The Clean Development Mechanism (CDM) is one of
towards meeting Kyoto targets. The CDM was intended to
the flexible mechanisms defined in the Kyoto Protocol
meet two objectives:
that provides for emissions reduction projects. The CDM
(1) To assist parties not included in Annex I in achieving
is one of the Protocol’s “project-based” mechanisms and
sustainable development and in contributing to the
designed to promote projects by developed countries among
ultimate objective of the United Nations Framework
developing countries that reduce emissions. This can help
Convention on Climate Change (UNFCCC), which is
developed countries getting carbon credit. Thus statement
to prevent dangerous climate change; and
(d) is incorrect.
(2) To assist parties included in Annex I in achieving
7. In the context of CO2 Emission and Global Warming, compliance with their quantified emission limitation
what is the name of a market-driven device under the and reduction commitments (greenhouse gas (GHG)
UNFCC that allows developing countries to get funds/ emission caps).
incentives from the developed countries to adopt better “Annex I” parties are the countries listed in Annex I of the
technologies that reduce greenhouse gas emissions? treaty, the industrialized countries. Non-Annex I parties are
(a) Carbon Footprint developing countries. Thus, projects under the clean

F–98 General Studies Environment and Ecology


Development Mechanism are related to 'Annex 1' and 'non- grow and maintain the urban forests.
Annex-1' countries. Which one of the following is correct in respect of the
above statements?
9. Which of the following is not true for India to combat
(a) Both Statement 1 and Statement 2 are correct and
the adverse impact of Climate Change?
Statement 2 is the correct explanation for Statement 1
(a) Establishment of the Clean Development Mechanism
(b) Both Statement 1 and Statement 2 are correct
(CDM)
but Statement 2 is not the correct explanation for
(b) Investment of Rs. 100 crores as an initial corpus under
Statement 1
National Adaptation Fund
(c) Statement 1 is correct by Statement 2 is not correct
(c) Not attended the 19th Meeting of BASIC at Sun City,
(d) Statement 1 is not correct but Statement 2 is correct
South Africa held recently
I.A.S (Pre) 2021
(d) All the above are true
Ans. (d)
U.P.P.C.S. (Pre) 2017
Ans. (c) Tree Cities of the World Programme is an international effort
to recognize cities and towns committed to ensuring that their
The Clean Development Mechanism (CDM) is defined
urban forests and trees are properly maintained, sustainably
in Article-12 of Kyoto Protocol of UNFCCC. It allows a
country with an emission-reduction or emission-limitation managed, and duly celebrated.
commitment under the Kyoto Protocol (Annex-B Party) The Arbor Day Foundation is a nonprofit conservation
to implement an emission-reduction project in developing and education organization founded in 1972 in Nebraska,
countries. Such projects earn saleable certified emission United States, by John Rosenow. It is the largest nonprofit
reduction (CER) credits, each equivalent to one tonne membership organization dedicated to tree planting.
of CO2 which can be counted towards meeting Kyoto Hyderabad is placed alongside 119 other cities from 63
targets. The National Adaptation Fund for Climate Change countries.Hyderabad has become the only city in India to
(NAFCC) was established in August 2015 to meet the cost
be recognized as a Tree City of the World by the Arbor Day
of adaptation to climate change for the States and Union
Foundation and the Food and Agriculture Organization (FAO)
Territories of India that are particularly vulnerable to the
adverse effects of climate change. There is no relation of the United Nations. Hence, Statement 1 is not correct.
between adverse effect of climate change and not attending 12. Consider the following pairs:
the 19th meeting of BASIC at Sun City, South Africa. Terms sometimes Their origin
10. Which of the following is not correctly matched? seen in the news
(a) First World Climate Conference - 1979 1. Annex-I Countries : Cartagena Protocol
(b) First Earth Summit - Agenda-21 2. Certified Emission : Nagoya Protocol
(c) Earth Summit Plus 5 - 1997 Reductions
(d) Carbon Trading - Montreal Protocol 3. Clean Development : Kyoto Protocol
Mechanism
M.P.P.C.S. (Pre) 2017
Which of the pairs given above is/are correctly
U.P.P.C.S. (Pre) (Re. Exam) 2015
matched?
Ans. (d)
(a) 1 and 2 only (b) 2 and 3 only
Carbon Trading is related to the Kyoto protocol. Thus option
(c) 3 only (d) 1, 2 and 3
(d) is not correctly matched.
I.A.S. (Pre) 2016
11. Consider the following statements: Ans. (c)
Statements 1 : The United Nations Capital Development
Annex-I countries participated in the 1st Kyoto protocol.
Fund (UNCDF) and the Arbor Day Foundation have
37 Annex-I countries and the EU agreed to second-round
recently recognized Hyderabad as 2020 Tree City of Kyoto targets. Certified Emission Reductions (CER) are
the world. a type of emissions unit (or carbon credits) issued by the
Statements 2 : Hyderabad was selected for the Clean Development Mechanism (CDM), Executive Board
recognition for a year following its commitment to for emission reductions achieved by CDM projects and

Environment and Ecology General Studies F–99


verified by a DOE (Designated Operational Entity) under 15. Which of the following statements best describes
the rules of the Kyoto Protocol. Cartagena Protocol is a "carbon fertilization"?
supplementary protocol of the Convention on Biological (a) Increased plant growth due to increased concentration
Diversity (CBD). It is committed to preserving bio-diversity of carbon dioxide in the atmosphere.
from Live Modified Organisms-LMO caused by the adoption (b) Increased temperature of Earth due to increased
of biotechnology. Cartagena protocol was adopted on 29 concentration of carbon dioxide in the atmosphere.
January, 2000. It came into effect on 11 September, 2003.
(c) Increased acidity of oceans as a result of increased
Nagoya Protocol came into effect on 12 October, 2014. It
concentration of carbon dioxide in the atmosphere.
is also a supplementary protocol of CBD. It is concerned
with access to genetic resources and fair distribution (d) Adaptation of all living beings on Earth to the climate
of benefits from them. Hence (c) is the correct answer. change brought about by the increased concentration
of carbon dioxide in the atmosphere.
13. In the context of mitigating the impending global
I.A.S. (Pre) 2018
warming due to anthropogenic emissions of carbon
dioxide, which of the following can be the potential Ans. (a)
sites for carbon sequestration? The carbon fertilization effect suggests that the increase
1. Abandoned and uneconomic coal seams of carbon dioxide in the atmosphere increases the rate of
2. Depleted oil and gas reservoirs photosynthesis in plants. The effect varies depending on the
3. Subterranean deep saline formations plant species, the temperature and the availability of water
Select the correct answer using codes given below: and nutrients. From a quarter to half of Earth's vegetated
(a) 1 and 2 only (b) 3 only lands has shown significant greening over the last 35 years
(c) 1 and 3 only (d) 1, 2 and 3 largely due to rising levels of atmospheric carbon dioxide.
I.A.S. (Pre) 2017
Ans. (d) 16. Regarding ‘carbon credits’ which one of the following
statements is not correct?
Carbon sequestration is the process of capturing and storing
(a) The carbon credit system was ratified in conjunction
atmospheric carbon in plants, geologic structures and sea. In
with the Kyoto Protocol
the context of mitigating the impending global warming due
to anthropogenic emissions of carbon dioxide, abandoned and (b) Carbon credits are awarded to countries or groups that
uneconomical coal seams, depleted oil and gas reservoirs, and have reduced greenhouse gases below their emission
subterranean deep saline formations could be the potential quota
sites for carbon sequestration. (c) The goal of the carbon credit system is to limit the
14. In rural road construction, the use of which of the increase of carbon dioxide emission
following is preferred for ensuring environmental (d) Carbon credits are traded at a price fixed from time to
sustainability or to reduce carbon footprint? time by the United Nations Environment Programme
1. Copper slag I.A.S. (Pre) 2011
2. Cold mix asphalt technology Ans. (d)
3. Geotextiles The Kyoto Protocol established caps on the maximum
4. Hot mix asphalt technology quantity of greenhouse gas emissions. These countries set
5. Portland cement internal quotas on emissions from installations countries
Select the correct answer using the code given below : run by local business and other organizations, generically
(a) 1, 2 and 3 only (b) 2, 3 and 4 only termed ‘operators’. Countries oversee this responsibility
(c) 4 and 5 only (d) 1 and 5 only through their own national ‘registries’ which are required to
I.A.S. (Pre.) 2020 be validated and monitored for compliance by the UNFCCC.
Ans. (a) Each operator is allocated an allowance of credits. Each
unit gives the owner the right to emit one metric ton of
In rural road construction, the use of copper slag, cold mix CO2. Operators that have not met their quotas can sell their
asphalt technology and Geotextiles are preferred for ensuring unused allowances as carbon credits, while businesses
environmental sustainability or to reduce carbon footprint. that are about to exceed their quotas can buy the extra

F–100 General Studies Environment and Ecology


allowances as credits, privately or on the open market. Reduce Short-Lived Climate Pollutants is a unique
Businesses alter their decision-making to find the most cost- initiative of the G20 group of countries.
effective way of operating under these regulations, either 2. The CCAC focuses on methane black carbon and
by investing in cleaner business practices or by purchasing hydrofluorocarbons.
credits from another operator with excess capacity. Which of the statements given above is/are correct?
17. The concept of carbon credit' originated from (a) 1 only (b) 2 only
(a) Earth Summit, Rio-de-Janeiro (c) Both 1 and 2 (d) Neither 1 nor 2
(b) Kyoto protocol I.A.S. (Pre) 2017
(c) Montreal protocolo Ans. (b)
(d) G-8 Summit, Heiligendam The Climate and Clean Air Coalition (CCAC) is a voluntary
U.P.P.C.S. (Pre) 2021 partnership of governments, inter-governmental organizations,
Ans. (b) business, scientific institutions and civil society organizations
The concept of carbon credit' originated from Kyoto protocol. committed to improving air quality and protecting the climate
through actions to reduce short-lived climate pollutants. It is
A carbon credit is a permit that allows the company that
a coalition formed by 53 countries and various institutions.
holds it to emit a certain amount of carbon dioxide or other
CCAC mainly focuses on methane, black carbon and
greenhouse gases. One credit permits the emission of a mass hydrofluorocarbons (HFCs). Along with spreading awareness
equal to one ton of carbon dioxide. about the effects of short-lived climate pollutants, the
The Kyoto Protocol was adopted on 11 December 1997. coalition also works for its mitigation strategy and to promote
Owing to a complex ratification process, it entered into force new operations at the regional and national level. Along with
on 16 February 2005. this, the allies of the coalition also recognize the fact that
minimization of short-lived climate pollutants is capable of
Currently, there are 192 Parties to the Kyoto Protocol
working as supplementary and supplement in global efforts
In short, the Kyoto Protocol operationalizes the United to minimize carbon dioxide.
Nations Framework Convention on Climate Change by
20. Concerning ‘Forest Carbon Partnership Facility’,
committing industrialized countries and economies in
which of the following statements is/are correct?
transition to limit and reduce greenhouse gases (GHG)
1. It is a global partnership of Governments,
emissions in accordance with agreed individual targets.
businesses, civil society and indigenous people
The Convention itself only asks those countries to
2. It provides financial aid to universities, individual
adopt policies and measures on mitigation and
scientists and institutions involved in scientific
to report periodically. forestry research to develop eco-friendly and
18. BioCarbon Fund Initiative for Sustainable Forest climate adaptation technologies for sustainable
Landscapes is managed by the: forest management
(a) Asian Development Bank 3. It assists the countries in their ‘REDD+ (Reducing
(b) International Monetary Fund Emission from Deforestation and Forest
(c) United Nations Environment Programme Degradation+)’ efforts by providing them with
(d) World Bank financial and technical assistance.
I.A.S. (Pre) 2015 Select the correct answer using codes given below:
Ans. (d) (a) 1 only (b) 2 and 3 only
The Bio Carbon Fund Initiative for Sustainable Forest (c) 1 and 3 only (d) 1, 2 and 3
Landscapes (ISFL) is a multilateral fund, supported by donor I.A.S. (Pre) 2015
Governments and managed by the World Bank. Norway has Ans. (c)
committed maximum $ 135 million, UK $ 120 Million and The Forest Carbon Partnership Facility (FCPF) is a World
the USA $25 million for this fund. This fund seeks to promote Bank program which was established in 2008 and consists
a reduction in Greenhouse gases from the land sector. of a Readiness Fund and a Carbon Fund. The FCPF was
19. Consider the following statements: created to assist developing countries to reduce emissions
1. Climate and Clean Air Coalition (CCAC) to from deforestation and forest degradation, enhance and

Environment and Ecology General Studies F–101


conserve forest carbon stocks and sustainably manage Without green house effect, the average temperature of earth
forests (REDD+) by providing value to standing forests. So, surface would be –18°C. The main green house gasses are
statement 2 is false as it does not provide funds to universities, Methane, Carbon dioxide, Nitrous Oxide, Water vapor and
individuals or institutes. Ozone.

21. The scientific view is that the increase in global 24. Human activities in the recent past have caused the
temperature should not exceed 20C above the pre- increased concentration of carbon dioxide in the
industrial level. If the global temperature increases atmosphere, but a lot of it does not remain in the lower
beyond 3 °C above the pre-industrial level, what can atmosphere because of:
be its possible impact/impacts on the world? 1. Its escape into the outer stratosphere
2. The photosynthesis by phytoplankton in the oceans.
1. Terrestrial biosphere tends toward a net carbon
3. The trapping of air in the polar ice caps.
source.
Which of the statements given above is/are correct?
2. Widespread coral mortality will occur.
(a) 1 and 2
3. All the global wetlands will permanently disappear.
(b) 2 only
4. Cultivation of cereals will not be possible anywhere
(c) 2 and 3
in the world.
(d) 3 only
Select the correct answer using the code given below:
I.A.S. (Pre) 2011
(a) 1 only (b) 1 and 2 only
Ans. (b)
(c) 2, 3 and 4 only (d) 1, 2, 3 and 4
Human activities in the recent past have caused the increased
I.A.S. (Pre) 2014
concentration of carbon dioxide in the atmosphere but a lot
Ans. (b)
of it does not remain in the lower atmosphere because of the
According to a report of “Assessing Key Vulnerabilities and photosynthesis by phytoplankton in the oceans and plants on
the Risk from Climate Change” published by International the land, in which carbon dioxide is absorbed.
Panel on Climate Change, if the increase in global temperature
25. What would happen if phytoplankton of an ocean is
exceeds 2 0C above pre-industrial level then Coral mortality
completely destroyed for some reason?
will occur around the globe. It will also result in the
1. The ocean as a carbon sink would be adversely
transformation of 1/6th of the earth's ecosystem and lead to
the extinction of 1/4th known species of the earth. If the earth's affected
temperature increases more than 30C above the pre-industrial 2. The food chains in the ocean would be adversely
level, the terrestrial biosphere will head towards a net carbon affected.
source and 1/5th of the ecosystem will get transformed. This 3. The density of ocean water would drastically
will lead to the extinction of 30% of the earth's known species. decrease.
22. Which of the following represents the average increase Select the correct answer using the code given below:
(a) 1 and 2
in earth’s temperature during the last century?
(b) Only 2
(a) 0.6 degree Celsius (b) 0.7 degree Celsius
(c) Only 3
(c) 0.8 degree Celsius (d) 0.9 degree Celsius
(d) 1, 2 and 3
U.P.P.C.S.(Pre) 2012
I.A.S. (Pre) 2012
Ans. (c)
Ans. (a)
The average increase in earth's temperature during the last
century was 0.80C. If phytoplankton of an ocean is completely destroyed for
some reason then the ocean as a carbon sink would be
23. Without green house effect, the average temperature adversely affected. Phytoplankton, also known as microalgae,
of earth surface would be: are similar to terrestrial plants in that they contain chlorophyll
(a) 0°C (b) – 18°C and require sunlight to live and grow. Hence food chains in
(c) 5°C (d) – 20°C the ocean would be adversely affected and if it is completely
U.P.P.C.S. (Pre) 2020 destroyed then the density of ocean water would not decrease
drastically.
Ans. (b)

F–102 General Studies Environment and Ecology


26. Which of the following is not related to the astronomical rate is declining due to excessive deforestation. Therefore,
theories of climate change? carbon concentrations are increasing in the atmosphere which
(a) Eccentricity of earth’s orbit is affecting the climate by increasing the greenhouse effect.
(b) Obliquity of earth’s rotational axis NASA and other studies do not consider the solar flare as a
(c) Precession of Equinoxes major cause of climate change. Thus, option (b) is correct.
(d) Solar irradiance
29. Which one of the following provides the cryogenic
U.P.P.C.S. (Pre) (Re. Exam) 2015
indicator of climate change?
Ans. (d)
(a) Ice core
Astronomical theory of climate change was propounded (b) Fossilized pollen
in 1920 by astronomer Milutin Milankovic. It states that (c) Tree ring growth
the variation in eccentricity, axial tilt and precession of the (d) Evaporite Deposits
Earth’s orbit determines climatic patterns on Earth through
U.P.P.C.S. (Pre) (Re Exam.) 2015
orbital forcing. Hence, solar irradiance is not concerned with
Ans. (a)
this theory.
An ice core is a core sample that is typically removed
27. Who had suggested that shifting Earth on its axis is
from an ice sheet, most commonly from the polar ice caps
one of the factors, responsible for the climatic change?
of Antarctica-Greenland or from high mountain glaciers
(a) Robert Hooke
elsewhere. As the ice forms the incremental building of
(b) Milutin Milankovic
annual layers of snow, lower layers are older than upper and
(c) George Simpson
an ice core contains ice formed over a range of years. The
(d) T. C. Chamberlain
properties of the ice and the recrystallized clarification needed
U.P.P.C.S. (Mains) 2015
inclusions within the ice can then be used to reconstruct a
Ans. (b)
climatic record over the age range of the core, normally
According to the astronomical theory of climate change, through isotopic analysis. This enables the reconstruction
Earth's movement produces an effect on climate change. of local temperature records and the history of atmospheric
In 1920, Serbian geophysicist and astronomer Milutin composition.
Milankovic theorized that variation in eccentricity, axial tilt
and precession of the earth's orbit results in cyclical variation 30. With reference to 'Global Climate Change Alliance',
in the solar radiation reaching the earth. This orbital forcing which of the following statements is/are correct?
influences climatic pattern on earth. 1. It is an initiative of the European Union.
2. It provides technical and financial support to
28. Which of the following are the primary causes of targeted developing countries to integrate climate
climate change? change into their development policies and
1. Excessive burning of fossil fuels. budgets.
2. Exploding numbers of automobiles driven by oil 3. It is coordinated by the World Resources Institute
3. Increasing solar flares (WRI) and World Business Council for Sustainable
4. Heavy deforestation Development (WBCSD).
Select the correct answer using codes given below: Select the correct answer using codes given below:
Codes : (a) 1 and 2 only
(a) 2 and 3 only (b) 1, 2 and 4 only (b) 3 only
(c) 1, 2, 3 and 4 (d) 1 and 4 only (c) 2 and 3 only
U.P.P.C.S. (Pre) 2017 (d) 1, 2 and 3
Ans. (b) I.A.S. (Pre)2017
There are two primary causes of climate change– factors that Ans. (a)
increase carbon emission and factors that increase carbon The GCCA was established by the European Union (EU) in
absorption. The causes of increase in carbon emission include 2007 to strengthen dialogue and cooperation with developing
excessive burning of the fossil fuels, exploding numbers of countries, in particular, least developed countries (LDCs) and
automobiles driven by oil etc., while the carbon absorption small island developing states (SIDS). It started its work in

Environment and Ecology General Studies F–103


just 4 countries. Today it has a budget of more than £ 300 (a) Solar Power
million and is one of the most significant climate initiatives (b) Afforestation
in the world. It supports 51 programmes around the world
(c) Nuclear power
and is active in 38 countries, 8 regions and sub-regions and at
the global level. It completes its goal through two interactive (d) Waste to energy conversion
strong pillars. These include providing technical and U.P.P.C.S. (Mains) 2016
financial assistance to targeted developing countries for the
Ans. (c)
integration of climate change in their development policies
and budgets. The overall aim of GCCA is to create a new See the explanation of the above question.
treaty between the European Union and the poor developing
countries, who have the least potential to deal with climate 34. When was Jharkhand Action Plan on Climate Change
change. It is coordinated by the European Commission (EC). published?
31. India’s first National Action Plan on Climate Change (a) 2013 (b) 2014
was released in: (c) 2015 (d) 2011
(a) 2008 A.D.
Jharkhand P.C.S. (Pre) 2016
(b) 2012 A.D.
(c) 2014 A.D. Ans. (a)
(d) 2015 A.D. Supported by UNDP, the state government of Jharkhand
U.P.P.C.S. (Pre) 2016 initiated the process to develop Jharkhand Action Plan on
Ans. (a) Climate Change. On May, 2011 the State Steering Group and
India's first National Action Plan on Climate Change was State Advisory Group (SAG) were formed through a state
released in June, 2008 by the then Prime Minister, Manmohan government notification. The Department of Environment
Singh. and Forests led the preparation of JAPCC in consultation
with various departments. On 23rd December 2013 final
32. Which of the following is not listed under eight action
public consultation meeting was organized at Jamshedpur
of the climate action plan of Government of India?
for sharing of the draft. JAPCC shared it with concerned
(a) Solar Energy
departments and the general public for comments and
(b) Atomic Energy
feedback. JAPCC endeavours reframing development
(c) Waste Energy Conversion
pathways with low carbon growth, at the same time ensuring
(d) Afforestation that development opportunities are protected, supported and
U.P.U.D.A./L.D.A. (Spl) (Mains) 2010 increased.
Ans. (b)
35. According to Jharkhand Action Plan on Climate
Climate Action Plan of Government of India was launched
on 30 June, 2008. Eight National Mission under this Action Change Report (2014) which is the most sensitive
Plan is as below: District?
1. National Solar Mission (a) E. Singhbhum
2. National Mission for Enhanced Energy Efficiency
3. National Mission on Sustainable Habitat (b) Saraikela Kharaswan
4. National Water Mission (c) Ranchi
5. National Mission for Sustaining the Himalaya Ecosystem (d) Bokaro
6. National Mission for Green India
Jharkhand P.C.S. (Pre) 2016
7. National Mission for Sustainable Agriculture
8. National Mission on Strategic Knowledge for Climate Ans. (b)
Change
According to Jharkhand Plan on Climate Change Report
Atomic energy or Nuclear Power is not included in this.
(2014) Saraikela Kharsawan is the most sensitive district
33. Which one of the following is not included in Eight with the score of 0.78 which shows its vulnerability. The
Missions under India’s National Action Plan on measurement is prepared on the scale of (–1) to (+1), which
Climate Change? shows respective vulnerability.

F–104 General Studies Environment and Ecology


36. Which of the following statements regarding global Ans. (d)
warming are correct? Methane hydrate is a crystalline solid which contains a
1. Melting of polar ice caps and subsequent rise in Methane molecule surrounded by water molecules. Methane
the sea level is the most important effect of global hydrate usually exist in form of 'Ice'. Global warming might
trigger the release of methane gas from these deposits.
warming
Large deposits of 'methane hydrate' have been found in Arctic
2. The sea-level is likely to rise by one meter by 2070 Tundra and under the seafloor.
A.D if the present level of global warming is not Methane is relatively shortlived in the atmosphere. It oxidizes
controlled in atmosphere into Carbon dioxide after a decade or two. It
3. All the coral islands in the world will be submerged also contributes into green house effect on the earth. Hence
option (d) is the correct answer.
4. By 2044 A.D, Fiji is likely to be submerged and
the rise in the sea level by the same year will pose 38. Scientists of Manchester University recently suggested
a grave danger to the Netherlands controlling global warming by cloud brightening over
Codes : the Pacific Ocean through geo-engineering. Which of
(a) 1, 2, 3 and 4 the following substances is used for this?
(b) 1, 2 and 3 (a) Silver salt
(c) 1, 3 and 4 (b) Iron powder
(d) 4 only (c) Sea water
Uttarakhand P.C.S. (Pre) 2002 (d) Gypsum
Ans. (c) U.P.P.C.S. (Mains) 2009
According to the Intergovernmental Panel on Climate Ans. (c)
Change, a rise in temperature between 1.4-5.8o C has been
Scientists of Manchester University have suggested using sea
estimated between 1990 to 2100 A.D. This would result in
water for creating brightening clouds for maximum reflection
melting polar ice caps and subsequent rise in sea level. As
of sun rays. This will help control global warming.
per IPCC, the sea level is expected to rise from 0.33 m to
0.45m from 1990 to 2100 A.D. Hence statement (1) is correct 39. Consider the following agricultural practices
while (2) is not correct. The rise in sea level is estimated to 1. Contour bunding
be 3 mm per year. At Fiji, its 0.15 meter per year. Hence by 2. Relay cropping
2044, Fiji is likely to be submerged.
3. Zero tillage
37. Which of the following statements are correct about In the context of global climate change, which of the
the deposits of 'methane hydrate'? above helps/help in carbon sequestration/storage in
1. Global warming might trigger the release of the soil?
methane gas from these deposits. (a) 1 and 2 (b) Only 3
2. Large deposits of 'methane hydrate' are found in
(c) 1, 2 and 3 (d) None of these
Arctic Tundra and under the seafloor.
I.A.S. (Pre) 2012
3. Methane in atmosphere oxidizes to carbon dioxide
Ans. (c)
after a decade or two.
Select the correct answer using codes given below: Any method which helps the soil to retain more organic matter
so that it may work as an effective Carbon sink would be
(a) 1 and 2 only
helpful in Carbon sequestration or storage. Contour bunding
(b) 2 and 3 only
is one of the extensively used soil and water conservation
(c) 1 and 3 only technique in several rainfed areas. Contour bunding is a
(d) 1, 2 and 3 Mechanical measure which minimizes the soil erosion.
I.A.S. (Pre) 2019 Relay cropping means that a new crop is planted or sown

Environment and Ecology General Studies F–105


before the previous one is harvested. This can provide 42. Which of the following statements is incorrect for
advantages for both crops as one of them may provide ‘Green Climate Fund” (GCF)?
nitrogen, shade, support or may discourage pests. The (a) It was established in the Durban Conference on
obvious benefit of Relay cropping is soil conservation. Climate change.
Zero tillage is also based on the premise of soil conservation, (b) It would support programs and policies in developing
the basic premise is to minimize the disturbances to the soil nations.
leading to an increase in retention of water, nutrients and
(c) The developed nations would provide the fund.
topsoil itself. Zero tillage has carbon sequestration potential
(d) The fund will start operating from, 2014.
through storage of soil organic matter in the soil of crop fields.
U.P.P.C.S. (Mains) 2013
Thus all the three help in carbon sequestration.
Ans. (a)
40. The United Nations Framework Convention on
Climate Change (UNFCCC) is an international treaty Green Climate Fund (GCF) was formally established during
drawn at: the 2010 United Nations Climate Change Conference in
Cancun, Mexico. It is a fund within the framework of the
(a) United Nations Conference on the Human
UNFCCC founded as a mechanism to assist developing
Environment, Stockholm, 1972
countries in adaptation and mitigation practices to counter
(b) UN Conference on Environment and Development,
climate change. The GCF is based in the New Songdo district
Rio-de-Janeiro, 1992
of Incheon, South Korea. It is intended to be the centrepiece
(c) World Summit on Sustainable Development,
of efforts to raise climate finance of $ 100 billion a year by
Johannesburg, 2002 2020.
(d) UN Climate Change Conference, Copenhagen, 2009
43. Which of the following statements regarding the
I.A.S. (Pre) 2010
Ans. (b) ‘Green Climate Fund’ is/are correct?
1. It is intended to assist the developing countries
The United Nations Framework Convention on Climate
Change (UNFCCC) is an international environmental treaty in adaptation and mitigation practices to counter
adopted at the United Nations Conference on Environment climate change.
and Development (UNCED), informally known as the Earth 2. It is founded under the aegis of UNEP, OECD,
Summit, held in Rio-de-Janeiro from 3 to 14 June, 1992. Asian Development Bank and World Bank.
41. 'Intended Nationally Determined Contribution' some Select the correct answer using codes given below:
time seen in news refers to – (a) 1 only (b) 2 only
(a) A promise made by European countries for (c) Both 1 and 2 (d) Neither 1 nor 2
rehabilitation of refugees from Middle east. I.A.S. (Pre) 2015
(b) World Action Plan to deal with climate change Ans. (a)
(c) Equity shareholding of member countries in AIIB
Green Climate Fund (GCF) was founded as a mechanism
(d) World action plan for sustainable development goals.
to assist developing countries in adaptation and mitigation
I.A.S. (Pre) 2016
practices to counter climate change. The fund is governed
Ans. (b)
by GCF Board. The board consists of 24 members. Hence
Countries of the world adopted a historic international climate statement (2) is not correct.
agreement at the U.N. framework convention on Climate
44. Which one of the following countries is the first country
Change Conference of Parties (COP21) in Paris in December
2015. Countries publicly outlined post -2020 climate action in the world to propose a carbon tax for its people to
they intended to take under this agreement. This was known address global warming?
as Intended Nationally Determined Contribution (INDCs). (a) Australia

F–106 General Studies Environment and Ecology


(b) Germany may be contributing to global warming. To what
(c) Japan possible reason/reasons is/are this attributable?
(d) New Zealand 1. The anaerobic conditions associated with rice
I.A.S. (Pre) 2006 cultivation cause the emission of methane.
Ans. (d) 2. When nitrogen-based fertilizers are used, nitrous
A Carbon Tax is a fee intended to make users of fossil oxide is emitted from the cultivated soil.
fuels pay for climate damage by their fuel use whereas Which of the statements given above is/are correct?
Sweden, Finland, the Netherlands and Norway have already (a) 1 only
introduced Carbon Tax. NewZealand, in 2005, introduced (b) 2 only
for the first time, a Carbon Tax to address global warming.
(c) Both 1 and 2
45. With reference to Deep Carbon Observatory (DCO) (d) Neither 1 nor 2
which of the following statements is/are correct? I.A.S. (Pre) 2010
1. It is a Global Research Programme to outreach role Ans. (c)
of Carbon on Earth.
The anaerobic conditions associated with rice cultivation
2. It conducts field observations of deep microbial cause the emission of methane. When nitrogen-based
ecosystem. fertilizers are used, nitrous oxide is emitted from the
Select the correct answer using codes given below: cultivated soil. Hence both statements given above are
(a) 1 only correct.
(b) 2 only
47. Which of the following statements are true regarding
(c) Both 1 and 2
the members of the Asia Pacific Partnership?
(d) Neither 1 nor 2
1. Their population account to 45% of the world’s
U.P.P.C.S. (Pre) 2019
population.
Ans. (c)
2. They consume the world’s 48% of energy.
The Deep Carbon Observatory (DCO) is a global community 3. They contribute nearly 48% of the world’s
of more than 1000 scientists on a ten-year quest to understand
greenhouse gasses.
the quantities, movements, forms and origins of carbon in
4. They intend to support the Kyoto Protocol.
Earth. From 2009-2019, DCO science was categorized into
following four broad theme-based communities- Select the correct answer from codes given below:
Extreme Physics and Chemistry- Dedicated to improving Codes :
our understanding of the physical and chemical behaviour of (a) 1 and 2 only
carbon at extreme conditions, as found in the deep interiors (b) 1, 2 and 3 only
of Earth and other planets. (c) 2, 3 and 4 only
Reservoirs and Fluxes - Dedicated to identifying deep
(d) All of the above
carbon reservoirs, determining how carbon moves among
U.P.P.C.S. (Mains) 2005
these reservoirs, and assessing Earth's total carbon budget.
Ans. (c)
Deep Energy - Dedicated to understanding the volume and
rates of abiogenic hydrocarbons and other organic species in The population of Asia Pacific Partnership countries is 41%
the crust and mantle through geological time. of the world’s population. They consume the world’s 48% of
Deep Life - Dedicated to assessing the nature and extent of energy and contribute nearly 48% of the world’s greenhouse
the deep microbial and viral biosphere. gases. They intend to support the Kyoto Protocol however
some of the countries have not ratified it.
46. Due to their extensive rice cultivation, some regions

Environment and Ecology General Studies F–107


Ozone Layer
*Ozone (O3) consists of the three atoms of the oxygen. Ozone varies seasonally. Its thickness is highest during spring season
gas is found in very less amount in the atmosphere. *Ozone is and lowest during the rainy season.
a pollutant on the surface of the earth i.e. at the lower levels of *The measuring unit of the ozone layer is "Dobson Unit" (DU).
the atmosphere it is harmful to the living organism, whereas at One DU is equal to 0.01 mm thickness of pure Ozone at 0oC
the higher levels of the atmosphere it protects the Earth from temperature and 1 atm pressure. The total mass of Ozone in
the ultraviolet (UV) radiations of the sun. atmosphere is about 3 billion metric tons.
Ozone Layer Depletion
*British team first discovered ozone hole over Antarctica by
using "total ozone mapping spectrometer" in 1985.
*CFC has the potential to destroy a large amount of ozone
by catalyzing the breakdown of ozone (O3) into oxygen (O2).
CFC is one of the major causes of the greenhouse effect.
Chlorofluorocarbons (CFC's) is a compound of man-made
chemicals, which is non-toxic, non-flammable, colourless and
odourless and easily converted to the fluid. It is a highly stable
compound and can sustain in the atmosphere for 80 to 100 years.
*It is used in refrigerators and air conditioners as a coolant.
*Since the 1960s, use of chlorofluorocarbon is increasing due
to the usage of refrigerators, air conditioners, spray cases,
solvents, foam constructions, pressed toiletries, micro-marine
works in industries, cleaning electronic components and other
applications.

*The earth’s atmosphere is divided into several layers and each


layer plays an important role.
*The layer extending about 10-50 Km is called the stratosphere.
The ozone layer is mainly found in the lower portion of
the stratosphere from approximately 15 to 30 kilometres
above the Earth, though the thickness varies seasonally and
geographically.
*The ozone layer protects the Earth from the sun UV rays.
*10% of the ozone layer is found in the troposphere and 90%
in the stratosphere.
*The lowest region on the atmosphere is called troposphere.
The thickness of the troposphere is maximum at the equator,
deepest in the tropics up to 18 to 20 km and shallowest near
the polar regions.
*The ozone layer in the stratosphere is naturaly regulated by
Nitrogen dioxide (NO2). The thickness of the ozone layer

F–108 General Studies Environment and Ecology


*The gases filled in the refrigerator are sold as Mafron. *The United Nation’s International Day for the Preservation
*Ammonia is also used as a coolant in big plants. of the Ozone Layer is observed on September 16 every year
*Ozone layer in the atmosphere prevents harmful ultraviolet since 1994.
rays from reaching to the Earth's surface. It thus safeguards
*The International Day of the preservation of the Ozone layer
life on Earth.
was celebrated on the 16th September 2020 with the theme
*Gases responsible for the depletion of the ozone layer are–
“Ozone for Life : 35 years of ozone layer protection”.
CFC, Halon - 1211,1301, Nitrous oxide and Tricloroethlyn
etc. Effects of Ozone Layer Depletion
*Ozone formation takes place at a very slow pace in the polar *The depletion of Ozone Layer has a very wide impact on
regions of the atmosphere. Therefore, the effect of Ozone layer the Earth. Harmful ultraviolet radiation coming from the Sun
depletion is maximum over the poles. causes skin cancer. DNA may have a genetic mutation if the
*Nitric Acid in the Polar stratospheric clouds reacts with
ultraviolet radiations from the Sun are absorbed by the skin for
chlorofluorocarbon. This reaction produces chlorine which is
long time. The absorption of the Ultra-Violet radiations by the
responsible for photo-chemical destruction of the ozone layer.
skin in larger amount can cause skin cancer.
Since stratospheric clouds help chlorine compound to convert
into chlorine atom depleting Ozone layer, their presence is also
a factor responsible for the formation of the ozone hole.

Measures for the Preservation of


the Ozone Layer
*A research team, led by G.W. Kent Moore from the University
of Toronto, Canada, discovered the Ozone Halo over the Tibetan
Plateau in 2005. The scientists have discovered the presence of Harmful effects of UV-B radiation
excess ozone around Tibetan Plateau. Ozone levels were low on the organism
over the centre of the Tibetan Plateau and a concentrated ring
*The ultraviolet radiation of the Sun is divided into three
of ozone sits around the Plateau. Ozone is a highly volatile gas.
regions called UV-A (315-400 nm) UV-B (280-315 nm)
At high concentrations, it can cause coughing, chest pain and
and UV-C (100-280 nm). Over exposure to UV-A and UV-B
damage to the lungs.
radiations not only can cause Sunburn but also some forms of
*The Montreal Protocol on substances that deplete the ozone
Skin Cancer. UV-C is also very harmful but UV-C is blocked
layer is an international treaty designed for the protection of the
by diatomic oxygen or by ozone in the atmosphere that is why
ozone layer by phasing out the production of various substances
they can’t reach the Earth’s surface.
that are responsible for ozone depletion. Chlorofluorocarbons,
*Ozone Layer blocks high wavelength ultraviolet light from
Halons and Carbon Tetrachloride are the main ozone-depleting
the Sun by 93% to 99% which are harmful to the living beings
substances.
on the Earth.
*It is a protocol to the Vienna Convention for the protection
of the ozone layer by phasing out the production or the use *Ozone is a poisonous gas, it causes direct and immediate harm
of the substances (such as chlorofluorocarbons etc) that are to the lungs and entire respiratory system. It is harmful to the
responsible for ozone depletion. biological life on the surface of the Earth.
*The Montreal Protocol was signed on 16 September 1987 *In the absence of the ozone layer, the Sun’s ultraviolet rays
and came into force on 1 January 1989. will cause excessive damage to biological life on Earth.

Environment and Ecology General Studies F–109


6. Which is the lowest layer of the Atmosphere?
Basic Questions (a) Stratosphere (b) Ozonosphere
1. Place where Ozone Layer is primarily found is – (c) Ionosphere (d) Troposphere
(a) Troposphere (b) Stratosphere Uttarakhand P.C.S. (Pre) 2016
(c) Mesosphere (d) Ionosphere Ans. (d)
U.P.P.C.S. (Pre) (Spl) 2008 The lowest region in the atmosphere is called troposphere.
Ans. (b) The thickness of the troposphere is maximum at the equator,
deepest in the tropics up to 18 to 20 km and shallowest
The earth’s atmosphere is divided into several layers and
each layer plays an important role. The layer extending near the polar regions. After this come to the stratosphere,
about 10-50 km is called the stratosphere. The ozone layer mesosphere, thermosphere and exosphere respectively.
is mainly found in the lower portion of the stratosphere 7. Which of the following is not true about chlorofluoro-
from approximately 15 to 30 kilometres above the Earth, carbon?
though the thickness varies seasonally and geographically. (a) It is used as a refrigerant.
The ozone layer protects the Earth from the sun UV rays. (b) Is not responsible for the greenhouse effect
10% of the ozone layer is found in the troposphere and 90% (c) It is responsible for reducing ozone in the stratosphere.
in the stratosphere. (d) It is inactive in the lower atmosphere
2. The maximum concentration of ozone is found in which U.P. Lower Sub. (Pre) 2004
of the following? Ans. (b)
(a) Troposhpere (b) Mesosphere Chlorofluorocarbons is an organic compound that contains
(c) Stratosphere (d) Exosphere carbon, chlorine and fluorine. One of the elements that make
U.P. P.C.S. (Pre) 2018 CFC is chlorine. Under certain conditions, this chlorine has
Ans. (c) the potential to destroy a large amount of ozone. Hence it is
See the explanation of the above question. one of the major causes of the greenhouse effect.

3. The ozone layer is located in – 8. Which is responsible for Ozone Hole?


(a) Troposphere (b) Stratosphere (a) CO2 (b) SO2
(c) Mesosphere (d) Ionosphere (c) O2 (d) CFC
U.P.U.D.A./L.D.A. (Spl) (Mains) 2010 (e) None of these
Ans. (b) Chhattisgarh P.C.S. (Pre) 2014
Ans. (d)
See the explanation of the above question.
CFC is responsible for the ozone hole.
4. Ozone layer is above the earth's crust around –
(a) 50 km. (b) 300 km. 9. The radiation that is absorbed by ozone present in the
(c) 2000 km. (d) 20 km. atmosphere is –
R.A.S./R.T.S.(Pre) 2012 (a) Infrared (b) Visible
Ans. (d) (c) Ultraviolet (d) Microwave
U.P.P.C.S.(Pre) 2013
The Ozone layer is mainly found in stratosphere from
Ans. (c)
approximately 15 to 30 km above the Earth's surface. It's
density varies seasonally and geographically. Ozone (O3) is a molecule formed by three atoms of oxygen.
While O2, which we normally refer to as oxygen, is essential
5. The ozone layer is found mainly in –
for all aerobic forms of life. Ozone is a deadly poison.
(a) Troposphere (b) Mesosphere
However, at the higher levels of the atmosphere, ozone
(c) Stratosphere (d) Exosphere
performs an essential function. It shields the surface of the
U.P.P.C.S. (Mains) 2012
Earth from ultraviolet (UV) radiation from the Sun. This
Ans. (c)
radiation is highly damaging to organisms and is known to
See the explanation of the above question. cause skin cancer in human beings.

F–110 General Studies Environment and Ecology


10. Ultraviolet radiation from sunlight causes the reaction 15. Harmful ultraviolet radiation coming from the sun can
that produce which of the following? cause –
(a) C O (b) SO2 (a) Liver cancer (b) Brain cancer
(c) O3 (d) Fluorides (c) Oral cancer (d) Skin cancer
U.P. P.C.S. (Pre) 2018 U.P.P.C.S. (Pre) 2014
Ans. (c) Ans. (d)

Stratospheric ozone is formed naturally by chemical reactions The ultraviolet radiation from the sun can cause skin cancer.
involving solar ultraviolet radiation and oxygen molecules. Exposure to ultraviolet rays for a long time can cause genetic
mutation which can cause skin cancer. The Ultraviolet rays
11. Ozone protects the biosphere from – of the sun are categorised as UV-A, UV-B and UV-C. First
(a) Infra-red rays (b) Ultraviolet rays two types of ultraviolet rays are harmful to the skin. UV-C
(c) X-rays (d) Gama rays is extremely dangerous however it does not reach the earth's
U.P.P.C.S. (Pre) 2014 surface.
Ans. (b)
16. “Ozone Layer Preservation Day” is celebrated on –
See the explanation of the above question. (a) 16 September (b) 5 June
(c) 23 March (d) 21 April
12. Ozone layer in atmosphere –
M.P.P.C.S. (Pre) 2014
(a) Produces rain
Ans. (a)
(b) Produces pollution
(c) Provides safety to life on earth from ultraviolet In 1994, the UN General Assembly declared 16 September as
radiation the International Day for the Preservation of the Ozone Layer,
(d) Produces oxygen in the atmosphere commemorating the date of the signing, of the Montreal
U.P.P.C.S. (Pre) 2006 Protocol on Substances that deplete the Ozone Layer in 1987.
U.P. Lower Sub. (Pre) 2004 17. The International Day for Preservation of Ozone layer
Ans. (c) is observed on:
(a) 15th August (b) 16th September
See the explanation of the above question.
(c) 24th October (d) 1st May
13. Ozone layers is useful to humanity because– U.P.P.C.S. (Pre) 1996
(a) It supplies oxygen to the atmosphere Ans. (b)
(b) It prevents the Sun’s ultraviolet rays from reaching 16 th September is observed as International Day for
the Earth. Preservation of Ozone Layer. The theme of World Ozone Day
(c) It controls the temperature of the Earth. 2020 is - 'Ozone for Life : 35 years of ozone layer protection'.
(d) None of these.
18. World Ozone day is celebrated on –
M.P.P.C.S. (Pre) 1994
(a) September, 16 (b) April, 21
Ans. (b)
(c) December, 25 (d) January, 30
See the explanation of the above question. U.P.P.C.S. (Pre) 2014
Ans. (a)
14. The ozone layer of atmosphere absorbs –
(a) Cosmic rays See the explanation of the above question.
(b) Infrared rays 19. World Ozone day is celebrated on –
(c) Ultraviolet rays (a) September, 16 (b) December, 1
(d) All radiations (c) March, 30 (d) April, 22
Uttarakhand P.C.S. (Pre) 2006 U.P.P.C.S. (Pre) 2015
Ans. (c) Ans. (a)
See the explanation of the above question. See the explanation of the above question.

Environment and Ecology General Studies F–111


20. The ozone hole is caused by 25. Which one of the following is not responsible for the
(a) Acetylene (b) Ethylene depletion of ozone layer:
(c) Chlorofluorocarbons (d) Methane (a) CFC-12 used in refrigerators
U.P.P.C.S. (Pre) 2016 (b) Methyl chloroform used as a solvent
Ans. (c) (c) HALON-1211 used in fire fighting
(d) Nitrous oxide
Ozone depletion occurs when chlorofluorocarbons (CFC) are U.P.P.C.S. (Pre) 2002
released into the atmosphere. These gases, through several U.P. Lower Sub. (Pre) 2002
chemical reactions, cause the ozone molecules to break down, Ans. (b)
reducing ozone’s ultraviolet radiation absorbing capacity. The ozone layer in the atmosphere prevents harmful
21. Which one of the following gases is responsible for the ultraviolet rays from reaching the Earth's surface. It thus
decrement in the ozone layer? safeguards life on Earth. Gases responsible for the depletion
of the ozone layer are– CFC, Halon - 1211,1301, Nitrous
(a) Nitrous Oxide (b) Chlorofluorocarbon
oxide and Tricloroethlyn etc.
(c) Carbon dioxide (d) Carbon Mono Oxide
U.P.R.O./A.R.O. (Pre) 2014 26. Among the following greenhouse gases, which does not
Ans. (b) cause ozone depletion in the troposphere?
(a) Methane (b) Carbon monooxide
See the explanation of the above question. (c) Nitrogen Oxides (NO) (d) Water vapours
22. Which of the following gas is responsible for the U.P.P.C.S. (Pre) 2008
depletion of Ozone Layer? Ans. (b)
(a) Carbon dioxide (b) Carbon monooxide The greenhouses gases responsible for ozone depletion in the
(c) Nitrous oxide (d) Chlorofluorocarbon troposphere are- Water vapour, Carbon dioxide, Methane,
42nd B.P.S.C. (Pre) 1997 Ozone, Nitrous oxide and Chlorofluorocarbon.
Uttarakhand P.C.S. (Pre) 2005 27. Formation of the ozone hole is maximum over –
Ans. (d) (a) India (b) Africa
(c) Antarctica (d) Europe
See the explanation of the above question.
M.P.P.C.S. (Pre) 2008
23. The Pollutants causing maximum damage to the ozone Ans. (c)
layer are –
The ozone hole is not technically a “hole” where no ozone
(a) Hydrocarbon (b) Carbon dioxide is present but is a region of exceptionally depleted ozone in
(c) Nitrous Oxides (d) Chlorofluorocarbon the stratosphere over Antarctica.
M.P.P.C.S. (Pre) 2015
28. Montreal Protocol is related to the protection of:
Ans. (d)
(a) Greenhouse gases
See the explanation of the above question. (b) Acid rain
(c) Ozone layer
24. The presence of which of the following degrades ozone
(d) Endangered species
from ozonosphere?
Uttarakhand P.C.S. (Pre) 2010
(a) Carbon monooxide
Ans. (c)
(b) Carbon dioxide
(c) Chlorofluorocarbons The Montreal Protocol on substances that deplete the Ozone
(d) Nitrogen Layer (a protocol to the Vienna Convention for the Protection
of the Ozone Layer) is an international treaty designed to
U.P.P.S.C. (GIC) 2009
protect the ozone layer by phasing out the production of
Ans. (c)
numerous substances that are responsible for ozone depletion.
See the explanation of the above question. It came into force on 1 January, 1989.

F–112 General Studies Environment and Ecology


29. The Montreal Protocol is related with: (c) 1, 3 and 4 (d) 1, 2, 3 and 4
(a) To protect the Ozone layer depletion I.A.S. (Pre) 2012
(b) Global warming Ans. (c)
(c) Acid rain Chlorofluorocarbons (CFCs) are nontoxic, nonflammable
(d) Photochemical smog chemicals containing atoms of carbon, chlorine, and fluorine.
(e) None of the these They are used in the manufacture of aerosol sprays, blowing
Chhattisgarh P.C.S. (Pre) 2015 agents for foams and packing materials, as solvents, and as
Ans. (a) refrigerants. Trichlorofluoromethane can dissolve grease
See the explanation of the above question. and used to clean electronic equipment. It is a highly stable
30. Montreal Protocol is related to? compound and can sustain in the atmosphere for 80 to 100
(a) White Lion (b) Chlorofluorocarbon years. Hence option (C) is the correct answer.
(b) Water pollution (d) Agriculture
3. Which of the following substances is/are ozone
M.P.P.C.S. (Pre) 2014
depleting?
Ans. (b)
Select the correct answer from codes given below:
The Montreal Protocol is related to protect the Ozone 1. Chlorofluorocarbons
layer which is depleted mainly by carbon dioxide and 2. Halans
chlorofluorocarbons. 3. Carbon tetrachloride
Codes :
High-Level Questions (a) 1 only (b) 1 and 2 only
(c) 1 and 3 only (d) 1, 2 and 3
1. In stratosphere, the level of ozone is naturally regulated
U.P.P.C.S.(Pre) 2012
by –
Ans. (d)
(a) Nitrous oxide
(b) Nitrogen dioxide Chlorofluorocarbons, Halans and Carbon tetrachloride, all are
(c) CFC ozone-depleting substances. All these substances are banned
(d) Water vapours after the Montreal Protocol.
U.P.P.C.S. (Mains) 2016 4. The formation of the ozone hole in the Antarctic region
Ans. (b) has been a cause of concern. What could be the reason
The ozone layer in the stratosphere is naturally regulated by for the formation of this hole?
Nitrogen dioxide. The thickness of the ozone layer varies (a) Presence of prominent tropospheric turbulence and
seasonally. Its thickness is highest during spring and lowest inflow of chlorofluorocarbons
during rain. The measurement unit of the ozone layer is (b) Presence of prominent polar front and stratospheric
'Dobson Unit' (DU). 1 DU is equal to 0.01 mm thickness of clouds and inflow of chlorofluorocarbons
pure ozone at 0oC temperature and 1 atm pressure. (c) Absence of polar front and stratospheric clouds and
inflow of methane and chlorofluorocarbons
2. Consider the following statements:
(d) Increased temperature in the polar region due to global
Chlorofluorocarbons, known as ozone-depleting
warming.
substances, are used
I.A.S. (Pre) 2011
1. In the production of plastic foams.
Ans. (b)
2. In the production of tubeless tires.
3. In cleaning certain electronic components. Nitric Acid in the Polar stratospheric clouds reacts with
4. As pressurizing agents in aerosol cans. clorofluorocarbon. This reaction produces chlorine which
Which of the statements given above is/are correct? is responsible for photo-chemical destruction of the ozone
(a) 1, 2 and 3 (b) Only 4 layer. Since stratospheric clouds help chlorine compound to

Environment and Ecology General Studies F–113


convert in chlorine particles depleting the ozone layer, their (b) Both (A) and (R) are true but (R) is not the correct

presence is also a factor responsible for the formation of the explanation of (A)
ozone hole. (c) (A) is true but (R) is false
(d) (A) is false but (R) is true
5. Which gas is filled in refrigerators?
U.P. U.D.A./L.D.A. (Mains) 2010
(a) Ammonia (b) Mafron
U.P.P.C.S. (Mains) 2010
(c) Methane (d) Acetylene
Ans. (a)
Uttarakhand P.C.S. (Pre) 2010
Ans. (b) The ozone layer is a layer in Earth’s atmosphere which
contains relatively high concentrations of ozone (O3). This
The gases filled in the refrigerator are sold as Mafron. These
layer absorbs 93-99% of the sun’s high-frequency ultraviolet
gases are a commonly halonic hydrocarbon. Although,
Ammonia is also used as a coolant in big plants, the correct light, which is potentially detrimental to life on Earth. Hence
answer to this question is option (b). both Assertion and Reason are true and (R) is the correct
explanation of (A). However, if ozone is understood as a gas
6. Which one of the following teams of scientists first
only, it is harmful to biotic life for being poisonous.
discovered ‘ozone hole’ over Antarctica?
(a) Russian Team
(b) German Team
(c) American Team Figure: Atomic structure of ozone
(d) British Team
9. Which of the following is not correctly matched?
U.P.P.C.S. (Mains) 2013
(a) Ozone - Chlorofluorocarbon (CFC)
Ans. (d)
(b) Acid rain - Nitric Acid
British team first discovered the ozone hole over Antarctica (c) Rocket fuel - Kerosene Oil
in 1985 using 'total ozone mapping spectrometer'. (d) Green House Effect - Carbon dioxide
7. Who among the following had discovered ‘ozone halo’ Uttarakhand P.C.S. (Mains) 2002
over Tibetan Plateau in 2005? Ans. (b)
(a) M. Molina Clorofluorocarbon is the main cause of ozone depletion.
(b) Joseph Farman Sulphur dioxide and oxides of nitrogen cause acid rain not
(c) G.W. Kent Moore
nitric acid. Rocket fuel is highly refined kerosene. Carbon
(d) Marcus Rex
dioxide is the second largest cause of greenhouse effect after
U.P.P.C.S. (Mains) 2013
water vapour.
Ans. (c)
10. Which one of the following is associated with the issue
In the leadership of G.W. Kent Moore, University of Toronto,
of control and phasing out of the use of ozone-depleting
in 2005, the researchers discovered ozone hole over Tibetan
substances?
Plateau.
(a) Bretton Woods Conference
8. Consider the following statements: (b) Montreal Protocol
Assertion (A) : Ozone is essential for biotic life. (c) Kyoto Protocol
Reason (R) : The ozone layer protects the earth’s
(d) Nagoya Protocol
surface from high energy radiation.
I.A.S. (Pre) 2015
Select the correct answer from codes given below:
Ans. (b)
(a) Both (A) and (R) are true and (R) is the correct
explanation of (A) See the explanation of the above question.

F–114 General Studies Environment and Ecology


Forest and Wildlife
Forest and their types Tropical Moist deciduous forests. Some of the important tree
species such as Bamboo, Shisham, Sandalwood etc., are
*Various types of forests are found in India. These include
found in Tropical Moist deciduous forests. Tropical deciduous
tropical evergreen forests, tropical deciduous forests, mangrove
forests are also called as the monsoon forests. These are found
forests, etc.
in India, Myanmar, Thailand and areas of south-east Asia. The
*The tropical evergreen forests are found in areas receiving
trees in these forests have a broad leaf which shed their leaves
more than 200 cm of rainfall and having a temperature of 15
once in a year.
to 30 degrees Celsius. They occupy about seven percent of the
*Temperate forests : In mountainous areas, the decrease in
Earth’s land surface and harbour more than half of the world’s
temperature with increasing altitude leads to a corresponding
plants and animals. They are found mostly near the equator.
change in natural vegetation. The Himalayan ranges show a
*In India, evergreen forests are found on the Western Ghat,
succession of vegetation from the tropical to the tundra, which
northeast India and Andaman & Nicobar Islands.
change in with the altitude. Deciduous forests are found in the
*Tropical Rainforest : This biome lies between 23.5ºN and
foothills of the Himalayas. It is succeeded by the wet temperate
23.5ºS. This is the largest biome of the world which covers
type of forests between an altitude of 1,000-2,000 m. In the
about 8% of the surface of the Earth. More than 50% of the
higher hill ranges of northeastern India, hilly areas of West
world’s biodiversity is found in this Biome. Rain continues
Bengal and Uttaranchal, evergreen broad leaf trees such as oak
throughout the year. Due to high temperature and high humidity
and chestnut are predominant. Between 1,500-1,750 m, pine
broad-leaved evergreen trees are dominant. There is so much
forests are also well-developed in this zone, with Chir Pine as
diversity in vegetation life that over 200 species of trees are seen
a very useful commercial tree. Blue pine and spruce appear at
in one-hectare area of the land. The vegetation here generally
altitudes of 2,225-3,048 m.
consists of shade- tolerant shrubs, herbs, small trees and large
*But in the higher reaches there is a transition to Alpine
woody vines which climb into the trees to capture sunlight.
forests and pastures. Silver firs, junipers, pines, birch and
These vines are called Kathalta or Lipana. About 70 to 80%
rhododendrons, etc. occur between 3,000-4,000 m.
of known species of insects are found in Tropical rain forest.
*The southern mountain forests include the forests found in
*Equatorial forests are found in such Tropical zones which
three distinct areas of Peninsular India viz; the Western Ghats,
receive more than 200cm rainfall. Tall, closely set trees
the Vindhyas and the Nilgiris. As they are closer to the tropics,
forming canopy is a characteristic of these forests. Tall and
and only 1,500 m above the sea level, vegetation is temperate in
broad - leaved trees constitute leaf canopy on the surface of
the higher regions, and subtropical on the lower regions of the
the forest. The upper portion of canopy often supports rich
Western Ghats, especially in Kerala, Tamil Nadu and Karnataka.
epiphytes. About 80% of the world’s biodiversity is found in
*Sandalwood is a Tropical deciduous tree which is not found
equatorial forests.
in the Himalayan region. While travelling through Himalayan
*Amazon rain forest is also known as Amazonia or the
region one can find Berg like shrubs growing naturally but the
Amazon forest. It is the moist broad-leaf forest that covers
possibility of growing sandalwood is negligible. Sandalwoods
most of the Amazon basin of South America. It is often called
are found mainly in South India.
the “Lungs of the Planet Earth” because they continuously
*Red Sanders (red Sandalwood) is the species of tree found in
absorb carbon dioxide and breathe out oxygen. More the 20%
the Tropical dry deciduous forests of South India. Its scientific
of Earth’s oxygen is produced by the Amazon Rain forest.
name is Pterocarpus Santalinus. It is found mainly in the
*Tropical Moist deciduous forests are found in areas receiving
Palkonda and Seshachalam hills of Andhra Pradesh. Its wood
100-200 cm of rainfall. Teak (Sagaun) is the main tree of the
Environment and Ecology General Studies F–115
is white in colour which gradually turns red due to exudation of
a red gummy juice. It is mainly exported to China and Japan by
India. It is used in traditional herbal medicine, manufacturing
toys and worshipping materials.
*Texas tree is found naturally in Himalayan regions. Texas
tree is listed in the Red Data Book. Taxol is an anti-cancer
drug which obtained by the Texas tree. This drug is also used
as the cure for Parkinson’s disease.
*Mangrove forest, Evergreen forest and Deciduous forests
are found in the Islands of Andaman and Nicobar. According
to the ISFR 2017, the Mangrove cover area in Andaman and from

Nicobar is 617 km2. Mangroves are found in Tropical and


Subtropical tidal areas. It grows in the saline water near the
coastal areas. These forests protect biodiversity. These forests
are an important link between the sea and the coast and protect
*Potential resources are those which are located in a particular
the coast from the destruction by intense waves coming from
area and can be used in the future whereas Actual resources are
the sea. With this, these forests keep the coastline stable and
those which have been surveyed and their quantity and quality
they protect the coast from erosions.
have been detected and are presently in use. Development of
*Mangrove acts as a reliable safety encloses against coastal
any actual resources depends on the cost and on the availability
disasters such as Tsunami and Cyclones. These trees do not
of technology.
break down by storm and tidal effects due to intense and deep
*Renewable resources are the resources which can be reused.
roots. Odisha coast is the most affected by the cyclones of the
Thus, forest is a renewable resource as it can be reused after
Bay of Bengal. Cyclones hit Odisha coast more frequently as
its sustainable exploitation. Forests enhance the quality of the
compared to any other state. Mangrove forest acts as a barrier
environment as it absorbs carbon dioxide and produces oxygen.
to the cyclones.
*Aluminum is called green metal due to its environment
*Bhitarkanika Mangrove is located on the deltas of Brahimi,
friendly nature because it is renewable.
Vaitarni and Mahanadi in the Kendrapada district of Odisha.
It is famous for mangrove forests. It was declared as a Ramsar Deforestation and Its Impacts
site in 2002. A mangrove commonly refers to two different *Deforestation of forest area in India is directly related to the
things a tidal swamp ecosystem found in tropical deltas, demographic transition of India because the pressure on the
estuaries, lagoons or islands and the characteristic tree species forest area is due to the increase in population. Forest areas are
populating this ecosystem. cut off due to agriculture, housing, urbanization, industry etc.
*The largest area of Mangrove vegetation is found in Sunderban *Forest area and population growth often have a negative
Delta. Sundari Plant in these forests is quite popular. relationship. Generally, population increase in developing
countries has a negative impact on the forest area whereas this
Benefits and Uses of Forest does not imply for the developed countries. An increase in
*Natural resources can be categorized on the basis of forest area is seen in developed countries due to environmental
development stage as : awareness and sensitivity of forests. However, in most of the
(i) Potential Resources countries, population growth affects the forest area.
(ii) Actual Resources *The main cause for the forest loss is industrialization.
(iii) Reserve Resources Deforestation occurs due to the urbanization and empty land
(iv) Stock Resources is being used for agriculture purpose.

F–116 General Studies Environment and Ecology


forest or tree cover. In mountain and hilly regions, the objective
is to maintain two-thirds of the area under such cover to prevent
erosion and land degradation and to ensure the stability of the
fragile ecosystem.
*The National Forest Policy 1988 includes-
(i) Afforestation and wasteland development
(ii) Reforestation and replantation in existing forests
(iii) Encouraging wood's substitute and supplying other types
of fuel.
*Promotion of wide use of insecticides and pesticides is not
included in the National Forest Policy.
*It is noteworthy that Nagaland hills are becoming barren due *Forestation of one-third of total country’s land and
to the use of shifting cultivation by the locals on a large scale. encouragement of public community participation in forest
Forest fire in Jharkhand forests is one of the major components management are main objectives of National Forest Policy.
that affects the climate. These areas include Hazaribagh, The draft of National Forest Policy, 2018 has been cleared by
Jamshedpur, Palamu, Bokaro etc. Forest fire is a major problem various ministeries at a meeting held on November 21, 2019.
of reserved and conserved forests of Jharkhand.
*Urbanization is not the effect of deforestation rather it is one India State of Forest Report, 2021
of the reasons of deforestation. The expansion of residential
*The Ministry of Environment, Forest and Climate Change
areas to meet the needs of growing population impacts
(MOEF&CC) released the biennial India State of Forest Report
forests and biodiversity. Forest areas have shrunk due to the
(ISFR) 2021 on January 13, 2021.
expansion of towns and cities. On the contrary, drying of water
*The ISFR 2021 is the seventeenth report in the series and
resources, loss of biodiversity and soil erosion are the impacts
of deforestation. looks at the growing stock in forests and trees outside forests,
bamboo resources, carbon stock and several other parameters.
*To achieve India’s aim of increasing the additional carbon sink
of 2.5 to 3 billion tonnes CO2 equivalent by 2030, the Nagar
Van Yojana has been introduced.
*This will help increase the tree cover and has been dovetailed
with the second phase of Green India Mission in the next five
years.
*Results of the nation-wide forest cover mapping exercise are
presented on 1:150000 scale in three canopy density classes viz
very dense forest (canopy density > 70%), moderately dense
forest (canopy density 40-70%), open forest (canopy density
10-40%) and scrubs (canopy density of less than 10%).
National Forest Policy (1952) *The increase in the forest cover shown in the report is not
*According to the National Forest Policy (1952), forests have uniform.
been classified as follows - *The forest cover as reported in ISFR includes all patches of
(i) Protected/Conserved Forests land with a tree canopy density of more than 10% and with an
(ii) National Forests area having more than 1 ha, irrespective of land use, ownership
(iii) Village Forests and species of trees.
(iv) Tree Lands
*National Parks are not in the category of forests. Key findings of ISFR 2021
*As per the National Forest Policy, one-third, which is 33% *India has set a target of bringing 33% of its geographical
of the total land area of the country, should be covered under area under forest cover as envisaged in the National Forest
Environment and Ecology General Studies F–117
Policy, 1988. lost their cover - Mizoram (1.03%), Arunachal Pradesh (0.39%),
*This was also one of the key targets enlisted in the Strategy Manipur (1.48 %), Meghalaya (0.43%), and Nagaland (1.88%).
for New India @ 75 document by NITI Aayog (2018). *This decline caused by natural calamities, shifting agriculture
*A total of 17 states/union territories’ have above 33% of the and deforestation will affect the region’s water resources and
geographical area under forest cover. will have an increased effect on landslides, as per the report.
*Out of these states and union territories, five namely *The Northeastern states have been consistently losing forest
Lakshadweep, Mizoram, Andaman & Nicobar Islands, cover, as per the last report published in 2019.
Arunachal Pradesh and Meghalaya have more than 75% forest *Between 2011 and 2019, forest cover of six states, excluding
cover. Assam, had decreased by nearly 18% and the region lost nearly
*Area-wise, Madhya Pradesh has the largest forest cover in the 25,012 sq km of forest cover in the preceding decade.
country followed by Arunachal Pradesh, Chhattisgarh, Odisha *The loss of India’s dense and moderately dense forest is a
and Maharashtra. matter of concern, because the recent amendments to the Forest
*The top five states in terms of forest cover as a percentage Act, 1980 are likely to further make the diversion of forest land
of total geographical area are Mizoram (84.53%), Arunachal for non-forest use easier.
Pradesh (79.33%), Meghalaya (76.00%), Manipur (74.34%)
and Nagaland (73.90%). Increase in forest fires
*Twelve states and union territories namely Manipur, Nagaland, *As per the ISFR 2021, India reported a total of 3,45,989 forest
Tripura, Goa, Kerala, Sikkim, Uttarakhand, Chhattisgarh, Dadra fires from November 2020 to June 2021. This is the highest
& Nagar Haveli and Daman & Diu, Assam, Odisha, have forest recorded in the country for this period so far.
cover between 33% to 75%. *Around 2,58,480 forest fires were reported during the same
time in 2018-19 (Rajya Sabha query response, December 16,
Increase in forest cover 2021) indicating a sharp rise.
*ISFR 2021 has found that there is an increase of 1,540 square *In India, severe fires occur in many forest types, particularly
kilometres (0.22%) of forest cover and 721 sq km (0.76%) of dry deciduous forest, while evergreen, semi-evergreen and
tree cover compared to the 2019 report. montane temperate forests are comparatively less prone. Around
*The total forest cover of the country is 21.71% of the 35.46% of the country’s forest cover has been estimated to be
geographical area; the tree cover is estimated at 2.91% of the prone to frequent forest fires.
geographical area. Thus, the total forest and tree cover comes *Nearly 4% of the country’s forest cover is extremely prone
to 24.62% of the geographical area of the country. to fire, whereas 6% of the forest cover is found to very highly
*Forest cover inside the recorded forest areas/greenwash (RFA/ fire-prone.
GW) has shown a slight increase of 31 sq km whereas there is *Odisha reported the maximum number of fires among all
an increase of 1509 sq km of forest cover outside the RFA/GW states (51,968), followed by Madhya Pradesh (47,795) and
as compared to the previous assessment of 2019. Chhattisgarh (38,106).
*The states with the highest increase in the forest cover are *Uttarakhand recorded the sixth-highest fire counts in the
Andhra Pradesh (647 sq km), Telangana (632 sq km), Odisha country; incidences were up 28.3 times this forest fire season
(537 sq km), Karnataka (155 sq km) and Jharkhand (110 sq km). compared to last.
*The reason for the increase in the forest cover in states like *Among districts, the maximum number of such incidents were
Andhra Pradesh is plantation and agroforestry. observed in Gadchiroli in Maharashtra at 10,577, followed by
*The ISFR 2021 report attributed the improvement in forest Kandhamal in Odisha at 6,156 and Bijapur in Chhattisgarh at
cover to “better conservation measures, protection, afforestation 5,499 incidents.
activities, tree plantation drives and agroforestry”.
Other findings
Decline in forest cover especially in *Forest cover in the hill districts of the country stands at 40.17%
the Northeast of the total geographical area of these districts.
*The Northeast reported the biggest overall loss of forest cover *This has shown a decline of 0.32% in the current assessment.
at 1,020 sq km. *While in the 2019 report, the forest cover in the hill regions
*Though the area has 23.75% of total forest cover, states have had an increase of 544 sq km, the 2021 report shows a decrease

F–118 General Studies Environment and Ecology


of 902 sq km. *The Kawal, Bhadra and the Sundarbans reserves have shown
*The total forest cover in the tribal districts which stood the maximum loss.
at 37.53% of the geographical area of the districts shows a *The total growing stock of wood in the country is estimated
decrease of 655 sq km inside the RFA/GW and an increase of as 6167.50 million cum comprising 4388.15 million cum
600 sq km outside. inside forest areas and 1779.35 million cum outside recorded
*With a total of 4,992 sq km of mangrove cover, the nation has forest areas.
had an increase of 17 sq km over the past two years. *The average growing stock per hectare in the forest has been
*The maximum increase in mangrove cover was reported estimated as 56.6 cum.
in Odisha (8 sq km) followed by Maharashtra (4 sq km) and *Total carbon stock in the country’s forest is estimated to be
Karnataka (3 sq km). 7,204 million tonnes and there is an increase of 79.4 million
*The total bamboo bearing area of the country is estimated as tonnes in the carbon stock of the country as compared to the
149443 sq km.
last assessment of 2019.
*There is a decrease of 10594 sq km in the bamboo bearing
*The annual increase in carbon stock is 39.7 million tonnes.
area as compared to the estimate of ISFR 2019. The bamboo
Soil organic carbon represents the largest pool of carbon stock
forests have increased to 53,336 million culms (stems) from
in forests, which has been estimated at 4010.2 million tonnes.
13,882 million culms in 2019.
*The soil organic carbon contributes 56% to the total forest
*ISFR 2021 assessed forest cover in the tiger reserves for the
carbon stock of the country.
first time.
*The report also estimates that by 2030, 45-64% of Indian forests
*Of 52 tiger reserves, 20 have recorded an increase in forest
cover since 2011 while it decreased in 32 tiger reserves during will be affected by climate change and rising temperatures, and
the same period. forests in all states (except Assam, Meghalaya, Tripura and
*While the tiger reserves have decreased by 22.6 sq km (0.04 Nagaland) will be highly vulnerable climate hot spots.
per cent), the tiger corridors have noted an increase by 37.15 *The report suggests that Ladakh (forest cover 0.1-0.2%) is
sq km (0.32%). likely to be the most affected by climate change.

State/UT wise Forest Cover


Forest Cover in the States & UTs of the country as per the 2021 assessment and change therein with respect to the
previous assessment (2019) has been presented in the Table
Geographical Area 2021 Assessment
State/UT
(G.A.) VDF MDF OF
Andhra Pradesh 1,62,968 1,994 13,929 13,861
Arunachal Pradesh 83,743 21,058 30,176 15,197
Assam 78,438 3,017 9,991 15,304
Bihar 94,163 333 3,286 3,762
Chhattisgarh 1,35,192 7,068 32,279 16,370
Delhi 1,483 6.72 56.60 131.68
Goa 3,702 538 576 1,130
Gujarat 1,96,244 378 5,032 9,516
Haryana 44,212 28 445 1,130
Himachal Pradesh 55,673 3,163 7,100 5,180
Jharkhand 79,716 2,601 9,689 11,431
Karnataka 1,91,791 4,533 20,985 13,212
Kerala 38,852 1,944 9,472 9,837
Madhya Pradesh 3,08,252 6,665 34,209 36,619
Maharashtra 3,07,713 8,734 20,589 21,475
Manipur 22,327 905 6,228 9,465
Meghalaya 22,429 560 9,160 7,326

Environment and Ecology General Studies F–119


Mizoram 21,081 157 5,715 11,948
Nagaland 16,579 1,272 4,449 6,530
Odisha 1,55,707 7,213 20,995 23,948
Punjab 50,362 11 793 1,043
Rajasthan 3,42,239 78 4,369 12,208
Sikkim 7,096 1,102 1,551 688
Tamil Nadu 1,30,060 3,593 11,034 11,792
Telangana 1,12,077 1,624 9,119 10,471
Tripura 10,486 647 5,212 1,863
Uttar Pradesh 2,40,928 2,627 4,029 8,162
Uttarakhand 53,483 5,055 12,768 6,482
West Bengal 88,752 3,037 4,208 9,587
A & N Islands 8,249 5,678 683 383
Chandigarh 114 1.36 13.51 8.01
Dadra & Nagar 602 1.40 85.56 140.79
Haveli and Daman
& Diu
Jammu & Kashmir 4,155 8,117 9,115
Shapefile Area*
(54,624)
2,22,236
Ladakh 2 512 1,758
Shapefile Area*
(1,68,055)
Lakshadweep 30 0.00 16.09 11.01
Puducherry 490 0.00 17.53 35.77
Total 32,87,469 99,779 3,06,890 3,07,120

Area of shapefile provided by Survey of India (August, 2021). Notified geographical areas for individual UTs from sol are
awaited.

Change Percentage
Percentage of Geo- Change in Forest Cover
Total Forest Cover w.r.t. 2019 assess- Scrub
graphical area w.r.t. ISFR 2019
ment
29,784 18.28 647 2.22 8,276
66,431 79.33 -257 -0.39 797
28,312 36.09 -15 -0.05 228
7,381 7.84 75 1.03 236
55,717 41.21 106 0.19 615
195.00 13.15 -0.44 -0.23 0.38
2,244 60.62 7 0.31 0
14,926 7.61 69 0.46 2,828
1,603 3.63 1 0.06 159
15,443 27.73 9 0.06 322
23,721 29.76 110 0.47 584

F–120 General Studies Environment and Ecology


38,730 20.19 155 0.40 4,611
21,253 54.70 109 0.52 30
77,493 25.14 11 0.01 5,457
50,798 16.51 20 0.04 4,247
16,598 74.34 -249 -1.48 1,215
17,046 76.00 -73 -0.43 663
17,820 84.53 -186 -1.03 1
12,251 73.90 -235 -1.88 824
52,156 33.50 537 1.04 4,924
1,847 3.67 -2 -0.11 34
16,655 4.87 25 0.15 4,809
3,341 47.08 -1 -0.03 296
26,419 20.31 55 0.21 758
21,214 18.93 632 3.07 2,911
7,722 73.64 -4 -0.05 33
14,818 6.15 12 0.08 563
24,305 45.44 2 0.01 392
16,832 18.96 -70 -0.41 156
6,744 81.75 1 0.01 1
22.88 20.07 0.85 3.86 0.38
227.75 37.83 0.10 0.04 4.85
21,387 39.15 29 0.14 284
2,272 1.35 18 0.80 279
27.10 90.33 0.00 0.00 0.00
53.30 10.88 0.89 1.70 0.00
7,13,789 21.71 1,540 0.22 46,539

Change in Forest Cover of States/UTs between 2019 and 2021 assessments


Geographical Area 2019 Assessment
State/UT
(G.A.) VDF MDF OF
Andhra Pradesh 1,62,968 1,994 13,938 13,205
Arunachal Pradesh 83,743 21,095 30,557 15,036
Assam 78,438 2,795 10,279 15,253
Bihar 94,163 333 3,280 3,693
Chhattisgarh 1,35,192 7,068 32,198 16,345
Delhi 1,483 6.72 56.42 132.30
Goa 3,702 538 576 1,123
Gujarat 1,96,244 378 5,092 9,387
Haryana 44,212 28 451 1,123
Himachal Pradesh 55,673 3,113 7,126 5,195
Jharkhand 79,716 2,603 9,687 11,321
Karnataka 1,91,791 4,501 21,048 13,026
Kerala 38,852 1,935 9,508 9,701

Environment and Ecology General Studies F–121


Madhya Pradesh 3,08,252 6,676 34,341 36,465
Maharashtra 3,07,713 8,721 20,572 21,485
Manipur 22,327 905 6,386 9,556
Meghalaya 22,429 489 9,267 7,363
Mizoram 21,081 157 5,801 12,048
Nagaland 16,579 1,273 4,534 6,679
Odisha 1,55,707 6,970 21,552 23,097
Punjab 50,362 8 801 1,040
Rajasthan 3,42,239 78 4,342 12,210
Sikkim 7,096 1,102 1,552 688
Tamil Nadu 1,30,060 3,605 11,030 11,729
Telangana 1,12,077 1,608 8,787 10,187
Tripura 10,486 654 5,236 1,836
Uttar Pradesh 2,40,928 2,617 4,080 8,109
Uttarakhand 53,483 5,047 12,805 6,451
West Bengal 88,752 3,019 4,160 9,723
A & N Islands 8,249 5,678 684 381
Chandigarh 114 1.36 14.24 6.43
Dadra & Nagar 602 1.40 85.62 140.63
Haveli and Daman
& Diu
Jammu & Kashmir 4,279 8,090 8,989
Shapefile
Area* (54,624) 2,22,236
Ladakh Shapefile 2 522 1,730
Area* (1,68,055)
Lakshadweep 30 0.00 16.09 11.01
Puducherry 490 0.00 17.66 34.75
Total 32,87,469 99,278 3,08,472 3,04,499

Area of shapefile provided by Survey of India (August, 2021). Notified geographical areas for individual UTs from sol are
awaited.
2021 Assessment Change
Total
Total VDF MDF OF Total VDF MDF OF
Change
29,137 1,994 13,929 13,861 29,784 0 -9 656 647
66,688 21,058 30,176 15,197 66,431 -37 -381 161 -257
28,327 3,017 9,991 15,304 28,312 222 -288 51 -15
7,306 333 3,286 3,762 7,381 0 6 69 75
55,611 7,068 32,279 16,370 55,717 0 81 25 106
195.44 6.72 56.60 131.68 195.00 0.00 0.18 -0.62 -0.44
2,237 538 576 1,130 2,244 0 0 7 7
14,857 378 5,032 9,516 14,926 0 -60 129 69
1,602 28 445 1,130 1,603 0 -6 7 1
15,434 3,163 7,100 5,180 15,443 50 -26 -15 9

F–122 General Studies Environment and Ecology


23,611 2,601 9,689 11,431 23,721 -2 2 110 110
38,575 4,533 20,985 13,212 38,730 32 -63 186 155
21,144 1,944 9,472 9,837 21,253 9 -36 136 109
77,482 6,665 34,209 36,619 77,493 -11 -132 154 11
50,778 8,734 20,589 21,475 50,798 13 17 -10 20
16,847 905 6,228 9,465 16,598 0 -158 -91 -249
17,119 560 9,160 7,326 17,046 71 -107 -37 -73
18,006 157 5,715 11,948 17,820 0 -86 -100 -186
12,486 1,272 4,449 6,530 12,251 -1 -85 -149 -235
51,619 7,213 20,995 23,948 52,156 243 -557 851 537
1,849 11 793 1,043 1,847 3 -8 3 -2
16,630 78 4,369 12,208 16,655 0 27 -2 25
3,342 1,102 1,551 688 3,341 0 -1 0 -1
26,364 3,593 11,034 11,792 26,419 -12 4 63 55
20,582 1,624 9,119 10,471 21,214 16 332 284 632
7,726 647 5,212 1,863 7,722 -7 -24 27 -4
14,806 2,627 4,029 8,162 14,818 10 -51 53 12
24,303 5,055 12,768 6,482 24,305 8 -37 31 2
16,902 3,037 4,208 9,587 16,832 18 48 -136 -70
6,743 5,678 683 383 6,744 0 -1 2 1
22.03 1.36 13.51 8.01 22.88 0.00 -0.73 1.58 0.85
227.65 1.40 85.56 140.79 227.75 0.00 -0.06 0.16 0.10
21,358 4,155 8,117 9,115 21,387 -124 27 126 29
2,254 2 512 1,758 2,272 0 -10 28 18
27.10 0.00 16.09 11.01 27.10 0.00 0.00 0.00 0.00
52.41 0.00 17.53 35.77 53.30 0.00 -0.13 1.02 0.89
7,12,249 99,779 3,06,890 3,07,120 7,13,789 501 -1,582 2,621 1,540

Top Ten Countries in Terms of Forest Area (2020) IUCN


Country Forest Percent of *The International Union for Conservation of Nature
Area Global (IUCN) is an international organization working in the field of
(Million forest nature conservation and sustainable use of natural resources.
Hectare) area The declared objective of the organization is to help in finding
1 Russia 815 20.1 practical solutions for the most vulnerable environmental and
2 Brazil 497 12.2 developmental challenges.
3 Canada 347 8.5 *The IUCN was established in the year 1948 and its
4 USA 310 7.6 headquarters is in Geneva, Switzerland.
5 China 220 5.4 *The Red list released by the IUCN gives information about
6 Australia 134 3.3 endangered plants and animal species.
7 Congo 126 3.1 *The elephant is in the category of endangered species.
8 Indonesia 92 2.3 *Gharial (Gavialis), a species of Crocodilia family is found
9 Peru 72 1.8 within the tributaries of Ganga River. It is mainly found in,
10 India 72 1.8 India and Nepal. It is best to see Gharial (Gavialis) in its natural
Total 2685 66.1 habitat in Chambal river. The scientific name of Gharial is

Environment and Ecology General Studies F–123


Gavialis Gangeticus. Most numbers of Gharial are found in Indian Wildlife (Protection) Act, 1972.
Chambal river and Girwa river. IUCN has placed Gharial in *A list of protected wildlife is provided under it. It is worth
the category of critically endangered species. mentioning that the tiger has been kept in the schedule (I) of
*Gharial, Leatherback turtle and Reindeer are listed in the Wildlife (Protection) Act, 1972. If a species of tortoise
endangered list of species. Kharai Camels are found in Kutch is declared protected under schedule I of the Indian Wildlife
(Gujarat). They are able to swim up to three km in the sea (Protection) Act, 1972, it will enjoy the same level of protection
water. They graze on Mangroves. These camels are declared as the tiger.
as Endangered Species. National Bureau of Animal Genetic *According to the Wildlife (Protection) Act, 1972 Gharial,
Resources has recognized the Kharai Camel as the ninth species Indian wild ass and wild buffalo all the three animals cannot
of camels found in India. The livelihood of the merchant be hunted by any person except under some provisions provided
community depends upon these camels. by law.
*Whale Shark is not only India’s largest fish but is largest fish *The Forest (Conservation) Act 1980 was enacted on 27
of the world. It can be up to 50 feet long. The natural habitat December, 1980 but, it was implemented on 25 October, 1980.
of the whale shark is on the shores of Australia and Africa but National Bureau of Plant Genetic Resources was established in
during the month of March to May a large number of whale 1977. It has its headquarters in New Delhi. This organization
sharks are seen on the shores of Gujarat. During this period the works as a nodal agency for activities of plant introduction and
whale sharks are pregnant. In 2004, a program was launched germplasm augmentation for use in crop improvement. It has
to safeguard the whale sharks coming to the coast of Gujarat. ten regional offices across the country. (1) Shimla (2) Jodhpur
*Spider monkeys are found in the tropical climatic zones. (3) Thrissur (4) Akola (5) Shillong (6) Bhovati (7) Cuttack (8)
These are mainly found in the evergreen forest of central and Hyderabad (9) Ranchi (10) Srinagar.
southern American region.
*The Animal Welfare Board of India is a statutory advisory
*It is noteworthy that Pugmark Technology is used to assess
body on animal welfare laws and promotes animal welfare in
the number of different wild animals. In this technology, these
the country. It was established in 1962 under section 4 of the
animals are identified with the help of footmarks of the animals.
Prevention of Cruelty to Animal Act, 1960.
*The various wild animals and their scientific names are as
*National Tiger Conservation Authority is a Statutory body
follows:
under the Ministry of Environment Forests and Climate Change
Animal Scientific Name
(Government of India).
Onager (Asiatic wild Ass) Equus hemionus
*The National Ganga River Basin Authority was set up on 20
Swamp deer (Barasingha) Rucervus duvaucelii February, 2009. This authority is chaired by the Prime Minister
Chinkara Gazella bennetti and its members are the Union Ministers concerned, the Chief
Nilgai Boselaphus tragocamelus Ministers of the States through which Ganga flows.
*The Forest Research Institute is an institute of the Indian
Indian efforts related to Council of Forestry Research and Education and is a premier
Conservation of Forest Resources institution in the field of forestry research in India. It is located at
*Geographical area of India is only 2.4% of the total land area Dehradun in Uttarakhand and is one of the oldest institutions of
of the world. India has about 8% of the world’s total known its kind. It was established in 1906 and was declared a deemed
Bio-diversity. university by the University Grants Commission.
*According the book 'Faunal Diversity in India' written by *The Rajiv Gandhi Wildlife Conservation Award is given
J R B Alfred about 7.28% of world’s known animal species annually for significant contribution in the field of wildlife
are found in India. conservation which has made or has the potential to make a
*The Wildlife (Protection) Act was enforced in India on major impact on the protection and conservation of wildlife
9th September, 1972. The main objectives of the Wildlife in the country.
(Protection) Act, 1972 are protection from illegal hunting, *In 1974, the Forest Department auctioned approximately 680
trafficking of wild animals, etc. In this Act, there are a total of hectares of forest in Raini village of Joshimath. Then under the
66 sections which are divided into 7 chapters and 6 schedules. leadership of Gaura Devi, hundreds of women opposed the
There were several provisions for the protection of Wildlife in deforestation by sticking to the trees. Chandi Prasad Bhatt

F–124 General Studies Environment and Ecology


was the protagonist of the Chipko Movement. He established Suriname 94.6
“Dasoli Gram Swaraj Mandal” in 1964. This institute later Micronesia 74.5
became the base of the Chipko Movement. Chipko Movement Seychelles 88.5
started in 1973 against large scale cutting of forests. Sundarlal Tuvalu 33.3
Bahuguna also played a leading role in this movement. Chandi Palau 87.6
Prasad Bhatt also received Ramon Magsaysay award in 1982. Gabon 81
*Amrita Devi Bishnoi award is given by the Forest Department Solomon Island 78.9
of Government of Rajasthan for significant contribution in the Pakistan 2.1
field of wildlife protection. A cash award of Rupees one Lakh Afghanistan 1.85
is presented to individual/institutions involved in wildlife Vietnam 45.0
protection. Venezuela 52.1
*Gond and Korku tribes plant a sapling of fruit-bearing trees *The Bhutan government has made a constitutional provision
every year during a month-long ecological campaign/festival. to maintain forest on at least 60% of the total geographical
*As a part of wildlife conservation efforts in Nepal and India area of the country. *At present, about 84% of Bhutan land is
a new association by the name “SAVE” was launched. The covered with forest.
objective of SAVE is to conserve tigers. Tibetan Buddhists use *To prevent desertification, the United Nations convention
tiger skin for decoration and as a seat while offering prayers to combat desertification was established in the year 1994.
which results in an increase in tiger hunting cases. India and *This is the only law-binding agreement, which jointly presents
Nepal jointly formed an organization named SAVE in July, environment, development and sustainable land management.
2010, for the conservation of tigers. It also has the objective *Its objective is to encourage effective action through
to work for increasing the number of tigers. innovative national programs and supportive international
*“Apna Van Apna Dhan” scheme has been started by partnerships. UNCCD is committed to the bottom-up approach
Himachal Pradesh government. The scheme aims to promote to encourage local participation in prevention of desertification.
afforestation. *The World Wildlife Fund has adopted Giant Panda as the
*Survey of India, the National Survey and Mapping symbol of the organization since its inception in the year 1961.
Organization under the Department of Science and Technology *Its scientific name is Ailuropoda melanoleuca.
has been assigned the role as the nation’s principal mapping *Its habitat is mainly found in broad-leaf and mixed forests.
Currently, their number is approximately 1600 around the
agency. It was set up in 1767. It is not related to the environment.
world.
Global efforts of the *In India, wildlife week is celebrated in the middle of 2nd to
Conservation of Forest 8th October.
*According to the world factbook (Recent) of the CIA, the state
*World Environment Day is celebrated on June 5, while World
of forestation in the countries with respect to their geographical
Forestry Day is celebrated on March 21.
area, is as follows :
*The nomination of 3rd March as the 'World Wildlife Day' was
Country Forestation (%) proposed in the 16th meeting of parties (CoP 16) of CITES
Japan 68.5 (Convention on International trade in Endangered species of
Indonesia 51.7 wild fauna and flora). The theme of World Wildlife Day, 2021
India 23.1 is 'Forests and Livelihoods : Sustaining People and Planet :
China 22.3 *On December 20, 2013 during the 68th annual session of the
Mozambique 43.7 United Nations General Assembly, it was decided to celebrate
Sweden 68.7 March 3rd as the “World Wildlife Day” every year.
Guinea-Bissau 55.2 *It is noteworthy that between the 21st and 24th of November
Belize 60.6 2010, the world's first Tiger Summit was organized in Saint
Laos 67.9 Petersburg (Russia).
Guyana 77.4 *World Wildlife Fund was established in April 1961.
Finland 72.9 *In July 2000, 13 lions died due to “Trypanosomiasis Disease”
Russia 49.4 in the Nandankanan Sanctuary of Odisha.

Environment and Ecology General Studies F–125


*The United Nations General Assembly proclaimed 21st March Equatorial forests are found in such tropical zones which
the International Day of Forests (IDF) in 2012. IDF celebrates receive more than 200 cm rainfall. Tall, closely set trees
and raises awareness of the importance of all types of forests. forming canopy is a characteristic of these forests. The upper
The theme of IDF 2018 was 'Forests and Sustainable Cities', portion of canopy often supports rich epiphytes. About 80%
whereas the theme of 2021 is "Forest Restoration: A path to of the world's biodiversity is found in equatorial forests.
recovery and well-being.'' 4. Consider the following States:
Basic Questions 1. Arunachal Pradesh
2. Himachal Pradesh
1. What is the theme of the 2018 International Day of 3. Mizoram
Forests? In which of the above States do ‘Tropical Wet
(a) Forests and Water Evergreen Forests’ occur?
(b) Forests and Energy (a) 1 only (b) 2 and 3 only
(c) Forests and Sustainable cities (c) 1 and 3 only (d) 1, 2 and 3
(d) Forests and Life I.A.S. (Pre) 2015
(e) None of the above/More than one of the above Ans. (c)
63rd B.P.S.C. (Pre) 2017 The tropical evergreen forests usually occur in areas receiving
Ans. (c) more than 200 cm of rainfall and having a temperature of 15
The United Nations General Assembly proclaimed 21st to 30 degrees Celsius. These forests occupy about seven per
March the International Day of Forests (IDF) in 2012. IDF cent of the Earth’s land surface and harbour more than half of
celebrates and raises awareness of the importance of all the world’s plants and animals. In India, the evergreen forest
types of forests. The theme of IDF 2018 was 'Forests and is found on the western slope of the Western Ghats in states
Sustainable Cities', whereas the theme of 2021 is "Forest such as Kerala and Karnataka. They are also found in hills of
Restoration: A path to recovery and well-being.'' Jaintia and Khasi (Meghalaya). While Himachal Pradesh is
mainly covered by Sub-Tropical Pine forest and Himalayan
2. Which one of the following has an evergreen forest? Moist Temperate forests. Hence (c) is the correct answer.
(a) Malwa Plateau (b) Eastern Ghat 5. The minimum land area recommended for forest cover
(c) Western Ghat (d) Chhotanagpur Plateau to maintain proper ecological balance in India is:
U.P.P.C.S. (Pre) 2015 (a) 25% (b) 33%
Ans. (c) (c) 43% (d) 53%
The tropical evergreen forests are found in areas receiving I.A.S. (Pre) 1999
more than 200 cm of rainfall and having a temperature of 15 Ans. (b)
to 30 degrees Celsius. They occupy about seven percent of As per the National Forest Policy, 1988, the national goal
the Earth’s land surface and harbour more than half of the should be to have a minimum of one-third (33%) of the
world’s plants and animals. They are found mostly near the total land area of the country under forest or tree cover. In
equator. In India, evergreen forests are found on the Western the mountain and hills region, the aim should be to maintain
Ghat, north-east India and Andaman and Nicobar Islands. two-thirds of the area under such cover to prevent erosion
3. Which of the following is/are unique characteristic/ and land degradation and to ensure the stability of the fragile
characteristics of equatorial forests? ecosystem.
1. The presence of tall, closely set trees with crowns 6. To maintain ecological balance, the area under forest
forming a continuous canopy. should be –
2. Co-existence of a large number of species. (a) 10% (b) 23%
3. The presence of numerous varieties of epiphytes. (c) 33% (d) 53%
Select the correct answer using codes given below : U.P.P.C.S. (Pre) 2014
(a) Only 1 (b) 2 and 3 Ans. (c)
(c) 1 and 3 (d) All of these See the explanation of the above question.
I.A.S. (Pre) 2013
Ans. (d) 7. The national forest policy aims at maintaining how

F–126 General Studies Environment and Ecology


much of the total geographical area under forests: cover has been observed as 712,249 km2 area of the country
(a) One-fourth (b) Half which is 21.67% of the geographical area of the country.
(c) One-fifth (d) One-third
11. According to the India State of Forest Report 2017,
U.P.P.C.S. (Pre) 2002
what percentage of the total geographical area of the
Ans. (d)
country is under forest?
See the explanation of the above question. (a) 20.34 (b) 22.34
8. Which of the following has not been categorized as (c) 21.54 (d) 23.54
forest under the National Forest Policy (1952)? U.P. P.C.S. (Pre) 2018
(a) National Forests (b) National Park Ans. (c)
(c) Protected Forests (d) Village Forests According to the India State of Forest Report 2017 & 2019,
U.P.R.O./A.R.O. (Pre) 2016 India has 21.54% and 21.67% forest cover respectively.
Ans. (b)
12. According to data obtained from a satellite survey,
According to the National Forest Policy (1952), forests have what percentage of India’s area is a forest?
been classified in India as follows - (a) 32 (b) 28
(i) Protected forests (i) National forests (iii) Village forests (c) 19 (d) 15
(iv) Tree-lands. R.A.S./R.T.S. (Pre) 2003
Thus it is clear that National Parks are not in the category
Ans. (c)
of forests.
Option (c) will be the most probable correct answer as per
9. What is the minimum land area recommended for
the explanation is given above.
forest cover to maintain proper ecological balance
India? 13. According to the Indian State of Forest Report 2015,
(a) 27% (b) 30% released by the Ministry of Environment, what is the
(c) 33% (d) 25% percentage of the total geographical area of India
Jharkhand P.C.S. (Pre) 2021 under forest and tree cover:
Ans. (c) (a) 22.48 (b) 23.00
The minimum land area recommended for forest cover (c) 24.16 (d) 24.48
to maintain proper ecological balance in India is 33%. In U.P.R.O./A.R.O. (Pre) 2016
the hills and in mountainous regions, the aim should be to Ans. (c)
maintain two-third of the area under such cover in order According to the Indian State of Forest Report-2015, the
to prevent erosion and land degradation and to ensure the total area of India under forest and tree cover is 79.42 million
stability of the fragile eco-system. As per IFSR 2021, The Hectare (794245 sq. km). This was 24.16 percent of the total
total forest and tree cover of the country is 80.9 million geographical area of the country. Recent data (ISFR, 2019)
hectare which is 24.62 percent of the geographical area of for the above question is 24.56 percent.
the country.
14. How much of Indian land is forest area?
10. What is the percentage of forest area in the total
(a) 33.5 percent (b) 22.7 percent
geographical area of India?
(c) 44.7 percent (d) 17.7 percent
(a) 24.5 % (b) 21 %
M.P.P.C.S. (Pre) 2010
(c) 20 % (d) 22 %
Ans. (b)
45th B.P.S.C. (Pre) 2001
U.P.P.C.S. (Pre) 1991 According to the India State of Forest Report-2017, 21.54
Ans. (*) percent of the geographical area is under forest cover and
2.85 percent under tree cover (total 24.40 percent). So, the
According to India State of Forest Report 2017 released
most possible answer is option (b). Recent data (ISFR, 2019)
in February, 2018. India is ranked 10th in the world, with
for the above question is 21.67 percent.
24.4% of land area under forest and tree cover. Land area
under forest cover was 21.54% and tree cover was 2.85%. 15. According to "India State of Forest Report 2017" what
According to the recent data of ISFR, 2019, the total Forest percentage of the total geographical area of the country

Environment and Ecology General Studies F–127


is under forest cover? According to India State of Forest Report (ISFR) 2017 &
(a) 21.04 (b) 21.54
2019, States are given in the option have an area under forest
(c) 20.54 (d) 20.04
as given below –
U.P. R.O./A.R.O. (Pre) 2017
Forest Cover (Percentage)
Ans. (b)
State 2017 2019
See the explanation of the above question.
Mizoram 86.27% 85.41%
16. According to India State of Forest Report released in Arunachal Pradesh 79.96% 79.63%
February 2018, how much area of India is covered by Nagaland 75.33% 75.31%
forests? Meghalaya 76.45% 76.33%
(a) 23.00% (b) 23.40%
(c) 24.00% (d) 24.40% 20. Which among the following States of India has the
(e) None of the above/More than one option largest percentage or geographical area under forest
64th B.P. S.C. (Pre) 2018 as per the report of the Forest Survey of India?
Ans. (e) (a) Manipur (b) Meghalaya
(c) Mizoram (d) Nagaland
See the explanation of the above question. U.P.R.O./A.R.O. (Pre) 2016
17. Which of the following States has the largest area Ans. (c)
under forests? See the explanation of the above question.
(a) Kerala (b) Uttar Pradesh
21. Concerning total geographical area the maximum
(c) Madhya Pradesh (d) Rajasthan
percent of forest found in :
M.P.P.C.S. (Pre) 2010
(a) Arunachal Pradesh (b) Nagaland
Uttarakhand P.C.S. (Pre) 2003
Ans. (c) (c) Tripura (d) Mizoram
U.P.P.C.S. (Mains) 2002
According to India State of Forest Report (ISFR) 2019, States U.P.U.D.A./L.D.A. (Pre) 2010
given in options have an area under forest as given below– Ans. (d)
State Forest Area
(Sq. km) According to India State of Forest Report (ISFR) 2017 &
Madhya Pradesh - 77482 2019, States are given in option has an area under forest-
Rajasthan - 16630 State 2017 2019
Kerala - 21144 Mizoram 86.27% 85.41%
Uttar Pradesh - 14806 Arunachal Pradesh 79.96% 79.63%
Hence, option (c) will be the correct answer. Nagaland 75.33% 75.31%
Tripura 73.68% 73.68%
18. Which Indian State has the largest forest cover area?
(a) Madhya Pradesh (b) West Bengal 22. Which among the following States had the largest
(c) Kerala (d) Assam percentage of its geographical area under forest in
R.A.S./R.T.S.(Pre) 2003 2015?
M.P.P.C.S. (Pre) 2000 (a) Arunachal Pradesh (b) Nagaland
Ans. (a) (c) Meghalaya (d) Mizoram
U.P. P.C.S. (Mains) 2017
See the explanation of the above question.
Ans. (d)
19. The percentage of forest cover concerning the total
geographical area is highest in See the explanation of the above question.
(a) Arunachal Pradesh (b) Meghalaya 23. The largest percentage of forest area is in :
(c) Mizoram (d) Nagaland (a) Arunachal Pradesh (b) Himachal Pradesh
U.P. P.C.S. (Mains) 2012 (c) Mizoram (d) Nagaland
Ans. (c) 44th B.P.S.C. (Pre) 2000

F–128 General Studies Environment and Ecology


Ans. (c) 27. In India the State with the largest area under very
dense forests is –
See the explanation of the above question.
(a) Arunachal Pradesh
24. As part of the New Forest Inventory Design implemented (b) Madhya Pradesh
in 2016, the revisit time has been reduced to - (c) Maharashtra
(a) 5 years (b) 2 years (d) Odisha
(c) 6 months (d) 10 years
U.P.P.C.S (Pre) 2011
U.P. R.O./A.R.O. (Mains) 2017
Ans. (a)
Ans. (a)
According to India State of Forest Report (ISFR) 2021, the
The New Forest Inventory Design implemented in 2016 has
area of very dense forest for States given in the options is as
reduced the revisit time of forest inventory from 20 years
follow –
to 5 years.
State Forest Area
25. National Mission for Green India has been initiated in (Sq. km)
the following States by Government of India with the Arunachal Pradesh - 21058
aim to improve the density of existing forests Madhya Pradesh - 6665
(a) Jharkhand Maharashtra - 8734
(b) Madhya Pradesh Odisha - 7213
(c) Both States 28. Which amongst the following States/UTs in India has
(d) None of the above the largest percentage of its geographical area under
Jharkhand P.C.S. (Pre) 2016
forest cover?
Ans. (a)
(a) Tripura (b) Andaman and Nicobar
One of the 8 missions outlined under the National Action (c) Nagaland (d) Mizoram
Plan on Climate Change (NAPCC), National Mission for U.P. Lower (Spl) (Pre) 2008
Green India (GIM) acknowledges the influence of forests Ans. (d)
on environmental amelioration through climate change
mitigation, water security, food security, biodiversity According to India State of Forest Report (ISFR) 2021, five
conservation and livelihood security of forest-dependent States/Union Territories with largest percentage of their
communities. In the second sitting of National Executive geographical area under forest cover are (1) Lakshadweep, (2)
Council (NEC) of National Mission for Green India (GIM) Mizoram, (3) Andaman and Nicobar Islands, (4) Arunachal
held in October 2015, the annual plan for the possible Pradesh, (5) Meghalaya.
schemes and operations proposed by the four states-Mizoram, 29. Among the following which statement is not correct?
Manipur, Kerala and Jharkhand was approved. (a) Madhya Pradesh has maximum forest area.
26. Which of the following is not the impact of deforestation (b) Arunachal Pradesh has maximum dense forest area.
in India? (c) Nagaland is the most forest covered State of India.
(a) Drying of water sources in Himalaya (d) Haryana is the least forest covered State of India.
(b) Loss of biodiversity U.P.P.S.C. (GIC) 2010
(c) Urbanization Ans. (c)
(d) Soil erosion
According to India State of Forest Report (ISFR) 2021,
Uttarakhand P.C.S. (Pre) 2006
Madhya Pradesh has maximum forest area while Arunachal
M.P.P.C.S. (Pre) 2013
Pradesh has maximum dense forest area. India’s most
Ans. (c)
forest covered state in terms of percentage is Mizoram not
Urbanization is not the effect for deforestation rather it is one Nagaland. State with minimum forest cover (as per area as
of the reasons of deforestation. The expansion of residential well as a percentage) is Haryana.
areas to meet the needs of growing population impacts
forests and biodiversity. Forest areas have shrunk due to the 30. Tree cover is maximum in –
expansion of towns and cities. On the contrary, drying of (a) East Deccan (b) Northern Plains
water resources in the Himalayas, loss of biodiversity and (c) West Coast (d) East Coast
soil erosion are the effects of deforestation. U.P.P.C.S. (Pre) 2009

Environment and Ecology General Studies F–129


Ans. (a) 33. In India, percentage coverage of dense forests to total
geographical area is about
The total tree-covered area of India is 95,748 sq.km which
(a) 8 percent (b) 10 percent
is 2.91% of total land. ISFR, 2021 has not mentioned such
(c) 12 percent (d) 14 percent
(physiographic region) data.
U.P.P.C.S. (Mains) 2008
31. Assertion (A) : Forests are a renewable resource. Ans. (c)
Reason (R) : They enhance the quality of environment
The percentage of the dense forest to India’s total geographical
Select the correct answer from codes given below:
area is 12.37. According to the India State of Forest Report
(a) Both (A) and (R) are true and (R) is the correct
2021, the dense forests of India is divided into very dense
explanation of (A)
forest and medium dense forest. The percentage of the very
(b) Both (A) and (R) are true but (R) is not the correct dense forest is 3.04% and the medium dense forest is 9.33.
explanation of (A)
(c) (A) is true but (R) is false 34. According to the National Remote Sensing Agency
(d) (A) is false but (R) is true (NRSA) which of the following state has its maximum
area covered by forest ?
U.P.P.C.S. (Pre) 2004
(a) Himachal Pradesh (b) Arunachal Pradesh
U.P. Lower Sub. (Spl) (Pre) 2002
(c) Meghalaya (d) Sikkim
Ans. (b)
U.P. Lower Sub. (Pre) 2004
Renewable resources are the resources which can be reused. Ans. (b)
Thus, forest is a renewable resource as it can be reused after
The land area identified by National Remote Sensing Agency
its sustainable exploitation. Forests enhance the quality of
(NRSA) as covered with forest is called forest cover. The
the environment as it absorbs carbon dioxide and produces
State of Forest Report issued by Forest Survey of India (FSI)
oxygen. Thus assertion (A) and Reason (R) both are correct
Dehradun, is based on data received through remote sensing.
but (R) is not the correct explanation of (A).
According to the report of the 2019 & 2021 percentage of
32. Consider the following statements: forest in the given States is as follows:
Assertion (A) : Natural resources are those that exist in State 2019 2021
a region and can also be used in future. Arunachal Pradesh - 79.63% 79.33%
Reason (R) : The development of an actual natural Meghalaya - 76.33% 76.00%
resource depends upon the technology Sikkim - 47.10% 47.08%
Himachal Pradesh - 27.72% 27.73%
and cost of production.
Thus, in the above mentioned States, Arunachal Pradesh has
Select the correct answer from codes given below:
the maximum percentage of forest covered area.
(a) Both (A) and (R) are true and (R) correctly explains
(A). 35. Consider the following statements:
(b) Both (A) and (R) are true but (R) does not correctly 1. The forest cover in India constitutes around 20% of
explains (A). its geographical area. Out of the total forest cover,
(c) (A) is true, but (R) is false. dense forest constitutes around 40%.
(d) (A) is false, but (R) is true. 2. The National Forestry Action Programme aims at
U.P.R.O./A.R.O. (Pre) 2016 bringing one-third of the area of India under tree/
Ans. (b) forest cover.
Which of these statements given above is/are correct?
On the basis of the development stage, natural resources are (a) Only 1 (b) Only 2
categorized as: (i) Potential Resources (ii) Actual Resources (c) Both 1 and 2 (d) Neither 1 nor 2
(iii) Reserve Resources (iv) Stock Resources. Potential
I.A.S. (Pre) 2005
Resources are those which exist in a particular area and can
Ans. (b)
be used in future. Actual Resources are those which have been
surveyed and their volume and quality have been determined According to ISFR, 2021 almost 21.71% area (7,13,789 square
and these are being used currently. The development of Km) of the geographical region of India is forest. Out of which
an actual resource depends upon technology and cost of near about 57% area (407750 square Km) is a dense forest,
production. Thus assertion and reason both are correct not 40%. The National Forestry Action Programme aims at
however reason does not explain the assertion. bringing one-third of the area of India under tree/forest cover.

F–130 General Studies Environment and Ecology


36. According to India State of Forest Report 2019, in Ans. (d)
which one of the following district, very dense forest
See the explanation of the above question.
area is minimum?
(a) Jaisalmer (b) Jodhpur 40. Among the following states, which has highest forest
(c) Alwar (d) Jaipur denisty in India?
R.A.S./R.T.S. (Pre) 2021 (a) Nagaland
Ans. (b) (b) Mizoram
(c) Assam
According to IFSR 2019, in Jodhpur district, very dense
(d) Gujrat
forest area is minimum. As per IFSR 2021, Jodhpur has zero
sq. Km. very dense forest area. Hence option (2) is correct. Jharkhand P.C.S. (Pre) 2021
Ans. (b)
37. Which one of the following groups of States has forest
coverage of more than 75% of the total geographical Mizoram has highest forest density in India among the given
area: states. As per IFSR 2021, Area-wise Madhya Pradesh has the
largest forest cover in the country followed by Arunachal
(a) Arunachal Pradesh, Assam, Nagaland
Pradesh, Chhattisgarh, Odisha and Maharashtra. In terms
(b) Arunachal Pradesh, Manipur, Nagaland
of forest cover as percentage of total geographical area, the
(c) Assam, Meghalaya, Nagaland
top five States are Mizoram (84.53%), Arunachal Pradesh
(d) Arunachal Pradesh, Nagaland (79.33%), Meghalaya (76.00%), Manipur (74.34%) and
U.P.P.C.S. (Pre) 1995 Nagaland (73.90%).
Ans. (b)
41. Consider the following States:
As per ISFR 2019 & 2021, the percentage forest in States 1. Chhattisgarh 2. Madhya Pradesh
mentioned above is as follows- 3. Maharashtra 4. Odisha
State 2019 2021 With reference to the States mentioned above, in
Arunachal Pradesh - 79.63% 79.33% terms of percentage of forest cover to the total area
Nagaland - 75.31% 73.90% of the State, which one of the following is the correct
Manipur - 75.46% 74.34%
ascending order?
Meghalaya - 76.33% 76.00%
(a) 2-3-1-4 (b) 2-3-4-1
Assam - 36.11% 36.09%
(c) 3-2-4-1 (d) 3-2-1-4
Madhya Pradesh - 25.14% 25.14%
Hence, we can see that Arunachal Pradesh and Meghalaya I.A.S. (Pre) 2019
are the States having more than 75% of the covered forest. Ans. (c)
Thus option (b) is the correct answer. According to ISFR-2021 percentage of forest area in
38. Which of the following state in India has maximum Maharashtra is 16.511%, in Madhya Pradesh 25.14%, in
number of tribal districts as per "India State of Forest Odisha 33.50% and in Chhattisgarh it is 41.21%.
Report 2019"? 42. Which State has more than 80% of its area covered
(a) Nagaland (b) Meghalaya by forest?
(c) Mizoram (d) Manipur (a) Arunachal Pradesh (b) Kerala
U.P.R.O./A.R.O. (Pre) 2021 (c) Madhya Pradesh (d) West Bengal
Ans. (*) U.P.P.C.S. (Spl.) (Mains) 2008
As per India State forest Report 2019, Nagaland and Ans. (a)
Meghalaya both have 11 district, all of which are tribal as According to India State of Forest Report (ISFR) 2019 &
well as hill districts. Hence both option (a&b) is correct. 2021, States in the given options with their percentage area
39. Which one of the following States has the highest under forest is given below –
percentage of area under forests? State Forest Cover(percentage)
(a) Himachal Pradesh (b) Assam 2019 2021
(c) Andhra Pradesh (d) Arunachal Pradesh Arunachal Pradesh – 79.63% 79.33%
(e) None of the above/More than one of the above Kerala – 54.42% 54.70%
63rd B.P.S.C. (Pre) 2017 Madhya Pradesh – 25.14% 25.14%

Environment and Ecology General Studies F–131


West Bengal – 19.04% 18.96% 47. Which one of the following regions of India has a
Therefore, the nearest answer can be option (a) but none of combination of mangrove forest, evergreen forest and
the given States has more than 80% of their area covered deciduous forest?
by forest. (a) North Coastal Andhra Pradesh
(b) South-West Bengal
43. Second largest mangrove area in India is found along
(c) Southern Saurashtra
the coast of –
(d) Andaman and Nicobar Island
(a) Andaman and Nicobar (b) Andhra Pradesh
I.A.S. (Pre) 2015
(c) Gujarat (d) Odisha
Ans. (d)
U.P.P.C.S. (Mains) 2008
Ans. (c) Mangrove Forest, Evergreen Forest and Deciduous Forests,
all are found in Andaman and Nicobar Island. According
According to ISFR 2021, the sequence of Mangrove Covered to ISFR, 2021 Andaman & Nicobar covers 616 sq. km. of
States/Union Territories in India are as follows- 1. West mangrove forest.
Bengal (2114 sq.km.) 2. Gujarat (1175 sq.km.) 3. Andaman
and Nicobar (616 sq.km) 4. Andhra Pradesh (405 sq.km.). 48. Fill in the blank with correct choice:
Note- Mangroves are shrubs or small trees that grow in The recorded forest area of U.P. in 2011 was around
coastal saline water. The term is also used for tropical coastal .... of its area :
vegetation consisting of such species. (a) 3% (b) 5%
44. Mangrove (Tidal Forests) vegetation in India is mostly (c) 7% (d) 9%
found in – U.P.P.C.S. (Mains) 2012
(a) Malabar coast (b) Sundarbans Ans. (c)
(c) Rann of Kachchh (d) Dandakaranya According to the Indian State of Forest Report 2017 &
Chhattisgarh P.C.S. (Pre) 2011 2019, recorded forest area in Uttar Pradesh is 16582 sq. Km.
Ans. (b) which is 6.88 (nearly 7) per cent of its total area. Out of the
total recorded forest about 73% is reserved forest, 6.98%
A mangrove commonly refers to two different things: a tidal
is protected forest and 20.22% is an unclassified forest. On
swamp ecosystem found in tropical deltas, estuaries, lagoons
the other hand satellite data shows that U.P. has the total
or islands and the characteristic tree species populating
forest cover of 14,818.65 sq. Km. which is 6.15% of its total
this ecosystem. Mangrove trees have developed unique
geographical area (According to ISFR of 2021).
adaptations to the harsh conditions of coastal environments.
Most of the region of Mangrove plants are found in Sundarban 49. Nagaland mountains are becoming increasingly barren
delta. Sundari plant in these forests is quite popular. mountains mainly due to –
45. Which one of the following areas of India is famous (a) Insurgency (b) Urbanization
for its mangrove vegetation? (c) Shifting cultivation (d) Rapid population growth
(a) Kaziranga (b) Silent Valley M.P.P.C.S. (Pre) 2010
(c) Sunderbans (d) Himalayan Terai Ans. (c)
U.P.P.C.S. (Spl) (Mains) 2004 Nagaland Mountains are becoming increasingly barren
Ans. (c) mountains mainly due to shifting cultivation.
See the explanation of the above question. 50. Which among the following States has launched ‘Apna
Van Apna Dhan’ scheme?
46. Which one of the following has a protected mangrove
(a) Uttar Pradesh (b) Madhya Pradesh
region?
(c) Himachal Pradesh (d) Arunachal Pradesh
(a) Eastern Ghats (b) Western Ghats
U.P.P.C.S.(Pre) 2012
(c) Goa (d) Chandra Tal
Ans. (c)
U.P.P.C.S. (Mains) 2016
Ans. (c) 'Apna Van Apna Dhan' scheme had been started by
Himachal Pradesh government. The scheme aims to promote
Chorao Island of Goa is a protected mangrove region. afforestation.

F–132 General Studies Environment and Ecology


51. Which of the following plant, grown in non-forest area Ans. (a)
has been omitted from the definition of trees in the
The Wildlife (Protection) Act, 1972 is for the protection of
Indian Forest (Amendment) Act, 2017?
wild animals in India.
(a) Palm (b) Canes
(c) Bamboo (d) Banana 56. In India, if a species of tortoise is declared protected
R.A.S./R.T.S. (Pre) 2018 under Schedule 1 of the Wildlife (Protection) Act 1972,
Ans. (c) what does it imply?
(a) It enjoys the same level of protection as the tiger.
Bamboo grown in non-forest area has been omitted from
(b) It no longer exists in the wild, a few individuals are
the definition of trees in the Indian Forest (Amendment)
under captive protection and now it is impossible to
Act 2017. Tribals and poor farmers can now use it for their
economic benefits. prevent its extinction.
(c) It is endemic to a particular region of India.
52. In which year the Wildlife Protection Act was (d) Both (b) and (c) stated above are correct in this context.
introduced in India? I.A.S. (Pre) 2017
(a) 1962 (b) 1970 Ans. (a)
(c) 1972 (d) 1982
There were several provisions for the protection of wildlife
R.A.S./R.T.S.(Pre) 2012
in Indian Wildlife (Protection) Act 1972. A list of protected
Ans. (c)
wildlife is provided under it. It is worth mentioning that the
The first comprehensive legislation relating to the protection tiger has been kept in the schedule (1). Therefore if a species
of wildlife was passed by the Parliament and it was assented of tortoise is declared protected under Schedule I of the Indian
to by the President on 9th September, 1972 and came to be Wildlife (Protection) Act, 1972, it will enjoy the same level
known as The Wild Life (Protection) Act, 1972. of protection as the tiger.

53. In which year was the Wildlife Protection Act enforced 57. According to the Wildlife (Protection) Act, 1972, which
in India? of the following animals cannot be hunted by any
(a) 1972 (b) 1980 person except under some provisions provided by law?
(c) 1970 (d) 1975 1. Gharial 2. Indian wild ass
M.P.P.C.S. (Pre) 2016 3. Wild buffalo
Ans. (a) Select the correct answer using the code given below:
(a) 1 only (b) 2 and 3 only
The Wildlife (Protection) Act was enforced in India on 9th
(c) 1 and 3 only (d) 1, 2 and 3
September, 1972.
I.A.S. (Pre) 2017
54. Wildlife Protection Act was passed in the year? Ans. (d)
(a) 1965 (b) 1970
According to the Wildlife (Protection) Act, 1972 Gharial,
(c) 1972 (d) 1975 Indian Wild Ass and Wild Buffalo all the three animals
U.P.P.C.S. (Pre) 2015 cannot be hunted by any person except under some provisions
Ans. (c) provided by law.
The Wildlife Protection Act, 1972 is an Act of the Parliament 58. If a particular plant species is placed under Schedule
of India enacted for the protection of plants and animal VI of the Wildlife Protection Act 1972. What is the
species.
implication?
55. Which of the following Acts provides for protection to (a) A licence is required to cultivate that plant.
wild animals in India? (b) Such a plant cannot be cultivated under any
(a) Wildlife Protection Act - 1972 circumstances.
(b) Forests Protection Act- 1982 (c) It is a Genetically Modified crop plant.
(c) Environment Protection Act - 1996 (d) Such a plant is invasive and harmful to the ecosystem
(d) West Bengal Wild Animal Protection Act - 1959 I.A.S. (Pre.) 2020
U.P.P.C.S. (Pre) 2016 Ans. (a)

Environment and Ecology General Studies F–133


According to ‘The Wildlife Protection Act, 1972’, the I. Wild Life (Protection) Act 1972
cultivation of the plant species listed under schedule VI are II. Biological Diversity Act 2002
prohibited except with a licence granted by the Chief Wild III. Project Tiger 1973
Life Warden or any other officer authorized by the state IV. Project Elephant 1992
government on this behalf. Following plant species are placed Code :
under schedule VI of ‘The Wildlife Protection Act, 1972’. (a) I, II, I, IV (b) I, III, IV, II
1. Beddomes cycad (Cycas beddomei) (c) II, III, IV, I (d) II, III, I, IV
2. Blue Vanda (Vanda soerulec) U.P.P.C.S. (Pre) 2019
3. Kuth (Saussurea lappa) U.P.P.C.S. (Pre) 2020
4. Slipper orchids (Paphiopedilum spp.) Ans. (b)
5. Pitcher plant (Nepenthes khasiana) Events Year
6. Red Vanda (Rananthera imschootiana) Wild Life (Protection) Act 1972
59. When was the Forest (Conservation) Act enacted? Biological Diversity Act 2002
(a) 1978 (b) 1979 Project Tiger 1973
(c) 1980 (d) 1981 Project Elephant 1992
U. P. Lower Sub. (Pre) 2013 Therefore correct chronological order will be I,III, IV, II.
Ans. (c) 64. Forest Research Institute is located at –
The Forest (Conservation) Act, 1980 was enacted on 27 (a) Hyderabad (b) Nainital
December, 1980. (c) Solan (d) Dehradun
U.P.P.C.S. (Spl) (Mains) 2008
60. The Government of India enacted the Forest
Ans. (d)
Conservation Act in the year:
(a) 1976 (b) 1980 The Forest Research Institute is an institute of the Indian
(c) 1983 (d) 1988 Council of Forestry Research and Education and is a premier
U.P.P.C.S. (Pre) 2017 institution in the field of forestry research in India. It is
Ans. (b) located at Dehradun in Uttarakhand and is one of the oldest
See the explanation of the above question. institutions of its kind. In 1991, it was declared a deemed
university by the University Grants Commission.
61. The National Afforestation and Eco-Development
65. Forest Research Institute is located at?
Board (NAEB) was set up by Government of India
(a) New Delhi (b) Shimla
in the year :
(c) Dehradun (d) Bhopal
(a) 1987 (b) 1992
Uttarakhand P.C.S. (Pre) 2006
(c) 1995 (d) 1998
Uttarakhand P.C.S. (Pre) 2007
U.P.R.O./A.R.O. (Re-Exam) (Pre) 2016
Ans. (c)
Ans. (b)
See the explanation of the above question.
The National Afforestation and Eco-Development Board
(NAEB) was set up by the Government of India in the year 66. Where is Forest Research Institute located?
1992. (a) Nagpur (b) Mysore
62. When was 'Project Tiger' launched in India? (c) Dehradun (d) Ernakulam
(a) 1968 (b) 1972 Uttarakhand P.C.S. (Pre) 2010
(c) 1984 (d) 1993 Ans. (c)
U.P. P.C.S. (Pre) 2018 See the explanation of the above question.
Ans. (*)
67. The Indian Council of Forestry Research and
Project Tiger is a tiger conservation programme launched Education (ICFRE) has signed MoU with TIFAC for
on 1 April, 1973 by the Government of India. The State of supporting forest-based communities. Where is the
Madhya Pradesh has the highest population of Tigers in India. headquarters of ICFRE located?
63. Arrange the following events in chronological order of (a) Hyderabad (b) Bengaluru
their commencement and select correct answer from (c) Allahabad (d) Dehradun
codes given below: (e) None of the above/More than one of the above

F–134 General Studies Environment and Ecology


63rd B.P.S.C. (Pre) 2017 72. Which animal is the symbol of the World Wildlife Fund:
Ans. (d) (a) Tiger (b) Giant Panda
(c) Hornbill (d) White Bear
Indian Council of Forestry Research and Education
U.P.P.C.S.(Pre) 2001
(ICFRE) is an apex body in the National forestry research
U.P. Lower Sub. (Spl) (Pre) 2002
system that promotes and undertakes need based forestry
U.P. Lower Sub. (Spl) (Pre) 2003
research extension. Its headquarters is located at Dehradun,
Ans. (b)
Uttarakhand.
Since its inception in 1961, the World Wildlife Fund adopted
68. Forest Research Institute is located in : Giant Panda as its organizational symbol. Its scientific
(a) Dehradun (b) Bhopal name is 'Ailuropoda melanoleuca'. It's habitat is Broad leaf
(c) New Delhi (d) Nagpur temperate forest.
U.P.R.O./A.R.O. (Re-Exam) (Pre) 2016
Ans. (a) 73. Gavialis crocodilia are found plenty in –
(a) Ganga (b) Godavari
See the explanation of the above question. (c) Krishna (d) Cauvery
69. Which of the following institutions is not related to R.A.S./R.T.S. (Pre) 1993
the environment? Ans. (a)
(a) Centre for Science and Environment The Gavialis is also known as the gavial and belong to
(b) Botanical Survey of India the fish-eating crocodile family. These are native to the
(c) Indian Wildlife Institute northern part of the Indian Sub-continent. They are mostly
(d) Survey of India found in Bangladesh, India and Nepal.
M.P.P.C.S. (Pre) 2016
74. Consider the following animals of India:
Ans. (d)
1. Crocodile
Survey of India, The National Survey and Mapping 2. Elephant
Organization under the Department of Science and Which of these is/are endangered species ?
Technology has been assigned the role as the nation’s (a) Only 1 (b) Only 2
principal mapping agency. It was set up in 1767. It is not (c) Both 1 and 2 (d) Neither 1 nor 2
related to the environment. I.A.S. (Pre) 2003
Ans. (c)
70. In world wildlife, India accounts for:
(a) 5 percent (b) 2 percent The species of crocodile found in India and Asian Elephant
(c) 6 percent (d) 4 percent has been listed in the endangered list by International Union
U.P.P.C.S. (Spl) (Pre) 2008 for Conversation of Nature.
Ans. (*) 75. ‘Chipko’ movement was basically against –
India constitutes only 2.4 per cent of the World's total land (a) Water pollution (b) Noise pollution
area. Notably, 8% of the world's total known bio-diversity (c) Deforestation (d) Cultural pollution
is found in India. According to 'Faunal Diversity of India' U.P.P.C.S. (Pre) 2012
by J.R.B. Alfred, 7.28% of the world's animal species are U.P.P.C.S. (Mains) 2003
found in India. Ans. (c)
71. India’s largest fish is: Chipko movement was against deforestation. Chandi Prasad
(a) Stonefish (b) Whale shark Bhatt and Sundarlal Bahuguna were the main leaders of this
(c) Marlin (d) Hilsa movement.
U.P.P.C.S. (Spl) (Pre) 2008
Ans. (b) 76. Who among the following is considered as the leader
of the Chipko Movement?
Whale Shark is the largest fish not only of India but the world. (a) Medha Patekar (b) Baba Amte
It may be up to 50 m long. They live on the coasts of Australia (c) Sundarlal Bahuguna (d) Kiran Bedi
and South Africa but every year in March they use to come to M.P.P.C.S.(Pre) 2014
the coasts of Gujarat. This time is gestation of Whale Sharks. Ans. (c)
A program was started in 2004 for the protection of Whale
Chipko movement was an organized movement against the
Sharks coming to the coasts of Gujarat.

Environment and Ecology General Studies F–135


destruction of forests during 1970. Sundarlal Bahuguna was Ans. (c)
the leader of this movement. The Environment (Protection) Act was enacted in 1986 with
77. Who is the protagonist of the Chipko Movement? the objective of providing for the protection and improvement
(a) Medha Patkar (b) M.S. Swaminathan of the environment. It empowers the Central Government
(c) Sunderlal Bahuguna (d) Chandi Prasad Bhatt to establish authorities [under section 3(3)] charged with
M.P.P.C.S. (Pre) 2016 the mandate of preventing environmental pollution in all its
Ans. (d) forms and to tackle specific environmental problems that are
peculiar to different parts of the country. The Act was last
Chandi Prasad Bhatt was the protagonist of the Chipko amended in 1991.
Movement. He established 'Dasoli Gram Swaraj Mandal'
in 1964. This institute later became the base of the Chipko 81. The pugmark technique is used –
Movement. Chipko movement started in 1973 against large (a) For bird watching in forests
scale cutting of forests. Sundarlal Bahuguna also played a (b) For breeding rare wildlife in captivity
leading role in this movement. Chandi Prasad Bhatt also (c) For estimation of population of various wild animals
received Ramon Magsaysay Award in 1982. Madhya Pradesh (d) For tattooing wildlife to distinguish one species from
Public Service Commission held option (c) as the correct the other
answer which is wrong. U.P.P.C.S. (Mains) 2008
78. Under whose leadership the movement against Ans. (c)
deforestation was launched in the Raini village of Pugmark is the term used to refer to the footprint of most
Chamoli? animals (especially megafauna). “Pug” means foot in Hindi.
(a) Sunder Lal Bahuguna (b) Chandi Prasad Bhatt Every individual animal species has a distinct pugmark and
(c) Gaura Devi (d) Kalyan Rawat as such, this is used for identification.
Uttarakhand P.C.S. (Pre) 2016
Ans. (c) 82. Consider the following statements:
In 1974, the Forest Department auctioned approximately 680 Assertion (A) : The loss of forest cover in India has been
hectares of forest in Raini Village of Joshimath. Then under directly associated with Demographic
the leadership of Gaura Devi, hundreds of women opposed Transition in India.
the deforestation by sticking to the trees. Reason (R) : The forest cover is negatively related
to population growth.
79. ‘World Wild Life Day’ is observed on:
Select the correct answer from codes given below:
(a) 21st March (b) 2nd May
(a) Both (A) and (R) are true and (R) explains (A).
(c) 5th June (d) 21st June
Uttarakhand P.C.S. (Pre) 2002 (b) Both (A) and (R) are true but (R) does not explain
Ans. (*) (A).
(c) (A) is true, but (R) is false.
World Wildlife Day is celebrated on different dates (d) (A) is false, but (R) is true.
in different countries. In America it is celebrated on U.P.R.O./A.R.O. (Pre) 2016
December 4, in India Wildlife week is celebrated from 2-8 Ans. (a)
October. World Environment Day on June 5 while World
Forest Day is celebrated on March 21. In the 16th (CoP A growing population means increased demand for food and
16) meeting of parties to the Convention on International a corresponding need to convert forests to agriculture land.
Trade in Endangered Species (CITES) held in Bangkok Demographic factors including population growth, density,
in March, 2013, a proposal was placed to declare March distribution, migration and urbanization are important drivers
3rd as World Wildlife Day. On December 20, 2013 in its of deforestation. Hence, both (A) and (R) are true and (R)
68th annual session, United Nations General Assembly explains (A).
decided to celebrate March 3rd as World Wildlife Day. 83. The main causes for the forest loss are –
80. In which of the following years, the Environment (a) Road Development
(Protection) Act passed by the Government of India? (b) River Valley projects
(a) 1982 (b) 1986 (c) Industrial Development
(c) 1990 (d) 1992 (d) Agricultural Development
U.P.P.C.S. (Pre) 2021 U.P.P.C.S. (Mains) 2011

F–136 General Studies Environment and Ecology


Ans. (c) across the country. (1) Shimla (2) Jodhpur (3) Thrissur (4)
The main cause for the forest loss is industrialization. Akola (5) Shillong (6) Bhowali (7) Cuttack (8) Hyderabad
Deforestation occurs due to the urbanization and empty land (9) Ranchi (10) Srinagar.
is being used for agriculture purpose.
87. National Research Centre for Agroforestry is located at:
84. Rajiv Gandhi Wildlife Conservation Award is given to: (a) Agra (b) Jhansi
(a) Educational and Research Institutions (c) Kanpur (d) Lucknow
(b) Forest and wildlife officers Chhattisgarh P.C.S. (Pre) 2019
(c) Wildlife conservationists Ans. (b)
(d) All the above
The National Research Centre for Agroforestry (NRCAF), as
U.P.P.C.S. (Pre) 1998 unit of ICAR (Indian Council of Agricultural Research) was
Ans. (d) established in 1988. The centre is located at Jhansi in Uttar
The Rajiv Gandhi Wildlife Conservation Award is given Pradesh, about 10 kms from Jhansi Railway Station and is
annually for significant contribution in the field of wildlife popularly known as "KRISHIVANIKI".
conservation which has made or has the potential to make, a 88. The greatest diversity of plants and animals is
major impact on the protection and conservation of wildlife characteristic of :
in the country. Two awards of Rupees One Lakh are given
(a) Temperate deciduous forests
to education and research institutions, organization, forest
and wildlife officers/research scholars or scientists/wildlife (b) Tropical moist forests
conservationists. Hence (d) is the correct answer. (c) Savana

85. Which of the following statement regarding Project (d) Temperate grasslands
Elephant launched by Government of India is NOT U.P.P.C.S. (Mains) 2013
correct? Ans. (b)
(a) It was launched in 1991-92 The greatest diversity of plants and animals is found in
(b) It addresses issues of man-animal conflict tropical moist forests.
(c) The welfare of only wild elephants is taken care off
89. In India, in which one of the following types of forests
(d) It was launched as a centrally sponsored scheme.
is teak a dominant tree species?
U.P. R.O./A.R.O. (Mains) 2017
(a) Tropical moist deciduous forest
Ans. (c)
(b) Tropical rain forest
Project Elephant was launched in February, 1992 as centre (c) Tropical thorn scrub forest
sponsored scheme.
(d) Temperate forest with grasslands
The main objectives were-
I.A.S. (Pre) 2015
a. To Protect elephants, their habitats and corridors.
b. To address issues of man-animal conflict. Ans. (a)
c. The welfare of domesticated elephants. Tropical moist deciduous forests are found in areas of
86. National Bureau of Plant Genetic Resources is moderate rainfall of 100 to 200 cm per annum. The trees of
these forests drop their leaves for about 6-8 weeks during
situated at–
the spring and early summer when sufficient moisture for
(a) New Delhi (b) Kolkata
the leaves is not available. Teak trees are the most dominant
(c) Mumbai (d) Chennai
species of trees found in these forests. Bamboos, Sal,
U.P. P.C.S. (Mains) 2013 Shisham, Sandalwood, Khair, Kusum, Arjun, Mulberry are
Ans. (a) some of the other commercially important species found here.
The National Bureau of Plant Introduction, which was
established in 1976, was renamed as 'National Bureau of Plant High-Level Questions
Genetic Resources' in January 1977. It has its headquarters
in New Delhi. The Organization works as a nodal agency for 1. Which of the following forests is known as the “lungs
activities of plant introduction and germplasm augmentation of the planet earth”?
for use in crop improvement. It has ten regional stations (a) Rainforests of North-East India

Environment and Ecology General Studies F–137


(b) Taiga forest Among the given options Paschim Champaran has larger
(c) Tundra forest area under dense deciduous forest cover.
(d) Amazon rain forest
R.A.S./R.T.S. (Pre) 2013 5. Consider the following programmes:
Ans. (d) 1. Afforestation and development of wastelands
2. Reforestation and replantation in existing forests
The Amazon is also known as Amazonia or the Amazon Jungle,
3. Encouraging the wood substitutes and supplying
is a moist broadleaf forest that covers most of the Amazon
other types of fuel
basin of South America. The Amazon rainforest functions
4. Promotion of the wide use of insecticides and
as a giant air machine that absorbs a large amount of carbon
dioxide and produces oxygen. That is why it is often called pesticides to restrict the loss of forest area from
the “Lungs of the Earth.” It is estimated that the Amazon degradation caused by pests and insects.
rainforest produces more than 20 per cent of world’s oxygen. The National Forest Policy of 1988 includes:
(a) 1,2,3 and 4 (b) 2 and 4
2. Which of the following leaf modifications occur(s) in
(c) 1, 3 and 4 (d) 1, 2 and 3
the desert areas to inhibit water loss?
I.A.S. (Pre) 1996
1. Hard and waxy leaves
Ans. (d)
2. Tiny leaves
3. Thorns instead of leaves The National Forest Policy 1988 includes –
Select the correct answer using codes given below : (i) Afforestation and Wasteland development.
(a) 2 and 3 only (b) 2 only (ii) Reforestation and Replantation in existing forests.
(c) 3 only (d) 1, 2 and 3 (iii) Encouraging wood substitute and supplying other types
I.A.S. (Pre) 2018 of fuel.
Ans. (d) Promotion of wide use of insecticides and pesticides is not
included in the National Forest Policy.
Structural Adaptation is important to the plant's survival in
the desert. Wax coating on leaves prevents water loss through 6. With reference to ‘Red Sanders’ sometimes seen in the
evaporation. Leaves are also smaller in desert plants further news, consider the following statements:
reducing the possibility for water loss. They have hard thick 1. It is a tree species found in a part of South India.
coatings and some are covered in prickly spines to prevent 2. It is one of the most important trees in the tropical
loss.
rainforest areas of South India.
3. In which of the following continents, the tropical Which of the statements given above is/are correct:
deciduous forests are most extensive? (a) 1 only (b) 2 only
(a) Africa (b) Asia (c) Both 1 and 2 (d) Neither 1 nor 2
(c) Australia (d) South America I.A.S. (Pre) 2016
U.P.R.O./A.R.O. (Pre) 2014 Ans. (a)
Ans. (b) Pterocarpus santalinus with the common name red sanders
Tropical deciduous forests or tropical broad-leaf forests are is a species of Pterocarpus endemic found in the southern
dominated by trees that lose their leaves each year. They Eastern Ghats mountain range of South India. This tree is
are found in areas with warm, moist summers and mild valued for the rich red colour of its wood. Hence statement
winters. Deciduous forests are mainly found in the Northern 1 is correct. Red Sanders grow on the Palkonda and
Hemisphere Eastern North America, East Asia, and Europe. Seshachalam mountain range in Andhra Pradesh.
4. Among the following districts, which one has larger 7. Consider the following statements ;
area under dense deciduous forest cover? 1. The definition of "Critical Wildlife Habitat" is
(a) Paschim Champaran (b) Gaya incorporated in the Forest Rights Act, 2006.
(c) Kaimur (d) Nawada 2. For the first time in India, Baigas have been given
(e) None of the above/More than one of the above Habitat Rights.
65th B.P.S.C. (Pre) 2019 3. Union Ministry of Environment, Forest and
Ans. (a) Climate Change officially decides and declare

F–138 General Studies Environment and Ecology


Habitat Rights for Primitive and Vulnerable Tribal permitted. Hence statement 1 is incorrect.
Groups in any part of India. Minor Forest Produce (MFP) is defined under the Scheduled
Which of the statements given above is/are correct? Tribes and other Traditional Forest Dwellers (Recognition of
(a) 1 and 2 only (b) 2 and 3 only Forest Rights) Act, 2006, as all non-timber forest produce of
(c) 3 only (d) 1, 2 and 3 plant origin including bamboo. Hence statement 2 is correct.
I.A.S. (Pre) 2018 The Scheduled Tribes and other Traditional Forest Dwellers
Ans. (a) (Recognition of Forest Rights) Act, 2006 allows ownership
of minor forest produce to forest dwellers. Hence statement
The phrase 'critical wildlife habitat is defined only in the
3 is also correct.
Scheduled Tribes and Other Traditional Forest Dwellers
So option (b) will be the correct answer.
(Recognition of the Forest Rights) Act, 2006. For the first
time in India, Baigas have been given Habitat Rights. Union 10. What were the main objectives of the National Forest
Ministry of Tribal Affair officially decides Habitat Rights for Policy? Choose the correct answer by using codes
Primitive and Vulnerable Tribal Groups in any part of India. given below-
8. Which of the following is NOT a major cause of 1. To ensure ecological balance.
Wildlife Extinction? 2. To promote social forestry.
(a) Loss of natural habitat 3. One-third of the country’s total land to be forested.
(b) To make fire in the forest 4. Encouraging public community participation in
(c) Illegal commercial trade of wildlife forest management.
(d) Fast pace of population growth Codes :
U.P.P.C.S. (Pre) 2020 (a) 1 and 2 (b) 1 and 3
Ans. (d) (c) 1 and 4 (d) 2 and 3
U.P.U.D.A./L.D.A. (Pre) 2001
Loss of natural habitat, illegal commercial trade of wildlife Ans. (c)
and to make fire in the forest, are the major causes of Wildlife
Extinction, whereas fast pace of population growth is not a The basic objectives that should govern the National forest
major cause of wildlife extinction. It increases competition Policy - are the following:
for food and shelter. Maintenance of environmental stability through preservation
9. Consider the following statements : and, where necessary, restoration of the ecological balance
1. As per recent amendment to the Indian Forest that has been adversely disturbed by serious depletion of the
Act, 1927, forest dwellers have the right to fell the forests of the country.
bamboos grown on forest areas. Conserving the natural heritage of the country by preserving
2. As per the Scheduled Tribes and Other Traditional the remaining natural forests with the vast variety of flora and
Forest Dwellers (Recognition of Forest Rights) Act, fauna, which represent the remarkable biological diversity
2006, bamboo is a minor forest produce. and genetic resources of the country.
3. The Scheduled Tribes and Other Traditional Forest
Checking soil erosion and denudation in the catchment areas
Dwellers (Recognition of Forest Rights) Act, 2006
of rivers, lakes, reservoirs in the "interest of soil and water
allows ownership of minor forest produce to forest
conservation, for mitigating floods and droughts and for the
dwellers.
retardation of siltation of reservoirs.
Which of the statements given above is/are correct?
Checking the extension of sand-dunes in the desert areas of
(a) 1 and 2 only (b) 2 and 3 only
(c) 3 only (d) 1, 2 and 3 Rajasthan and along the coastal tracts.
I.A.S. (Pre) 2019 Increasing substantially the forest/tree cover in the country
Ans. (b) through massive afforestation and social forestry programmes,
especially on all denuded degraded and unproductive lands.
As per recent amendment to the Indian Forest Act, 1927,
Meeting the requirements of fuel-wood, fodder, minor forest
'Bamboo' plant growing on non-forest areas are exempted
from definition of tree. According to the changed law produce and small timber of the rural and tribal populations.
(Indian Forest (Amendment) Ordinance, 2017) felling or Increasing the productivity of forests to meet essential
transportation of bamboo grown on non-forest areas is legally national needs.

Environment and Ecology General Studies F–139


Encouraging efficient utilisation of forest produce and 13. Which of the following city of India is specialized in
maximising substitution of wood. the plantation?
Creating a massive people's movement with the involvement (a) Vijayawada (b) Chandigarh
of women, for achieving these objectives and to minimise (c) Shillong (d) Valparai
pressure on existing forests. U.P.R.O./A.R.O. (Mains) 2013
11. Arrange the following States of India in descending Ans. (d)
order of their cover (percent of forest area to total area)
Valparai is located in Coimbatore district of Tamil Nadu.
and select the correct answer from codes.
This Indian city has specialization in plantation among the
1. Haryana 2. Maharashtra
given options.
3. Manipur 4. Orissa
Code : 14. Which country has the highest percentage of its
(a) 3, 4, 2, 1 (b) 3, 2, 4, 1 geographical area under forest?
(c) 1, 4, 3, 2 (d) 4, 3, 2, 1 (a) China (b) India
U.P.P.C.S. (Mains) 2002 (c) Indonesia (d) Japan
Ans. (a) U.P.P.C.S.(Pre) 2013
According to ISFR, 2019 & 2021, the States of India in Ans. (d)
descending order of their forest cover (percentage of forest According to the World Factbook (Recent) of the CIA, the
area to total area) are as follows- percentage of forestation in relation to their geographical
Percent of Forest Area area, for the countries given in options is as follows-
State 2019 2021
Country Forest cover (in%)
Manipur - 75.46% 74.34%
Japan - 68.6
Odisha - 33.15% 33.50%
Maharashtra - 16.50% 16.51% Indonesia - 51.7
Haryana - 3.62% 3.63% India - 23.1
China - 22.3
12. Arrange the following State of India in descending
order of their Forest-area and select the correct answer 15. Which of the following countries has passed a
from codes given below: resolution to maintain forest on 60% land of its
1. Andhra Pradesh 2. Arunachal Pradesh geographical area?
3. Chhattisgarh 4. Odisha (a) Maldives (b) Nepal
Codes: (c) Bhutan (d) Afghanistan
(a) 1, 3, 4, 2 (b) 1, 2, 3, 4 U.P.P.C.S. (Mains) 2016
(c) 4, 3, 1, 2 (d) 2, 1, 4, 3
Ans. (c)
U.P.P.C.S (Pre) 2011
Ans. (*) The government of Bhutan has passed a resolution to
maintain forest on 60% land of its total geographical area.
The question is based on recorded forest cover of these States.
Presently, around 84% of its territory is covered with forests.
The recorded forest cover (in sq. km) of the given States in
2017 and 2019 was as below – 16. Assertion (A) : Aluminium is a green metal.
Reason (R) : It substitutes wood for saving the forest.
State 2017 2019
Select the correct answer from codes given below:
Andhra Pradesh - 28147 37258
Chhattisgarh - 55547 59772 (a) Both (A) and (R) are true and (R) is the correct
Odisha - 51345 61204 explanation of (A).
Arunachal Pradesh - 66964 51407 (b) Both (A) and (R) are true, but (R) is the correct
Notably, the question was asked before the bifurcation of explanation of (A).
Andhra Pradesh. During that period Telangana was a part of (c) (A) is false, but (R) is true.
Andhra Pradesh and the total recorded forest area of Andhra (d) (A) is true, but (R) is false.
Pradesh was 63814 sq.km. According to this the correct U.P.P.C.S. (Mains) 2009
answer to this question was option (a). Ans. (d)

F–140 General Studies Environment and Ecology


Aluminium is known as green metal due to its eco-friendly I.A.S. (Pre) 2011
and renewable specialities. Aluminium can be a substitute of Ans. (d)
wood, but the reason behind this substitution is not to save Mangroves can serve as a reliable safety hedge against
the forests (at least in a substantive way). Thus, (A) is correct coastal calamities. The mangrove trees do not get uprooted
while (R) is not correct. by storms and tides because of their extensive roots. Hence
(d) is the correct answer.
17. Which one of the following eco-regions of India is not
correctly matched? 20. Assertion (A) : Coastal Odisha is most cyclone-prone
(a) South-Western Ghats - Moist forests area of India.
(b) Tarai Duar - Broadleaf forests Reason (R) : There is a large amount of deforestation
(c) Rann of Kachchh - Grasslands of Mangroves in Mahanadi delta area.
(d) Eastern Deccan Plateau - Moist forests Choose the correct answer in the given codes :
U.P. Lower Sub. (Pre) 2013 Code :
Ans. (d) (a) Both (A) and (R) are true and (R) is the correct
explanation of (A).
It is moist deciduos forests which are found in Eastern Ghats,
(b) Both (A) and (R) are true, but (R) is the correct
not moist forests. Other options are correctly matched.
explanation of (A).
18. Monoculture of commercially viable trees is destroying (c) (A) is false, but (R) is true.
the unique natural profile of – Thoughtless exploitation (d) (A) is true, but (R) is false.
of timber, deforesting vast tracts for palm cultivation, U.P.P.S.C. (GIC) 2010
destruction of mangroves, illegal logging by tribals U.P. Lower Sub. (Spl) (Pre) 2006
and poaching only compound the problem. Freshwater Ans. (a)
pockets are fast drying up due to deforestation and
Coastal Odisha is the most affected area due to the cyclones
destruction of mangroves”. The place referred to this
generated in the Bay of Bengal. Here, the frequency of
quotation is: cyclone is more as compared to the other States. Hence,
(a) Sunderbans Assertion (A) is correct. Mangrove can serve as a reliable
(b) Kerala coast safety hedge against coastal calamities. Recently, there was
(c) Orissa coast massive deforestation along the Mahanadi delta area, which
(d) Andaman and Nicobar Island has added the risks arising due to cyclones. Hence, both the
I.A.S. (Pre) 1995 assertion and reason are correct and the reason is the correct
Ans. (a) explanation of the assertion.
The place mentioned in the quotation of the question seems 21. Consider the following statements about Bhitarkanika
near Sundarban. Effects are on to conserve mangrove in Mangroves:
Sundarbans. 1. It is located in the deltaic region of Vansadhara
19. The 2004 Tsunami made people realize that mangroves and Subarnarekha rivers.
can serve as a reliable safety hedge against coastal 2. It lies in the state of West Bengal.
calamities. How do mangroves function as a safety Of these:
hedge? (a) Only 1 is correct.
(a) The mangrove swamps separate the human settlements (b) Only 2 is correct.
from the sea by a wide zone in which people neither (c) 1 and 2 both are correct
live nor venture out (d) Neither 1 nor 2 is correct.
(b) The mangroves provide both food and medicines U.P.R.O./A.R.O. (Pre) 2016
which people are in need of after any natural disaster. Ans. (d)
(c) The mangrove trees are tall with dense canopies See the explanation of the above question.
and serve as an excellent shelter during a cyclone or
Tsunami 22. Consider the following statements:
(d) The mangrove trees do not get uprooted by storms 1. The Taxus tree naturally found in the Himalayas
and tides because of their extensive roots 2. The Taxus tree is listed in the Red Data Book.

Environment and Ecology General Studies F–141


3. A drug called ‘taxol’ is obtained from Taxus tree (a) 1 only (b) 2 only
which is effective against Parkinson’s disease. (c) Both 1 and 2 (d) Neither 1 nor 2
Which of the statements given above is/are correct ? I.A.S. (Pre) 2017
(a) 1 only (b) 1 and 2 Ans. (b)
(c) 2 and 3 (d) 1, 2 and 3
Trade Related Analysis of Fauna and Flora in Commerce
I.A.S. (Pre) 2010
(TRAFFIC) is a non-government organization. It serves to
Ans. (d)
monitor the trade of wild plants and animals in the context
The Taxus tree is naturally found in the Himalayas. The of sustainable development and biodiversity conservation.
Taxus tree is listed in the Red Data Book. A drug called The mission of TRAFFIC is to ensure that the trade of wild
Taxol is obtained from Taxus tree which is used in breast and plants and animals does not threaten the conservation of
lung cancer treatment and also effective against Parkinson’s nature. TRAFFIC was established in 1976. It is a strategic
disease. Hence all the statements are correct. alliance of WWF and IUCN. It is not a bureau under UNEP.
23. Consider the following statement : 25. Consider the following:
1. The Worldwide Fund for Nature was established 1. Star Tortoise 2. Monitor lizard
in 1961. 3. Pygmy hog 4. Spider monkey
2. The death of 13 lions in Nandan Forest Sanctuary Which of the above are naturally found in India?
of Odisha in July, 2000 was due to trypanosomiasis
(a) 1, 2 and 3 (b) 2 and 3
disease.
(c) 1 and 4 (d) All of these
3. The tree eucalyptus is named as eco-friend.
I.A.S. (Pre) 2013
4. The biggest aquarium in India is situated in
Ans. (a)
Kolkata.
Of these statements: The spider monkies are found in tropical forests of Central
(a) 1, 2 and 3 are correct and South America, from southern Mexico to Brazil. Rest all
(b) 2, 3 and 4 are correct are naturally found in India. Hence (a) is the correct answer.
(c) 1, 2 and 4 are correct 26. Match List-I (Indian Wild Life species) with List-II
(d) 1 and 2 are correct (Scientific names) and select the correct answer using
U.P.P.C.S. (Mains) 2002 codes given below:
Ans. (c)
List - I List-II
The World Wide Fund for Nature (WWF) is an international (Indian Wild Life Species) (Scientific Names)
non-governmental organization founded in 1961. The period A. Asiatic Wild Ass 1. Boselaphus
of June and July, 2000 was an ominous period for the Tigers tragocamelus
in zoos of India because as many as 13 tigers died in Nandan B. Barasingha 2. Cervus duvauceli
Kanan Zoo (Eastern India) due to Trypanosomiasis. The C. Chinkara 3. Equus hemionus
biggest aquarium in India is situated in Kolkata. Eucalyptus D. Nilgai 4. Gazella Benetty
is called as the environmental enemy, not friend. Hence Code :
option(c) will be the correct answer. A B C D
(a) 2 3 1 4
24. Consider the following statements in respect of Trade
(b) 3 2 4 1
Related Analysis of Fauna and Flora in Commerce
(c) 2 3 4 1
(TRAFFIC) :
(d) 3 2 1 4
1. TRAFFIC is a bureau under the United Nations
I.A.S. (Pre) 2002
Environment Programme (UNEP).
Ans. (b)
2. The mission of TRAFFIC is to ensure that trade
in wild plants and animals is not a threat to the The correct match of list A with list B is as follows-
conservation of nature. Asiatic wild ass - Equus hemionus
Which of the above statements is/are correct? Barasingha - Cervus duvauceli

F–142 General Studies Environment and Ecology


Chinkara - Gazella bennetty 30. Which country has the largest percentage of its total
area under forests?
Nilgai - Boselaphus tragocamelus
(a) Japan (b) Indonesia
27. In which one of the following States is Pakhui Wildlife (c) Suriname (d) Guyana
Sanctuary located? U.P.P.C.S. (Mains) 2009
(a) Arunachal Pradesh (b) Manipur Ans. (c)
(c) Meghalaya (d) Nagaland According to the World Factbook, (Recent) of CIA. Countries
I.A.S. (Pre) 2018 with the highest percentage of forest cover of their total area
Ans. (a) are as follows-
Pakhui is a wildlife sanctuary and a dedicated tiger reserve Suriname - 94.6%
(also known as the pakke tiger reserve) located in the district Guyana - 77.4%
of East Kameng in Arunachal Pradesh. Japan - 68.5%
28. Consider the following fauna of India : Indonesia - 51.7%
1. Gharial 31. Given below are two statements, one labelled as
2. Leatherback turtle Assertion (A) and the other labelled as Reason (R):
3. Swamp deer Assertion (A): Mangroves are very specialized forest
Which of the above is/are endangered? ecosystems of tropical and subtropical
(a) 1 and 2 (b) Only 3 regions bordering certain sea coasts.
(c) All of these (d) None of these Reason (R) : They stabilize the shoreline and act as
I.A.S. (Pre) 2013 a bulwark against encroachments by
Ans. (c) sea.
In the context of the above two statements which one
Gharial, Leatherback turtle and Swamp deer all of three come
of the following is correct?
under the endangered category.
(a) Both A and R are true and R is the correct explanation
29. What is/are unique about ‘Kharai camel’, a breed of A
found in India? (b) Both A and R are true but R is not a correct explanation
1. It is capable of swimming up to three kilometres of A
in seawater. (c) A is true but R is false
2. It survives by grazing on mangroves. (d) A is false but R is true
3. It lives in the wild and cannot be domesticated. I.A.S. (Pre) 1996
Select the correct answer using the codes given below: Ans. (a)
(a) 1 and 2 only (b) 3 only Mangroves are very specialized forest eco-system of
(c) 1 and 3 only (d) 1, 2 and 3 tropical and subtropical areas bordering certain sea coasts.
I.A.S. (Pre) 2016 Mangroves stabilize the shoreline and act as a bulwark against
Ans. (a)
encroachments by the sea which make them specialized
Kharai Camel or Swimming Camels are found only in forest ecosystem. That is why the Government is protecting
Gujarat’s Bhuj area. It has been recently recognized as Sundarban mangrove. Hence both the Assertion and Reason
a separate breed (one among nine such breeds found in are correct and Reason is the correct explanation of Assertion.
India) of a camel for better conservation. It can swim up
32. Amrita Devi Smriti Award is given for:
to three kilometres into the sea in search of mangroves, its
(a) Best afforestation efforts
primary food. Kharai Camel can live in both coastal and
(b) Protection of forests and wildlife
dry ecosystems. Given the breed’s ability to survive both on
(c) Best literature on forest protection
land and sea, the Kharai camel is one of the most preferred (d) Beautification of Parks
choices of graziers in the arid coastal region of Kachchh. R.A.S./R.T.S.(Pre) 2003
Hence statement 3 is not correct. Ans. (b)

Environment and Ecology General Studies F–143


Amrita Devi Bishnoi award is given by the forest department rhododendron are also found in these areas. Sandalwood is a
of Government of Rajasthan for significant contribution tropical deciduous tree which is not found in the Himalayan
in the field of wildlife protection. A cash award of Rupees region. If you travel through Himalaya, it is quite possible
One lakh is presented to individuals/institutions involved in to see the trees like Rhododendron and Oak but sandalwood
wildlife protection. trees would not be seen here. For examining the forest of
Sandalwood, you have to travel towards South India.
33. The World Tiger Summit, 2010 was held at –
(a) Bangkok (b) Nairobi 36. In Uttarakhand, Oak-Rhododendron are characteristic
(c) New Delhi (d) Petersburg plants of
U.P.P.C.S (Pre) 2011 (a) Sub-tropical forest (b) Sub-alpine forest
Ans. (d) (c) Temperate forest (d) All of the above
Uttarakhand P.C.S. (Pre) 2016
The first World Tiger Summit was held in 2010 between 21
to 24 November at Saint Petersburg (Russia). Ans. (c)
In Uttarakhand, the Oak-Rhododendron are characteristic
34. As a part of wildlife conservation efforts in Nepal
plants found in temperate forests. The temperate zone is
and India, a new association by the name ‘SAVE’ was
marked by the presence of Quercus leucotrichophora (Banj
launched recently. The objective of SAVE is to conserve Oak), Rhododendron arboretum (Burans), Myrica esculent
(a) Vultures (b) Tigers (Kaphal), Lyonia ovalifolia (Aynor), Ilex dipyrena, Quercus
(c) Parrots (d) Elephants semecarpifolia (Kharsu oak) etc.
U.P.P.C.S (Mains) 2011
37. Every year, a month long ecologically important
Ans. (b)
campaign/festival is held during which certain
As a part of wildlife conservation efforts in Nepal and communities/ tribes plant saplings of fruit-bearing
India, a new association by the name ‘SAVE’ was launched
trees. Which of the following are such communities/
recently. The objective of SAVE is to conserve tigers. Tibetan
tribes?
Buddhists use tiger skin for decoration and as a seat while
(a) Bhutia and Lepcha (b) Gond and Korku
offering prayers which results in an increase in tiger hunting
cases. India and Nepal jointly formed an organization named (c) Irula and Toda (d) Sahariya and Agariya
‘SAVE’ in July, 2010, for the conservation of tigers. It also I.A.S. (Pre) 2014
has the objective to work for increasing the number of tigers. Ans. (b)

35. If you travel through the Himalayas, you are likely to Gond and Korku tribes plant a sapling of fruit-bearing trees
every year during a month-long ecological company/festival.
see which of the following plants naturally growing
there? 38. In a particular region in India, the local people train
1. Oak the roots of living trees into robust bridges across the
2. Rhododendron streams. As time passes, these bridges become stronger.
3. Sandalwood These unique living root bridges’ are found in
Select the correct answer using the Code given below: (a) Meghalaya (b) Himachal Pradesh
(a) 1 and 2 only (b) 3 only (c) Jharkhand (d) Tamil Nadu
(c) 1 and 3 only (d) 1, 2 and 3 I.A.S. (Pre) 2015
I.A.S. (Pre) 2014 Ans. (a)
Ans. (a) Living root bridges are a form of tree shaping common in
Subtropical forests are found in North-West (except Kashmir) the southern part of the Northeast Indian State of Meghalaya.
Khasi hills, Nagaland and Manipur. Pine is the main tree in They are hand made from the aerial roots of rubber trees by
these forests but the trees like Oak having broad leaves can be the Khasi and Jaintia peoples of the mountainous terrain
easily seen in highly humid areas. Apart from this, shrubs like along the southern part of the Shillong Plateau.

F–144 General Studies Environment and Ecology


39. Consider the following statements: 41. Match list-I with list-II and select the correct answer
The Environment Protection Act, 1986 empowers the using the code given below -
Government of India to : List-I List-II
1. state the requirement of public participation in (Name of the Act (Year)
the process of environmental protection, and the in India)
procedure and manner in which it is sought (A) Forest Conservation Act (1) 1980
2. lay down the standards for emission or discharge (B) The Environment (2) 1986
of environmental pollutants from various sources Protection Act
Which of the statements given above is/are correct? (C) The Air (Prevention and (3) 1981
(a) 1 only (b) 2 only Control of Pollution) Act
(c) Both 1 and 2 (d) Neither 1 nor 2 (D) The Water Pollution (4) 1974
I.A.S. (Pre) 2019 Control Act
Ans. (b) Code :
The Environment Protection Act, 1986 does not empower A B C D
the Government of India to state the requirement of Public (a) 3 1 4 2
Participation in the process of environmental protection (b) 1 4 2 3
and the procedure and manner in which it is sought. Hence (c) 4 3 2 1
statement 1 is incorrect. This Act lays down the standard (d) 1 2 3 4
for emission or discharge of environmental pollutants from U.P.R.O./A.R.O. (Pre) 2021
various sources. Hence statement 2 is correct. Therefore
Ans. (d)
option (b) will be the correct answer.
The Forest (Conservation) Act, 1980 an Act of the Parliament
40. Consider the following statements : of India to provide for the conservation of forests and for
1. Animal Welfare Board of India is established under matters connected therewith or ancillary or incidental thereto.
the Environment (Protection) Act, 1986. It was further amended in 1988. It was enacted by Parliament
2. National Tiger Conservation Authority is a of India to control further deforestation of Forest Areas in
statutory body. India. The act came into force on 25 October 1980.
3. National Ganga River Basin Authority is chaired The Environment (Protection) Act was enacted in 1986 with
by the Prime Minister. the objective of providing for the protection and improvement
Which of the statements given above is/are correct? of the environment. It empowers the Central Government
(a) 1 only (b) 2 and 3 only to establish authorities [under section 3(3) charged with the
(c) 2 only (d) 1, 2 and 3 mandate of preventing environmental pollution in all its
I.A.S. (Pre) 2014 forms and to tackle specific environmental problems that
are peculiar to different parts of the country. The Act was
Ans. (b)
last amended in 1991.
The Animal Welfare Board of India is a statutory advisory Air (Prevention and control of pollution) Act, 1981 an act
body on animal Welfare Laws and promotes animal welfare to provide for the prevention, control and abatement of air
in the country, established in 1962 under Section 4 of the pollution, for the establishment, with a view to carrying out
Prevention of Cruelty to Animals Act, 1960. Hence Statement the aforesaid purposes, of Boards, for conferring on and
1 is not correct. National Tiger Conservation Authority is assigning to such Boards powers and functions relating there
Statutory Body under the Ministry of Environment, Forests to and for matters connected therewith.
and Climate Change (Government of India). Hence Statement The Water (Prevention and Control of Pollution) Act was
2 is correct. The National Ganga River Basin Authority was enacted in 1974 to provide for the prevention and control
set up on 20 February, 2009. This authority is chaired by the of water pollution, and for the maintaining or restoring of
Prime Minister and has its members the Union Ministers wholesomeness of water in the country. The Act was amended
concerned, the Chief Ministers of the States through which in 1988.
Ganga flows. Hence Statement 3 is correct. Hence option (d) is correct.

Environment and Ecology General Studies F–145


Sanctuaries/Biosphere Reserves
National Park
*Various methods are adopted for the conservation of
biodiversity. Establishing protected / conserved area is an
important method. Protected areas include National Parks,
Wildlife Sanctuaries, Conservation Reserves and Community
Reserves. National Parks and Wildlife Sancturies have been
established for the Protection and Conservation of the wildlife
in the country. A network of 981 protected areas (PAs)
has been established, extending over 1,71,921 km2 (5.03%
of the total geographic area of the country) comprising 104
National Parks, 566 Wildlife Sanctuaries, 97 Conservation
Reserves and 214 Community Reserves. Most of the wildlife
conservation areas are surrounded by the forest which is very *Biosphere Reserve is an area having flora and fauna of natural
beneficial. and scientific interest. Biosphere Reserves entertain no human
According to the data shown by 'ENVIS centre on Wildlife activity in their sphere. In a sanctuary, hunting is prohibited
without permission but grazing of cattle is regular.
Protected Areas', number of protected areas in India during
*In the National Parks hunting and grazing of the cattle is
2020 are as follows –
completely prohibited. Some restricted human activity is
Status in permitted in the sanctuary but no human activity is permitted
Protected areas/Area
Dec., 2020 in the National Park except for the ones permitted by the
Total number of protected areas 981 Chief Wildlife Warden of the State. There are 104 National
Total area of protected areas 1,71,921 Parks in India. Madhya Pradesh has the largest number of
Percentage of protected areas to National Parks (11) followed by Andaman & Nicobar, Kerala,
5.03%
the total geographical area of the country Maharashtra, Uttarakhand and West Bengal each having 6.
National Parks 104 Andaman & Nicobar Islands have the maximum number (96)
of Wildlife Sanctuaries.
Wildlife Sanctuaries 566
*Jim Corbett National Park is the first National Park of
Conservation Reserves 97
India. It was established in 1936 as Hailey National Park. It
Community Reserves 214 is located in Nainital District of Uttarakhand. In 1973, it was
*A National Park is an area, whether within a sanctuary or not, chosen as the location for launching Project Tiger. Both Jim
Corbett and Rajaji National Park, located in Uttrakhand use
that can be notified by the State Government to be constituted
remote sensing and GIS approach for Wildlife Management.
as a National Park. Any ecosystem of rich biodiversity and
*Keoladeo National Park was established as a National Park in
almost pristine habitats can be declared National Parks
1981. It is located in Bharatpur district of Rajasthan. In 1985,
provided that such natural area is of ecological, faunal, floral, Keoladeo National Park has been declared a World Heritage
geomorphological and hydrological importance and needs site under the World Heritage convention by UNESCO.
protection and conservation. *Keoladeo Ghana National Park was formerly known as the

F–146 General Studies Environment and Ecology


Bharatpur Bird Sanctuary. 366 bird species, 379 floral Sanctuary (established in 1977) were merged in 1983 to form
species, fish, snakes, lizards, amphibians, turtles and a variety Rajaji National Park. Rajaji National Park was declared as a
of other invertebrates are found in Keoladeo National Park. But Tiger Reserve in 2015.
Tiger is not a protected species here. *The Great Himalayan National Park is spread in the Kullu
*Ramganga and Kosi rivers flow through the Corbett National region of Himachal Pradesh and Van Vihar National Park is
Park. Brahmaputra, Diphlu, Mora Diphlu and Mora Dhansiri spread in the Bhopal region of Madhya Pradesh.
rivers flow through Kaziranga National Park. *Gulf of Mannar is a Marine National Park. It is situated in
*The Kuntipuzha river flows through Silent Valley National Tamil Nadu and was established in 1980.
Park. *Garo Hill is a part of Garo-Khasi range located in Meghalaya.
*The rehabilitation of rhinoceros is going on in Dudhwa Nokrek is the highest peak of the Garo hills. Nokrek Biosphere
National Park. It is located in Lakhimpur Kheri district of Reserve is established here.
Uttar Pradesh. The First Rhino rehabilitation programme *Loktak Lake is the largest freshwater lake in North-East
was initiated in Dudhwa in 1984. Second Rhino rehabilitation India and it is situated in Manipur.
programme has been initiated in Dudhwa in 2018. *Namdapha National Park is situated in Arunachal Pradesh.
*Bandipur National Park is located in the South Indian State It is the largest protected area of an eastern Himalayan
of Karnataka. biodiversity hotspot.
*Pin Valley National Park is located within the Lahaul and *Buxa Tiger Project is located near Darjeeling in West Bengal.
Spiti district in the State of Himachal Pradesh. It was given the status of National Park in 1992.
*Sariska Tiger Reserve and Ranthambore National Park *Nagarhole National Park is located at a distance of 94 km
both are located in Rajasthan and both are Tiger Reserves. from Mysore in Karnataka. It is also known as Rajiv Gandhi
*Sariska Tiger reserve is located in Alwar district of Rajasthan. National Park. It is spread in Kodagu and Mysore district of
It was declared a Wildlife Sanctuary in 1955 and given the status Karnataka.
of Tiger Reserve in 1978. *Betla (Palamu) National Park was established in Palamu
*Salim Ali National Park is located in Jammu and Kashmir. district of Bihar (present-day Jharkhand) in 1986. It falls under
City Forest National Park was renamed as Salim Ali National the Project Tiger.
Park. *Gir National Park is located in Junagarh district of Gujarat.
*Indravati National Park is located in the Dantewada district It is famous for being the habitat of Asiatic Lion.
of Bastar region of Chattisgarh. Indravati was declared as a *Hoolock Gibbon is found is the north-eastern states of India
National Park in the year 1982. The total area of the Park is especially Assam.
about 1258.37 km . In the year 1983, Indravati National Park
2
*Bhitarkanika National Park is situated in Odisha and is
has declared a Tiger Reserve under the Project Tiger of India. home to saltwater crocodile.
*Bandhavgarh National Park is located in the Umaria in *Great India bustard is the centre of attraction of the Desert
Madhya Pradesh. In 1968, it was declared a National Park. It National Park located in Rajasthan.
is famous for White Tigers. *The first National Park of USA is Yellow Stone National Park.
*Dandeli Sanctuary is located in Karnataka. It is spread over It was established in 1872 spread across Wyoming, Montana
an area of 886.41 km . 2
and Idaho states of USA. Old Faithful Geyser is located in this
*Rajaji National Park is spread in three districts of National Park. Grand Canyon is also situated here.
Uttarakhand. These three districts of Uttarakhand are Dehradun,
Haridwar and Pauri Garhwal. It has been named after C.
National Park List
Rajagopalachari, who is also known as Rajaji, become the *National Parks has been established in India from time to
only Indian and last governor-general after, independence. time. The first National Park which was established in India
*Three wildlife sanctuaries Rajaji Sanctuary (established in was Jim Corbett National Park. Simlipal National Park is
1948), Motichur Sanctuary (established in 1964) and Chilla located in Mayurbhanj district of Odisha which is 320 km from

Environment and Ecology General Studies F–147


Bhubaneswar. It was established in 1980. tiger reserve and elephant reserve.
*Some National Parks and their establishment year- *Keibul Lamjao National Park is located in Manipur. It is the
National Park Year only floating park in the world. Keibul Lamjao National Park
Corbett (Uttarakhand) 1936 is characterized by many floating decomposed plant materials
Kanha (Madhya Pradesh) 1955 which are known as Phumdis.
Dudhwa (Uttar Pradesh) 1977 *Great Himalayan National Park (GHNP) is located in
Rajaji (Uttrakhand) 1983 Kullu district of Himachal Pradesh. This park is famous for its
Namdapha (Arunachal Pradesh) 1983 biodiversity. Himalayan Brown bear is found in the temperate
Guindy (Tamil Nadu) 1976 and alpine forest in GHNP. More than 30 mammals and more
Kaziranga (Assam) 1974 than 200 birds reside here.
Silent Valley (Kerala) 1984 *World Heritage Natural site status was conferred on GHNP on
23rd June 2014 in the proceedings of the 38th World Heritage
National Park State
Committee meeting in Doha Qatar.
Gir Forest National Park Gujarat
*Sultanpur National Park (formerly Sultanpur Bird
Bharatpur Bird Santuary Rajasthan
Sanctuary) is located in Gurgaon district of Haryana.
Bandhavgarh National Park Madhya Pradesh
*National Parks of Bhutan is spread in an area of about 12,922
Kaziranga National Park Assam
km2 which is approximately 33.66% of the total area of Bhutan
Corbett National Park Uttrakhand
Bandipur National Park Karnataka (38,394 km2).
Sanjay National Park Madhya Pradesh *Delhi Zoo was officially given the status of National
Rajaji National Park Uttrakhand Zoological Park in 1982.
Simlipal National Park Odisha Sanctuary / National Park Wild Life
Manas National Park Assam Kaziranga Rhinoceros
Anshi National Park Karnataka
Gir Lion
Betla National Park Jharkhand
Sunderban Tiger
Indravati National Park Chhattisgarh
Periyar Elephant/Tiger
Gugamal National Park Maharashtra
Intanki National Park Nagaland National Park Reserve
Keibul-Lamjao National Park Manipur Bandipur Tiger Sanctuary
Guindy National Park Tamil Nadu Kaziranga One Horned Rhinoceros Sanctuary
Dachigam National Park Jammu and Kashmir Sunderban Biosphere and Tiger Reserve
Papikonda National Park Andhra Pradesh Simlipal Elephant Sanctuary
Sariska National Park Rajasthan

National Park District Biosphere Reserve


Kaziranga Golaghat, Nagaon and Sonitpur *The term Biological diversity was first time used by
Silent Valley Palghat Raymond F. Dasmann in 1968 in his book “A Different kind
Pench Valley Nagpur of Country”. The contracted form of Biological Diversity is
Kanha National Park Mandla, Balaghat and Dindori (MP) biodiversity which was coined by W.G. Rosen in 1985.
Kudremukh Chikkamaglur *The biosphere reserve programme is an effective method of
*The largest surviving population of Nilgiri Tahr is found in the conservation of genetic variation of the species. So far
Eravikulam National Park which is located along Western Ghats 18 Biosphere reserve has been established by the Ministry of
in the Idukki district of Kerala. Environment, Forest and Climate Change of Government of
*Periyar National Park is located in Kerala. It is mainly a India of which 12 Biosphere reserves are within the framework

F–148 General Studies Environment and Ecology


of UNESCO’s Man and Biosphere (MAB) programme. *Seshachalam Hills (Situated in Andhra Pradesh) of Eastern
Ghats was declared as 17th Biosphere Reserve of the country
on 20 September 2010.
Transition area *Panna (located in Madhya Pradesh) was declared as
18th Biosphere Reserve of the country on 25 August 2011.
Buffer zones
Establishment of Biosphere Reserve Areas is helpful in
Core area conservation and sustainable development of biodiversity.
List of Biosphere Reserves in India
Education & Training Year of Area (in
Biosphere Reserves
Establishment Km2)
1. Nilgiri 1986 5520
2. Nanda Devi 1988 5860.69
*These 12 Biosphere Reserves are :
3. Gulf of Mannar 1989 10500
UNESCO - MAB Biosphere Reserve 4. Nokrek 1988 820
Biosphere Reserve Designated Year
5. Sunderban 1989 9630
1. Nilgiri 2000
6. Manas 1989 2837
2. Gulf of Mannar 2001
7. Simlipal 1994 4374
3. Sunderban 2001
4. Nanda Devi 2004 8. Dihang-Dibang 1998 5112
5. Nokrek 2009 9. Pachmarhi 1999 4926
6. Pachmarhi 2009 10. Achanakmar-Amarkantak 2005 3835.51
7. Similipal 2009 11. Kachchh 2008 12454
8. Achanakmar-Amarkantak 2012 12. Cold Desert 2009 7770
9. Great Nicobar 2013 13. Khangchendzonga 2000 2620
10. Agasthyamalai 2016
14. Agasthyamalai 2001 3500
11. Khangchendzonga 2018
15. Great Nicobar 1989 885
12. Panna 2020
16. Dibru-Saikhowa 1997 765
*Nanda Devi Biosphere Reserve is situated around the peak 17. Seshachalam Hills 2010 4756
of Nanda Devi in the state of Uttrakhand. It was inscribed 18. Panna 2011 2998.98
as a World Heritage site by UNESCO in 1988. Nanda Devi
*Rann of Kachchh was initially not listed in Biosphere Reserve
Biosphere reserve has been included within the framework of
declared by Government of India but in January 2008, it was
UNESCO’s Man and Biosphere programme.
given the status of Biosphere Reserve.
*In 2016, Agasthyamalai was included in the framework of
*Man and the Biosphere Programme (MAB) is an
UNESCO’s Man and Biosphere programme.
intergovernmental scientific programme, launched in 1971
*Khangchendzonga and Panna was included in the
by UNESCO, that aims to establish a scientific basis for
framework of UNESCO’s Man and Biosphere programme in
the improvement of relationships between people and their
2018 and 2020 respectively. environments.
Important Biosphere Reserve Area *Nilgiri, Sundarban, Gulf of Mannar and Nanda Devi were
*Biosphere Reserves are the major centres for the protection selected from India in the first list of the Man and the Biosphere
of ecosystems, races and genetic resources. Other facts related Programme.
to biosphere reserve areas are as follows: *UNESCO declared on May 2007 that 22 new Bio sanctuary
*Cold desert is declared as 16th Biosphere Reserve area of will be selected for the Man and Biosphere programme. These
India on 28 August, 2009 and is located in Himachal Pradesh. 22 new Bio Sanctuary will be selected from 17 countries.

Environment and Ecology General Studies F–149


Simlipal, Pachmarhi and Nokrek are from India.
Agasthyamalai Kerala-Tamil Nadu
*Later in the year 2012 Achankamar - Amarkantak Biosphere
Namdapha Arunachal Pradesh
Reserve was selected from India for MAB. Great Nicobar was
Sariska Rajasthan
selected for the Man and Biosphere programme in 2013.
*By March 2016 the MAB list of UNESCO had 669 Biosphere Garampani Assam
Reserves from 120 different countries. Pakhal Telangana
*By February 2021, there are 714 Biosphere Reserves in 129 Mahuadanr Jharkhand
countries have been included in the world network of biosphere Dachigam Jammu and Kashmir
reserves.
Kahna National Park Madhya Pradesh
Biosphere Reserve State Periyar Kerala
Nokrek Meghalaya Keoladeo Ghana Bird Sanctuary Rajasthan
Manas Assam
*Bondla Wildlife Sanctuary - Goa (located in North-East of
Dihang-Dibang Arunachal Pradesh
Goa and is spread over an area of 8 km2 is the centre of attraction
Agasthyamalai Kerala
for school students).
Similipal Odisha
*Kanger Ghati National Park is located in Chhattisgarh.
Khangchendzonga Sikkim *Orang Sanctuary is located in Assam and it is also known
Cold Desert Himachal Pradesh as mini Kaziranga.
*Gulf of Mannar (area = 10500 Km2) is the largest UNESCO *Ushakothi Sanctuary is situated at a distance of about 45 km
certified (according to the area) biosphere reserve of India but east of Sambalpur in Odisha.
it is the second largest biosphere reserve of India. *Shuklaphanta wildlife sanctuary is a protected area in Terai

*The largest biosphere reserve of India is Rann of Kachchh (South-West area) of Kanchan district of Nepal covering an

(Gujarat) with an area of 12454 km2. Nokrek Biosphere is area of 305 km2. Hunting was prohibited in this area in 1969. It

spread over part of Garo Hills while Agasthyamalai is on the was gazetted in 1976 as Royal Shuklaphanta Wildlife Reserve.

Western Ghats, Dihang-Dibang is spread in upper Siang, *The One Horned Rhino is found in West Bengal and Assam.

Western Siang and Debang Valley in Arunachal Pradesh. Nanda Kaziranga National Park is famous for One Horned Rhino.

Devi is located in Uttrakhand. *Gir, Ranthambore and Corbett are famous for Tigers.

*Bundala Biosphere Reserve is situated on the South-East *Kaziranga National Park is located on the bank of river

coast of Sri Lanka. There is also a Central area National Park Bramhmaputra in Assam. It is spread over an area of 1173.58

in Bundala Reserve which includes four saline water lagoons. km2 with the habitat of mainly One Horned Rhino and Elephant.

The Central Area is home to native as well as migrating species It is a Tiger Reserve.

of Birds. The closest town near Bundala is Hambantota. *Kanha (Madhya Pradesh), Ranthambore (Rajasthan) and
Bandhavgarh (Madhya Pradesh) are the part of Project Tiger.
Wildlife Sanctuary *Manas Wildlife Sanctuary in Assam is famous for Tigers
For the protection and conservation of wildlife, Wildlife
and Rhinos. It is a project Tiger Reserve, Elephant Reserve
Sanctuary has been established. Some important Sanctuaries
and Biosphere Reserve. It is also included in the list of Natural
and their parent state are as follows-
World Heritage site by UNESCO in 1985. On 21 June 2011,
Sanctuary State
it was removed from the list of World Heritage in danger. (In
Similipal Odisha 1992 UNESCO declared it as a World Heritage site in danger.)
Nokrek Meghalaya *Chandra Prabha Wildlife Sanctuary is situated in Chandauli
Dihang Dibang Arunachal Pradesh district of Uttar Pradesh at a distance of 70 km from Varanasi.

F–150 General Studies Environment and Ecology


It is spread over an area of 78 km2. adventure seekers.
*Karera Wildlife Sanctuary is situated in Shivpuri district of
Madhya Pradesh. It is spread over an area of 202.21 km2 and is
Bird Sanctuaries and
only 44 km away from Jhansi (U.P.). Butterfly Conservation
*Jaisamand Wildlife Sanctuary is spread over an area of 52
*Various attempts have been made for the protection and
km2 in Udaipur district of Rajasthan.
conservation of the birds in India. Establishment of the Bird
*Nahargarh Wildlife Sanctuary is a small sanctuary (50 km2 Sanctuary was a major step for the protection and conservation
area) which is located in the Jaipur district of Rajasthan. of the birds.
*Periyar Game Sanctuary is located in Kerala and is famous *The natural habitat for the Great Indian Hornbill is the
for wild elephants. Western Ghat. The scientific name of this bird is Buceros
*Askot Wildlife Sanctuary is 54 km far from Pithoragarh bicornis. Great hornbills are found in the forests of India,
district of Uttarakhand. The Wildlife sanctuary of Uttarakhand Bhutan, Nepal, China, Indonesia, etc. Due to habitat loss and
from West to East - Kedarnath-Nanda Devi- Binsar-Askot. hunting in some areas, the great hornbill is listed as vulnerable
*Kedarnath Wildlife Sanctuary is situated in Chamoli on the IUCN Red List of Threatened Species.
and Rudraprayag districts of Uttarakhand. Nanda Devi is in
Bird Sanctuary District
Chamoli, Binsar is in Almora and Askot wildlife sanctuary is
Nawabganj Bird Sanctuary Unnao
located in Pithoragarh.
Okhala/Okhla Bird Sanctuary Gautam Buddha Nagar
*Kuno Wildlife Sanctuary is located in Sheopur district of
Madhya Pradesh near a place called Palpur. It was selected as a Samaspur Bird Sanctuary Rae Bareli
site to implement the Asiatic Lion Reintroduction Project. This Parvati Arga Bird Sanctuary Jaiprakash Nagar (Gonda)
site is selected because before 1873 it was the habitat of Asiatic Udhwa Bird Sanctuary Sahibganj
Lion. An environment ministry’s expert committee on March
*India's first butterfly park was established in Bannerghatta
2017, has approved the translocation of Asiatic Lions from Gir National Park in 2006 which is situated in Karnataka.
National Park of Gujarat to Kuno Palpur Wildlife Sanctuary *The scientific name of Blue Mormon is Papilio Polyester. It
in Madhya Pradesh as the second home for the Asiatic Lions. is a large swallowtail butterfly found in South India and Sri
*Wild Ass Wildlife Sanctuary is located in the Rann of Lanka. It is the State Butterfly of Maharashtra. In India, it is
Kachchh in Gujarat. It is spread over an area of 4953.71 km2. found in Maharashtra and Western Ghat (South India). It is the
The wildlife sanctuary was established in 1973. It is the largest third largest butterfly of India. The largest butterfly is Golden
wildlife sanctuary of India. Birdwing.
*Mahuadanr Wolf Wildlife Sanctuary is situated in Latehar *There are only four states in India which have declared their
district of Jharkhand. This wildlife sanctuary was created state butterflies Maharashtra and Karnataka, Uttarakhand and
over an area of 63.26 km in order to conserve the endangered
2 Arunachal Pradesh.
wolf species. The other creatures found here are rabbits, mice,
squirrels, small deer, etc. Project Tiger
*The Neyyar Wildlife Sanctuary in the Southern State of Kerala *The Government of India has taken a pioneering initiative
is spread over the southeast corner of the Western Ghats, and for conserving the tiger by launching the "Project Tiger" in
covers a total area of 128 km . It was declared as a wildlife
2
1973. The project aims at ensuring a viable population of Tigers
sanctuary in 1958. in their natural habitats and protecting them from extinction.
*'Gomarda' Wildlife Sanctuary is situated near Sarangarh, *The population of the Tigers around the world is expected to
in Raigarh district of Chhattisgarh. The wildlife sanctuary be between 3000 to 4000. The population of the Tigers in India
was established in the year 1975. Spread over more than 275 is 2967 (According to latest Tiger census).
square kilometer area, it is one of the ideal destination for the *International Tiger Day, is an annual celebration to raise

Environment and Ecology General Studies F–151


awareness for tiger conservation. It was decided in 2010 at the 23. Dampa Mizoram
Saint Petersburg Tiger Summit. 24. Bhadra Karnataka
*International Tiger Day is celebrated on 29 July every year. 25. Pench Maharashtra
*M-STrIPES (Monitoring system for Tigers - Intensive 26. Pakke Arunachal Pradesh
Protection and Ecological Status) is a software-based 27. Nameri Assam
monitoring system launched across Indian tiger reserves.
28. Satpura Madhya Pradesh
The System consists of two components
29. Annamalai Tamil Nadu
(a) Field based protocols for patrolling, law enforcement,
30. Udanti-Sitamadi Chhattisgarh
recording wild life crimes and ecological monitoring.
31. Satkosia Odisha
(b) A customized software for storage, retrieval, analysis and
32. Kaziranga Assam
reporting.
33. Achanakmar Chhattisgarh
*M-STrIPES produces easily interpretable reports and maps
34. Dandeli - Anshi (Kali) Karnataka
that are useful for management and policy decisions. It enables
35. Sanjay-Dubri Madhya pradesh
managers to assess the intensity and spatial coverage of patrols
36. Mudumalai Tamil Nadu
in a GIS-based tool. The system reduces the response time to
37. Nagarhole Karnataka
detrimental events like poaching or habitat degradation and
38. Paranbikulam Kerala
becomes a comprehensive tool to keep the pulse of a tiger
39. Sahyadri Maharashtra
reserve [GIS - Geographic Information system].
40. Biligiri Ranganatha Karnataka
As on February 2021, there are 51 Tiger Reserves in India 41. Karnel Telangana
Name of Tiger Reserve State
42. Satyamangalam Tamil Nadu
1. Bandipur Karnataka
43. Mukandra Hills Rajasthan
2. Corbett Uttarakhand
44. Navegaon nagzira Maharashtra
3. Manas Assam
45. Amrabad Telangana
4. Palamau Jharkhand
46. Pilibhit Uttar Pradesh
5. Ranthambore Rajasthan
47. Bor Maharashtra
6. Similipal Odisha
48. Rajaji Uttarakhand
7. Sundarban West Bengal
49. Orang Assam
8. Kanha Madhya Pradesh
50. Kamlang Arunachal Pradesh
9. Melghat Maharashtra
51. Srivilliputhur Megamalai Tamil Nadu
10. Periyar Kerala
11. Sariska Rajasthan *Rajaji National Park which is situated in Uttarakhand was

12. Namdapha Arunachal Pradesh declared as a tiger reserve in 2015. It became the second tiger
13. Nagarjuna Sagar Srisailam Andhra Pradesh reserve in Uttarakhand. The first tiger reserve of Uttarakhand
14. Buxa West Bengal was Corbett Tiger Reserve.
15. Indravali Chhattisgarh *Madhya Pradesh has the maximum number of tigers where
16. Dudhawa Uttar Pradesh as Karnataka comes second.
17. Kalakad - Mundanthurai Tamil Nadu *Kaziranga National Park was declared as a tiger reserve
18. Valmiki Bihar in 2006.
19. Pench Madhya Pradesh *Orang National Park is the 49th Tiger reserve which is
20. Tadoba-Andhari Maharashtra situated in Assam.
21. Bandhavgarh Madhya Pradesh *Kamlang National Park which is situated in Arunachal
22. Panna Madhya Pradesh Pradesh is the 50th Tiger reserve of the country.

F–152 General Studies Environment and Ecology


*Srivilliputhur Megamali Tiger Reserve is 51st tiger reserve Reserve of Andhra Pradesh. In the 40th session of the World
which is situated in Tamil Nadu. Heritage Committee held in Istanbul, Turkey in 2016, Nalanda
*Nagarjuna Sagar - Srisailam Tiger Reserve is the largest Mahavihar located in Nalanda of Bihar, the historical capital
Tiger Reserve in India. The total area of this tiger reserve complex in Chandigarh and Kanchenjunga of Sikkim were
is 3296.31km which comes under the boundary of Andhra
2 declared World Heritage site.
Pradesh whereas 2611.39 km area comes in Telangana which
2
UNESCO World Heritage Sites in India
is now known as Amarabad Tiger Reserve. Drones give
Name of Sites Designated year
a better monitoring of the wildlife situations than on-land
Agra Fort (1983)
traditional kind.
Ajanta Caves (1983)
*Bandipur Tiger Reserve now is being monitored by the
Buddhist Monuments at Sanchi (1989)
drones for the protection and conservation of the wildlife.
Champaner-Pavagadh Archaeological Park (2004)
*Dampa Tiger Reserve was declared a Tiger Reserve in 1994.
Chhatrapati Shivaji Terminus (formerly (2004)
It is situated in the western part of Mizoram State.
Victoria Terminus)
*Gumti Wild Life Sanctuary is located in Tripura. Saramati is
Churches and Convents of Goa (1986)
a peak rising above the surrounding peaks at the mountainous
Elephanta Caves (1987)
border of Nagaland and the Sagaing Region of Burma.
Ellora Caves (1983)
*Namdapha National Park is a Tiger Reserve in Arunachal
Fatehpur Sikri (1986)
Pradesh. Buxa Tiger Reserve is situated in West Bengal Sariska Great Living Chola Temples (1987)
Tiger Reserve is a National Park and Tiger Reserve located in Group of Monuments at Hampi (1986)
Alwar district of Rajasthan. Group of Monuments at Mahabalipuram (1984)
*Pench Tiger Reserve is 90% in Madhya Pradesh and 10% Group of Monuments at Pattadakal (1987)
in Maharashtra. Hill Forts of Rajasthan (2013)
*Kanha National Park is located in Madhya Pradesh and Manas Humayun’s Tomb, Delhi (1993)
National Park is located in Assam. Khajuraho Group of Monuments (1986)
Miscellaneous Mahabodhi Temple Complex at Bodh (2002)
The Royal Botanic Garden was established in England's Gaya
'Kew' in 1759. It became a UNESCO World Heritage site in Mountain Railways of India (1999)
2003. It is the world's largest garden with more than 40,000 Qutb Minar and its Monuments, Delhi (1993)
species of plants. Rani-ki-Vav (the Queen’s Stepwell) at (2014)
Patan, Gujarat
*Project Elephant was launched in 1992 by the Ministry
Red Fort Complex (2007)
of Environment and Forest of the Government of India. The
Rock Shelters of Bhimbetka (2003)
project aims to ensure long term survival of elephants in their
Sun Temple, Konârak (1984)
natural habitats by protecting the elephants, their habitats and
Taj Mahal (1983)
migration corridors. According to 2017 census, synchronized
The Jantar Mantar, Jaipur (2010)
population of the elephants in India is about 29964.
Archaeological Site of Nalanda (2016)
*Satya Manglam Tiger Reserve Area is located at the meeting
Mahavihara(Nalanda University) at
site of the Eastern Ghats and the Western Ghats. Elephants,
Nalanda, Bihar
Hyenas, Leopards, etc. are also found in this Tiger Reserve. It
The Architectural Work of Le Corbusier, (2016)
is the largest wildlife sanctuary of Tamil Nadu.
an Outstanding Contribution to the Modern
*Nallamala forest is located in Andhra Pradesh.
Movement
*Nagarhole National Park is located in Kudagu district and
Historic City of Ahmadabad (2017)
Mysore district of Karnataka.
Great Himalayan National Park (2014)
*Seshachalam Biosphere Reserve is the first Biosphere

Environment and Ecology General Studies F–153


Kaziranga National Park (1985) Uttarakhand P.C.S. (Pre) 2006
Ans. (a)
Keoladeo National Park (1985)
Manas Wildlife Sanctuary (1985) A National Board for Wildlife (NBWL), chaired by the Prime
Nanda Devi and Valley of Flowers National (1988) Minister of India provides a policy framework for wildlife
Parks conservation in the country. A network of 981 Protected
Area’s has been established extending over 1,71,921 km2
Sundarban National Park (1987)
(5.03% of the total geographic area), comprising 104 National
Western Ghats (2012)
Parks, 566 Wildlife Sanctuaries, 97 Conservation Reserves
Khangchendzonga National Park (2016)
and 214 Community Reserves. In India, most of the wildlife
The Victorian and Art Deco Ensemble of (2018) protected areas are surrounded by dense forests.
Mumbai
Jaipur (2019)

*Harike wetland is located on the confluence of the Beas and


Sutlej rivers of Punjab province.
*Keoladeo Ghana National Park is situated at the confluence
of the Gambhir and Banganga rivers in the Bharatpur district
of Rajasthan.
*Kolleru Lake is the largest freshwater lake in India located
in State of Andhra Pradesh. Kolleru Lake is located between
Krishna and Godavari deltas. Ramganga River flows through
the Corbett National Park.
*The first Marine sanctuary was established in the Gulf
of Kutch. Dolphin, Mollusca, Turtle, etc are some of the sea
organisms conserved in this Marine Sanctuary.
3. Today how many national parks are there in India,
*Chinnar Wild Life Sanctuary is located in the Idukki district
which has been established to protect wild animals?
of Kerala.
(a) 39 (b) 49
*Vedanthangal Bird Sanctuary and Karikili Bird Sanctuary
(c) 59 (d) 96
are located in Tamil Nadu. Karikilli is located at a distance of
R.A.S./R.T.S. (Pre) 1993
86 km from Chennai. Ans. (*)

Basic Questions At the time when the question was asked, there were 96
National Park in India. Currently, there are 104 National
1. International ‘Tiger Day’ is observed on – Parks in India.
(a) 24th July (b) 29th July
4. ‘Project Tiger’ of the Government envisages:
(c) 20th July (d) 25th July
(a) To observe the habits of the tiger
U.P.P.C.S. (Mains) 2014
(b) Collect vital information about different species
Ans. (b) (c) To save the Indian tiger from extinction
International Tiger Day is observed on July, 29 catching (d) None of these
worldwide attention to the conservation of tigers. It is both U.P.P.C.S. (Pre) 1994
an awareness as well as celebration day. It was founded at Ans. (c)
Saint Petersburg Tiger Summit in 2010.
The Government of India has taken a pioneering initiative
2. Most of the wildlife protected areas in India are for conserving its national animal, the tiger, by launching the
surrounded by – ‘Project Tiger’ in 1973. The Project Tiger aims to save the
(a) Dense forests (b) Rivers and lakes tiger from extinction.
(c) Human settlements (d) Mountains and hills 5. ‘Project Tiger’ in India was started in -

F–154 General Studies Environment and Ecology


(a) 1970 (b) 1973 U.P.P.C.S. (Mains) 2014
(c) 1981 (d) 1984 Ans. (c)
M.P.P.C.S. (Pre) 2008 In 1982, Delhi Zoo was officially renamed to National
U.P.P.C.S. (Pre) 2002 Zoological Park, with hopes that it could become a model for
Ans. (b) other zoos in the country. Wikipedia in its article ‘Sightseeing
See the explanation of the above question. in Delhi’ termed the same park as National Biological
Park. This could be the reason why it has been described as
6. To protect the Indian Tigers, ‘Project Tiger’ was National Biological Garden. However, in the official website
launched in the year? of this garden in Hindi it is entitled as ‘Rashtriya Prani Udyan’
(a) 1971 (b) 1973 and in English, it is entitled as National Zoological Park’.
(c) 1977 (d) 1991 10. Which one of the following is the first National Park
M.P.P.C.S. (Pre) 2012 of India?
Ans. (b) (a) Kanha National Park
See the explanation of the above question. (b) Dudhwa National Park
(c) Rajaji National Park
7. The term 'M-STrIPES' is sometimes seen in the news
(d) Corbett National Park
in the context of
M.P.P.C.S. (Pre) 2012
(a) Captive breeding of Wild Fauna
Chhattisgarh P.C.S. (Pre) 2011
(b) Maintenance of Tiger Reserves
(c) Indigenous Satellite Navigation System Ans. (d)
(d) Security of National Highways Jim Corbett National Park is the first National Park of India.
I.A.S. (Pre) 2017 The other National Parks mentioned in the question with their
Ans. (b) respective establishment year are as follows-
The term 'M-STrIPES stands for Monitoring System National Park Year of establishment
for Tigers- Intensive Protection and Ecological Status is Corbett (Uttarakhand) - 1936
sometimes seen in the news in the context of maintenance Kanha (Madhya Pradesh) - 1955
of Tiger Reserves. There are two main components of this
Dudhwa (Uttar Pradesh) - 1977
mechanism –
Rajaji (Uttarakhand) - 1983
1. Field-based protocol for patrolling, law enforcement,
recording wildlife crimes and ecological monitoring.
11. Which one of the following is the first National Park
2. A customized software for storage, retrieval, analysis
and reporting. On proper implementation, it ensures quick established in India?
response in the perilous conditions like tiger hunting or (a) Chandoli National Park
habitat destruction. It uses a system based on Geographical (b) Jim Corbett National Park
Information System (GIS). (c) Gir Forest National Park
8. India's first Dolphin Observatory is being built in: (d) Dudhwa National Park
(a) Delhi (b) Mumbai U.P.P.C.S. (Pre) 2017
(c) Bihar (d) Odisha Ans. (b)
(e) None of the above/More than one of the above See the explanation of the above question.
66th BPSC Re-Exam 2020
Ans. (c) 12. First National Park established in India is
(a) Jim Corbett National Park
India's first Dolphin Observatory 'Vikramshila Gangetic
(b) Kanha National Park
Dolphin Sanctuary' is located in Bhagalpur District of Bihar.
(c) Kaziranga National Park
9. National Biological Garden of India is situated in – (d) Gir National Park
(a) Mumbai (b) Lucknow R.A.S./R.T.S. (Pre) 2016
(c) New Delhi (d) Bengaluru Ans. (a)
Environment and Ecology General Studies F–155
Jim Corbett National Park, which is a part of the larger Nagarhole Forest (also called the Rajiv Gandhi National
Corbett Tiger Reserve lies in the Nainital district of Park) is one of the most popular tourist attractions of Coorg
Uttarakhand. Established in the year 1936 as Hailey National (Karnataka). Located in the districts of Mysore and Coorg,
Park, Corbett has the glory of being India’s oldest and most at a distance of 94 km from the city of Mysore, Nagarhole
prestigious National Park. is one of the best-managed parks of India.
13. Which of the following is the Oldest National Park 17. Maintenance of genetic diversity in National Parks is
in India? done by :
(a) Kaziranga National Park (a) In-situ conservation
(b) Hemis National Park (b) Ex-situ conservation
(c) Rajaji National Park (c) Gene pool
(d) Jim Corbett National Park (d) None of the above
Chhattisgarh P.C.S. (Pre) 2019 Uttarakhand P.C.S. (Pre) 2016
Ans. (d) Ans. (a)

See the explanation of the above question. The maintenance of Genetic Diversity, Wildlife Sanctuary
and National Parks is done by In-situ conservation. When a
14. First National Park of India is plant or organism is preserved in its original natural habitat
(a) Nanda Devi National Park area, it is called in-situ conservation, while when they are
(b) Jim Corbett National Park preserved somewhere else it is called Ex-situ conservation.
(c) Rajaji National Park 18. The Periyar Game Sanctuary is famous for –
(d) Kanha National Park (a) Lions (b) Spotted deer
Uttarakhand P.C.S. (Pre) 2016 (c) Tigers (d) Wild Elephants
Ans. (b) U.P.P.C.S (Pre) 2011
See the explanation of the above question.
Ans. (d)
15. The first National Park to be established in India is- The Periyar Game Sanctuary is located in Kerala which is
(a) Namdapha National Park famous for wild elephants.
(b) Corbett National Park
(c) Dudhwa National Park 19. Betla National Park is situated at –
(d) Guindy National Park (a) U.P (b) Bihar
U.P.P.C.S. (Mains) 2014 (c) M.P. (d) Odisha
Ans. (b) M.P.P.C.S. (Pre) 1990
Ans. (b)
Among the options mentioned above, Jim Corbett National
Park is the first to be established in India. The other parks Betla (Palamu) National Park is located in the Chota
with their year of establishment are as follows- Nagpur Plateau of the Palamu district of the Indian State of
National Park Year of Establishment Jharkhand, erstwhile Bihar, in India. It was established in
Corbett - 1936 1986 and is associated with 'Project Tiger'.
Namdapha - 1983
20. Rani Jhansi Maritime National Park was founded in
Dudhwa - 1977
(a) 1994 (b) 1995
Guindy - 1976
(c) 1996 (d) 1997
16. The Rajiv Gandhi National Park is located in: U.P.P.C.S. (Pre) 2017
(a) Andhra Pradesh (b) Rajasthan Ans. (c)
(c) Madhya Pradesh (d) Karnataka
Rani Jhansi Maritime National Park was founded in 1996 at
U.P. Lower Sub. (Pre) 2002
Andaman and Nicobar Island. The area of the park is spread
U.P.P.C.S. (Pre) 2002 over 256.14 sq. km.

Ans. (d) 21. The largest Tiger Habitat in India is in –

F–156 General Studies Environment and Ecology


(a) Andhra Pradesh (b) Karnataka In the year 1995, when the question was asked, Great Rann
(c) Madhya Pradesh (d) Uttar Pradesh of Kutch was not listed in Biosphere Reserves declared
U.P.P.C.S. (Mains) 2011 by Government of India. But in 2008, it was also listed in
Ans. (a) Biosphere Reserves of India.
Total 18 biosphere reserves have been listed so far. These
Nagarjunasagar Srisailam Tiger Reserve of Andhra Pradesh are –
is the largest tiger habitat in India. It is notable that after 1. Nilgiri (*) 1986 5520 km2
the reorganization of the State, 3296.31 sq. km area of this 2. Nanda Devi(*) 1988 5860.69 km2
Reserve falls under Andhra Pradesh while 2611.39 sq. km 3. Nokrek(*) 1988 820 km2
area falls under Telangana. 4. Great Nicobar(*) 1989 885 km2
22. Dhoopgarh Peak is situated in which of the following 5. Gulf of Mannar(*) 1989 10,500 km2
National Parks? 6. Manas 1989 2837 km2
7. Sunderban(*) 1989 9630 km2
(a) Satupra National Park
8. Similipal(*) 1994 4374 km2
(b) Kanha National Park
9. Dibru-Saikhowa 1997 765 km2
(c) Bandhavgarh National Park 10. Dehang-Debang 1998 5111.50 km2
(d) Madhav National Park 11. Pachmarhi(*) 1999 4926 km2
U.P.R.O./A.R.O. (Pre) 2021 12. Khangchendzonga(*) 2000 2619.92 km2
Ans. (a) 13. Agasthyamalai(*) 2001 1828 km2
14. Achanakmar – Amarkantak(*)2005 3835.51 km2
Dhoopgarh Peak is situated in Satpura National Park. Satpura
15. Kachchh 2008 12,454 km2
National Park is located in the Hoshangabad district of
16. Cold Desert 2009 7770 km2
Madhya Pradesh. Satpura national park covers an area of
17. Seshachalam Hills 2010 4755.997km2
524 km. sq and established in 1981.
18. Panna * 2011 2998.98 km2
23. Among the following Tiger Reserves, which one has the Note – (*)These have been included in UNESCO's World
largest area under ‘‘Critical Tiger Habitat’’? biosphere reserve list under MAB Programme.
(a) Corbett
26. Match List-I with List-II and select the correct answer
(b) Ranthambore
from codes given below the lists :
(c) Nagarjunasagar-Srisailam
List-I List-II
(d) Sunderban
(Biosphere sites) (Year of setting up)
I.A.S. (Pre.) 2020
A. Nilgiri 1. 2000
Ans. (c)
B. Nanda Devi 2. 1989
See the explanation of the above question. C. Sunderban 3. 1988
24. Where is the home of the Asiatic Lion? D. Kanchenjunga 4. 1986
(a) Gir Forest (b) Kanha Code :
(c) Corbett Park (d) Dudhwa A B C D
M.P.P.C.S. (Pre) 1998 (a) 1 2 3 4
Ans. (a) (b) 4 3 2 1
(c) 3 4 1 2
Gir National Park is located in Junagadh, Gujarat. It is famous (d) 2 3 1 4
for the habitat of Asiatic Lions. Besides, Sambhar and Cheetal
U.P.P.C.S.(Pre) 2012
are also found in Gir National Park.
Ans. (b)
25. Which one of the following does not belong to biosphere See the explanation of the above question.
reserves set-up so far?
(a) Great Nicobar (b) Sunderban 27. Which one of the following is not a Biosphere Reserve?
(c) Nanda Devi (d) Gulf of Kachchh (a) Agasthyamali (b) Nallamalai
I.A.S. (Pre) 1995 (c) Nilgiri (d) Panchmarhi
Ans. (*) I.A.S. (Pre) 2005

Environment and Ecology General Studies F–157


Ans. (b) Ans. (b)

See the explanation of the above question. The correct match of both the lists is as follows –
Nokrek - Meghalaya
28. Which of the following sanctuaries is not correctly
Manas - Assam
matched with the specified protected species?
Dehang Debang - Arunachal Pradesh
(a) Jim Corbett - Tiger
Agasthyamalai - Kerala
(b) Ghana - Lion
(c) Kaziranga - Rhinoceros 31. Match List–I with List-II and select the correct answer
(d) Periyar - Elephants from the codes given below the lists :
U.P. Lower Sub. (Pre) 2009 List–I List-II
Ans. (b) (Biosphere Reserve) (States of India)
A. Nokrek 1. Assam
Keoladeo Ghana National Park formerly known as Bharatpur
Bird Sanctuary is situated in Bharatpur, Rajasthan. It is home B. Manas 2. Kerala
to 366 bird species, 379 floral species and different species C. Similipal 3. Meghalaya
of fish, snakes, lizards and amphibians. However, lion is not D. Agasthyamalai 4. Odisha
a protected species of this park. Codes :
29. Biosphere reserves are areas to preserve – A B C D
(a) Grasslands (a) 3 4 2 1
(b) Agricultural produce (b) 4 2 3 1
(c) Atmospheric balance (c) 3 1 4 2
(d) Genetic diversity (d) 2 3 4 1
U.P. Lower Sub. (Spl) (Pre) 2004 U.P.B.E.O. (Pre) 2019
Ans. (d) Ans. (c)

Biosphere reserves are areas comprising terrestrial, See the explanation of the above question.
marine and coastal ecosystems. Each reserve promotes
32. Which one of the following pairs is not correctly
solutions reconciling the conservation of biodiversity with
its sustainable use. The core area(s) comprises a strictly matched?
protected ecosystem that contributes to the conservation of (Biosphere Reserve) (Location)
landscapes, ecosystems, species and genetic variation. (a) Manas Meghalaya
30. Match List-I with List-II and select the correct answer (b) Nanda Devi Uttarakhand
by using codes given below the lists: (c) Kanchanjunga Sikkim
List-I List-II (d) Agasthyamalai Kerala
(Biosphere Reserve) (Location) U.P.P.C.S. (Mains) 2013
A. Nokrek 1. Kerala Ans. (a)
B. Manas 2. Assam Manas Biosphere is situated in Assam. While rest of the
C. Dehang Debang 3. Meghalaya options are matched correctly.
D. Agasthyamalai 4.Arunachal Pradesh
33. Out of all the Biosphere Reserve in India, four have
Code :
been recognized on the World Network by UNESCO.
A B C D
Which one of the following is not one of them?
(a) 2 3 1 4
(a) Gulf on Mannar
(b) 3 2 4 1
(b) Khangchendzonga
(c) 4 1 3 2
(c) Nanda Devi
(d) 1 4 2 3
(d) Sunderbans
U.P.P.C.S.(Pre) 2013

F–158 General Studies Environment and Ecology


I.A.S. (Pre) 2008 Uttarakhand P.C.S. (Pre) 2012
Ans. (*) Ans. (d)

Today Twelve of the eighteen biosphere reserves are a part The correct match is as follows:
of the World Network of Biosphere Reserves based on Similipal - Odisha
the UNESCO Man and the Biosphere (MAB) Programme list Nokrek - Meghalaya
Name Agasthyamalai - Kerala - Tamil Nadu
(1) Nilgiri Biosphere Reserve Khangchendzonga - Sikkim
(2) Gulf of Mannar Biosphere Reserve
37. Match List-I with List-II and select the correct answer
(3) Sundarbans Biosphere Reserve
from codes given below the lists:
(4) Nanda Devi Biosphere Reserve
List-I List-II
(5) Nokrek Biosphere Reserve
(Biosphere Reserve) (States)
(6) Pachmarhi Biosphere Reserve
A. Similipal 1. Himachal Pradesh
(7) Similipal Biosphere Reserve
B. Dihang-Dibang 2. Uttarakhand
(8) Great Nicobar Biosphere Reserve
C. Nokrek 3. Arunachal Pradesh
(9) Achanakmar-Amarkantak Biosphere Reserve
D. Cold Desert 4. Odisha
(10) Agasthyamalai Biosphere Reserve
5. Meghalaya
(11) Khangchendzonga Biosphere Reserve
(12) Panna Biosphere Reserve Code :
A B C D
34. Which one of the following does not find a place in the (a) 1 3 5 4
UNESCO network of biosphere reserve? (b) 1 5 2 4
(a) Sunderban (c) 4 5 2 1
(b) Gulf of Mannar (d) 4 3 5 1
(c) Rann of Kutch U.P.R.O./A.R.O. (Pre) 2016
(d) Nilgiri Ans. (d)
U.P.P.C.S. (Mains) 2009 The correctly matched list is as follows:
Ans. (c) (Biosphere Reserve) (States)
Similipal - Odisha
See the explanation of the above question.
Dihang-Dibang - Arunachal Pradesh
35. Which one of the following pairs is correctly matched? Nokrek - Meghalaya
(a) Similipal - Assam Cold Desert - Himachal Pradesh
(b) Nokrek - Meghalaya 38. Which of the following pairs is NOT correctly
(c) Dehang Debang - Sikkim matched?
(d) Agasthyamalai - Karnataka (a) Bandipur National Park - Karnataka
U.P.P.C.S. (Mains) 2007 (b) Manas Wildlife Sanctuary - Assam
Ans. (b) (c) Periyar Wildlife Sanctuary - Kerala
(d) Similipal National Park - Madhya Pradesh
Biosphere reserves and related States are as follows: U.P. P.C.S. (Pre) 2018
Similipal - Odisha Ans. (d)
Nokrek - Meghalaya
See the explanation of the above question.
Dehang Debang - Arunachal Pradesh
Agasthyamalai - Kerala, Tamil Nadu 39. Which of the following biosphere reserves of India is
spread over Garo Hills?
36. Which of the following is not properly matched?
(a) Nokrek
Biosphere Reserves State (b) Agasthyamalai
(a) Similipal - Odisha (c) Dehang Debang
(b) Nokrek - Meghalaya (d) Nanda Devi
(c) Agasthyamalai - Kerala U.P.P.S.C. (R.I.) 2014
(d) Khangchendzonga - Himachal Pradesh Ans. (a)

Environment and Ecology General Studies F–159


The Nokrek Biosphere Reserve is located in the north-east Ans. (d)
of India on Garo Hills which forms part of the Meghalaya Nanda Devi biosphere reserve is located in Uttarakhand. It
Plateau. While Agasthyamalai is spread over the Western is listed in World Network of Biosphere Reserves, based on
Ghats, Dehang Debang over Upper Siyang, Debang Valley the UNESCO Man and the Biosphere (MAB) Programme.
in Arunachal Pradesh and Nanda Devi lies in Uttarakhand.
43. Which one of the following is a ‘World Heritage’ site?
40. Study the following statements and choose the correct (a) Nanda Devi Biosphere Reserve
option: (b) Corbett National Park
Statement I : Kanger Ghati is a National Park (c) Rajaji National Park
Statement II : Kanger Ghati is not a Biosphere Reserve (d) Gir Forest
Statement III : Kanger Ghati is spreaded starting from U.P.P.C.S. (Pre) 2008
Tirathgarh Waterfalls to the Kolab river, the boundary U.P.P.C.S. (Mains) 2005
of Odisha in the east Ans. (a)
(a) Statement I, II and III are true
Nanda Devi Biosphere Reserve was inscribed a World
(b) Statement I, II and III all are false Heritage Site Status by UNESCO in 1988. In 2005, it was
(c) Statement I and III are true, but statement II is false extended to the valley of flowers as a World Heritage Site.
(d) Statement I and II are true, but statement III is false
44. India has recently added one more Biosphere Reserve.
Chhattisgarh P.C.S. (Pre) 2020
Which of these is the latest one in the list?
Ans. (a)
(a) Nokrek (b) Dehang Debang
Kanger Ghati National Park (Kanger Valley National Park) (c) Similipal (d) Cold desert
covers an area of 200 square kilometers. It was declared as R.A.S./R.T.S.(Pre) 2010
a national park in 1982. It is situated in Chhattisgarh and is Ans. (d)
spreaded starting from Tirathgarh Waterfalls to the Kolab
river, the boundary of Odisha in the east. At present Kanger See the explanation of the above question.
Ghati is not a Biosphere Reserve. 45. Consider the following statements:
1. The boundaries of a National Park are defined by
41. What will be the decreasing order of the following
legislation.
biosphere reserves from the point of view of the length?
2. A Biosphere Reserve is declared to conserve a few
(a) Sunderban, Gulf of Mannar, Pachmarhi, Kanchenjunga
specific species of flora and fauna.
(b) Gulf of Mannar, Sunderban, Pachmarhi, Kanchenjunga
3. In a Wildlife Sanctuary, limited biotic interference
(c) Gulf of Mannar, Sunderban, Kanchenjunga, Pachmarhi
is permitted.
(d) Sunderban, Pachmarhi, Kanchenjunga, Gulf of
Which of the statements given above is/are correct?
Mannar
(a) 1 only
Chhattisgarh P.C.S. (Pre) 2020
(b) 2 and 3
Ans. (b)
(c) 1 and 3
Gulf of Mannar have largest area (10,500 km2) under (d) 1, 2 and 3
Biosphere Reserve followed by the Sunderban which I.A.S. (Pre) 2010
covers about 9630 km2 area. Pachmarhi Biosphere Reserve Ans. (c)
has an area of about 4926 km2 and covers parts of Betul,
Hoshangabad and Chindwara districts of Madhya Pradesh. The boundaries of a National Park are defined by legislation.
Kanchenjunga Biosphere Reserve has an area of about No alteration to the boundaries of a National Park shall be
2619.92 km2 and covers parts of Kanchenjunga hills and made except on a resolution passed by the Legislature of
Sikkim. the State. Biosphere Reserve are representative parts of
natural and cultural landscapes extending over a large area
42. In which State, Nandadevi Biosphere Reserve is of terrestrial or coastal/marine ecosystems. The purpose of
located? the formation of biosphere reserves is to conserve in situ all
(a) Chhattisgarh (b) Assam forms of life along with its support system. The difference
(c) Himachal (d) Uttarakhand between a national park and a sanctuary is that no human
Chhattisgarh P.C.S. (Pre) 2011 activity is allowed inside a national park (except for the ones

F–160 General Studies Environment and Ecology


permitted by the Chief Wildlife warden of the state) while of 40 years only.
limited activities are permitted within the sanctuary. Each 4. Among the States in India, the highest elephant
reserve promotes solutions reconciling the conservation of population is in Kerala.
biodiversity with its sustainable use. Thus only statement 1 Which of the statements given above is/are correct?
and 3 are correct. (a) 1 and 2 only (b) 2 and 4 only
(c) 3 only (d) 1, 3 and 4 only
46. In which one among the following categories of
I.A.S. (Pre.) 2020
protected areas in India are local people not allowed
Ans. (a)
to collect and use the biomass?
(a) Biosphere Reserve An elephant is placed in the order Proboscidea, the natural
(b) National Parks group of animals with a proboscis or trunk as their distinct
(c) Wetlands declared under Ramsar Convention physical feature. The zoological name of African elephant
is Loxodonta africana and that of Asian or Indian elephant
(d) Wildlife Sanctuaries
is Elephas maximus. Generally, the leader of an elephant
I.A.S. (Pre) 2012
group is a female. The gestation period of an elephant is about
Ans. (b)
18-22 months. Elephants have the longest gestation period
See the explanation of the above question. of all mammals. Most of the female elephants give birth for
the first time between 16 and 17 years, and they can calve
47. Which of the following National Parks/Sanctuary is until the age of 60 or more. According to the elephant census
known as “World Natural Heritage?” 2017, Karnataka has the highest number of elephants (6,049),
(a) Ranthambore National Park, Sawai Madhopur followed by Assam (5,719) and Kerala (3,054).
(b) Keoladeo National Park, Bharatpur
50. Where is the wild ass sanctuary?
(c) Desert National Park, Jaisalmer
(a) U.P (b) Assam
(d) Tal Chhapar Sanctuary, Churu
(c) Gujarat (d) Rajasthan
R.A.S./R.T.S. (Pre) 1992
U.P.P.C.S (Pre) 2010
Ans. (b)
Ans. (c)
Keoladeo National Park is formally known as Bharatpur
Indian Wild Ass Sanctuary is also known as the Wild Ass
Bird Sanctuary in Bharatpur, Rajasthan was established as
Wildlife Sanctuary is located in the Rann of Kachchh in
a National Park in 1981. In 1985, the Park was declared
Gujarat. Spread over 4953.71 km², it is the largest wildlife
a World Heritage Site under the World Heritage Convention.
sanctuary in India. It was established in 1973.
48. In India, Project Elephant was launched in the year –
51. One horned rhinoceros is found in the following States-
(a) 1968 (b) 1970
(a) Arunachal Pradesh and Tripura
(c) 1972 (d) 1974
(b) West Bengal and Assam
U.P.P.C.S. (Mains) 2007
(c) Arunachal Pradesh and Assam
Ans. (*)
(d) West Bengal and Tripura
Project Elephant was launched by the Government of India M.P.P.C.S. (Pre) 2008
in 1992 as a centrally sponsored scheme with the aim to
Ans. (b)
protect elephants, their habitat, to address issues of man-
animal conflict and welfare of captive elephants. Under this One-horned Rhinoceros is found in West Bengal and Assam.
project, financial and technical support is being provided to
52. Sanctuary famous for Rhinoceros:
major elephant bearing States in the country. Synchronized
Elephant population in India is estimated at 29964 as per the (a) Kaziranga (b) Gir
census conducted in 2017. (c) Ranthambore (d) Corbett
R.A.S./R.T.S.(Pre) 2010
49. With reference to Indian elephants, consider the
Ans. (a)
following statements :
1. The leader of an elephant group is a female. Kaziranga National Park hosts two-thirds of the world’s Great
2. The maximum gestation period can be 22 months. One-horned rhinoceros.
3. An elephant can normally go on calving till the age 53. In which of the following National Parks of India

Environment and Ecology General Studies F–161


Rhinoceros rehabilitation is being done? 56. Which of the following is not correctly matched?
(a) Corbett National Park (a) Mahuatai Sanctuary - Palamu
(b) Dudhwa National Park (b) Topchanchi Sanctuary - Dhanbad
(c) Kanha National Park (c) Udhwa Bird Sanctuary - Kodarma
(d) Kaziranga National Park (d) Lavalong Sanctuary - Chatra
U.P.P.C.S. (Spl) (Mains) 2004 Jharkhand P.C.S. (Pre) 2003
Ans. (b) Ans. (c)
As reported by the State Government of Uttar Pradesh, Udhwa Lake Bird Sanctuary is situated in Sahebganj District
rhinoceroses have been relocated/rehabilitated in Dudhwa of Jharkhand. While other pairs are correctly matched.
National Park in Uttar Pradesh from Assam in 1984. The
reintroduction of rhino in Dudhwa took place in two phases. 57. Consider the following statements and select the
In the first phase (1984), five rhinos (2 males and 3 females) correct answer from codes given below –
were captured and translocated to Dudhwa from Pobitora 1. Similipal National Park is situated in Odisha.
Wild Life Sanctuary, Assam while the second phase is being 2. Thailand was known as Siam during ancient time.
implemented. 3. Astadiggajas was the gem of the court of Shivaji
54. Recently there was a proposal to translocate some of 4. Astachap was a group of devotees who were a
the lions from their natural habitat in Gujarat to which disciple of Adi Shankara.
one of the following sites? Codes :
(a) Corbett National Park (a) 1 and 2 (b) 2 and 3
(b) Kuno Palpur Wildlife Sanctuary (c) 3 and 4 (d) All four
(c) Mudumalai Wildlife Sanctuary U.P. Lower Sub. (Pre) 2002
(d) Sariska National Park Ans. (a)
I.A.S. (Pre) 2017
Similipal National Park is located in Mayurbhanj
Ans. (b)
district of Odisha. Siam is the ancient name of Thailand
The expert committee of the Ministry of Environment had which was changed in the actual name on 23rd June 1939.
given its approval in March 2017 to transfer Asian lions to Ashtadiggajas is the collective title given to the eight Telugu
Kuno Palpur Wildlife Sanctuary from Gir National Park, poets in the court of the emperor Krishna Deva Raya and
Gujarat. According to the committee, the conditions of Kuno Ashtachap designates the eight devotional poets of Braj,
Palpur Wildlife Sanctuary are suitable for another house of who were a disciple of Vallabhcharya. Hence, statement 1
Asian lions. However, translocation project was contested and 2 are correct.
by the Government of Gujarat.
58. Match List-I to List-II and select the correct answer
55. Consider the following statements : as per the given codes –
1. Asiatic lion is naturally found in India only. List -I List -II
2. Double-humped camel is naturally found in India (National Park) (State)
only. A. Bandhavgarh 1. Himachal Pradesh
3. One-horned rhinoceros is naturally found in India National Park
only. B. Bandipur National 2. Gujarat
Which of the statements given above is/are correct? Park
(a) 1 only (b) 2 only C. Rohla National Park 3. Madhya Pradesh
(c) 1 and 2 only (d 1 and 3 only D. Gir National Park 4. Karnataka
I.A.S. (Pre) 2019 Codes :
Ans. (a) A B C D
Asiatic lion is found naturally in Gir National Park situated (a) 3 4 1 2
in Gujarat. The Bactrian camel or Double-humped camel is (b) 4 3 2 1
native of Central Asia. In India it is found in Nubra Valley. (c) 1 2 3 4
One-horned Rhino is found in India and Nepal both. Hence (d) 3 2 1 4
option (a) is correct answer. U.P. Lower Sub. (Pre) 2002

F–162 General Studies Environment and Ecology


Ans. (a) (c) Yellow Mountain Park
(d) Mizo Hills Park
The correct match is as follows:
M.P.P.C.S. (Pre) 2020
Bandhavgarh National Park – Madhya Pradesh
Bandipur National Park – Karnataka Ans. (b)
Rohla National Park – Himachal Pradesh Phawngpul national Park in Mizoram is also known as Blue
Gir National Park – Gujarat Mountain Park. It is located in the Lawngtlai district, towards
Hence option (a) is the correct answer. the southeast of Mizoram and relatively close to Burma.
The Kolodyne river flows along the eastern side of the Park
59. Select the State from amongst the following, with the and also forms the international border with Myanmar. The
maximum number of National Park– highest point in Mizoram, Phawngpui Peak (2,157 m), is
(a) Uttar Pradesh (b) Rajasthan located within this Park. Most of the Park is covered with
(c) Madhya Pradesh (d) West Bengal Sub-tropical Broadleaf and Tropical Evergreen forests.
U.P.P.C.S. (Pre) 2008 63. Match List I with List II and select the correct answer
U.P. Lower Sub. (Pre) 2004 using codes given below:
Ans. (c) List-I (Protected List-II (Number
There are 104 national parks in India. Andaman & Nicobar, areas of India) in year 2018)
Kerala, Maharashtra, Uttarakhand and West Bengal have 6 A. Community Reserves i. 103
National Parks each. Madhya Pradesh has maximum number B. Conservation Reserves ii. 46
of National parks (11). Thus, option (c) is the correct answer. C. National Parks iii. 544
60. Which of the following is not correctly matched? D. Wildlife Sanctuaries iv. 76
(a) Anshi National Park - Karnataka Codes :
(b) Balphakram National Park - Meghalaya A B C D
(c) Chandoli National Park - Gujarat (a) ii iv i iii
(d) Hemis National Park - Ladakh (b) iii ii i iv
M.P.P.C.S. (Pre) 2020 (c) iv iii ii i
Ans. (c) (d) iii ii iv i
R.A.S./R.T.S. (Pre) 2018
Chandoli National Park is a national park spread over Satara,
Kolhapur and Sangli Districts of Maharashtra state, India. Ans. (*)
It was established in May 2004. Earlier it was a Wildlife According to ENVIS centre on Wildlife & Protected Areas,
Sanctuary declared in 1985. Chandoli Park is imminent as the different protected areas of India and their strength in the
southern portion of the Sahyadri Tiger Reserve, with Koyna
year 2019 & 2020, is given below :
Wildlife Sanctuary forming the northern part of the reserve.
Hence option(c) is not correctly matched. 2019 2020
Community Reserve - 163 214
61. Which one among the following has the maximum
Conservation Reserve - 86 97
number of National Parks?
(a) Andaman and Nicobar Islands National Parks - 101 104
(b) Arunachal Pradesh Wild Life Sanctuaries - 553 566
(c) Assam 64. The State of India with maximum number of wildlife
(d) Meghalaya sanctuaries is :
I.A.S. (Pre) 2008 (a) Karnataka (b) Tamil Nadu
Ans. (a) (c) Maharashtra (d) Madhya Pradesh
See the explanation of the above question. (e) None of the above/More than one of the above
66th B.P.S.C. (Pre.) 2020
62. Phawngpul National Park in Mizoram is also known
Ans. (e)
as which of the following?
(a) Black Mountain Park Andman and Nicobar Island (96) has the maximum number
(b) Blue Mountain Park of wildlife sanctuaries.

Environment and Ecology General Studies F–163


65. Match List-I with List-II and select the correct answer (c) 4 3 2 1
from codes given below: (d) 2 3 1 4
List- I List- II U.P.P.C.S. (Pre) 1998
A. Nawabganj Bird 1. Gonda
Sanctuary Ans. (b)
B. Okhla Bird 2. Unnao The correctly matched list of National Parks and their
Sanctuary respective States is given below –
C. Samaspur Bird 3. Ghaziabad Karnataka - Bandipur National Park
Sanctuary
Uttarakhand - Rajaji National Park
D. Parvati Arga 4. Raibareli
Odisha - Similipal National Park
Bird Sanctuary
Codes : Assam - Manas National Park
A B C D 68. Which of the following is not correctly matched?
(a) 2 4 3 1 National Park State
(b) 2 3 4 1 (a) Bandipur Karnataka
(c) 4 3 1 2 (b) Rajaji Uttarakhand
(d) 3 4 2 1 (c) Similipal Odisha
U.P.U.D.A./L.D.A. (Pre) 2001 (d) Pin Valley Jammu and Kashmir
Ans. (b) U.P.P.C.S. (Mains) 2016
Ans. (d)
The correctly matched list of bird sanctuaries of Uttar Pradesh
and their respective districts is given below : Pin Valley National Park is located in the Lahaul and Spiti
Nawabganj Bird Sanctuary - Unnao district in Himachal Pradesh. Rest all are correctly matched.
Okhla Bird Sanctuary - Ghaziabad
69. Read the list of National Parks and Animals housed
Samaspur Bird Sanctuary - Raebareli for conservation:
Parvati Arga Bird Sanctuary - Gonda National Parks Animals
66. Which one of the following States has the ideal habitat A. Bandipur (i) Tiger Reserve
for the Siberian crane? B. Kaziranga (ii) Elephant Reserve
(a) Rajasthan (b) Arunachal Pradesh C. Sunderbans (iii)One-horned Rhinoceros
(c) Andhra Pradesh (d) Odisha Reserve
U.P.P.C.S. (Pre) 1996 D. Similipal (iv) Biosphere and Tiger Reserve
U.P.P.S.C. (GIC) 2010 Which one of the following is correctly matched?
Ans. (a) Codes :
A B C D
Keoladeo National Park situated in Bharatpur, Rajasthan
(a) ii iv i iii
provides ideal habitat for the Siberian crane. (b) i iii iv ii
67. Match the following List- I with List- II and select the (c) iv i iii ii
correct answer from codes given below the lists: (d) iii i ii iv
List- I List- II R.A.S./R.T.S.(Pre) 2013
(States) (National Park) Ans. (b)
A. Uttarakhand 1. Bandipur The correctly matched list of National Parks and animals
B. Assam 2. Rajaji National Park house for conservation is given below –
C. Odisha 3. Similipal
National Parks Animals
D. Karnataka 4. Manas
Bandipur - Tiger Reserve
Codes :
Kaziranga - One horn Rhinoceros Reserve
A B C D
(a) 1 2 3 4 Sunderbans - Biosphere and Tiger Reserve
(b) 2 4 3 1 Similipal - Elephant Reserve

F–164 General Studies Environment and Ecology


70. Sariska and Ranthambore are the reserves for which I.A.S. (Pre) 2006
of the following animals? Ans. (c)
(a) Lion (b) Deer
This question seems to assess information about nearby cities
(c) Tiger (d) Bear
of the wildlife sanctuaries given in the options. (Although
R.A.S./R.T.S.(Pre) 2010
these sanctuaries are not located in these cities/districts).
U.P.P.C.S. (Main) 2012
Chandra Prabha Wildlife Sanctuary is located in Chandauli
U.P. Lower Sub. (Pre) 2015
district of U.P. It is 70 km. away from Varanasi.
Ans. (c)
Karera wildlife sanctuary is located in Shivpuri district of
Both Sariska and Ranthambore are the national parks for the Madhya Pradesh. It is only 44 km. away from Jhansi.
protection of tigers. Jaisamand wildlife sanctuary is situated in Udaipur district
and is 48 km. away from the district headquarter.
71. Which of the following is not correctly matched? Nahargarh wildlife sanctuary is a small sanctuary situated
(a) Bandipur National Park - Karnataka in Bara district of Rajasthan. It is at a distance of 300 km.
(b) Manas Wildlife Sanctuary - Assam from Jaipur.
(c) Periyar Wildlife Sanctuary - Kerala
74. Match List-I with List-II and select the correct answer
(d) Simlipal National Park - Madhya Pradesh from codes given below the lists :
U.P. Lower Sub. (Pre) 2009
List-I List-II
Ans. (d)
(National Parks) (States)
Simlipal National Park is located in Odisha, not in Madhya A. Intanki 1. Jharkhand
Pradesh. Rest are correctly matched. B. Betla 2. Tamil Nadu
C. Sirohi 3. Nagaland
72. ‘Salim Ali National Park’ is located in –
D. Guindy 4. Manipur
(a) Maharashtra
Codes :
(b) Jammu and Kashmir
A B C D
(c) Madhya Pradesh
(d) Andhra Pradesh (a) 2 1 3 4
Uttarakhand P.C.S. (Pre) 2005 (b) 4 3 2 1
Ans. (b) (c) 3 1 4 2
(d) 3 4 1 2
Salim Ali National Park is located in Srinagar, Kashmir. U.P. R.O./A.R.O. (Pre) 2017
The park earlier known as City Forest National Park was Ans. (c)
christened after Indian ornithologist and naturalist Salim Ali.
The correct match of List-I with List-II is as follows-
73. Match List-I (National Park/Wildlife Sanctuary) with
National Parks States
List-II (Nearby Town) and select the correct answer
using codes given below the lists: Intanki Nagaland
List-I List-II Betla (Palamu) Jharkhand
(National Park/Wildlife (Nearby Town) Guindy Tamil Nadu
Sanctuary) Sirohi Manipur
A. Chandra Prabha 1. Jaipur Note : Sirohi National Park is not included in the list provided
B. Karera 2. Jhansi by Envis Centre on Wildlife & Protected Areas.
C. Jaisamand 3. Agra
D. Nahargarh 4.Varanasi 75. Match List–I with List–II and select the correct answer
5. Udaipur using the codes given below the lists :
Code : List–I List-II
A B C D (National Parks) (States)
(a) 4 1 5 2 A. Indravati 1. Jharkhand
(b) 5 2 3 1 B. Mollem 2. Haryana
(c) 4 2 5 1 C. Kalesar 3. Goa
(d) 5 1 3 2 D. Betva 4. Chhattisgarh

Environment and Ecology General Studies F–165


Codes : 21. Panna Madhya Pradesh
A B C D 22. Dampa Mizoram
(a) 4 3 2 1 23. Bhadra Karnataka
(b) 4 2 3 1 24. Pench Maharashtra
(c) 4 1 3 2 25. Pakke Arunachal Pradesh
(d) 3 4 2 1 26. Nameri Assam
U.P.P.S.C. (Pre) 2020 27. Satpura Madhya Pradesh
Ans. (*) 28. Annamalai Tamil Nadu
Indravati National Park is located in Chhattisgarh, Mollem 29. Udanti-Sitanadi Chhattisgarh
National Park in Goa and Kalesar National Park is in Haryana. 30. Satkosia Odisha
Betla (Palamu) National Park is located in Jharkhand. In the 31. Kaziranga Assam
question, the name Betwa is mentioned which does not exist
32. Achanakmar Chhattisgarh
in the list of 104 national parks provided by ENVIS centre
on wildlife and protected areas. 33. Dandeli-Anshi Karnataka
34. Sanjay-Dubri Madhya Pradesh
76. Which one of the following national parks is not listed
35. Mudumalai Tamil Nadu
in “Project Tiger”?
36. Nagarahole Karnataka
(a) Kanha (b) Ranthambhore
37. Parambikulam Kerala
(c) Corbett (d) Bandhavgarh
38. Sahyadri Maharashtra
M.P.P.C.S. (Pre) 1994
Ans. (*) 39. Biligiri Ranganatha Karnataka Temple
40. Kawal Telangana
All the four parks given in the options are listed under Project
41. Sathyamangalam Tamil Nadu
Tiger. As on February 2021, there are 51 parks which are
42. Mukandra Hills Rajasthan
governed by Project Tiger.
43. Nawegaon-Nagzira Maharashtra
Sl. No. Name of Tiger Reserve State
44. Nagarjunsagar Srisailam Andhra Pradesh
1. Bandipur Karnataka
45. Amrabad Telangana
2. Corbett Uttarakhand
Amangarh (buffer 46. Pilibhit Uttar Pradesh
of Corbett TR) 47. Bor Maharashtra
3. Kanha Madhya Pradesh 48. Rajaji Tiger Reserve Uttarakhand
4. Manas Assam 49. Orang Tiger Reserve Assam
5. Melghat Maharashtra 50. Kamlang Tiger Reserve Arunachal Pradesh
6. Palamau Jharkhand 51. Srivilliputhur Megamalai Tamil Nadu
7. Ranthambore Rajasthan 77. Match List-I with List-II and select the correct answer
8. Similipal Odisha using codes given below the Lists :
9. Sunderbans West Bengal List-I List-II
10. Periyar Kerala (Tiger Reserve) (State)
11. Sariska Rajasthan A. Bandipur 1. Assam
12. Buxa West Bengal B. Manas 2. Arunachal Pradesh
13. Indravati Chhattisgarh C. Namdapha 3. Karnataka
14. Namdapha Arunachal Pradesh D. Simlipal 4. Orissa
15. Dudhwa Uttar Pradesh Codes :
16. Kalakad-Mundanthurai Tamil Nadu A B C D
17. Valmiki Bihar (a) 3 1 2 4
18. Pench Madhya Pradesh (b) 1 2 3 4
19. Tadoba-Andhari Maharashtra (c) 3 1 4 2
20. Bandhavgarh Madhya Pradesh (d) 4 3 2 1

F–166 General Studies Environment and Ecology


U.P.P.C.S. (Mains) 2006 The Pench Tiger Reserve is spread over two States namely
Ans. (a) Madhya Pradesh (90%) and Maharashtra (10%).
See the explanation of the above question.
83. The famous Tiger Reserve ‘Sariska’ is situated in which
78. Which one of the following is not a Project Tiger of the following States?
Reserve in India? (a) Uttar Pradesh (b) Rajasthan
(a) Dudhwa (b) Chilka (c) Uttarakhand (d) Madhya Pradesh
(c) Kanha (d) Manas U.P.P.C.S. (Mains) 2012
R.A.S./R.T.S.(Pre) 2012 Ans. (b)
Ans. (b)
The Sariska Tiger Reserve is located in the Alwar district of
See the explanation of the above question. Rajasthan. It was declared a Wild Life Sanctuary in 1955 and
Tiger Reserve in 1978.
79. Consider the following areas –
1. Bandipur 2. Bhitarkanika 84. Which of the following is not a tiger sanctuary?
3. Manas 4. Sunderbans (a) Kanha (b) Ranthambhore
Which of the above are Tiger Reserves? (c) Kaziranga (d) Bandhogarh
(a) 1 and 2 (b) 1, 3 and 4 M.P.P.C.S. (Pre) 1997
(c) 2, 3 and 4 (d) 1, 2, 3 and 4 Ans. (*)
I.A.S. (Pre) 2012
Kaziranga wildlife sanctuary is a national park in Assam
Ans. (b) which hosts two- thirds of the world’s great one-horned
See the explanation of the above question. rhinoceroses. It was not a tiger Resergve when this question
was asked. But now it is a tiger Reserve (2006). Rest are
80. Which one of the following Tiger reserves of India is tiger reserves.
located in Mizoram?
(a) Melghat (b) Buxa 85. Kaziranga is known for –
(c) Dampha (d) Bhadra (a) Rhinoceros (b) Tiger
U.P.P.C.S. (Mains) 2012 (c) Birds (d) Lion
Ans. (c) U.P.P.C.S.(Pre) 2012
Jharkhand P.C.S. (Pre) 2013
See the explanation of the above question. Ans. (a)
81. Which one of the following pairs is not correctly See the explanation of the above question.
matched?
Tiger Reserve State 86. Match List-I with List-II and select the correct answer
(a) Buxa - Bihar from codes given below the lists :
(b) Dampha - Mizoram List-I (Sanctuary) List-II (State)
(c) Nameri - Assam A. Garmpani 1. Andhra Pradesh
(d) Namdapha - Arunachal Pradesh B. Nam Dafa 2. Arunachal Pradesh
U.P.P.C.S. (Mains) 2010 C. Pakhal 3. Assam
Ans. (a) D. Sariska 4. Rajasthan
Codes :
Buxa Tiger Reserve is in West Bengal not in Bihar. Rest
A B C D
others are correctly matched.
(a) 2 3 1 4
82. Which of the following tiger reserve lies in two States ? (b) 4 3 1 2
(a) Kanha (b) Manas (c) 1 2 4 3
(c) Pench (d) Sariska (d) 3 2 1 4
U.P.P.C.S. (Mains) Spl. 2008 U.P.P.C.S. (Mains) 2003
Ans. (c) Ans. (d)

Environment and Ecology General Studies F–167


The correctly matched list of sanctuaries and their respective Manas Wildlife Sanctuary located in Assam is one of the
States is given below : first reserves included in the network of the tiger reserves
Sanctuary State under Project Tiger in 1973. In 1985, it was declared as
Garmpani - Assam World Heritage Site (Natural) by UNESCO for outstanding
Nam Dafa - Arunachal Pradesh universal value.
Pakhal - Andhra Pradesh
91. Match the following:
Sariska - Rajasthan
(A) Kanha National Park (1) Shivpuri
87. Which one of the following is not correctly matched? (B) Bandhavgarh National Park (2) Mandla
Wildlife Sanctuary State (C) Madhav National Park (3) Bastar
(a) Mukambika Karnataka (D) Indravati National Park (4) Shahdol
(b) Dalma Jharkhand Codes :
(c) Neyyar Chhattisgarh A B C D
(d) Kotigaon Goa (a) 2 4 1 3
M.P.P.C.S. (Pre) 2019 (b) 1 2 4 3
Ans. (c) (c) 3 2 1 4
(d) 4 3 2 1
The Neyyar Wildlife Sanctuary in the southern state of Kerala
M.P.P.C.S. (Pre) 1999
is spread over the southeast corner of the Western Ghats, and
Ans. (a)
covers a total area of 128 km2. It was declared as a wildlife
sanctuary in 1958. All other options given in the question, National Parks mentioned in the question and their respective
are correctly matched. districts are as follows –
Kanha National Park - Mandla
88. 'Gomarda' Wildlife Sanctuary is situated ?
Bandhavgarh National Park - Shahdol
(a) Dhamtari district (b) Raipur district Madhav National Park - Shivpuri
(c) Raigarh district (d) Sarguja district Indravati National Park - Bastar
Chhattisgarh P.C.S. (Pre) 2019
92. Which one of the following protected areas is well-
Ans. (c)
known for the conservation of a sub-species of the
'Gomarda' Wildlife Sanctuary is situated near Sarangarh, in Indian swamp deer (Barasingha) that thrives well on
Raigarh district of Chhattisgarh. The wildlife sanctuary was hard ground and is exclusively graminivorous?
established in the year 1975. Spread over more than 275 (a) Kanha National Park
square kilometer area, it is one of the ideal destination for
(b) Manas National Park
the adventure seekers.
(c) Mudumalai Wildlife Sanctuary
89. The National Chambal Sanctuary does NOT fall in (d) Tal Chhapar Wildlife Sanctuary
which of the following states? I.A.S. (Pre.) 2020
(a) Uttar Pradesh (b) Madhya Pradesh Ans. (a)
(c) Haryana (d) Rajasthan Kanha National Park is located in Madhya Pradesh.
U.P.P.C.S. (Pre) 2020 Besides tiger, this protected area is also well-known for
Ans. (c) the conservation of a sub-species of the Indian swamp
The National Chambal Sanctuary is located along the deer (Barasingha) that thrives well on hard ground and is
Chambal River. The sanctuary falls in the tri-point of exclusively graminivorous.
Rajasthan, Madhya Pradesh and Uttar Pradesh. It does not 93. Match the following –
fall in Haryana. A. Gir Forest 1. Rajasthan
90. For what is the Manas sanctuary in Assam known? B. Bharatpur Bird Sanctuary 2. Madhya Pradesh
(a) Bears (b) Tigers C. Bandhavgarh Sanctuary 3. Assam
(c) Elephants (d) Birds D. Kaziranga National Park 4. Gujarat
Uttarakhand P.C.S. (Pre) 2010 Codes :
Ans. (b) A B C D

F–168 General Studies Environment and Ecology


(a) 1 2 4 3 96. Match List – I with List – II and select the correct
(b) 4 1 2 3 answer from codes given below :
(c) 2 4 3 1 List – I List – II
(d) 2 3 1 4 A. Dochigam 1. Andhra Pradesh
M.P.P.C.S. (Pre) 1993 B. Papikonda 2. Jammu and Kashmir
Ans. (b) C. Sariska 3. Rajasthan
The correct match is as follows – D. Bandipur 4. Karnataka
Gir Forest - Gujarat Codes :
Bharatpur Bird Sanctuary - Rajasthan A B C D
Bandhavgarh Sanctuary - Madhya Pradesh (a) 3 1 2 4
Kaziranga National Park - Assam (b) 2 1 3 4
94. Match List – I with List – II and select the correct (c) 1 2 4 3
answer from codes given below: (d) 3 4 2 1
List – I List – II U.P.P.C.S. (Pre) 2015
(Park/Sanctuary) (State) Ans. (b)
A. Dochigam Wildlife 1. Madhya Pradesh The correctly matched list is as follows –
Sanctuary National Park State
B. Keoladeo Ghana Bird 2. Rajasthan
Dochigam - Jammu and Kashmir
Sanctuary
Papikonda - Andhra Pradesh
C. Kanha National Park 3. Kerala
D. Periyar Wildlife 4. Jammu and Kashmir Sariska - Rajasthan
Sanctuary Bandipur - Karnataka
Codes : 97. Which of the following is not correctly matched?
A B C D
(a) Kaziranga National Park - Assam
(a) 4 2 1 3
(b) Corbett National Park - Uttarakhand
(b) 1 3 2 4
(c) Bandipur National Park - Tamil Nadu
(c) 2 1 4 3
(d) Sanjay National Park - Madhya Pradesh
(d) 3 4 2 1
U.P.P.S.C. (GIC) 2010
U.P.P.C.S. (Pre) 2014
Ans. (c)
Ans. (a)
National Parks mentioned in the question and their respective
Below is the correct match of the list-I with list-II –
Dochigam Wildlife Sanctuary - Jammu and Kashmir States are follows –
Keoladeo Ghana Bird Sanctuary - Rajasthan National Park State
Kanha National Park - Madhya Pradesh Kaziranga National Park - Assam
Periyar Wildlife Sanctuary - Kerala Corbett National Park - Uttarakhand
95. Dachigam National Park is associated with which of Bandipur National Park - Karnataka
the following? Sanjay National Park - Madhya Pradesh
(a) Musk Deer
(b) Golden Oriole 98. From the ecological point of view, which one of the
(c) Yellow-Throated Marten following assumes importance in being a good link
(d) Hangul or Kashmir Stag between the Eastern Ghats and the Western Ghats?
U.P. P.C.S. (Pre) 2018 (a) Sathyamangalam Tiger Reserve
Ans. (d) (b) Nallamala Forest
Dachigam National Park is located in Jammu and Kashmir, (c) Nagarhole National Park
near Srinagar. It is located in Zabarwan Range of the western (d) Seshachalam Biosphere Reserve
Himalayas. The park is famous for the Kashmiri stag also I.A.S. (Pre) 2017
known as Hangul. Ans. (a)

Environment and Ecology General Studies F–169


Sathyamangalam Tiger Reserve is located on the meeting Motichur Sanctuary (1964) and (iii) Chila Sanctuary (1977).
site of the Eastern Ghats and the Western Ghats. Apart from It was notified as a tiger reserve in 2015.
tiger, elephant, hyena, leopard etc are also found in this tiger
reserve. This is the largest wildlife sanctuary in Tamil Nadu. 101. Which one of the following is located in Chhattisgarh?
Nallamala forest is located in Andhra Pradesh. Nagarhole (a) Anshi National Park
National park is located in Karnataka. Sechachalam (b) Betla National Park
Biosphere Reserve is the first biosphere reserve of Andhra (c) Indravati National Park
Pradesh. (d) Gugamal National Park
99. Arrange the following National Parks in their U.P.R.O./A.R.O. (Pre) 2016
chronological order from the earliest to the latest and Ans. (c)
select the correct answer from codes given below the See the explanation of the above question.
parks:
1. Silent Valley 2. Jim Corbett 102. Which of the following animals can live for the longest
3. Kaziranga 4. Kanha duration without drinking water?
Codes : (a) Giraffe (b) Camel
(a) 1, 3, 2, 4 (b) 4, 2, 1, 3 (c) Kangaroo (d) Kangaroo rat
(c) 3, 1, 4, 2 (d) 2, 4, 3, 1 U.P. P.C.S. (Pre) 2018
U.P.R.O./A.R.O. (Pre) 2016 Ans. (d)
Ans. (d) Among the given options Kangaroo rat can live for the longest
National Park Establishment year duration without drinking water. It is due to the special
Silent Valley - 1984 structure and working system of its excretory system (kidney).
Jim Corbett - 1936 103. Which of the following has been recently declared as
Kaziranga - 1974 ‘Tiger Reserve’?
Kanha - 1978 (a) Balpakram National Park
(b) Rajaji National Park
100. Which one of the following is located in the Bastar
(c) Betala National Park
region?
(d) Kaziranga National Park
(a) Bandhavgarh National Park
U.P.P.C.S. (Mains) 2015
(b) Dandeli Sanctuary
(c) Rajaji National Park Ans. (b)
(d) Indravati National Park In 2015, Rajaji National Park in Uttarakhand was declared as
U.P. Lower Sub. (Pre) 2015 a tiger reserve. It is India’s 48th tiger reserve. After that Orang
I.A.S. (Pre) 2007 National Park of Assam was declared 49th tiger reserve of
Ans. (d) India. Notably, Kaziranga Park was declared tiger reserve
in 2006.
Indravati National Park is located in Dantewada district of
Bastar region of Chhattisgarh spread over 1258.37 sq. km. 104. Match List-I with List-II and select the correct answer
This was declared National Park in 1982 and tiger reserve from codes given below the Lists:
in 1983. List-I List-II
Bandhavgarh national park is located in Umaria district (Sanctuary / National (Main Protected
of Shahdol zone in Madhya Pradesh. It was declared Park) Animal)
National Park in 1981. It is mainly famous for white tigers.
A. Kaziranga 1. Tiger
Dandeli Sanctuary is located in Karnataka and spreads over
B. Gir 2. Elephant
886.41 sq.km.
C. Sundarban 3. Lion
Rajaji National Park, named after C. Rajgopalachari, spreads
D. Periyar 4. Rhinoceros
over three districts of Uttarakhand- Dehradun, Haridwar and
Codes :
Pauri Garhwal. This national park was formed merging three
A B C D
sanctuaries in 1983 - (i) Rajaji Sanctuary (1948), (ii)
(a) 1 4 3 2

F–170 General Studies Environment and Ecology


(b) 2 4 1 3 Ans. (d)
(c) 4 3 1 2
Yellowstone National Park is a located in the United States of
(d) 4 3 2 1
America (USA) in the state of Wyoming, Montana and Idaho.
U.P.P.C.S. (Mains) 2006 It was established on March 1, 1872. The park is known for
Ans. (c) its wildlife and its many geothermal features, especially Old
Faithful Geyser. It is the first National Park of U.S.A.
The correctly matched list is as follows :
(Sanctuary / National (Main Protected 108. Which of the following is a marine national park?
Park) Animal) (a) Bhitar-Kanika
(b) Sundarban
Kaziranga - Rhinoceros
(c) Gahirmatha
Gir - Lion
(d) Gulf of Mannar
Sundarban - Tiger
U.P.P.C.S. (Pre) (Re. Exam) 2015
Periyar - Elephant Ans. (d)
105. Which one of the following is not correctly matched? Gulf of Mannar is a marine national park in Tamil Nadu,
(a) Rajaji National Park : Elephant established in the year 1980.
(b) Periyar National Park : Hangul
(c) Manas National Park : Elephant 109. The UNESCO declared which one of the following
(d) Dudhwa National Park : Tiger National Park of India as the World Heritage Site in
U.P.P.C.S. (Pre) 2014 July, 2016?
Ans. (b) (a) Kaziranga National Park
(b) Kanchanjunga (Khangchengtzonga National Park)
Periyar National Park is home to tigers and a significant (c) Corbett National Park
elephant population. Hence (b) is not correctly matched.
(d) Valley of Flowers National Park
106. Which one of the following pairs is correctly matched? U.P.R.O./A.R.O. (Pre) 2016
(a) Great Himalayan National Park - Manali Ans. (b)
(b) Rajaji National Park - Dehradun The 40th World Heritage Committee at Istanbul, Turkey on
(c) Keoladeo National Park - Bharatpur the 17th of July, 2016 inscribed Khangchendzonga National
(d) Van Vihar National Park - Jabalpur Park of Sikkim as a UNESCO’s World Heritage Site on
U.P.P.C.S. (Mains) 2009 ‘Mixed’ criteria recognizing the outstanding universal values
Ans. (c) for its both natural and cultural significance. This status was
also conferred on Nalanda Mahavihar of Bihar and Capital
The Great Himalayan National Park is in Kullu, Himachal Complex of Chandigarh.
Pradesh, Rajaji National Park is spread over three districts of
Uttarakhand namely Haridwar, Dehradun and Pauri Garhwal. 110. Jaipur was listed as 'World Heritage Site' by the
Keoladeo National Park is located in Bharatpur, Rajasthan. UNESCO 'World Heritage Committee' in its 43rd
Van Vihar National Park is in Bhopal, Madhya Pradesh. Session held in the city
Hence (c) is the correct answer. (a) Baku (Azerbaijan)
107. The Yellowstone National Park is located in (b) Bishkek (Kyrgyzstan)
(a) Canada (c) Istanbul (Turkey)
(b) New Zealand (d) Marrakesh (Morocco)
(c) South Africa U.P.P.C.S. (Pre) 2019
(d) The U.S.A. Ans. (a)
U.P.P.C.S. (Mains) 2011 Jaipur was listed as 'World Heritage Site' by the UNESCO
U.P.P.C.S. (Spl) (Mains) 2004 'World Heritage Committee" in its 43rd session held in Baku,
Uttarakhand P.C.S. (Pre) 2005 (Capital of Azerbaijan).
Uttarakhand P.C.S. (Pre) 2006 111. Which of the following is the only UNESCO World
U.P.P.C.S. (GIC) 2010 Heritage Site present in Bihar State of India?

Environment and Ecology General Studies F–171


(a) Mahabodhi Vihar Small scale photographs are suitable for wildlife management
(b) Buddhist Monastery in Nalanda in Jim Corbett National Park situated at Uttarakhand and
(c) Ancient site of Vikramashila monastery Rajaji National Park.
(d) Tomb of Sher Shah Suri
3. Which of the following National Parks is unique in
(e) None of the above/More than one of the above
being swamp with floating vegetation that supports a
63rd B.P.S.C. (Pre) 2017
rich biodiversity?
Ans. (e)
(a) Bhitarkanika National Park
Mahabodhi Temple Complex at Bodh Gaya (2002) and (b) Keibul Lamjao National Park
Archaeological Site of Nalanda Mahavihar at Nalanada, (c) Keoladeo Ghana National Park
(2016) are included in the list of UNESCO World Heritage (d) Sultanpur National Park
Site in Bihar state of India. Hence option (e) is correct answer. I.A.S. (Pre) 2015
Ans. (b)
High-Level Questions The Keibul Lamjao National Park is situated in Bishnupur
1. Which one of the following pairs is correctly matched? district of Manipur. This national park is characterized by
many floating decomposed plant materials locally called
(a) Biosphere reserves - Edward Suess
phumdis.
(b) Ecosystem - A.P. de Candolle
(c) Ecology - A.G. Tansley 4. Recently, for the first time in our country, which of the
(d) Biodiversity - Reiter following State has declared a particular butterfly as
U.P.P.C.S.(Pre) 2013 ‘State Butterfly’?
Ans. (a) (a) Arunachal Pradesh (b) Himachal Pradesh
(c) Karnataka (d) Maharashtra
The term ‘biosphere reserve’ was first used by Edward Suess
I.A.S. (Pre) 2016
whereas the term Ecosystem was first used by A.G. Tansley. Ans. (d)
Reiter and Ernst Haeckel first used the term ' Ecology' and the
Maharashtra has become the first state in the country to have
word ‘Biodiversity’ was first used by Ramond F. Dasmann.
a ‘State butterfly’. State Government has declared the Blue
Hence (a) is the correct answer. Mormon as the State Butterfly. The Blue Mormon is a large
swallowtail butterfly found primarily in Sri Lanka and India
Transition area mainly restricted to the Western Ghats of Maharashtra, South
India and Coastal belts. It is the second largest butterfly of
Buffer zones India. Southern Birdwing is India's largest butterfly which
has been declared as State butterfly of Karnataka. Notably,
Core area
Maharashtra is the first and Karnataka the second state to
declare 'State butterfly'.
Education & Training 5. Which one of the following biosphere reserves are not
included in the list of a world network of biosphere
reserves of UNESCO?
(a) Similipal (b) Sunderban
2. Which of the following aerial photographs are suitable (c) Gulf of Mannar (d) Nilgiri
for wildlife management in Corbett and Rajaji National U.P.P.C.S (Pre) 2010
Park?
(a) Small scale photographs Ans. (*)
(b) Medium scale photographs
Initially, UNESCO's World Biosphere reserve list only
(c) Large scale photographs included India's Nilgiri, Sundarban, Bay of Mannar and
(d) Extra large scale photographs Nanda Devi Biosphere reserves. In May, 2009 UNESCO
Uttarakhand P.C.S. (Pre) 2006 announced to include 22 new biosphere reserves of 17
Ans. (a) countries. In the new list India's Similipal, Pachmadi and

F–172 General Studies Environment and Ecology


Nokrek biosphere reserves were also included. After that Gulf of Mannar (10500 km 2 ) is the largest biosphere reserve
in July, 2012, UNESCO also included India's Achanakmar- in term of the area approved by UNESCO. However, terms
Amarkantak biosphere reserve. In 2013, Great Nicobar of the area it is India's second largest biosphere reserve. The
biosphere reserve also became a part of this list. By February, largest bio-reserve in India is Rann of Kachchh with a total
2021, UNESCO's list included 714 biosphere reserves of 129 area of 12454 sq. km.
countries belongs to World Network of Biosphere Reserves.
10. Consider the following pairs
6. ‘Man and Biosphere Programme’ (MAB) by UNESCO 1. Nokrek Biosphere Reserve : Garo Hills
was started in – 2. Logtak (Loktak) Lake : Barail Range
(a) 1991 (b) 1970 3. Namdapha National Park : Dafla Hills
(c) 1972 (d) 1986 Which of the above pairs is/are correctly matched?
R.A.S./R.T.S. (Pre) 2013 (a) Only 1 (b) 2 and 3
Ans. (*) (c) All of these (d) None of these
I.A.S. (Pre) 2013
Launched in 1971, UNESCO’s Man and the Biosphere
Ans. (a)
Programme (MAB) is an Intergovernmental Scientific
Programme that aims to establish a scientific basis for the Nokrek National Park is located 2 km from Tura Peak in
improvement of relationships between people and their West Garo Hills district of Meghalaya. UNESCO added this
environments. park to its list of Biosphere Reserves in May, 2009. Loktak
7. Great Himalayan National Park, which has been Lake is the largest freshwater lake of India. It is located in
accorded the UNESCO World Heritage Site status, Manipur and is famous for the phumdis floating over it.
is located in: Namdapha National Park is the largest protected area in
the Eastern Himalaya biodiversity hotspot and is located in
(a) Uttarakhand (b) Himachal Pradesh
Arunachal Pradesh.
(c) Jammu-Kashmir (d) Nagaland
Uttarakhand P.C.S. (Pre) 2012 11. The 16th Biosphere Reserve of India’ Cold Desert, lies
Ans. (b) in–
(a) Arunachal Pradesh (b) Himachal Pradesh
The Great Himalayan National Park (GHNP) covering
(c) Jammu and Kashmir (d) Uttarakhand
an area of 754.4 sq. km is located in the Kullu District of
U.P.P.C.S. (Mains) 2010
Himachal Pradesh, India. The Great Himalayan National
Park Conservation Area (GHNPCA) as a Natural Site was Ans. (b)
inscribed into the UNESCO World Heritage List during the Cold Desert Biosphere Reserve located in Himachal Pradesh
session of the 38th World Heritage Committee in Doha, the was on 28th August, 2009 declared as 16th Biosphere Reserve
State of Qatar, on 23rd June, 2014. of India. On 20th September, 2010, Seshachalam Hills of
Andhra Pradesh was declared as 17th Biosphere Reserve of
8. The ‘‘Oceanic National Park’’ of India is located in :
India and on 25th August, 2011, Panna (Madhya Pradesh) was
(a) Sunderbans (b) Chilika Lake
declared as 18th Biosphere Reserve of India. Establishment of
(c) Nicobar Island (d) Kutch Biosphere Reserves help to protect biodiversity and achieve
U.P.R.O./A.R.O. (Re-Exam) (Pre) 2016 sustainable development.
Ans. (d)
12. Which of the following are in Agasthyamala Biosphere
The ‘Oceanic National Park’ of India is located in Gulf of Reserve?
Kutch in Jamnagar, Gujarat. The park was established in (a) Neyyar, Peppara and Shendurney Wildlife Sanctuaries
1995 and spread in an area of 162.89 sq. km.
and Kalakad Mundanthurai Tiger Reserve
9. 8. Which one of the following is (regarding area) the (b) Mudumalai, Sathyamangalam and Wayanad Wildlife
largest UNESCO approved biosphere reserve of India? Sanctuaries; and Silent Valley National Park
(a) Nilgiri (b) Nanda Devi (c) Kaundinya, Gundla Brahmeswaram and Papikonda
(c) Sunderban (d) Gulf of Mannar Wildlife Sanctuaries, and Mukurthi National Park
U.P.R.O./A.R.O. (Mains) 2013 (d) Kawal and Sri Venkateswara Wildlife Sanctuaries;
Ans. (d) and Nagarjunasagar-Srisailam Tiger Reserve

Environment and Ecology General Studies F–173


I.A.S. (Pre) 2019 16. The world’s largest population of tigers is in India. It
Ans. (a) is estimated –
In the Agasthyamala Biosphere Reserve Neyyar, Peppara (a) 6000 (b) 16000
and Shendurney Wildlife Sanctuaries are located. Moreover (c) 60000 (d) 10600
Kalakad Mundanthurai Tiger Reserve is also situated here. 40th B.P.S.C. (Pre) 1995
Ans. (*)
13. The Government of India has set a target of achieving
how much solar power capacity by the year 2022? The total estimated population of tigers in the word is 3000-
(a) 20 Giga watt (b) 40 Giga watt 4000. India is home to 2967 (According to latest census)
(c) 80 Giga watt (d) 100 Giga watt tigers which is highest as compared to other countries in the
world. Hence, none of the options is correct.
R.A.S./R.T.S. (Pre) 2018
Ans. (d) 17. With reference to the detailed report of Tiger Census
2018 released by Government of India in July 2020,
The Government has set a target of installing 100GW of solar
which of the following Statement(s) is/are correct?
capacity by 2022 in the country. A target of installing 175
GW of renewable energy capacity by the year 2022 has been 1. India has 70% of total global tiger’s population.
set, which includes 100 GW from solar, 60 GW from wind, 2. Nearly 30% of India’s tigers live outside tiger
10 GW from bio-power and 5 GW from small hydropower. reserves.
Select the correct answer from the codes given below.
14. The garden of five seasons is located near –
Codes :
(a) Dal Lake (b) Mehrauli
(a) 1 only (b) 2 only
(c) Silent Valley (d) Ooty
U.P.P.C.S. (Mains) 2009 (c) 1 and 2 both (d) Neither 1 nor 2
Ans. (b) U.P.P.S.C. (Pre) 2020
Ans. (c)
The garden of five seasons is located near Mehrauli. The
Garden was inaugurated in February, 2003. It spans over 20 According to the latest tiger census, there are 2967 tigers
acres of land. alive in India. With reference to the detailed report of Tiger
Census 2018, released by the Government of India in July
15. “India has the largest population of Asian X. Today, 2020, India has 70% of world’s tiger population and nearly
there are just about 20,000 to 25,000 X in their natural 30% of India’s tigers live outside tiger reserves.
habitat spreading across the evergreen forests, dry
18. Which of the following National Parks has started to
thorn forest, swamps and grasslands. Their prime
use a drone or unmanned aerial vehicle for wildlife
habitats are however the moist deciduous forests. The
management?
X population in India ranges from North-West India
(a) Bandipur Tiger Reserve
where they are found in Dehradun and Nainital of
(b) Corbett Tiger Reserve
Uttarakhand to forest areas of Bijnor in Uttar Pradesh
(c) Ranthambore Tiger Reserve
to the Western Ghats in the States of Karnataka,
(d) Periyar Tiger Reserve
Kerala and Tamil Nadu.
U.P.R.O/A.R.O. (Pre) 2014
In Central India, their population is distributed in
Ans. (a)
Southern Bihar and Orissa. In the East, they are seen
in North Bengal, Assam and a few other states”. Recently, Bandipur Tiger Reserve of Karnataka has started
The animal “X” referred to in this question is: to use a drone or unmanned aerial vehicle for wildlife
(a) Lion (b) Elephant management.
(c) Tiger (d) One-horned rhinoceros 19. Which National Park/Sanctuary has been selected to
I.A.S. (Pre) 1999 house Gir lions?
Ans. (b) (a) Pench (b) Kanha
The description given in the question is referring to Asian (c) Bandhavgarh (d) Palpur Kuno
Elephants and areas mentioned in the above paragraph are M.P.P.C.S. (Pre) 2008
the habitat of Asian Elephants. Ans. (d)

F–174 General Studies Environment and Ecology


Kuno Wildlife Sanctuary or Palpur-Kuno Wildlife Ans. (c)
Sanctuary lies in the Sheopur district of Madhya Pradesh. The Desert National Park is spread in two districts of Rajasthan-
Kuno Wildlife Sanctuary was selected as the reintroduction site Jaisalmer and Barmer. It is one of the natural habitats of Great
for the endangered Asiatic lion because it was home to Asiatic Indian Bustard (locally called Godawan). So, statements 1
lions before they were hunted to extinction in about 1873. and 3 are correct. There is human habitation inside the park,
so statement 2 is not correct.
20. Consider the following pairs-
Protected area : Well- known for 23. If you want to see gharials in their natural habitat,
1. Bhitarkanika, Orissa : Salt Water Crocodile which one of the following is the best place to visit?
2. Desert National Park, : Great Indian Bustard (a) Bhitarkanika Mangroves
Rajasthan (b) Chambal River
3. Eravikulam, Kerala : Hoolak Gibbon (c) Pulicat Lake
Which of the pairs given above is/are correctly (d) Deepor Beel
matched? I.A.S. (Pre) 2017
(a) 1 only (b) 1 and 2 only Ans. (b)
(c) 2 only (d) 1, 2 and 3
To see gharials in their natural habitat, Chambal river is the
I.A.S. (Pre) 2010
best place to visit. The scientific name of gharial is 'Gavialis
Ans. (b)
Gnageticus'. The most of gharials in India are found in
Bhitarkanika National Park has one of the largest populations Chambal and Girwa river. IUCN has placed gharials in the
of endangered saltwater crocodiles, in India. Desert National category of critically endangered.
Park is a beautiful place, located in the Jaisalmer district of the
24. Match List - I with List - II and select the correct
State of Rajasthan. The great Indian Bustard is a magnificent
bird and can be seen in considerably good numbers in this answer using codes given below the lists:
park. Hoolak Gibbon is found in the north-eastern States of List- I List II
India mainly in Assam. (National Park) (Location)
A. Kaziranga 1. Chikmagalur
21. In which of the following regions of India are you most
B. Kudremukh 2. Palghat
likely to come across the ‘Great Indian Hornbill’ in its
C. Silent Valley 3. Nagpur
natural habitat?
D. Pench Valley 4. Golaghat-Nawgaon
(a) Sand deserts of northwest India
Codes :
(b) Higher Himalayas of Jammu and Kashmir
A B C D
(c) Salt marshes of western Gujarat
(a) 3 4 2 1
(d) Western Ghats
(b) 4 1 2 3
I.A.S. (Pre) 2016
(c) 2 4 1 3
Ans. (d)
(d) 1 3 4 2
The natural habitat of 'Great Indian Hornbill' is Western U.P. Lower Sub. (Pre) 2013
Ghat. Besides India, great hornbills are also found in Nepal, Ans. (b)
Bhutan, China and Indonesia.
Correctly matched list is as follows –
22. With reference to India’s Desert National Park, which Kaziranga - Golaghat-Nawgaon
of the following statements are correct? Kudremukh - Chikmagalur
1. It is spread over two districts.
Silent Valley - Palghat
2. There is no human habitation inside the Park.
Pench Valley - Nagpur
3. It is one of the natural habitats of Great Indian
Bustard. 25. Which of the following Protected Areas are located in
Select the correct answer using the code given below : Cauvery basin?
(a) 1 and 2 only (b) 2 and 3 only 1. Nagarhole National Park
(c) 1 and 3 only (d) 1, 2 and 3 2. Papikonda National Park
I.A.S. (Pre.) 2020 3. Sathyamangalam Tiger Reserve

Environment and Ecology General Studies F–175


4. Wayanad Wildlife Sanctuary Askot Wildlife Sanctuary and Gangotri National Park are
Select the correct answer using the code given below : located in Uttarakhand State. Both of these protected areas are
(a) 1 and 2 only (b) 3 and 4 only most likely places to find the musk deer in its natural habitat.
(c) 1, 3 and 4 only (d) 1, 2, 3 and 4
29. Match List-I and List-II and select the correct answer
I.A.S. (Pre.) 2020
using codes given below the Lists :
Ans. (c)
List-I List-II
The Cauvery basin extends over states of Tamil Nadu, (Tiger Reserve) (State)
Karnataka, Kerala and Union Territory of Puducherry. A. Bandipur 1. West Bengal
Nagarhole National Park (Karnataka), Sathyamangalam
B. Melghat 2. Karnataka
Tiger Reserve (Tamil Nadu) and Wayanad Wildlife Sanctuary
C. Buxa 3. Arunachal Pradesh
(Kerala) are located in Cauvery basin. Whereas Papikonda
D. Pakke 4. Maharashtra
National Park is located in Andhra Pradesh. So, option (c)
Codes :
is correct answer.
A B C D
26. At which place India’s first Butterfly Park has been
(a) 3 4 2 1
established?
(b) 2 4 1 3
(a) Bannerghatta Biological Park, Bangalore (c) 2 3 1 4
(b) National Zoological Park, Kolkata (d) 1 2 4 3
(c) Kaziranga National Park. U.P.P.C.S. (Mains) 2008
(d) None of the above Ans. (b)
Chhattisgarh P.C.S. (Pre) 2008
See the explanation of the above question.
Ans. (a)
India's first Butterfly Park is Bannerghatta National Park near 30. The Corbett National Park gets its water from
Bengaluru, Karnataka. It was founded in 1970 and declared (a) Alaknanda river (b) Ramganga river
as a National Park in 1974. (c) Dhauliganga river (d) Sarda River
U.P.P.C.S. (Mains) 2006
27. Askot Wildlife Sanctuary is situated in the district of –
Ans. (b)
(a) Almora (b) Chamoli
Corbett National Park gets its water from the Ramganga river.
(c) Uttarkashi (d) Pithoragarh
Uttarakhand U.D.A./L.D.A. (Pre) 2003 31. Which one of the following National Park has a climate
Ans. (d) that varies from tropical to subtropical, temperature
Askot Wildlife Sanctuary is located 54 km from Pithoragarh and Arctic?
near Askot in Uttarakhand. (a) Khangchendzonga National Park
(b) Nandadevi National Park
28. Which of the following are the most likely places to (c) Neora Valley National Park
find the musk deer in its natural habitat? (d) Namdapha National Park
1. Askot Wildlife Sanctuary I.A.S. (Pre) 2015
2. Gangotri National Park Ans. (d)
3. Kishanpur Wildlife Sanctuary Namdapha National Park, situated in Arunachal Pradesh, has
4. Manas National Park a climate that varies from tropical to subtropical, temperature
Select the correct answer using the code given below : and arctic.
(a) 1 and 2 only (b) 2 and 3 only 32. Buxa Tiger Reserve is located in which State of India?
(c) 3 and 4 only (d) 1 and 4 only (a) Madhya Pradesh (b) Rajasthan
I.A.S. (Pre.) 2020 (c) Gujarat (d) West Bengal
Ans. (a) U.P.P.C.S. (Pre) 2003

F–176 General Studies Environment and Ecology


Ans. (d) are the rivers which flow through Kaziranga National Park.
Buxa Tiger Reserve is situated in Alipur Duar sub-division Kunthipuzha river flows through Silent Valley National Park.
of Jalpaiguri district, West Bengal. Hence none of them is correctly matched.
36. Consider the following pairs:
33. Suklaphanta Wildlife Reserve is located in
Wetlands Confluence of rivers
(a) Nepal (b) Myanmar
1. Harike Wetlands : Confluence of Vyas and
(c) Bhutan (d) Sri Lanka
Satluj/Sutlej
U.P.P.C.S. (Mains) 2016
2. Keoladeo Ghana : Confluence of Banas and
Ans. (a)
National Park Chambal
Suklaphanta Wildlife Sanctuary is located in Kanchanpur 3. Kolleru Lake : Confluence of Musi
district of south-west Nepal. It is spread over 305 sq. km. and Krishna
area. This area was banned for hunting till 1969. In 1976 it
Which of the above pairs is/are correctly matched?
was recognized as wildlife sanctuary.
(a) 1 only (b) 2 and 3 only
34. Consider the following pairs : (c) 1 and 3 only (d) 1, 2 and 3
1. Dampa Tiger Reserve : Mizoram I.A.S. (Pre) 2014
2. Gumti Wildlife Sanctuary : Sikkim Ans. (a)
3. Saramati Peak : Nagaland Harike Wetland is located downstream the confluence of
Which of the above pairs is/are correctly matched? the Vyas and Satluj rivers. Keoladeo Ghana National Park
(a) 1 only (b) 2 and 3 only is located at the confluence of two rivers, the Gambhir and
(c) 1 and 3 only (d) 1, 2 and 3 Banganga. Kolleru Lake is one of the largest freshwater lakes
I.A.S. (Pre) 2014 in India in Andhra Pradesh between Krishna and Godavari
Ans. (c) deltas. Hence, only option (1) is correctly matched.
Dampa Tiger Reserve, the largest wildlife sanctuary in 37. The first marine sanctuary in India with coral reefs,
Mizoram was notified in 1985 and declared Tiger Reserve molluscs, dolphins, tortoises and various kinds of sea
in 1994. Gumti Wildlife Sanctuary is situated in Tripura.
birds, within its bounds, has been established in:
Saramati is a peak rising above the surrounding peaks at the
(a) Sundarbans (b) Chilka Lake
mountainous border of Nagaland State, India and the Sagaing
(c) Gulf of Kachchh (d) Lakshadweep
region of Burma. Hence 1 and 3 are correctly matched.
I.A.S. (Pre) 1999
35. Consider the following pairs:
Ans. (c)
National Park River flowing through
Marine National Park in the Gulf of Kachchh is situated
the park
on the southern shore of the Gulf of Kachchh. There are 42
1. Corbett National Park : Ganga
islands on the Jamnagar coast in the Marine National Park,
2. Kaziranga National : Manas
most of them surrounded by reefs. 52 coral species, molluscs,
Park
dolphins, tortoises and various kinds of sea birds are
3. Silent Valley National : Kaveri found here.
Park
38. The “Cloud Goats” of the Nilgiri is found in-
Which of the above pairs is/are correctly matched?
(a) Eravikulam National Park (b) Mann Forest
(a) 1 and 2 (b) Only 3
(c) Periyar Reserve (d) Silent Valley
(c) 1 and 3 (d) None of these
U.P.P.C.S. (Mains) 2005
I.A.S. (Pre) 2013
Ans. (a)
Ans. (d)
Ramganga and Kosi rivers flow through Corbett National The Cloud Goats of Nilgiri is found in Eravikulam National
Park but river Ganga does not flow through Corbett National Park. They are rare species and called Nilgiri Tahr.
Park. Brahmaputra, Diphlu, Mora Diphlu and Mora Dhansiri 39. Match List-I with List-II and select the correct answer

Environment and Ecology General Studies F–177


using codes given below in the lists : 42. Sultanpur Bird Sanctuary is situated in –
List–I (National Park/Wildlife Sanctuary) (a) Chandigarh (b) Bharatpur
A. Bondla Wildlife Sanctuary (c) Gurgaon (d) Gandhi Nagar
B. Kangerghat National Park 56 to 59th B.P.S.C. (Pre) 2015
th

C. Orang Sanctuary Ans. (c)


D. Ushkothi Wildlife Sanctuary Sultanpur Bird Sanctuary is situated in Gurgaon, Haryana.
List–II (State)
43. The bird sanctuary of Tamil Nadu is located in –
1. Orissa 2. Assam
(a) Karikili (b) Kalakadu
3. Chhattisgarh 4. Goa
(c) Kunthakulum (d) Mudumalai
5. Tripura
U.P.P.C.S. (Mains) 2008
Codes :
Ans. (a)
A B C D
(a) 2 1 5 3 Bird Sanctuary of Tamil Nadu is located in Karikili. It is 40
(b) 4 3 2 1 km away from Chennai
(c) 2 3 5 1 44. Which of the following countries has more than 30%
(d) 4 1 2 3 of its total area under National Parks?
I.A.S. (Pre) 2005 (a) Israel (b) Bhutan
Ans. (b) (c) Iceland (d) Gabon
U.P.P.C.S. (Pre) (Re. Exam) 2015
Bondla Wildlife Sanctuary is located in the northeast of Goa.
Ans. (b)
Spread over 8 sq. km area, this sanctuary is particularly a
centre of attraction for students. Kangerghat National Park In Bhutan, National Parks spread over 12,922 sq. km. which
is situated in Chhattisgarh. Orang sanctuary, also known is 33.66% of the total geographical area of the country.
as Mini Kaziranga is located in Assam. Ushkothi Wildlife
45. The largest Botanical garden of the world is at –
Sanctuary is located in Odisha.
(a) Kew (England) (b) Paris (France)
40. Which one of the following National Parks lies (c) Howrah (India) (d) Tokyo (Japan)
completely in the temperate alpine zone? U.P.P.C.S. (Pre) 2015
(a) Manas National Park Ans. (a)
(b) Namdapha National Park World's largest botanical garden- Royal Botanical Garden is
(c) Neora Valley National Park located in Kew, England. It was formed in 1759 and declared
(d) Valley of Flowers National Park a world heritage site by UNESCO in 2003. It is habitat to
I.A.S. (Pre) 2019 more than 40 thousand floral species.
Ans. (d) 46. Bundala Biosphere Reserve which has been recently
Valley of Flowers National Park lies completely in the added to the UNESCO’s Man and Biosphere (MAB)
temperate alpine zone. Valley of flowers is situated in network is located in ?
Chamoli District of Uttarakhand. This valley is situated in (a) Russia (b) India
backdrop of Zaskar range of Himalayas. (c) Sri Lanka (d) Bangladesh
I.A.S. (Pre) 2006
41. The Chinar Wild Life Sanctuary is located in –
Ans. (c)
(a) Jammu and Kashmir (b) Kerala
(c) Sikkim (d) Tamil Nadu Bundala Biosphere Reserve is located on the southeast coast
U.P.P.C.S. (Mains) 2005 of Sri Lanka. It is an international wintering ground for
Ans. (b) migratory birds. In 2005, it was designated as a biosphere
reserve and included in Man and Biosphere Reserve Network
Chinar Wildlife Sanctuary is situated in Kerala. by UNESCO.

F–178 General Studies Environment and Ecology


Non-Conventional Energy
Energy and Energy Resources *The danger of exhaustion of fossil fuels such as crude oil and
natural gas is named as the Energy crisis.
*Energy resources can be classified into two categories:
1. Renewable or Non Conventional Energy *Uranium is used as a raw material for nuclear power

2. Non Renewable or Conventional Energy. generation. Nuclear energy is energy that is obtained from
*All the activities of nature are controlled by some form of the nucleus of an atom. The energy is released from an atom
energy. This energy comes from different resources. Renewable through one of the two processes - 1. Nuclear Fusion or 2.
energy is energy from sources that are naturally replenishing but Nuclear Fission.
flow-limited; renewable resources are virtually inexhaustible in
duration but limited in the amount of energy that is available
Renewable and Non-Renewable
per unit of time. Energy Resources
*The United Nations General Assembly unanimously declared *Some examples of renewable energy sources are solar energy,
the decade 2014-2024 as the "Decade of Sustainable Energy
wind energy, tidal energy, hydroelectric energy, etc.
for all".
*It is noteworthy that the best source of the production of
electricity in the perspective of sustainable development is
Hydroelectric energy as compared to coal energy, petroleum
energy, natural gas and nuclear energy.
*As on 28th February, 2021 the main Renewable Energy Sources
and their capacity is given below :

Renewable Energy Sources Capacity (in MW)


Wind Power 38789.15
Solar Power 39083.71
Bio Power 10314.56
Geothermal Small Hydro Power 4783.06

Alternative Energy/Solar Energy


*Sun is the most comprehensive and unlimited source of
energy. Solar Energy is a renewable resource, which is
obtained by solar radiation. Solar energy is the largest source
*The fuels we get by the decomposition of buried plants or dead
organisms over the course of millions of years are called Fossil of alternative energy. The light and heat coming from the sun

fuels. Fossil fuels are non-renewable energy resources. Fossil is necessary for the existence of life. Solar energy is essential

fuels are currently the major sources of energy. Once they are for the Photosynthesis of plants and the survival of another
used, they cannot be recovered again. Examples of fossil fuels living organism. Photovoltaic technology is used to convert
are coal, natural gas, petroleum etc. sunlight into electricity.

Environment and Ecology General Studies F–179


31 October, 2019 in New Delhi. According to ISA declaration,
its member has set a target of raising USD 1 trillion for solar
energy by 2030.

Biodiesel
*It is noteworthy that energy from biomass can be obtained
through disintegration of chemical bonds between biological
atoms. These chemical bond are formed through the process
of photosynthesis. Hence biomass is also an indirect source of
*The Ministry of New and Renewable Energy (MNRE) solar power.
provides a 70% subsidy on installation cost for a solar power
*Jatropha is a Biodiesel crop. Jatropha is a species of flowering
plant in the North - Eastern States.
plant, which is resistant to dry environment and harmful insects.
*The National Solar Mission is one of the several initiatives
that are part of National Action Plan on Climate Change. Jatropha’s oil can be converted to biodiesel to use it in diesel
*The solar power source of renewable energy has the greatest engine. But Bio-fuels are not cost effective as compared to fossil
potential in India. The role of new and renewable energy has fuels. Bio-fuel can be obtained from Jatropha.
been assuming increasing significance in recent times with the *Mahua and Neem etc. can also be used as bio-fuels.
growing concern for the country’s energy security.
*Other than Jatropha, Pongamia plant is considered as a
*Solar panels are most eco-friendly power generation systems.
resource that can meet the growing demand of biodiesel in India
They are also called as the fuel of the future. This is why
due to their high productivity and less maturity cycle. Pongamia
the Government has set a target of 100GW MW solar power
pinnata grows naturally in most of the arid regions in India. It
generation by 2022 under Jawahar Lal Nehru Solar Mission
can be grown in waterlogged, saline and alkaline soil and can
started on 11 January, 2010.
withstand harsh agro-climates. The seeds of Pongamia pinnata
*The world's largest solar plant at a single location, has been
are rich in liquid content of which nearly half is oleic acid.
established in Tamil Nadu. Domestic Content Requirement
(DCR) is related to the development of the production of solar
energy in the country.
*The objective of the National Solar Mission is to establish
India as a global leader in solar energy. In January 2010 National
Solar Mission was renamed Jawaharlal Nehru National Solar
Mission.
*International Solar Alliance-(ISA) is a group of 102 (latest
signatory is Antigua and Barbuda) countries located between
the Tropic of Cancer and Tropic of Capricorn using solar energy
from the sun for their energy needs.
*The initiative of ISA was launched at United Nations Climate
Change conference in Paris on 30 November, 2015 in the
presence of Prime Minister Narendra Modi, Francois Hollande
(then President of France) and Ban Ki - Moon (then Secretary-
General of UNO). Its headquarter is located in Gurugram, *Chemical reactions in this process biological substances or
Haryana. fossil-based carbonic substances can be transformed to Carbon
The first International Solar Alliance Summit was held on 11 th
Monoxide, Hydrogen, Carbon dioxide and Methane. Obtained
March, 2018 in New Delhi. The 2 ISA Summit was held on
nd
gaseous mixture is known as Producer Gas or Synthetic Gas

F–180 General Studies Environment and Ecology


which itself is a fuel. It’s significance lies in the fact that it can from non-petroleum based renewable resources. In these
serve as an important source of obtainable renewable energy. sources sugar, molasses, paddy, maize, lignin, cellulose, waste
of palm oil, waste of white mustard oil are included.
Green Fuel *Non-petroleum based tar (i.e. Asphalt) is helpful to reduce
*Use of Green Fuels leads to a reduction in Environmental the surface temperature of roads. Hence, the construction of
Pollution. Ethanol is a famous alcohol. It is also known as ethyl upper layer of roads from bioasphalt is ecologically favourable.
alcohol. Ethanol is obtained from any sugary substance (like *A fuel cell is an electrochemical cell that converts the chemical
sugarcane’s sugar, glucose, saltpetre, Mahua’s flower etc.) or energy from a fuel into electricity through an electrochemical
starch. Ethanol is obtained by fermentation of Pine, Karanja reaction of hydrogen with oxygen.
(Pongamia pinnata) and Fern. These are used as a source of *Stationary fuel cells are used for commercial, industrial and
green fuels. residential purposes and backup power generation. Fuel cells
*It is noteworthy that bio-ethanol can be obtained from generate Direct Current (DC).
sugarcane crops. This can be used to obtain energy in the form *It is noteworthy that when hydrogen burns in air, then it
of fuel. combines with oxygen to produce water and in this process, a
huge amount of energy (150 KJ/g) is released.
*Microbiological Fuel Cells (MFC) can convert the available
chemical energy present in organic substrate directly into the *Hydrogen gas is considered to be a pollutionless extremely
electrical energy. Micro-organisms like bacteria are used as good fuel due to its highest calorific value.
a catalyst in the conversion of substrate into an electron. For *Hydrogen is produced by temperature differentiation,
purification of wastewater, use of microbiological fuel cells photolysis and electrical reduction of water.
not only optimise the cost of purification but it is an attractive *In order to accelerate the development and utilization of
option for the production of electricity. hydrogen energy in the country, a National Hydrogen Energy
*In Micro-biological fuel cells, primarily carbonic substances Board was set up in 3 October, 2003.
like glucose, sucrose, cellulose, starch, formate, acetate, ethanol *Up to 2020 it targets to set up 1000 MW Hydrogen based
methanol, amino acid, protein etc are used in the form of power generation capacity. Hydrogen gas is called as the
substrate. Also some inorganic substances like sulphides are future fuel.
used in the form of substrate.
Miscellaneous
*The high level of expertise is required for the production and
engineering of these fuels, as well as knowledge and experience *Primary forms of Biogas are methane and carbon dioxide.
are required for higher production and innovation. It is difficult *Geothermal energy is the natural heat of the earth.
to produce such type of fuel in developing countries because *In India sources of geothermal energy is found primarily
it requires heavy capital investment apart from establishing in : Himalaya, Khambhat Basin, Sonata (SO-NA-TA)
facilities for large scale production, ecology and society can (Son-Narmada-Tapi), Western Ghat, Godavari Basin and
be adversely affected. Mahanadi Basin.
*Diu is the first Union Territory of India to run 100 percent
on solar energy. In the Union Territory, solar plants had been
installed over more than 50 acres of land in 2018.
*Sulphur present in the coal is responsible for the formation
of sulphur dioxide.
*Sulphur dioxide gas is primarily responsible for acid rain.
*Sikkim is the first 100 percent Organic State in the world.
The State won the prestigious Future Policy Gold Award
from the UN Food and Agriculture (FAO), after beating 51
*Bioasphalt is a substitute to Asphalt which can be generated nominations from around the world.

Environment and Ecology General Studies F–181


U.P.P.C.S.(Mains) 2004
Basic Questions Ans. (b)

1. Which one of the following is a renewable source of Solar energy is used most in organic form. Light and heat
energy? from the Sun are essential for life on Earth. Solar energy
(a) Coal is required for the process of photosynthesis in plants.
(b) Nuclear energy Photovoltaic systems convert sun rays into electricity.
(c) Petrol 6. Which of the following energy source is most eco-
(d) Solar energy friendly?
M.P.P.C.S. (Pre) 2016 (a) Petroleum Product (b) Forest Product
Ans. (d) (c) Nuclear Fission (d) Solar Cell
Solar energy is radiant light and heat from the sun that is U.P.P.C.S. (Pre) 2015
harnessed using a range of ever-evolving technologies such U.P.P.C.S.(Mains) 2006
as solar heating, photovoltaics, solar thermal energy etc. It is Ans. (d)
an important source of renewable energy and its technologies
are broadly characterized as either passive solar or active Among the energy sources given in the option, Solar cell is
solar depending on how they capture and distribute solar most eco-friendly as it converts the energy of light directly
energy or convert it into solar power. into electricity by the photovoltaic effect. Rest options affect
the environment in some way.
2. Which of the following is the largest storehouse of
7. Which among the following places was the first 100
alternative energy?
(a) Solar Energy percent Organic State in the world?
(b) Tidal Energy (a) Queensland (Australia)
(c) Nuclear Energy (b) California (United States of America)
(d) Geo-Thermal Energy (c) Sikkim (India)
40th B.P.S.C. (Pre) 1995 (d) Quebec (Canada)
Ans. (a) U.P.P.C.S. (Pre) 2019
Ans. (c)
See the explanation of the above question.
Sikkim is the first 100 percent Organic State in the world.
3. Solar energy is drawn from which of the following :
The State won the prestigious Future Policy Gold Award
(a) Moon (b) Ocean from the UN Food and Agriculture (FAO), after beating 51
(c) Sun (d) Wind nominations from around the world.
44th B.P.S.C. (Pre) 2000
8. Which one of the following is not a fossil fuel?
Ans. (c)
(a) Coal (b) Petroleum
See the explanation of the above question. (c) Natural gas (d) Uranium
4. Conventional source of energy is M.P.P.C.S. (Pre) 2016
(a) Wind energy (b) Solar energy Ans. (d)
(c) Coal (d) Nuclear energy Fossil fuels are non-renewable sources of energy. Fossil fuel
Jharkhand P.C.S. (Mains) 2016 was formed by natural processes through decomposition of
Ans. (*) the buried organism. Examples of fossil fuel include oil,
Coal, Petroleum, Natural gas and Atomic or Nuclear energy coal and natural gas. Fossil fuels are the main sources of
are conventional source of energy. Whereas Solar, Wind, energy presently.
Hydroelectric etc. are non-conventional source of energy. 9. An ultimate source of energy stored in fossil fuels is :
5. Among the following energy sources which one is used (a) Earth (b) Sun
maximum in organic form? (c) Sea (d) Moon
(a) Nuclear Energy (b) Solar Energy U.P.R.O./A.R.O. (Re-Exam) (Pre) 2016
(c) Geo-Thermal Energy (d) Tidal Energy Ans. (b)

F–182 General Studies Environment and Ecology


Fossil fuels were formed from the dead remains of living U.P.P.C.S. (Mains) 2015
organisms. An ultimate source of energy stored in fossil Ans. (a)
fuels is Sun. See the explanation of the above question.
10. Which form of energy does not create environment 14. What is meant by energy crisis?
issues? (a) Shortage of hydroelectricity.
(a) Coal (b) Nuclear (b) Malnutrition leading to a short supply of energy in the
(c) Petrol (d) Solar body.
U.P. Lower Sub. (Pre) 2004 (c) Shortage of thermal power.
Ans. (d) (d) The danger of the extinction of fossil fuel like coal
Solar energy is a unique source of energy which is pollution and petrol.
free and does not give rise to environmental issues. Uttarakhand P.C.S. (Pre) 2006
11. Which one of the following fuels creates the least Ans. (d)
pollution? The danger of the extinction of fossil fuels like coal and crude
(a) Diesel (b) Coal oil is called energy crises.
(c) Hydrogen (d) Kerosene
15. Which one of the following is the best source of
U.P.P.C.S. (Pre) 2017
generating electricity in India from the viewpoint of
U.P. Lower Sub. (Pre) 2015
sustainable development?
U.P. Lower Sub. (Pre) 2013
(a) Coal
U.P.U.D.A./L.D.A. (Spl.) (Pre) 2010 (b) Mineral oil and gas
Ans. (c) (c) Hydro- electricity
Diesel, Coal and Kerosene, when comes in contact with (d) Atomic energy
atmospheric oxygen, produce carbon dioxide, a greenhouse U.P.P.C.S.(Pre) 2012
gas which is a major factor for pollution. On the other hand, Ans. (c)
hydrogen, when comes in contact with oxygen, forms water
Sustainable development is economic development that is
and produces a huge amount of energy. Thus, it is a highly
conducted without the depletion of natural resources. Coal,
efficient fuel with minimum pollution. Notably, India has set a
Mineral Oil, gas and atomic energy are natural resources
target to generate 1000 MW of electricity through hydrogen-
which are limited. Hydroelectricity uses water to produce
based technology.
electricity without depleting water. Hence, Hydro-electricity
12. The second most important source after fossil fuels is the best source to generate electricity for sustainable
contributing to Indian energy need is - development.
(a) Hydropower energy 16. Which one of the following sources of renewable energy
(b) Wind energy has the greatest potential in India?
(c) Nuclear energy (a) Solar power
(d) Solar energy (b) Biomass power
U.P.R.O./A.R.O. (Pre) 2021 (c) Small hydel power
Ans. (d) (d) Energy recovery from waste
The second most important source after fossil fuels U.P.P.C.S (Mains) 2011
contributing to Indian energy need is solar energy. The solar Ans. (a)
energy share in total non-fossil fuel is 12.5 per cent. Hence The solar power source of renewable energy has the greatest
option(d) is correct. potential in India. The role of new and renewable energy has
13. Which of the following fuels produces least been assuming increasing significance in recent times with
environmental pollution? the growing concern for the country’s energy security. Solar
(a) Hydrogen panels are the most eco-friendly power generation systems.
(b) Power alcohol They are also called as the fuel of the future. This is why
(c) Petrol the Government has set a target of 100 GW Solar power
(d) Compressed Natural Gas (CNG) generation by 2022 under Jawahar Lal Nehru Solar Mission

Environment and Ecology General Studies F–183


started on 11 January, 2010. It is to be noted that world's 66th BPSC Re-Exam 2020
largest solar plant at a single location has been established Ans. (d)
in Tamil Nadu. The use of diesel and petrol as fuels in automobiles is being
17. Which of the following statements about bio-fuel is not replaced by CNG (Compressed Natural Gas), because CNG
correct? produces the harmful products in very small amounts. The
(a) Bio-fuel is eco-friendly. major component of CNG is Methane (CH4).
(b) Bio-fuel is cost-effective.
22. Which is the first Indian airline company that used
(c) Bio-fuel can contribute to remedy energy crisis.
biofuel as ATF in August 2018?
(d) Bio-fuel is also made from corn.
(a) Jet Airways (b) Vistara
U.P.P.C.S. (Mains) 2014
(c) Air India (d) Spicejet
Ans. (b)
(e) None of the above/More than one of the above
Even with all the benefits associated with biofuels, they 64th B.P.S.C. (Pre) 2018
are quite expensive in the market. Hence, they are not cost- Ans. (d)
effective.
On August 27, 2018, Spicejet, successfully conducted India's
18. Which one of the following is a bio-diesel crop? first ever Bio Fuel powered flight. The airline successfully
(a) Cotton (b) Sugarcane undertook the flight on the Dehradun-Delhi route. Spicejet
(c) Jatropha (d) Potato used its Bombardier Q400 aircraft for this flight.
U.P.P.C.S. (Pre) 2015
23. Under the Nuclear Power Projects, Environmental
Ans. (c)
Effects that have to be studied and resolved are
Biofuel development in India centres mainly around the (a) Radioactive contamination of air, soil and water.
cultivation and processing of Jatropha plant seeds which (b) Deforestation and loss of flora and fauna.
are very rich in oil (40%). Jatropha oil can be used after (c) Radioactive waste disposal.
extraction in diesel generators and engines. Jatropha oil has (d) All of the above.
been used in India for several decades as biodiesel for the U.P. Lower Sub. (Pre) 2013
fuel requirements of the remote rural and forest communities.
Ans. (d)
19. Which of the following is the source of ‘Green Fuel’?
During the establishment of nuclear power projects,
(a) Pine (b) Karanja
radioactive radiation and radioactive waste disposal are to
(c) Fern (d) None of the above
be taken into consideration. Radioactive radiation have a
U.P.U.D.A./L.D.A. (Spl) (Pre) 2010
disastrous effect on flora and fauna.
Ans. (*)
24. The first summit of 'International Solar Alliance' was
Ethanol also known as ethyl alcohol is used as a green fuel.
held in which of the following cities?
Ethanol is mainly formed from sugar cane, glucose etc.
(a) New Delhi (b) New York
Ethanol is also formed from the fermentation of Pine, Karanja
(c) Paris (d) None of the above
and Fern. These are also included in the green fuel sources.
U.P. R.O./A.R.O. (Pre) 2017
20. Given below are the names of four energy crops. Which Ans. (a)
one of them can be cultivated for ethanol?
(a) Jatropha (b) Maize The first International Solar Alliance (ISA) Summit was held
(c) Pongamia (d) Sunflower in New Delhi on 11 March, 2018.
I.A.S. (Pre) 2010 25. At which of the following places is the Secretariat of the
Ans. (b) International Solar Alliance (I.S.A.) located at present?
Maize is cultivated for ethanol while Jatropha, Pongamia and (a) Paris (b) New Delhi
Sunflower are cultivated for biodiesel. (c) Gurugram (d) New York
U.P.P.C.S. (Pre) 2019
21. The major component of CNG is: Ans. (c)
(a) CO2 (b) N2
(c) H2 (d) CH4 The Secretariat of the International Solar Alliance (I.S.A.) is
(e) None of the above/More than one of the above located in Gurugram, Haryana.

F–184 General Studies Environment and Ecology


26. The International Solar Alliance (ISA) has recently 30. The First Union Territory of India to run 100 percent
signed a pact with which international organization to on solar energy is
promote solar energy deployment in Asia and Pacific? (a) Andaman-Nicobar
(a) Asian Infrastructure Investment Bank (b) Chandigarh
(b) New Development Bank (c) Diu
(c) African Development Bank (d) Puducherry
(d) Asian Development Bank U.P.P.C.S. (Pre) 2019
(e) None of the above/More than one of the above Ans. (c)
63rd B.P.S.C. (Pre) 2017 Diu is the first Union Territory of India to run 100 percent
Ans. (d) on solar energy. In the Union Territory, solar plants had been
In March 2018, the International Solar Alliance (ISA) has installed over more than 50 acres of land in 2018.
signed a pact with the Asian Development Bank (ADB) to 31. Which of the following smart cities has become the
promote solar energy deployment in Asia and the Pacific. India's first city to run on 100% renewable energy
27. Incinerators are used for which of the following? during daytime?
(a) Burning wastes (a) Bengaluru
(b) Putting wastes into them (b) Jaipur
(c) For cutting green trees (c) Indore
(d) For making fertilizers (d) Diu
U.P. P.C.S. (Pre) 2018 (e) None of the above/More than one of the above
Ans. (a) 63rd B.P.S.C. (Pre) 2017
Ans. (d)
Incineration is a waste treatment process that involves
the combustion of organic substances contained in waste Diu Smart City has become the first city in India, that runs
materials. Incinerators are used in this process. on 100 percent renewable energy during daytime.
28. Which district has become the India's first district to
have 100% solar-powered health centres?
(a) Kolkata (b) Chennai
High-Level Questions
(c) Bengaluru (d) Surat 1. Consider the following statements:
(e) None of the above/More than one of the above 1. The International Solar Alliance was launched at
63rd B.P.S.C. (Pre) 2017 the United Nations Climate Change Conference in
Ans. (d) 2015.
2. The Alliance includes all the member countries of
Surat district of the Gujrat, becomes the first district in the
the United Nations.
country to have 100 percent solar powered Primary Health
Centres (PHC). As on 26th March, 2018, all 52 PHCs in the Which of the statements given above is/are correct?
district are powered by a solar system. (a) 1 only
(b) 2 only
29. The first Union Territory of India to run 100 percent (c) Both 1 and 2
on solar energy is- (d) Neither 1 nor 2
(a) Chandigarh I.A.S. (Pre) 2016
(b) Diu Ans. (a)
(c) Andaman-Nicobar
The International Solar Alliance is a common platform for
(d) Puducherry
co-operation among sun-rich countries lying fully or partially
U.P. R.O./A.R.O. (Pre) 2017
between the Tropic of Cancer and Capricorn who are seeking
Ans. (b)
to massively ramp up solar energy thereby helping to bend
The Union Territory of Diu became India's first 100% the global greenhouse emission curve while providing clean
renewable State in March, 2018. It is reportedly generating and cheap energy. The initiative was launched at the UN
up to 13 MW of solar energy every day which is more than Climate Change Conference in Paris at the end of 2015 by
its daily electricity demand. the President of France and the Prime Minister of India. Its

Environment and Ecology General Studies F–185


secretariat is based in Gurugram, Haryana, India. According Earthworms are a part of worm composting and thus they
to ISA declaration, it's members have set a target of raising speed up the process of decomposition. Worm composting
USD 1 Trillion for solar energy by 2030. Hence statement 1 is is using worms to recycle food scraps and other organic
correct. The alliance includes about 102 countries that support material into a valuable soil amendment called vermicompost
a common declaration. Hence, statement 2 is not correct. or worm compost.

2. Biomass gasification is considered to be one of the 4. Which of the following renewable power resources had
highest installed capacity in the country in 2014?
sustainable solutions to the power crisis in India. In
(a) Biomass power (b) Solar power
this context, which of the following statement(s) is/are
(c) Waste to energy (d) Wind power
correct?
Chhattisgarh P.C.S. (Pre) 2015
1. Coconut shells, groundnut shells and rice husk can
Ans. (d)
be used in biomass gasification.
2. The combustible gases generated from biomass As of 28th February 2021, solar, wind, biomass and small
hydropower (Renewable Energy Sources) contribute about
gasification consist of hydrogen and carbon dioxide
24.52 percent of the total installed capacity for electricity. The
only. total installed capacity touched the figure of 379130.41 MW
3. The combustible gases generated from biomass with wind power contributing 38789.15 MW, Solar energy
gasification can be used for direct heat generation 39083.71 MW, Bioenergy 10314.56 MW and small hydro
energy 4783.06 MW. As on 28th February 2021, renewable
but not in internal combustion engines.
energy sources constitute 24.52 percent of the total energy
Select the correct answer using codes given below: generation capacity.
(a) Only 1
5. Concerning ‘fuel cells’ in which hydrogen-rich fuel
(b) 2 and 3
and oxygen are used to generate electricity. Consider
(c) 1 and 3
the following statements:
(d) 1, 2 and 3
1. If pure hydrogen is used as a fuel, the fuel cell emits
I.A.S. (Pre) 2012
heat and water as a by-product.
Ans. (a)
2. Fuel cells can be used for powering buildings and
Biomass gasification means incomplete combustion of not for small devices like laptop computers.
biomass resulting in the production of combustible gases 3. Fuel cells produce electricity in the form of
consisting of carbon monoxide, hydrogen and traces of
Alternating Current (AC).
methane. Hence, statement (2) is not correct. The resulting
Which of the statements given above is/are correct :
gas from the gasification process is called producer gas
(a) 1 Only (b) 2 and 3 only
which is itself a fuel and can be used in internal combustion
engines. Hence, statement (3) is also incorrect. Coconut (c) 1 and 3 (d) 1, 2 and 3
shells, groundnut shells and rice husk can be used in biomass I.A.S. (Pre) 2015
gasification. Hence statement (1) is correct. Ans. (a)
3. Which of the following statement is incorrect in A fuel cell is an electrochemical cell that converts the
reference to compost? chemical energy from a fuel into electricity through an
(a) In this, biodegradable waste is decomposed in pits. electrochemical reaction of hydrogen with oxygen. In this
process, fuel cell produces electricity and as by-product,
(b) It is rich in organic matter and nutrients.
water and heat. Hence statement (1) is correct. Stationary
(c) The process of decomposition in pits is slowed down fuel cells are used for commercial, industrial and residential
by earthworms. purposes and backup power generation. Hence statement
(d) The waste material decomposed is farm waste material (2) is not correct. Fuel cells generate Direct Current (DC).
like livestock excreta, vegetable waste etc. Hence, statement (3) is incorrect.
R.A.S./R.T.S. (Pre) 2018 6. Assertion (A) : Coal-based thermal power stations
Ans. (c) contribute to acid rain.

F–186 General Studies Environment and Ecology


Reason (R) : Oxides of carbon are emitted when coal Select the correct answer using codes given below:
burns. (a) 1 and 2 only (b) 2 and 3 only
(a) Both (A) and (R) are true and (R) is the correct (c) 3 only (d) 1, 2 and 3
explanation of (A). I.A.S. (Pre) 2017
(b) Both (A) and (R) are true, but (R) is not the correct Ans. (b)
explanation of (A). The production of algae-based biofuel is possible in both
(c) (A) is false, but (R) is true. seas and continents. Thus, statement (1) is incorrect. The
(d) (A) is true, but (R) is false. high level of expertise is required for the production and
I.A.S. (Pre) 2003 engineering of these fuels, as well as knowledge and
Ans. (b) experience are required for higher production and innovation.
Sulphur present in coal produces sulphur dioxide which It is difficult to produce such type of fuel in developing
is mainly responsible for acid rain. Oxides of carbon are countries because it requires heavy capital investment, apart
emitted when coal burns. Hence both (A) and (R) are true from this, establishing facilities for large scale production,
but (R) is not the correct explanation of (A). Hence, (b) is ecology and society can be adversely affected.
the correct answer. 9. Consider the following statements:
7. Microbial fuel cells are considered a source of At the present level of technology available in India,
sustainable energy. Why? solar energy can be conveniently used to
1. They use living organisms as catalysts to generate 1. Supply hot water to residential buildings.
electricity from certain substrates. 2. Supply water for minor irrigation projects.
2. They use a variety of inorganic materials as 3. Provide street lighting.
substrates. 4. Electrify a cluster of villages and small towns.
3. They can be installed in wastewater treatment Of these statements:
plants to cleanse water and produce electricity. (a) 1, 2, 3 and 4 are correct
Which of the statements given above is/are correct? (b) 2 and 4 are correct
(a) 1 only (b) 2 and 3 (c) 1 and 3 are correct
(c) 1 and 3 (d) 1, 2 and 3 (d) 1, 2 and 3 are correct
I.A.S. (Pre) 2011 I.A.S. (Pre) 1996
Ans. (c) Ans. (d)

A Microbial Fuel Cell (MFC) is a bio-electrochemical device Sun is the greatest and limitless source of energy. Solar energy
that harnesses the power of respiring microbes to convert is a natural resource received through solar radiation. It can
organic substrates directly into electrical energy. Hence, be used in street lighting, minor irrigation project, supplying
statement 1 is correct and statement 2 is incorrect. The most hot water to residential buildings, railway signal and solar
immediately foreseeable application of MFC is in waste- cooker etc. Nevertheless, the use of solar energy to electrify
water treatment. Microbes love sewage and the conditions of small villages and towns is not cost effective with a view of
a wastewater treatment plant are ideal for the types of bacteria the technology constraints. The government, however, has
that can be used in MFC. Hence statement (3) is correct. started a few pilot projects for this.

8. It is possible to produce algae-based biofuels, but what 10. Other than Jatropha curcas, why is Pongamia pinnata
is/are the likely limitations of developing countries in also considered a good option for the production of
promoting this industry? bio-diesel in India?
1. Production of algae-based biofuels is possible in 1. Pongamia pinnata grows naturally in most of the
seas only and not on continents. arid regions of India.
2. Setting up and engineering the algae-based biofuel 2. The seeds of Pongamia pinnata are rich in lipid
production requires a high level of expertise/ content of which nearly half is oleic acid.
technology until the construction is completed. Which of the statements given above are/is correct?
3. Economically viable production necessitates the (a) 1 only (b) 2 only
setting up of large scale facilities which may raise (c) Both 1 and 2 (d) Neither 1 nor 2
ecological and social concerns. I.A.S. (Pre) 2010

Environment and Ecology General Studies F–187


Ans. (c) manufacture of Silicon Wafers used in photovoltaic
Other than Jatropha, Pongamia plant is considered as a units.
resource that can meet the growing demand of biodiesel in 2. The solar power tariffs are determined by the Solar
India due to their high productivity and less maturity cycle. Energy Corporation of India.
Pongamia pinnata growes naturally in most of the arid regions Which of the statements given above is/are correct?
in India. It can be grown in waterlogged, saline and alkaline (a) 1 only
soil and can withstand harsh agro-climates. The seeds of (b) 2 only
Pongamia pinnata are rich in liquid content of which nearly (c) Both 1 and 2
half is oleic acid. Hence, both the statements are correct. (d) Neither 1 nor 2
11. In which of the following areas of India geothermal I.A.S. (Pre) 2018
energy sources have not been found? Ans. (d)
(a) Godavari Delta (b) Ganga Delta
Silicon chip is not manufactured in India. Every solar panel
(c) Himalayas (d) West Coast which is made in India is only assembled here while all the
U.P.P.C.S. (Mains) 2016 material comes from China, Europe and some other countries.
Ans. (b) India is yet to develop semiconductor clusters. Central
Geothermal energy is the natural heat of the earth. Earth’s Electricity Regulatory Commission (CERC) determines solar
interior heat originated from its fiery consolidation of dust power tariffs and regulates the tariffs of generating companies
and gas over 4 billion years ago. It is continually regenerated owned or controlled by the central government.
by the decay of radioactive elements that occurs in all rocks. 14. In the Union Budget, 2011-12, a full exemption from
There are seven geothermal provinces in India namely the basic customs duty was extended to the bio-based
Himalayas, Sohana, West Coast, Cambay, Son-Narmada-Tapi asphalt (bio-asphalt). What is the importance of this
(SONATA), Godavari and Mahanadi. material?
12. Match List -I with List -II and select the correct 1. Unlike traditional asphalt, bio-asphalt is not based
answer using codes given below lists. on fossil fuels.
List– I List–II 2. Bio-asphalt can be made from non-renewable
A. Minerals 1. Rainfall resources.
B. Solar energy 2. Methane and Carbon 3. Bi-asphalt can be made from non-organic waste
dioxide materials.
C. Biogas 3. Exhaustible 4. It is eco-friendly to use bio-asphalt for surfacing
D. Deforestation 4. Inexhaustible of the roads.
Codes: Which of the statements given above is/are correct?
A B C D (a) 1, 2 and 3
(a) 3 4 2 1 (b) 1, 3 and 4
(b) 3 4 1 2 (c) 2 and 4
(c) 2 1 3 4 (d) 1, 2, 3, and 4
(d) 1 2 3 4 I.A.S. (Pre) 2011
U.P.P.C.S. (Pre) 2015 Ans. (b)
Ans. (a) Conventional asphalt is petroleum based and contains toxic
Minerals are exhaustible which means if we extract this and heavy metals. Today's green revolution has brought
resource, one day it will run out while solar energy is about an interesting and more environmental friendly
inexhaustible. Methane and carbon dioxides are Biogas and alternative known as 'Bio Asphalt'. Bio Asphalt is a renewable
deforestation would affect rainfall. Hence, (a) is the correct construction material that is manufactured without the use
answer. of petroleum. Employing everything from potato starches,
corn, tree resins, natural rubber, sugar and vegetable oils,
13. With reference to solar power production in India, Bio Asphalt finds application in roofing and street paving. In
consider the following statements : 2007, two streets of Norway were paved using Bio Asphalt.
1. India is the third largest in the world in the Hence, option (b) is the correct answer.

F–188 General Studies Environment and Ecology


Pollution
Pollution and its types (3) Land Pollution
*When due to man-made or natural reasons the gases present
*Pollution occurs as a result of unexpected change in physical,
in the atmosphere in a definite proportion change an into
chemical and biological characteristics of soil, water and air.
undesirable proportion (due to poisonous gases or due to
It occurs due to disproportionate accumulation of pollutants.
particulate matter) then it is known as Air pollution.
Pollution is of different kinds. These can be categorised as –
*There are two sources of air pollution :
(1) Natural Sources
(2) Man-made Sources
*Natural sources comprise of forest fires and volcanic
eruptions, the gases like sulphur dioxide (SO2) nitrogen oxides
or oxides of nitrogen (NOx) generated due to degradation of
biological wastes, etc.
*Man-made sources include gases, smoke, particulate matter
and heat (produced from the combustion of fuel) etc.
*It is noteworthy that on combustion of wood, coal, diesel,
petrol etc. different types of gases and smoke are generated.
*Primarily carbon and nitrogen oxides are generated in the air
which are main sources of air pollution.
*Rain and dew clean atmosphere polluted due to smoke, dust
and gaseous waste.
*But the pollutants of particle diameter less than 2.5 μ, cannot
be cleaned through the rain by the atmosphere. *Photochemical
smog is generated by the combination of smoke and fog.
*Photochemical smog is a state of air pollution.
*The smoke present in the waste generated from automobiles
*Pollution caused by humans is called Anthropogenic and industries contain nitrogen oxide and nitrogen dioxide
pollution. Anthropogenic pollution mainly gives rise to non- gases, which react with hydrocarbon in the presence of
biodegradable pollutants which usually not get decomposed by sunlight to produce many secondary pollutants, like ozone,
organic decomposers or decompose very slowly. formaldehyde and PAN (Peroxy Acetyl Nitrate).
*Those pollutants which can be broken down into simpler, *Photochemical smog is a secondary air pollutant and
harmless substances in nature in the due course of time (by generally formed in traffic congested metropolitan cities where
the action of micro-organisms like certain bacteria) are called warm conditions and intense solar radiation are present.
Bio- degradable pollutants. Domestic waste (garbage), faecal *Photochemical smog is especially formed by the reaction of
matter, sewage, cattle dung, animal bones, etc are biodegradable Nitrogen oxides (NOx), Ozone (O3) and Peroxyacetyl nitrate.
pollutants. *It is noteworthy that automobile exhausts contain
hydrocarbon and NO and it plays an important role in the
Air Pollution
formation of ozone (O3) and PAN (peroxyacetyl nitrate) in
On the basis of the part of the Environment, Pollution can urban areas.
primarily be catageorised into following three main types: *Photochemical reactions involved in smog formation are as
(1) Air Pollution follows :
(2) Water Pollution Reactions inside an engine of the automobile :

Environment and Ecology General Studies F–189


N2 + O2 o 2NO sulphur dioxide), oxides of Nitrogen, carbon monoxide are
Reactions that occur in the atmosphere primary pollutants.
2NO + O2 o 2NO2 *Burning of petrol in vehicles releases lead, which pollutes
NO2 NO + O the air.
O + O 2 o O3 *Lead is used as an anti-knocking agent in engines. Lead
NO + O3 o NO2 + O2 is an air pollutant that leads to serious health disorders. Lead
HC + NO + O2 o NO2 + PAN hinders development of the brain in children, reduces their
*This causes irritation in eye and nose and generate throat and IQ (Intelligence Quotient) and is responsible for heart and
respiratory problems in human beings. respiratory diseases in adults.
*These processes are known as photochemical because *The colourless gas released from the motor car and incomplete
sunlight and chemical pollutants both are present in them. combustion of cigaratte is carbon monoxide. Carbon monoxide
*The brown-orange film formed due to air pollution is known (CO) is a primary air pollutant. It combines with haemoglobin
as photochemical smog. present in the blood to form carboxy-haemoglobin, due to
*It is noteworthy that oxygen and nitrogen combine to form which oxygen carrying capacity of blood reduces.
Nitric oxide (NO). *When approximately 50 percent haemoglobin transforms into
*This gas combines with air to form Nitrogen dioxide (NO2). carboxyhaemoglobin then due to a reduction in oxygen level,
*NO2 is a reddish brown coloured gas that has a sharp, harsh there is an obstruction in respiration which leads to death.
odor at higher concentrations. *Fly ash is a pollutant obtained from the combustion of coal.
*NO2 forms a single molecule of Nitric Oxide (NO) and Nascent *The fine powder produced from coal-based thermal power
Oxygen (O) by absorbing light present in the atmosphere. plant causes respiratory diseases in organisms.
*It is notable that Nascent Oxygen (O) is highly reactive. *It accumulates on leaves of plants and hinders photochemical
Nascent oxygen reacts with one molecule of oxygen to form reaction.
ozone (O3). Thereafter O3 (Ozone) through open stomata of *Smoke of a cigarette is a complex mixture of chemical
leaves enters into the cells of mesophyll tissue. substances.
*It reacts with water present in the cell to form harmful 'oxygen *The cigarette smoke contains gases like carbon monoxide,
free radicals' and peroxy radicals like O2–, OH–, O–, HO2– etc. hydrogen cyanide and nitrogen oxide as well as with unstable
*Peroxy radicals either react with oxygen atoms to form ozone chemical compounds like Formaldehyde, Acrolein and Benzene
or react with Nitrogen dioxide to form PAN (peroxyacetyl are also present in them.
nitrate). *Nicotine is also present in the cigarette smoke but it is present
*PAN destroys chloroplast leading to the loss of capacity of in very small amounts. It is extremely harmful and causes
photochemical reaction and development of plants. cancer. This causes birth to diseases concerning irritation in
*This hinders Electron Transport Chain (ETC) occurring in eye, nose, throat, and harmful diseases like cancer, bronchitis
mitochondria of the cell. and asthma.
*This affects enzyme system as well. In human eyes, PAN *Asian Brown Cloud was mainly spread over South Asia due
produces excessive irritation to pollution. A detailed report was published on this in 2007.
*PAN combines with air and oxygen to form smog. Visibility *Brown clouds are formed due to air pollution and spread over
decreases due to excessive smog formation. particularly South Asian region during January-March. Brown
*Those air pollutants which are injected in the atmosphere clouds consist of approximately 85% aerosol. It also contains
directly from pollutant sources are known as primary sulphate, black carbon and other deadly chemicals which cause
pollutants e.g. CO, CO2, SO2, etc. Those air pollutants, which respiratory problems and cancer.
are produced due to the reaction between primary air pollutants *Significant, change in solar radiation in India and China,
and common atmospheric substances are known as secondary uneven atmospheric temperature and change in monsoonal
pollutants. e.g. Ozone, peroxyacetyl nitrate etc. rainfall pattern are caused by brown clouds.
*Hence Peroxyacetyl Nitrate (PAN), Ozone and Smog are *Fly ash is used for the manufacturing of bricks. The most
secondary pollutants, whereas oxides of sulphur (primarily common use of fly ash is as a replacement for Portland cement

F–190 General Studies Environment and Ecology


used in producing concrete. due to weight. The gas free from particulate particles, released
through the upper part of cylinder enters into the atmosphere.
*Cyclone separator is used for prevention of this type of air
pollution.
*Notable that bacteria responsible for nitrification converts
ammonia present in the soil to nitrate.
*The plants absorb this through roots in the form of nitrite
s ions, nitrate ions etc. In return, plants provide these bacteria
food and shelter.
*In this way the effect of mutualism is seen on plants and the
atmosphere around them.

*Bio-indicators are living organisms that respond in an


specially clear way to a change in the environment. The lichens
*SiO2, Al2O3 and occasionally CaO are the main chemical are useful bioindicators for air pollution, especially sulphur
components present in fly ashes. In addition to this, it also dioxide pollution since they derive their water and essential
contains some toxic elements like lead, arsenic and copper nutrients mainly from the atmosphere rather than from the soil.
particles.
*It is noteworthy that many types of mechanisms are used to *Lichens are sensitive to air pollution due to which they are
prevent air pollution due to gaseous and particulate particles. not found in the nearby areas of cities.
*The special filter, know as bag filter, is used to separate *In a report released on 2 nd May, 2018, World Health
particulate particles mixed with smoke and released from of Organisation (WHO) highlighted that eight of ten most
factories. Bag filter is used to separate particulate substances pollutant cities were in India and four of them (Kanpur,
less than 50 micrometre diameter. Varanasi, Lucknow and Agra) were in Uttar Pradesh. Kanpur
*Whereas to filter particulate particles greater than 50 was named the world's most polluted city in WHO's list.
micrometre some equipment named a Cyclone separator or
*It is noteworthy that in February, 2020, I.Q. Airvisual
Cyclone collector and Wet Scrubber are used.
published 'World Air Quality Report, 2019'. According to
*Cyclone divider is not used to check air pollution.
the report Ghaziabad is the most polluted city of the World.
*India's Air Quality Index is based on 8 major pollutants-
(a) < 10 micro meter size particulate matter (PM10)
(b) < 2.5 micro meter size particulate matter (PM)
(c) Nitrogen dioxide (NO2)
(d) Carbon Mono oxide (CO)
(e) Ozone (O3)
(f) Sulphur dioxide (SO2)
(g) Ammonia (NH3)
(h) Lead (Pb)
*CO and O3 are measured on an average of 8 hours and
other pollutants are measured on an average of 24 hours,
Measurement unit for NO2 and O3 is microgram per qubic metre
and for CO miligram per cubic metre.
*It is notable that carbon dioxide (CO2) and methane (CH4) are
used for the measurement of global warming.
*Smoke is churned in a cyclone separator through cyclone *It is noteworthy that six series of Air Quality Index is
or whirling. Eventually solid particulate particles settle down prescribed.

Environment and Ecology General Studies F–191


Index Series Colour Possible Health Hydrofluorocarbons.
*In addition to this, member countries of the alliance also
Shown Impacts recognizes the fact that minimization of short-lived climatic
0-50 Good Green Minimum Impact pollutants are capable of evaluating the pros and cons of global
efforts for minimization of carbon dioxide.
51-100 Satisfactory Light green Minor breathing *The equipment named Catalytic Converter is used for
discomfort to conversion of NOx, CO and unburnt hydrocarbons released
from vehicles, into non-harmful molecules.
sensitive people.
*In this released gas is flown over the catalyst (Platinum -
101-200 Moderately Yellow Breathing Palladium and rhodium) in which the gaseous mixture present
polluted discomfort to the converts to as CO o CO2, NOx o N2 and HC5 o CO2 + water
(H2O)
people with
2CO + O2 o 2CO2 + O2
asthma and heart
*Air pollution caused due to domestic activities is known as
diseases. Domestic air pollution.
*Most significant pollutant of indoor air pollution is Radon gas.
201-300 Poor Orange Breathing *Radon is a colourless, odourless radioactive noble gas. *Radon
discomfort to gas is released from Radium.
*Due to the presence of radon gas there is a probability of lung
most people on cancer and blood cancer in human beings.
prolonged *It is noteworthy that radon gas releases naturally from soil.
exposure. *In cities due to a reduction in ventilation this gas accumulates
in houses to give birth to diseases like lung cancer.
301-400 Very Poor Red Respiratory *According to WHO, every year lakhs of people die due to
illness on domestic air pollution.

prolonged
'Acid Rain'
exposure *Acid rain is caused by a chemical reaction that begins when
401-500 Severe Dark Red/ Affects healthy a compound like sulphur dioxide and nitrogen oxides are
released into the air. These substances can rise very high into
Maroon people and
the atmosphere where they mix and react with water, oxygen
seriously and other chemical to form more acidic pollutants. When these
impacts those pollutants fall on the surface with rain, it is termed acid rain.
*Normally the pH value of acid rain is less than 5.6. Notably,
with existing the pH value of pure water is 7.0.
diseases. *Due to this reason, acid gets accumulated on earth. It happens
in two forms.
*It is noteworthy that to prevent increasing air pollution in
cities, Ministry of Environment and Forests on 17th October,
2014 released the National Air Quality Index: NAQI.
*This index indicates the level of air pollution in the urban cities
by adopting (One Number - One Colour - One Description).
*It is notable that this was started under ‘Swachh Bharat
Abhiyaan’.
*It is notable that (CCAC) Climate and Clean Air Coalition is
a global effort of different countries, civil societies and private
groups, which improve the quality of atmosphere by minimising
short-lived climatic pollutants.
*This is an alliance formed by 53 countries and different
organizations.
*CCAC focuses primarily on Methane, Black Carbon and

F–192 General Studies Environment and Ecology


*It is noteworthy that approximately half of the acid gets and in nearby Agra from 9.1 (1963) to 6.3 (1984).
transferred from atmosphere to earth and gets accumulated in *Acid rain also occurs in Canada and other Scandinavian
dry form. Countries which is extremely harmful there.
*It is noteworthy that the Acid Rain Information Centre
(ARIC) is situated in Manchester, England.

Acidic Gas Natural Sources Man made


Sources
N2 and their Volcanic eruption, Oil, Coal and
oxides. biological decay and combustion of
lightning strikes natural gas, Forest
Fire
Sulphur and Volcanic eruption Combustion of
their oxides Sea, fires, phyto- coal, petroleum
plankton products, by
refining of crude oil
and by Chemical
and Fertilizer
Industry.
Carbon Respiration, Deforestation,
*SO2 gas is produced from different sources and after absorbing
dioxide Decomposing through Industrial
moisture forms Sulphuric Acid (H2SO4).
*SO2 combines with rainwater causing acid rain. Because of vegetation, Processes, by
this layer and layers of red powder get corroded from bricks outgassing from combustion of
of buildings. oceans Fossil Fuel.
*It is noteworthy that SO2 is also known as Cracking gas
because if it is continuously flown on a stone, the stone gets
mangled.
*SO2 gas excreted from Mathura Oil Refinery is responsible
for the corrosion of Taj Mahal of Agra. Acidic raindrops after
coming in contact with the soil on the earth causes irreparable
damage.
*Due to excessive acidity there is an exchange of hydrogen-ion
and positive cations (k+ and Mg++) nurturing the soil. *Because
of these reasons nurturing elements of soil gets leached and the
fertility of soil degrades.
*Norway is the most affected country from Acid Rain. Its half
of the southern part is affected from acid rain because most of
the lakes and river water become acidic. Lakes are becoming
biologically dead. Because of these reasons Acid rain is known
as Lake Killer. The BAPMON (Background Air Pollution
Monitoring Network), data collected during 1974-1984 shows
Water Pollution
that a few areas are already under stress conditions. During two *Pollution in water is an extensive problem. Various types of
decades, the acidic content of rain in Delhi increased, which pollutants cause pollution in water. Even the excess of nutrients
means its pH level decreased from 7.0 (1965) to 6.1 (1984), in the water also influences the organism.

Environment and Ecology General Studies F–193


*When both organic and inorganic nutrients become excess in *These are identified as pollutant species of polluted water.
water then this phenomenon is called Eutrophication. *It is noteworthy that when the value of DO of any reservoir
*Eutrophication leads to algal bloom. In this process on the goes below 8.0 mgL–1 then that water is called contaminated
surface of water body gets covered by blue-green algae along water. When this level goes below 4.0 mgL–1 then that water
with a number of plants.
is called highly polluted water.
*Algae and other aquatic plants use most of the oxygen of water
*The highest level of BOD in the Ganga river is found between
for the respiration process.
Kanpur and Allahabad. In fact, because of pouring down the
*Along with this they produce toxin in the reservoir. Due to lack
of O2 aquatic organisms begin to die in the reservoir. highest amount of pollutants in the Ganga river from Kanpur,
*Therefore, due to the excessive increase in nutrition, the the level of BOD becomes highest there.
destruction of bio-diversity occurs. Hence, eutrophication *Abiotic pollutant Arsenic pollutes the underground water.
decreases water quality and reduces the oxygen level leading Arsenic in the state of the compound, is found at many places on
to water pollution. the earth. It is in the mixed form with the vapours of volcanoes,
oceanic water and other mineral water.
*In India underground water at many places is contaminated
with Arsenic.
*The continuous contact with Arsenic leads to in Black Foot
disease.
aquatic *It is remarkable that at the coastal areas of Ganga river, the
highest level of Arsenic is found in the underground water.
*It is noteworthy that Arsenic also pervades in the ecosystem
by the spray of chemical pesticides and weedicides.
*According to the standards of the World Health Organization,
the level of Arsenic should be 0.01 mg/litre. Unfortunately
the absorbability of Arsenic in paddy plants is much more. It
reaches to grains through ground water. By this, the whole food
chain is getting affected.
*Increasing the quantity of nitrogen and phosphorus in water
by unnatural means is called the eutrophication. *Underground water in the plains of Ganga-Brahmaputra in
*BOD: Biological Oxygen Demand is the main unit of India and plains of Padma-Meghna of Bangladesh is mostly
measurement of water pollution. affected by Arsenic pollution.
*The greater the B.O.D. the greater is the infection level of *Underground water in seven states of India i.e. West Bengal,
water. Jharkhand, Bihar, Uttar Pradesh, Assam, Manipur and
*It is remarkable that the rate of decomposition increases as Rajnandgaon of Chhattisgarh has been extremely polluted
the amount of organic waste (i.e. Sewage) increases and use with Arsenic.
of O2 increases as well. As a result, the level of dissolved *In the year 1978, the first case of water pollution was reported
oxygen (DO) also decreases. Therefore, the demand for O2 by Arsenic in West Bengal in India.
is directly related to the amount of increased waste. This *Damodar river has become the most polluted river of the
demand is called Biological Oxygen Demand (B.O.D.). country as it flows through the coal mines and industrial areas.
*Therefore, the greaer, the B.O.D., the lower will be D.O. It has become 'Biological Desert' in 300 km. long way between
Giridih and Durgapur.
B.O.D. α (directly proportional)
*The most chemical pollution from industrial debris occurs
*It is remarkable that just a few tolerant species of organisms from the leather industry. This industry is mainly responsible
like Annelida and eggs of some other insects may survive in for water pollution and soil pollution.
highly polluted and less DO water. *Quality of drinking water in Manipur, Tripura, Odisha,

F–194 General Studies Environment and Ecology


Meghalaya, Jharkhand and Madhya Pradesh is highly *Excess of nitrogen is responsible for fast growth of Algae.
worrisome. *It is notable that in the year 1948 in an International meet
*85 percent rural population of India is dependant on ground- in Geneva, International Sea Organization was constituted
water. Remaining 15 percent population gets potable water formally, the original name of which was Inter-governmental
from open wells or open sources of water. Maritime Consultative Organization – IMCO. *But in the
*Deadly chemicals like Arsenic, Fluoride and Uranium have year 1982 organization's name was converted to International
been found in groundwater. Maritime Organization - IMO.
*Normal voice generated from an object is known as Sound.
"Knock-knee syndrome" is produced by the pollution of
*When the intensity of sound increases, then it is known as
Fluoride. Although fluoride element is available in small
noise.
amounts in water, which provides protection to gums and
*The unwanted sound is responsible for creating unrest and
teeth, but its excessive concentration causes a disease called
discomfort in human beings.
Fluorosis. The intake of excessive Fluoride increases the
*Unit of sound is Decibel (dB).
likelihood of 'Humped back'. Its continuous consumption
*This name was given with a vision for praising works of
leads to weakness in the joint of bones, especially in the spinal
Alexander Graham Bell.
cord.
*Unplanned Industrial development, use of excessive motor
*High Fluoride storage is the only cause of bending of the feet,
vehicles and by operating different types of vehicles contributes
which is called 'Knock-knee' syndrome.
significantly to noise pollution.
Sea Pollution *During take-off and landing of Jet Aircraft excessive noise
pollution is produced.
*Increasing sea pollution is a worrisome subject.
*It is noteworthy that sea acts as a storehouse by absorbing Source of Sound Sound (In Decibel)
one-third of CO2 generated from man-made activities. Whispering sound of 20 dB
*Ocean acidification is the ongoing decrease in the PH value leaves
of the earth’s oceans, caused by the intake of carbon dioxide Murmuring 30 dB
from the atmosphere.
Room/Peaceful sound
*The acidification of seawater would adversely affect the
of a workplace 40 dB
growth and survival of calcareous phytoplankton, the growth
Sound generated during
and survival of coral reefs and survival of some animals that
have phytoplankton larvae. normal conversation. 60 dB
Noise due to trucks. 80-85 dB
Noise of Jet Engine 120 dB
Landing of Jet Plane 150 dB
Rocket Engine 180 dB

*It is noteworthy that noise produced from Jet Aircraft moving


faster than the speed of sound is known as Sonic boom.
bleaching *Sonic Boom can be expressed in Mach Unit. *Notable that
is affected those objects which move faster than sound, the noise produced
from them is known as Mach-1. *If this speed is double of
Sound's speed then it is known as Mach-2.
*Huge green plants are grown in excessive sound polluted
areas because they have the capacity to absorb sound waves.
The green muffler is a control measure for noise pollution by
planting green plants.

Environment and Ecology General Studies F–195


Soil Pollution *Plastic packets takes excessive time to decay.*Plastic bag is a
thermoplastic which is obtained by polymerisation of ethylene.
*Excessive use of fertilizers produce different types of pollution *It is a non-biodegradable pollutant, which pollutes atmosphere
which primarily includes soil pollution, water pollution and air
as well as soil.
pollution.*This pollution enters into the food chain of man and
animals through the medium of different types of crops and 'Radioactive and other Gaseous
affects them with different types of serious illnesses. Pollution'
*Inorganic materials like phosphate and nitrate dissolve in the *On April, 1986, a nuclear accident took place at a place named
water ecological system. Chernobyl in Ukraine. Due to the deadly radioactive effect
*This increases Eutrophication in hydroecological system. many people lost their lives and a large number of people got
*Nitrate pollutes the drinking water as well. injured.
*Fertilizer increases infertility and decreases water absorbing *It is noteworthy that radiations produced from disintegration
capacity of soil.*Some parts of them remain in the crops which of radioactive nuclides served as radioactive pollution.* This
acts as a slow poison for human beings. radiation is harmful to tissues and organs of organisms and
affects their genetic properties.

*Radioactive pollution causes hereditary changes in the


all living organism. It hinders blood circulation and causes
carcinogenesis.

lead

*Soil pollution is more dangerous in comparison to Industrial


Pollution.

'Plastic Pollution' *Bhopal disaster, also referred to as the Bhopal Gas Tragedy
was a gas leak incident in India, considered the world’s worst
*Primary constituents of polythene are atoms of carbon and
industrial disaster. It occurred on the night of 3 December, 1984
Hydrogen.
at the Union Carbide India Limited pesticide plant in Bhopal,
*It is a polymer of ethylene C2H4. *In polythene, molecules of
Madhya Pradesh.
ethylene are combined together in such a manner that they do
*Over 500,000 people were exposed to Methyl Isocyanate
not get biologically decayed. *Because of this reason plastic
gas. According to state government records, 3787 people died
packets thrown after use do not decompose and serve as a source
in this tragedy.
of environmental pollution.
*It was discovered in 1953 by Italy's Chemist Giulio Natta and 'Bio-Remediation'
Germany's Carl Zeigler. *They first saw that atoms of Carbon
*Bio-remediation is a process of detoxification of environment
and Hydrogen together form a series and combines together in
through the use of small organisms. This technique can help
the form of single bonds and double bonds.
to overcome adverse effects of pollution at particular places.
*For this discovery Giulio Natta and Carl Zeigler were awarded
Bio-remediation is used in cleaning surface and groundwater
the Nobel Prize for Chemistry in the year 1963. and soil, etc.

F–196 General Studies Environment and Ecology


*TERI has developed Oilzapper which is a consortium of crude
oil and oily sludge degrading bacteria deriving from various
bacterial cultures existing in the natural environment.
*In August, 2010 in portal areas of Mumbai, in an accident
between M.V. Khalijia and MSC Chitra named ships, this
technique was used for cleaning spilt oil.

Euro Standards
*European Emission Standards define the acceptable limits to
exhaust emission for new vehicles sold by the European Union
and its member states. At present nitrogen oxides (NOx), total
hydrocarbon (THC), carbon monoxide (CO) and particulate
matter (PM) related to emission are regulated under European
Emission Standards.
*There are various polluted gas emissions from motor vehicles.
To control these gases and particulate matters, European
Union laid down emission standards, which is known as Euro
Standards.
*It is notable that European countries have applied Euro
Standards I in the year 1992 and Euro Standards II in 1997.
*Euro-I Standards are more liberal than Euro-II Standards.
*The quantity of Sulphur should be less than or equal to 0.05%
in the ultra-low sulphur diesel to fulfil the Euro-II Standards.
*Dr. Mashelkar's Committee was constituted for "Auto fuel
policy" to control vehicle pollution. Its recommendation was to
implement euro standard in India in a phased manner in order
to control air pollutants.
*It is notable that the International Council on Clean
Transportation (ICCT) has given relaxation for India to adopt
Euro VI directly in the year 2020 instead of Euro-V.
*The process of expulsion of toxic pollutants from the *It is noteworthy that recently New Delhi Government has
environment by using micro-organism is called bio-remediation. implemented the odd-even formula to control pollution.
*It is a technique for cleaning up pollution by enhancing the *It is known that BS-IV Standards has been implemented in
same biodegradation process that occurs in nature. India since April 1, 2017. Emission of Nitrogen oxides (NOx)
*Pollutants containing many types of heavy metals cannot Total Hydro carbons (THC), Non-methane hydro carbon
be treated by the process of bio-remediation. Any pollutant (NMHC), carbon mono oxides (CO) and suspended particulate
containing heavy metals like cadmium and lead for citation matter are regulated for most vehicle types, including cars,
cannot be treated comfortably and completely by bio-
trucks (lorries), locomotives, tractors and similar machinery.
remediation using micro-organisms.
*Genetic engineering can be used to create microorganisms Pollution and Disease
specially designed for bio-remediation, for example, superbug
Pseudomonas putida which is created by genetic engineering *There are various types of diseases generated by pollution in
techniques. It plays an important role in ending the oil spill in human and plants both. One in eight deaths in India in 2017 was
the ocean. attributed to air pollution, which contributes to more disease
*Extensive research at The Energy and Research Institute burden than tobacco use.
(TERI) has resulted in the development of an easy and cost-ef- *Automobiles use lead as an anti knocking agent. Lead when
fective bio-remediation process. released in the air with exhaust are most toxic. Lead affects the

Environment and Ecology General Studies F–197


nervous system, brain and digestive system of the human body. entrance in the lungs andcausing various disease. While the
*Cadmium works as respiratory poison. It becomes the main particle size greater than 0.5-3 micron (P) are stopped by Nasal
factor of heart disease by increasing blood pressure. Excess membrane and could not enter the lungs.
of cadmium in drinking water leads to Itai Itai or ouch-ouch
disease. It leads to severe pain in bones and joints. Pollution (Miscellaneous)
*Drinking of contaminated water mixed with Mercury (Hg)
*Asbestosis is a chronic lung disease caused by inhaling
leads to Minamata disease. An amalgam of Mercury present in
asbestos fibres. Prolonged exposure to these fibres can cause
wastewater gets converted into toxic matter (methyl mercury).
lung tissue scarring and shortness of breath. It can cause
Symptoms include ataxia, numbness in the hands and feet
Mesothelioma cancer disease.
blurred eyes, general muscle weakness, narrowing of the field
*La-Nino represents periods of below-average sea surface
vision and damage to hearing and speech.
temperatures across the east-central equatorial pacific.
*Fluoride pollution is responsible for 'Knock-Knee-Syndrome'. *Mustard's seed can be adulterated by argemone seeds.
Fluoride has an affinity for calcium and gets accumulated in *Argemone mexicana is a species of poppy found in Mexico.
bones resulting pain in bones and joints and outward bending Adultaration in mustard oil can cause epidemic dropsy.
of legs from the knees.
*The air (prevention and control of pollution) Act, 1981 Basic Questions
provide for preventing control smoke and standards of dirtiness
1. Human-made pollutants are called –
coming out from motor vehicles and other factors. In 1987,
(a) Xenobiotics (b) Antibiotics
sound pollution was also included in this Act. *Monosodium
Glutamate is a sodium salt of Glutamic acid. It is used in (c) Humalins (d) Anthropogenic
increasing taste of Chinese fast food. U.P.P.C.S. (Pre) 2005
*Asbestos is poisonous material whose dust could cause lung Ans. (d)
cancer. Pollution occurs as a result of unexpected change in physical,
*Mercury affects the human stomach. chemical and biological characteristics of soil, water and
*Carbon mono oxide displaces the oxygen in the human air. It occurs due to the disproportionate accumulation of
bloodstream and lead to poisoning. pollutants.
*Nitrogen's oxides can produce cancer in the human body. Dust
Pollution caused by humans is called anthropogenic
particles cause breathing-related disease. pollution. Anthropogenic pollution gives rise to non-
*Carbon mono Oxide, Oxides of sulphur, Nitrogen Oxide and biodegradable pollutants which do not get decomposed
Carbon dioxide (CO2) all four major pollutants are from steel by organic decomposers or are decomposed very slowly.
industries in India. Therefore, it is very difficult to reign in non-biodegradable
*Aflatoxins are poisonous and cancer-causing chemicals pollutants caused by anthropogenic pollution.
produced by certain molds which grow in soil, decaying
2. The environment modified by human activities is called
vegetation and grain. They are regularly found in improperly
(a) natural environment
stored staple commodities such as cassava, chilli peppers, corn,
(b) Anthropogenic environment
cotton seed, millet, peanut, rice, wheat etc.
*Persistent Organic Pollutants (POPs) are organic (c) Urban environment
compounds that are resistant to environmental degradation (d) Modern environment
through chemical, biological and photolytic processes. U.P.P.C.S. (Pre) 2019
*Some brominated flame retardants are included in the list of Ans. (b)
persistent organic pollutants. The environment modified by human activities is called
*Nanoparticles pollute water, soil and air. 'Anthropogenic environment'.
*Nanoparticles of Zinc oxide and Titanium oxide produce free
3. Bio-degradable pollutant is
radicals, which can be harmful.
*Respirable micro particles are those particles which can enter (a) Mercury (b) Sewage
the lungs during respiration. The floating microparticles of less (c) Plastic (d) Asbestos
than 0.5P cannot be stopped by nasal membrane leading to their U.P.P.C.S. (Pre) 2014

F–198 General Studies Environment and Ecology


Ans. (b) Photochemical smog is a mixture of pollutants that are formed
Those pollutants which can be broken down into simpler, when nitrogen oxides and volatile organic compounds react
harmless substances in nature in the due course of time (by to sunlight, creating a brown haze above cities. It tends
the action of micro-organisms like certain bacteria) are called to occur more often in summer because that is when we
bio-degradable pollutants. Domestic waste (garbage), urine, have the most sunlight. Photochemical smog is formed by
faecal matter, sewage, cattle dung, animal bones, etc. are oxidesof nitrogen (NOx) and Ozone (O3). The common
biodegradable pollutants. components of photochemical smog are ozone, nitric oxide,
acrolein, formaldehyde and peroxyacetyl nitrate (PAN)
4. The combustion of coal, petrol and diesel etc. is the
8. Photochemical smog is a resultant of the reaction among.
basic source of –
(a) NO2, O3 and peroxyacetyl nitrate in the presence of
(a) Water pollution (b) Land pollution
sunlight
(c) Air pollution (d) Noise pollution
(b) CO, O2 and peroxyacetyl nitrate in the presence of
U.P.P.C.S. (Mains) 2011
sunlight
Ans. (c)
(c) CO, CO2 and NO2 at low temperature
Coal, petrol and diesel burn to produce oxides of carbon
(d) High concentration of NO2, O3 and CO in the evening
and nitrogen which are the main factor behind air pollution.
I.A.S. (Pre) 2013
5. Which of the following energy sources produces NO
Ans. (a)
atmospheric pollution?
Photochemical smog is a mixture of pollutants that are formed
(a) Nuclear energy (b) Solar energy when nitrogen oxides (NO2), ozone (O3) and peroxyacetyl
(c) Petroleum energy (d) Coal energy nitrate (PAN) reacts with sunlight, creating a brown haze
U.P. P.C.S. (Pre) 2018 above cities.
Ans. (b) 9. Smog is essentially caused by the atmospheric presence
Solar energy is an important source of renewable energy. of :
There is no atmospheric pollution during the conversion of (a) Oxygen and ozone
solar energy into electrical energy.
(b) Ozone and nitrogen
6. Which of the following is a biodegradable pollutant? (c) Oxygen and nitrogen
(a) Sewage (b) Asbestos (d) Oxide of nitrogen and sulphur
(c) Plastic (d) Polythene U.P.P.C.S. (Pre) 2019
U.P.P.C.S. (Mains) 2016 Ans. (d)
Ans. (a)
Smog is a type of intense air pollution. The word 'smog' is a
Those pollutants which can be broken down into simpler, contraction of the words smoke and fog to refer the smoky
harmless substances in nature in due course of time (by the fog. It is a kind of visible air pollution and is composed
action of micro-organism like certain bacteria) are called of nitrogen oxides, sulphur oxides, ozone, smoke and
biodegradable pollutants. Domestic wastes (garbage), urine, other particulates. Man-made smog is derived from coal
sewage, agriculture residues, cattle dung, animal bones, combustion emissions, vehicular emissions, industrial
vegetable stuff are biodegradable pollutants. Hence, (a) is emissions, forest and agricultural fires and photochemical
the correct answer. reactions of these emissions.

7. Which of the following is produced during the 10. Which of the following is the reason behind the
formation of photochemical smog? maximum noise pollution?
(a) Hydrocarbons (b) Nitrogen Oxides (a) Heavy Traffic (b) Election Meetings
(c) Ozone (d) Methane (c) Rock Music (d) Jet Flight
I.A.S. (Pre) 2003 U.P. Lower Sub. (Pre) 2003
Ans. (b) Ans. (d)

Environment and Ecology General Studies F–199


Noise pollution or noise disturbance is disturbing or excessive U.P. P.C.S. (Pre) 2018
noise that may harm the activity or balance of human or Ans. (a)
animal life. The source of most outdoor noise worldwide See the explanation of the above question.
is mainly caused by machines and transportation systems,
motor vehicles, aircraft and train. Among the options given 14. Colourless gas coming out from motor vehicle and
above, jet flight create the most the intense noise pollution. cigarette due to incomplete combustion is –
Sound Source Sound (in dB) (a) Carbon dioxide
Rustling of leaves 20 dB (b) Nitrous oxide
(c) Carbon monoxide
Whispering 30 dB
(d) Methane
Room noise 40 dB
Uttarakhand U.D.A./L.D.A. (Pre) 2003
Normal conversation 60 dB
Ans. (c)
Truck sound 80-85 dB
Colourless gas coming out from a motor vehicle and cigarette
Jet engine sound 120 dB due to incomplete cumbustion is carbon monoxide. Carbon
Landing of Jet Plane 150 dB monoxide is harmful when breathed because it displaces
oxygen in the blood causing even death.
Rocket engine 180 dB
15. Which one of the following air pollutants can affect
11. During normal conditions, the gas which pollutes the
blood- stream leading to death?
atmosphere is –
(a) Asbestos dust (b) Cadmium
(a) Carbon Monoxide (CO)
(c) Nitrogen oxides (d) Carbon monoxide
(b) Carbon dioxide (CO2)
(c) Nitrogen (N2) U.P.P.C.S.(Pre) 2013
(d) Oxygen (O2) Ans. (d)
U.P.U.D.A./L.D.A. (Pre) 2006 See the explanation of the above question.
Ans. (a) 16. Pollutant emission gas in vehicle exhaust is mainly –

Carbon monoxide (CO) is the main primary air pollutant. (a) Carbon dioxide (b) Carbon monoxide
Carbon monoxide is an odourless, colourless gas formed by (c) Marsh gas (d) Nitrogen oxide
the incomplete combustion of fuels. When people are exposed Uttarakhand P.C.S. (Pre) 2002
to CO gas, the CO molecules displace the oxygen in their 44th B.P.S.C. (Pre) 2000
bodies and lead to poisoning. Ans. (b)
12. Which one of the following is not a secondary Carbon monoxide is main pollutant emission gas in vehicle
pollutant? exhaust.
(a) PAN (b) Smog
17. Combustion of petrol in automobiles pollutes the air
(c) Sulphur dioxide (d) Ozone
by producing the following metal.
U.P.P.C.S. (Pre) 2014
(a) Mercury (b) Cadmium
Ans. (c)
(c) Lead (d) Carbon Dioxide
PAN (Peroxyacetyl Nitrate), Ozone and Smog are secondary (e) None of these
pollutants because these are formed after reaction of primary
Chhattisgarh P.C.S. (Pre) 2015
pollutants with atmosphere. While oxide of sulphur (mainly
sulphur dioxide), an oxide of nitrogen, carbon monoxide Ans. (c)
are primary pollutants as these directly spread in the air and Combustion of petrol in automobiles pollutes the air by
pollute the atmosphere. producing lead which causes long term harm in adults including
13. Which among the following is the secondary pollutant? increased risk of high blood pressure and kidney damage.
(a) Smog (b) Carbon dioxide 18. Which of the following air pollutants get dissolved
(c) Carbon mono-oxide (d) Fly ash in the haemoglobin of the blood more rapidly than

F–200 General Studies Environment and Ecology


oxygen? (a) Soil Pollution
(a) PAN (b) Carbon dioxide (b) Water pollution
(c) Carbon monoxide (d) Ozone (c) Air pollution
U.P.P.C.S. (Pre) 2016 (d) All of the above
Ans. (c) U.P.P.C.S. (Pre) 2016
The molecular weight of carbon monoxide is similar to that Ans. (d)
of air. It mixes freely with air in any proportion. Carbon The use of chemical fertilizer is a cause of nitrogen loss
monoxide is not detectable by humans either by taste or contributing to soil and land degradation. Surface water and
smell. In the human body, it reacts with haemoglobin to form groundwater sources are polluted from chemical fertilizer
carboxyhemoglobin (COHb). used in irrigation farming, thus contaminating water quality.
19. Which of the following air pollutants is most The most common chemical fertilizer, nitrogen is produced
dangerous? using large amounts of oil and natural gas. As these are
fossil fuels, their use generates air pollution. Hence (d) is
(a) Ozone (b) Hydrogen sulphide
the correct answer.
(c) Carbon dioxide (d) Carbon monoxide
U.P.R.O./A.R.O. (Mains) 2014
Ans. (d)
See the explanation of the above question.
20. The non-biotic pollutant in underground water is –
(a) Bacteria (b) Algae
(c) Arsenic (d) Viruses
U.P.P.C.S.(Pre) 2012
Ans. (c)

Arsenic is a non-biotic pollutant of underground water. The


introduction of Arsenic into drinking water can occur as a 23. The highest chemical pollution is caused by the
result of its natural geological presence in the local bedrock. industrial effluents of :
Presence of Arsenic in groundwater is relatively high in areas (a) Leather industry
adjoining banks of Ganga. Pesticides and insecticides also (b) Paper industry
add to its presence in the ecosystem. According to WHO the (c) Rayon industry
amount of arsenic should be 0.01 mg/litre. (d) Textile industry
21. Water pollution of river is measured by - U.P.P.C.S. (Pre) 2005
(a) Amount of Chlorine dissolved in water Ans. (a)
(b) Amount of Ozone dissolved in water
The highest chemical pollution caused by the industrial
(c) Amount of Nitrogen dissolved in water effluents is from the leather industry. The leather industry is
(d) Amount of Oxygen dissolve in water mainly responsible for water and soil pollution.
U.P.R.O./A.R.O. (Pre) 2021 24. Acid rain is due to air pollution by–
Ans. (d) (a) Carbon dioxide
Water pollution of river is measured by the amount of Oxygen (b) Carbon monoxide
dissolved in water. The test Dissolved oxygen (DO) measures (c) Methane
that amount of oxygen dissolved in the water. Oxygen is (d) Nitrous oxide and Sulphur dioxide
essential for both plants and animals, but high levels in U.P.P.C.S.(Pre) 2013
water can be harmful to fish and other aquatic organisms.
Ans. (d)
Nonpoint-source pollution can decrease the amount of
dissolved oxygen in water, which can be harmful to fish and Acid rain is caused by a chemical reaction that begins when
other aquatic organisms. a compound like sulphur dioxide and nitrogen oxides are
22. Indiscriminate use of fertilizers has led to : released into the air. These substances can rise very high into

Environment and Ecology General Studies F–201


the atmosphere where they mix and react with water, oxygen (a) Acetic acid and Phosphoric acid
and other chemical to form more acidic pollutants. When (b) Acetic acid and Sulphuric acid
these pollutants fall on the surface with rain, it is termed (c) Nitric acid and Sulphuric acid
acid rain. Normally, the pH value of acid rain is less than (d) Hydrogen chloride and Acetic acid
5.6. Notably, the pH value of pure water is 7.0. Chhattisgarh P.C.S. (Pre) 2020
25. The component(s) of acid rain is/are: Ans. (c)
(a) HNO3 (b) H2SO4
See the explanation of above question.
(d) CO2 (d) Both (a) and (b)
(e) None of the above/More than one of the above 30. Acid rain is caused by pollution of environment by –
66th BPSC Re-Exam 2020 (a) Carbon monoxide and Carbon dioxide
Ans. (d) (b) Carbon dioxide and Nitrogen
(c) Nitrous oxide and Sulphur dioxide
See the explanation of above question.
(d) Carbon dioxide
26. Which among the following environment pollutants U.P.P.S.C. (GIC) 2010
are responsible for "acid rain"? U.P.P.C.S. (Mains) 2007
(a) Carbon dioxide and Nitrogen Uttarakhand P.C.S. (Pre) 2006
(b) Carbon monoxide and Carbon dioxide U.P.U.D.A./L.D.A. (Pre) 2001
(c) Nitrous oxide and sulphur dioxide Ans. (c)
(d) Ozone and Carbon dioxide
See the explanation of above question.
R.A.S./R.T.S. (Pre) 2018
Ans. (c) 31. In the acid rain, the rainwater and snow is contaminated
by which of the following pollutants?
Acid rain is the rainfall that has been acidified. It is formed
when oxides of sulphur and nitrogen react with the moisture 1. Sulphur-dioxide 2. Nitrogen oxide
in the atmosphere. It is rain with a PH of less than 6. Acid 3. Carbon-dioxide 4. Methane
rain particularly damages lakes, streams, forests, plants and Codes :
animals that live in that ecosystem. (a) 1, 2 and 4 (b) 1 and 2 only
27. Which of the following is/are responsible for acid rain? (c) 1, 2 and 4 (d) 2 and 3 only
(a) Carbon dioxide R.A.S./R.T.S. (Pre) 2015
(b) Carbon monoxide Ans. (b)
(c) Propane Sulphur dioxide and nitrogen oxide are the main factors for
(d) Nitrous oxide and sulphur dioxide acid rain. Acid rain contaminates rainwater and snow. This
Jharkhand P.C.S. (Pre) 2013 produces an adverse impact on rivers, lakes and ponds.
Ans. (d) 32. Acid rain occurs due to –
See the explanation of the above question. (a) acid vapour reaching the clouds
28. Acid rain is caused by the pollution of the environment (b) reaction between rainwater and carbon dioxide
by: pollutants
(a) Carbon dioxide and nitrogen (c) reaction between clouds and sulphur dioxide pollutants
(b) Carbon monoxide and carbon dioxide (d) reaction between water vapour and electric ions during
(c) Ozone and carbon dioxide lightening and cloud burst.
(d) Nitrous oxide and sulphur dioxide U.P.P.C.S. (Pre) 2001
I.A.S. (Pre) 2013 Ans. (c)
Ans. (d) Acid rain is caused by the reaction of the cloud's water and
sulphur dioxide. Acid rain reduces pH value of soil. It also
See the explanation of above question.
impacts micro-organism as their growth is dependent on pH
29. 'Acid rain' consists of the following: value. Acid rain also increases the corrosion rate of metals in

F–202 General Studies Environment and Ecology


particular iron, steel, copper and bronze. These enter in the Ans. (c)
drinking water and soil and thus affect the nervous system. Eutrophication is a form of water pollution. It occurs when
33. Which of the following, when dissolved in rainwater excessive fertilizers run into lakes and rivers. Eutrophication
turns rainwater acidic (acid rain) – decreases water quality and reduces oxygen. Asbestosis is
(a) Oxide of Sulphur (b) Oxide of boron a disease caused by the presence of asbestos in the air for a
long time. Rest are related to air pollution.
(c) Hydrochloric acid (d) Nitric acid
R.A.S./R.T.S. (Pre) 1997 38. Fly ash pollution is caused by
Ans. (a) (a) Oil refining (b) Fertilizer industry
See the explanation of the above question. (c) Thermal power plant (d) Mining

34. Acid rain is caused by – U.P.P.C.S. (Mains) 2004


(a) Industries (b) Petrol U.P.P.C.S. (Pre) 2021
(c) Burning coal (d) Wood Ans. (c)
47th B.P.S.C. (Pre) 2005 Fly ash is a by-product generated as the coal is burnt at
Ans. (a)
thermal power plants. The low-grade coal used in thermal
While a small portion of sulphur dioxide and nitrogen power generation carries 30-45% ash content.
oxide that cause acid rain is from natural resources such as The high-grade imported coal has a low ash content of 10-
volcanoes, most of it comes from the burning of fossil fuels. 15%. Since most of the coal used in thermal plants in low-
The major sources of sulphur dioxide and nitrogen oxide in grade, it generates a large quantity of ash which requires a
the atmosphere are -
large area as landfill or ponds for disposal.
Burning of fossil fuels
Vehicles and heavy equipment 39. Green Muffler’ is related to –
Manufacturing, oil refineries and other industries. (a) Soil pollution (b) Air pollution

35. Which one of the following gases is responsible for (c) Noise pollution (d) Water pollution
causing acid rain in the atmosphere: U.P.P.C.S. (Pre) 2014
(a) CFC (b) CH4 Ans. (c)
(c) O3 (d) SO2 Green Muffler is a control measure for noise pollution by
U.P.U.D.A./L.D.A. (Pre) 2002 planting green plants. Normally 4-5 rows of plants are grown
U.P.P.C.S. (Pre) 2003 near the noisy places like roadsides and industrial areas so
Ans. (d) that they can obstruct noise.
See the explanation of the above question. 40. Bhopal Gas Tragedy (leakage of methyl isocyanate
36. Which one of the following is an air pollutant gas and ‘MIC’) accident happened on –
is released by burning fossil fuel? (a) December 2, 1982
(a) Hydrogen (b) Nitrogen
(b) December 3, 1985
(c) Oxygen (d) Sulphur dioxide
(c) December 3, 1984
U.P.P.C.S. (Mains) 2011
Ans. (d) (d) December 4, 1986

Sulphur dioxide is an air pollutant gas and is released by Uttarakhand U.D.A./L.D.A. (Pre) 2007

burning fossil fuel. Ans. (c)

37. Which one of the following is not related to air Bhopal disaster, also referred to as the Bhopal Gas Tragedy
was a gas leak incident in India, considered the world’s worst
pollution?
industrial disaster. It occurred on the night of 3 December,
(a) Smog (b) Acid Rain 1984 at the Union Carbide India Limited pesticide plant
(c) Eutrophication (d) Asbestosis in Bhopal, Madhya Pradesh. Over 500,000 people were
U.P.P.C.S. (Mains) 2013 exposed to Methyl Isocyanate (MIC) gas. According to state

Environment and Ecology General Studies F–203


government record, 3787 people died in this tragedy. Later, not destroyed by bacteria. Hence they pollute the atmosphere.
due to disease caused by the gas around 25000 people were Rest of all are easily disintegrated by the micro-organism.
killed. 46. Which one of the following is biodegradable?
41. The gas which leaked from Union Carbide Factory, (a) Plastic (b) Polythene
Bhopal was– (c) Mercury (d) Rubber
(a) Carbon monoxide (b) Methane U.P.R.O./A.R.O. (Pre) 2014
(c) Methyl isocyanate (d) Sulpher dioxide Ans. (d)
U.P.P.C.S. (Pre) 2008 A substance capable of being decomposed by bacteria or
U.P. Lower Sub. (Pre) 2002 other living organism and thereby avoiding pollution is
Ans. (c) called biodegradable. Among the given options, rubber is
See the explanation of the above question. biodegradable.
47. Which of the following takes maximum time to decay?
42. Which gas leaked out, in the Bhopal Gas Tragedy
(a) Cigarette bud (b) Leather shoe
causing numerous deaths:
(c) Photo film (d) Plastic bag
(a) Chlorine (b) M.I.C.
U.P.P.C.S. (Spl) (Pre) 2008
(c) Ammonia (d) Phosgene
Ans. (d)
M.P.P.C.S. (Pre) 1991
U.P.P.C.S.(Pre) 2001 A plastic bag takes maximum time to decay as polyethylene
Ans. (b) are non-biodegradable.

See the explanation of the above question. 48. Which one of the following is a bioindicator of air
pollution?
43. The compound associated with the Bhopal Gas Tragedy
(a) Lichen (b) Fern
was?
(c) Money plant (d) Dodder
(a) Methyl alcohol
U.P.P.C.S.(Pre) 2013
(b) Phosphazene
U.P.U.D.A./L.D.A. (Spl) (Pre) 2010
(c) Methyl isocyanate
Ans. (a)
(d) Methylamine
U.P.R.O./A.R.O. (Pre) 2014 Bioindicators are living organisms that respond in a specially
Ans. (c) clear way to a change in the environment. The hardy lichens are
useful bioindicators for air pollution, especially sulphur dioxide
See the explanation of the above question. pollution since they derive their water and essential nutrients
44. Polythene bags cannot be destroyed because they are mainly from the atmosphere rather than from the soil.
made of – 49. Which one of the following is the well known example
(a) Unbreakable molecules of bioindicator of air pollution?
(b) Inorganic compounds (a) Lichens (b) Methyl mercury
(c) Polymers (c) Rose plant (d) Sunflower
(d) Proteins U.P.P.C.S. (Pre) 2019
U.P.P.C.S. (Pre) 2007 Ans. (a)
Ans. (c)
See the expalantion of above question.
Polythene is made up of molecules of carbon and hydrogen.
It is a polymer of ethylene (C2H4). It is non-biodegradable. 50. Which of the following is indicator of air
This is the reason why polythene bags cannot be destroyed. pollution ?
(a) Puffballs (b) Algae
45. Which of the following is not destroyed by bacteria?
(c) Lichen (d) Moss
(a) Cow dung (b) Leafs
U.P.P.C.S. (Pre) 2021
(c) Food ingredient (d) Plastic
Ans. (c)
R.A.S./R.T.S. (Pre) 1992
Ans. (d) Lichens are symbiotic associations i.e. mutually useful
associations, between algae and fungi. The algal component
Plastic, Iron, lead are non-biodegradable pollutants which are is known as phycobiont and fungal component as mycobiont,

F–204 General Studies Environment and Ecology


which are autotrophic and heterotrophic, respectively. Algae Ans. (d)
prepare food for fungi and fungi provide shelter and absorb See the explanation of the above question.
mineral nutrients and water for its partner. So close is their
association that if one saw a lichen in nature one would never 55. With reference to Eutrophication, which of the
imagine that they had two different organisms within them. following statements is/are correct?
Lichens are very good pollution indicators - they do not grow 1. It is a phenomenon of nutrient enrichment of a
in polluted areas. water body.
51. Lichens are the best indicator of – 2. It depletes dissolved oxygen of the water.
(a) Air pollution (b) Water pollution Select the correct answer using codes given below :
(c) Soil pollution (d) pollution Codes :
U.P.P.C.S.(Pre) 2012 (a) 1 only
Ans. (a) (b) 2 only
See the explanation of the above question. (c) Both 1 and 2
(d) Neither 1 nor 2
52. Biological Oxygen Demand (BOD) is an indication of
U.P.P.C.S. (Pre) 2019
pollution in :
Ans. (c)
(a) Aquatic environment
(b) Soil Eutrophication is excessive richness of nutrients in a lake
(c) Air or other body of water, frequently due to run-off from the
(d) All the above land, which causes a dense growth of plant life. This induces
Uttrakhand U.D.A./L.D.A. (Mains) 2007 excessive growth of algae and this process depletes dissolved
Ans. (a) oxygen of the water. Eutrophication is often induced by the
discharge of nitrate or phosphate-containing detergents,
Biological oxygen demand is the amount of dissolved oxygen
fertilizers, or sewage into an aquatic system.
needed by an anaerobic biological organism to break down
organic material present in a given water sample at a certain 56. BOD is maximum in the Ganga river between :
temperature over a specific period. Increase in BOD shows (a) Haridwar and Kanpur
a high level of organic pollution of water since all organic (b) Kanpur and Allahabad
and inorganic waste use dissolved oxygen for decomposition. (c) Allahabad and Patna
This means if there is less pollution, the dissolved oxygen (d) Patna and Uluberia
will be high and vice versa. Thus BOD works as an indicator
U.P.P.C.S. (Spl) (Mains) 2008
of pollution in the aquatic environment.
Ans. (b)
53. A water body having high B.O.D. indicates that its
BOD is maximum in the Ganga river between Kanpur and
water is –
Allahabad as Kanpur discharge heavy amount of pollutants
(a) Receiving minerals
in Ganga. As a result of which BOD is high here.
(b) Being aerated
(c) Being contaminated by sewage 57. Bioremediation means:
(d) Atrophic (a) Elimination of toxic pollutant from the environment
U.P.P.C.S. (Pre) 2007 by organisms
(b) Biocontrol of pathogens and pests
Ans. (c)
(c) Transplantation of organs in the body
BOD indicates the amount of organic matter present in water. (d) Diagnosis of diseases by the help of microorganisms
Therefore, a low BOD is an indicator of good quality water, R.A.S./R.T.S.(Pre) 2007
while a high BOD indicates polluted water. Ans. (a)
54. The water pollution in a river is determined by Bioremediation is a process of detoxification of environment
measuring the dissolved amount of : through the use of small organisms. This technique can help
(a) Chlorine (b) Nitrogen to overcome adverse effects of pollution at particular places.
(c) Ozone (d) Oxygen Bioremediation is used in cleaning surface and groundwater
U.P.P.C.S. (Pre) 2011 and soil, etc.

Environment and Ecology General Studies F–205


58. In the context of solving pollution problems, what Groundwater in Brahmaputra, Gangetic plains of India and
is/are the advantage/advantages of bioremediation Padma Meghna plains of Bangladesh is highly polluted
technique? with arsenic. Groundwater in seven states of India- West
Bengal, Jharkhand, Bihar, Uttar Pradesh, Assam, Manipur
1. It is a technique for cleaning up pollution by
and Chhattisgarh is polluted with arsenic. The first instance
enhancing the same biodegradation process that of water pollution with arsenic came to notice in 1978.
occurs in nature. Notably, the permissible range of arsenic in groundwater is
2. Any contaminant with heavy metals such as 10 Microgram per liter.
cadmium and lead can be readily and completely 61. Chernobyl accident is related with
treated by bioremediation using microorganisms. (a) Nuclear accident (b) Earthquake
3. Genetic engineering can be used to create micro- (c) Flood (d) Acid rain
organisms specifically designed for bioremediation. (e) None of these
Select the correct answer using codes given below: Chhattisgarh P.C.S. (Pre) 2015
(a) 1 only (b) 2 and 3 only Ans. (a)
(c) 1 and 3 only (d) 1, 2 and 3
The Chernobyl disaster was a nuclear accident that took place
I.A.S. (Pre) 2017 on 26 April, 1986 in Ukraine. Due to the deadly radioactive
Ans. (c) affect, many people lost their lives. Importantly, decomposing
radioactive nuclides cause radioactive pollution. It harms
The process of expulsion of toxic pollutants from the
organisms and affects their genetic properties.
environment by using microorganism is called bioremediation.
It is a technique for cleaning up pollution by enhancing the 62. Which of the following is a biodegradable pollutant?
same biodegradation process that occurs in nature. But (a) Asbestos (b) DDT
pollutants containing many types of heavy metals cannot
(c) Plastic (d) Sewage
be treated by the process of bioremediation. Any pollutant-
Uttarakhand P.C.S. (Pre) 2010
containing heavy metals like cadmium and lead cannot be
treated comfortably and completely by bioremediation using Ans. (d)
microorganisms. Genetic engineering can be used to create Sewage is a biodegradable pollutant. Rest are nonbio-
microorganisms specially designed for bioremediation, for degradable substances.
example, superbug Pseudomonas putida which is created by
genetic engineering techniques. It plays an important role in 63. The most toxic metal pollutant of automobile exhaust
ending the oil spill in the ocean. is
(a) Copper
59. Which of the following is not a water pollutant?
(b) Lead
(a) Zinc (b) Copper
(c) Cadmium
(c) Nickel (d) Sulphur Dioxide
(d) None of the above
U.P.P.C.S. (Mains) 2011
U.P.P.C.S. (Mains) 2009
Ans. (d)
U.P.P.C.S. (Pre) 2006
Sulphur dioxide is an air pollutant. When it rises very high U.P.P.C.S. (Mains) 2006
into the atmosphere, it reacts with water, oxygen and other U.P. U.D.A./L.D.A. (Pre) 2006
chemicals to form more acidic pollutants. Rain with such
Ans. (b)
harmful pollutants is known as acid rain.
Automobiles use lead as an anti-knocking agent. Lead when
60. The problem of water pollution with Arsenic is released in the air with exhaust are most toxic. Lead affects
maximum in – the nervous system, brain and digestive system of the human
(a) Haryana (b) Rajasthan body.
(c) Madhya Pradesh (d) West Bengal 64. The most polluted city of India is –
U.P.P.C.S. (Pre) (Re. Exam.) 2015 (a) Ankleshwar (b) Lucknow
Ans. (d) (c) Ludhiana (d) Raipur

F–206 General Studies Environment and Ecology


U.P.P.C.S. (Mains) 2009 (a) Noyale (b) Damodar
Ans. (a) (c) Bhiwani (d) Bhadar
In 2009, Ankleshwar was the most polluted city in India. U.P.P.S.C. (R.I.) 2014
However, according to 2016 data Wapi (Gujarat) has become Ans. (b)
India's most polluted city. Its Comprehensive Environmental
See the explanation of the above question.
Pollution Index score was 85.31 followed by Ghaziabad with
a score of 84.13. It is noteworthy that in February, 2020, 69. Which of the following is used as an adulterant of a
IQ Airvisual published 'World Air Quality Report, 2019'. mustard seed?
According to the report Ghaziabad is the most polluted city
(a) Argimon seed (b) Papaya seed
of the world.
(c) Cumin seed (d) Coriander seed
65. According to the report released by Greenpeace India U.P.U.D.A/L.D.A. (Spl) (Pre) 2010
in January, 2020, amongst 287 Indian cities, which of Ans. (a)
the following is the most polluted city?
Argimon seed is usually mixed with mustard seed as an
(a) Raniganj (b) Jharia adulterant. It is a form of poppy found in Mexico. Mixing
(c) Bajrang Nagar (d) Sohagpur of this in mustard oil could yield an epidemic called dropsy.
U.P.B.E.O. (Pre) 2019
70. Why is there a great concern about the 'microbeads'
Ans. (b)
that are released into environment?
Greenpeace India has released Airpocalypse Report in (a) They are considered harmful to marine ecosystem.
January, 2020 and identified 231 out of 287 Indian cities (b) The are considered to cause skin cancer in children.
severely polluted. According to the report, coal-belching
(c) They are small enough to be absorbed by crop plants
Jharia in Jharkhand is the most polluted city in India.
in irrigated fields.
66. Which was the seasonal factor responsible for the (d) They are often found to be used as food adulterants.
intense cold in northern India in January, 2012? I.A.S. (Pre) 2019
(a) Deforestation (b) Atmospheric pollution Ans. (a)
(c) La Nina (d) Al Nino Microbeads are considered harmful to marine ecosystem
U.P.U.D.A./L.D.A. (Spl) (Mains) 2010 as they are usually made up of polyethylene. They are
Ans. (c) extensively used in shampoo, face cream and baby lotions.
They are usually toxic to marine life. Hence, there is great
La Nina is characterized by unusually cold oceanic
concern over releasing them into environment as they are
temperature in the Equatorial Pacific. Due to La Nina Wind,
harmful to ecosystems.
January 2012 witnessed intense cold in Northern India.
71. Which one of the following is known for cleaning a
67. Which of the following rivers is called ‘biological
polluted environment?
desert’ on account of its pollutants?
(a) Oxygen (b) Rain
(a) Yamuna (b) Periyar
(c) Nitrogen (d) Air
(c) Damodar (d) Mahanadi
U.P.U.D.A/L.D.A. (Pre) 2013
U.P.P.C.S.(Pre) 2012
Ans. (b)
Ans. (c)
Rain can wash out particles from the air. Washing works
Damodar, considered as ‘Sorrow of Bengal’ due to its
much better on larger particles (above 10 microns or above
devastating floods, has now become the sorrow of both
2.5 microns) than small particles. Rains also help by making
Jharkhand and Bengal owing to excessive pollution. The
dust particles stick to the ground.
enormous quantity of pollutants coming out from the
mines, industries, cities and agricultural fields have so much 72. The Union Cabinet has approved MoU between India
degraded the quality of river that it has virtually become a and SACEP for cooperation on oil and chemical
biological desert. pollution in the South Asian seas region. What does
68. Which Indian River is called ‘biological desert’? SACEP stand for?

Environment and Ecology General Studies F–207


(a) South Asia Compatibility Environment Programme I.A.S. (Pre) 2019
(b) South Asia Cooperative Environment Programme Ans. (c)
(c) South Asia Connectivity Environment Programme Solid Waste Management Rules, 2016 in India provide for
(d) South Asia Coercive Environment Programme exact and elaborate criteria for the identification of sites for
(e) None of the above/More than one of the above landfills and waste processing facilities. Hence, option (c)
63rd B.P.S.C. (Pre) 2017 is correct answer.
Ans. (b) 3. Asian Brown Cloud, 2002 was mainly spared over –
(a) East Asia (b) South-East Asia
On 28th March 2018, the Union Cabinet has approved MoU
(c) South Asia (d) West Asia
(Memorandum of Understanding) between India and SACEP
for cooperation on Oil and Chemical Pollution in the South Uttarakhand U.D.A./L.D.A. (Pre) 2003
Asian Seas Region. Here SACEP stands for 'South Asia Ans. (c)
Cooperative Environment Programme'. Asian Brown Cloud was mainly spread over South Asia
due to pollution. A detailed report was published on this
in 2007. Brown clouds are formed due to air pollution and
High-Level Questions spread over particularly South Asian region during January-
1. There is a concern over the increase in harmful algal March. Brown clouds consist of approximately 85% aerosol.
It also contains sulphate, black carbon and other deadly
blooms in the sea-waters of India. What could be the
chemicals which cause respiratory problems and cancer.
causative factors for this phenomenon?
Significantly, change in solar radiation in India and China,
1. Discharge of nutrients from the estuaries. uneven atmospheric temperature and change in monsoonal
2. Run-off from the land during the monsoon. rainfall pattern are caused by brown clouds.
3. Upwelling in the seas.
4. In the context of which of the following do some
Select the correct answer using codes given below: scientists suggest the use of cirrus cloud thinning
(a) 1 only (b) 1 and 2 technique and the injection of sulphate aerosol into
(c) 2 and 3 (d) 1, 2 and 3 stratosphere?
I.A.S. (Pre) 2011 (a) Creating the artificial rains in some regions
Ans. (d) (b) Reducing the frequency and intensity of tropical
Harmful algal blooms, lethal for a human being and marine cyclones
ecosystems alike, are steadily increasing in intensity in Indian (c) Reducing the adverse effects of solar wind on the Earth
waters. Upwelling, the formation of mud banks, nutrient (d) Reducing the global warming
discharges from estuaries and run-off from land during I.A.S. (Pre) 2019
the south-west and north-east monsoons cause some algae Ans. (d)
blooms in coastal waters. Hence (d) is the correct answer.
For reducing the global warming geo-engineering solutions
2. As per the Solid Waste Management Rules, 2016 in have often been suggested by the scientists. Injection
India, which one of the following statements is correct? of sulphate aerosols, very fine solid particles, into the
(a) Waste generator has to segregate waste into five stratosphere is one of the most widely discussed ideas.
categories Sulphate aerosol particles are very good reflectors of sunlight,
and it has been shown, through various climate models, that
(b) The Rules are applicable to notified urban local bodies,
even if 1% of current incident solar radiation is reflected
notified towns and all industrial townships only. back in space, a very significant amount of temperature rise
(c) The Rules provide for exact and elaborate criteria on earth can be offset. Another way of instantly cooling the
for the identification of sites for landfills and waste planet is the use of cirrus cloud thinning technique. These
processing facilities. clouds are composed mainly of ice crystals and they have
(d) It is mandatory on the part of waste generator that greenhouse property. They let the solar radiation pass through
and reach the surface, but trap the higher wavelength infrared
the waste generated in one district cannot be moved
radiation emitted from the earth, thereby contributing to the
to another district. heating. If these clouds are reduced by some engineering

F–208 General Studies Environment and Ecology


interventions, it would allow the IR radiation from the earth, (c) Nitrogen Dioxide (d) Radon gas
too, to pass through to space, thus allowing some of the heat (e) None of these
to dissipate and hence cool the planet. Chhattisgarh P.C.S. (Pre) 2015
5. In the context of which one of the following are the Ans. (d)
terms ‘pyrolysis and plasma gasification’ mentioned? Radon is a colourless, odourless, radioactive gas. It occurs
(a) Extraction of rare earth elements naturally from the soil as a decay product of radium. Radon
(b) Natural gas extraction technologies gas is a health hazard. It causes lung cancer.
(c) Hydrogen fuel-based automobiles 8. Cigarette smoke contains –
(d) Waste-to-energy technologies (a) Carbon monoxide dioxycine
I.A.S. (Pre) 2019 (b) Carbon monoxide and nicotine
Ans. (d) (c) Carbon monoxide and Benzene
Like incineration, plasma pyrolysis and gasification (d) Dioxycine and Benzene
technologies are thermal processes that use high temperatures U.P.P.C.S. (Pre) (Re. Exam) 2015
to break down waste. Plasma gasification system facilitates Ans. (c)
safe disposal of all type of organic wastes. The inorganic
Cigarette smoke contains complex compounds like carbon
materials like glass, metals can also be minimized to slag.
monoxide, hydrogen cyanide, arsenic, formaldehyde and
In plasma pyrolysis the most likely compounds which are
benzene etc. It also contains nicotine in minute quantity.
produced include carbonaceous matter, methane, carbon
monoxide, hydrogen, carbon dioxide and water molecules. 9. Lead, ingested or inhaled, is a health hazard. After the
The product gases recovered has calorific and commercial addition of lead to petrol has been banned, what still
values. These gases can be used as a heat source or as fuel. are the sources of lead poisoning?
NITI Ayog in his three-year Action Agenda (2017-2020), has
1. Smelting units
also assessed the benefit-cost ratio of thermal pyrolysis and
2. Pens and pencils
plasma gasification as waste-to-energy technologies.
3. Paints
6. Assertion : During winter, air pollution in Delhi
4. Hair oils and cosmetics
reaches to the maximum level.
Select the correct answer using the code given below:
Reason : Combustion process in motor vehicles
(a) 1, 2 and 3 (b) 1 and 3
increases during winter.
(c) 2 and 4 (d) 1, 2, 3 and 4
(a) Both (A) and (R) are true and (R) is the correct
I.A.S. (Pre) 2012
explanation of (A). Ans. (b)
(b) Both (A) and (R) are true, but (R) is not the correct
Lead is a common environmental pollutant. Children living
explanation of (A).
near factories that process lead, such as lead smelters, have
(c) (A) is true, but (R) is false. been found to have unusually high blood lead levels. Some
(d) (A) is false, but (R) is true. lead compounds are colourful and are widely used in paints
U.P. Lower Sub. (Pre) 2004 and lead paint is a major route of lead exposure in children.
Ans. (c) Except these, soil, water, lead-containing products, bullets are
other sources of lead. Hence, option (b) is the correct answer.
Cold air being relatively heavy than the warm air does not
rise high in the atmosphere. Hence, dust particles, pollutants 10. Match List-I with List-II and select the correct answer
and environmental dirt form a layer above the surface in the by using codes given below the lists:
atmosphere. This is the reason for increased pollution level List-I List-II
during winter in Delhi. Therefore, assertion (A) is correct. (Air Pollutant) (Part affected)
However, Reason (R) is incorrect as combustion process in A. Asbestos Dust 1. Brain
motor vehicles decreases during winter.
B. Lead 2. Stomach
7. The most important indoor air pollutant is – C. Mercury 3. Lung
(a) Sulpher Dioxide (b) Carbon Dioxide D. Carbon Monoxide 4. Bloodstream

Environment and Ecology General Studies F–209


Codes : I.A.S. (Pre) 2014
A B C D Ans. (d)
(a) 1 2 3 4 In steel furnace, coke reacts with iron ore releasing iron and
(b) 3 1 2 4 generating Carbon monoxide and Carbon dioxide gases.
(c) 3 2 4 1 Due to the use of coal, pollutants such as oxides of sulphur
(d) 2 3 1 4 and nitrogen are released. Thus, all the options are correct.
U.P.P.C.S.(Pre) 2012 Hence, option(d) is the correct answer.
Ans. (b) 13. Which of the following countries suffer from the acid
rains?
Asbestos is a poisonous material whose dust could cause
lung cancer. Lead affects the nervous system and brain of the 1. Canada 2. France
human body. Mercury affects the human stomach, Carbon 3. Norway 4. Germany
monoxide displaces the oxygen in the human bloodstream Select the correct answer from codes given below:
and lead to poisoning. Codes :
11. Match List-I with List-II and select the correct answer (a) 1 and 2 (b) 1 and 3
from codes given below the lists : (c) 2 and 3 (d) 3 and 4
List-I List-II U.P.P.C.S (Pre) 2011
(Pollutants) (Effects) Ans. (b)
A. Carbon monoxide 1. Damage to liver and kidney Among the countries given in the options, Canada and
B. Oxides of Nitrogen 2. Cancer Norway suffer most from acid rain. Notably, Sulphur dioxide
C. Soil Particles 3. Ailment related to breathing and nitrogen oxide emitted by factories in the U.K. and
D. Lead 4. Central nervous system Germany affect Norway and Sweden through acid rain.
Codes: 14. Which one of the following countries receive the most
A B C D acid rain:
(a) 2 3 4 1 (a) China (b) Japan
(b) 4 3 2 1 (c) Norway (d) the USA
(c) 1 2 3 4 U.P.P.C.S. (Pre) 2000
(d) 4 3 1 2 Ans. (c)
U.P.P.C.S. (Pre) 2008
Norway receives most acid rain among the given options.
Ans. (c)
15. Consider the following
(Pollutants) (Effects)
1. Carbon dioxide
Carbon monoxide Damage to liver and kidney
2. Oxides of Nitrogen
Oxides of Nitrogen Cancer
3. Oxides of Sulphur
Soil Particles Ailments related to breathing
Which of the given above is/are the emission/emissions
Lead Central nervous system
from coal combustion at thermal power plants ?
12. Which of the following are some important pollutants (a) 1 only (b) 2 and 3
released by the steel industry in India? (c) 1 and 3 (d) 1, 2 and 3
1. Oxides of sulphur I.A.S. (Pre) 2011
2. Oxides of nitrogen Ans. (d)
3. Carbon monoxide Pollution from coal combustion thermal power plants is
4. Carbon dioxide caused by the emission of gases such as carbon dioxide,
Select the correct answer using codes given below. nitrogen oxides and sulphur dioxide into the air. Hence (d)
(a) 1, 3 and 4 only (b) 2 and 3 only is the correct answer.
(c) 1 and 4 only (d) 1, 2, 3 and 4 16. Match List–I with List–II and select the correct answer

F–210 General Studies Environment and Ecology


from the codes given below the lists : emissions. Hydrogen upto one-fifth by volume can be blended
List–I List–II with CNG as fuel for buses. Hence statement 3 is correct.
(Phenomenon) (Compounds) However H-CNG is not cheaper fuel than CNG. So statement
A. Acid Rain 1. Chlorofluoro Carbon 4 is not correct. Hence (b) will be the correct answer.
B. Photochemical Smog 2. Carbon Monoxide 18. Concerning ‘fly ash’ produced by the power plants
C. Combination 3. Sulphur dioxide using coal as fuel, which of the following statements
with Haemoglobin is/are correct?
D. Depletion of 4. Unsaturated 1. Fly ash can be used in the production of bricks for
Ozone Layer Hydrocarbons building construction.
A B C D 2. Fly ash can be used as a replacement for some of
(a) 1 2 3 4 the Portland cement concrete.
(b) 3 2 4 1 3. Fly ash is made up of silicon dioxide and calcium
(c) 3 4 2 1 oxide only and does not contain any toxic elements.
(d) 1 3 2 4 Select the correct answer using codes given below:
U.P.B.E.O. (Pre) 2019 (a) 1 and 2 (b) 2 only
Ans. (c) (c) 1 and 3 (d) 3 only
I.A.S. (Pre) 2015
The correctly matched list is as follows:
Ans. (a)
Phenomenon Compounds
Fly ash is one of the coal combustion products composed of
Acid Rain Sulphur dioxide
the fine particles that are driven out of the boiler with the flue
Photochemical Smog Unsaturated gases. Fly ash is used for the manufacturing of construction
Hydrocarbons bricks. The most common use of fly ash is as a replacement
Combination with Haemoglobin Carbon Monoxide for Portland cement used in producing concrete. SiO2, Al2O3
and occasionally CaO are the main chemical components
Depletion of Ozone layer Chlorofluoro Carbon present in fly ashes. In addition to this, it also contains some
17. In the context of proposals to the use of hydrogen- toxic elements like lead, arsenic and copper particles. Hence,
enriched CNG (H-CNG) as fuel for buses in public option (a) is the correct answer.
transport, consider the following statements: 19. Which of the following are the reasons/factors for
1. The main advantage of the use of H-CNG is the exposure to benzene pollution?
elimination of carbon monoxide emissions. 1. Automobile exhaust
2. H-CNG as fuel reduces carbon dioxide and 2. Tobacco smoke
hydrocarbon emissions. 3. Wood burning
3. Hydrogen up to one-fifth by volume can be blended 4. Using varnished wooden furniture
with CNG as fuel for buses. 5. Using products made of polyurethane
4. H-CNG makes the fuel less expensive than CNG. Select the correct answer using the code given below :
Which of the statements given above is/are correct? (a) 1, 2 and 3 only (b) 2 and 4 only
(a) 1 only (b) 2 and 3 only (c) 1, 3 and 4 only (d) 1, 2, 3, 4 and 5
(c) 4 only (d) 1, 2, 3, and 4 I.A.S. (Pre.) 2020
Ans. (d)
I.A.S. (Pre) 2019
Ans. (b) Benzene is a chemical that is a colourless or light yellow
liquid at room temperature. Benzene is formed from both
Compared to Conventional CNG, use of H-CNG (which is a
natural processes and human activities and it evaporates
blend of hydrogen and CNG) can reduce emission of Carbon
into air very quickly. Automobile exhaust, tobacco smoke,
monoxide (CO) upto 70%. It will not eliminate CO emission.
wood burning and using varnished wooden furniture and
Hence, statement 1 is not correct.
products made of polyurethane, etc. are the factors/reasons
Moreover, H-CNG as fuel reduces CO2 and hydrocarbon for exposure to benzene pollution.

Environment and Ecology General Studies F–211


20. Improper handling and storage of cereal grains and exploration
oilseeds result in the production of toxins known as (c) It is a genetically engineered high biofuel-yielding
aflatoxins, which are not destroyed by the normal Maize variety
cooking process? Aflatoxins are produced by: (d) It is the latest technology to control the accidentally
(a) Bacteria (b) Protozoa caused flames from oil wells.
(c) Moulds (d) Viruses I.A.S. (Pre) 2011
I.A.S. (Pre) 2013 Ans. (a)
Ans. (c) Extensive research at the Energy and Research Institute
Aflatoxins are poisonous and cancer-causing chemicals (TERI) has resulted in the development of an easy and
produced by certain moulds which grow in soil, decaying cost- effective bio-remediation process. TERI has developed
vegetation and grain. They are regularly found in improperly Oilzapper which is a consortium of crude oil and oily sludge
stored staple commodities such as cassava, chili peppers, degrading bacteria deriving from various bacterial cultures
corn, cottonseed, millet, peanuts, rice, wheat etc. existing in the natural environment.

21. Which one of the following is not an instrumental 24. The headquarter of International Maritime Organization
device to check air pollution? is located in :
(a) Bag Filter (b) Cyclone Collector (a) London (b) Geneva
(c) Cyclone Separator (d) Cyclone Divider (c) Paris (d) Rome
U.P.P.C.S. (Mains) 2013 U.P.P.C.S. (Mains) 2016
Ans. (d) Ans. (a)

The cyclone divider is not used to check air pollution. Filter International Maritime Organization is a specialized agency
bags are used to filter pollutant particles of less than 50 of the United Nations. It is global standard-setting authority
micrometer size. Cyclone separator or cyclone collector is for the safety, security and environmental performance of
used to filter particles of more than 50 micrometer size. international shipping. Its main role is to create a regulatory
framework for the shipping industry that is universally
22. Which of the following statements about Radioactive adopted and implemented. Its headquarters is in London.
pollution are correct?
25. Concerning bio-toilets used by the Indian Railways,
1. It causes a hereditary change in the animals.
consider the following statements:
2. It causes disbalance among different minerals in
1. The decomposition of human waste in their toilets
the soil.
is initiated by fungal inoculum.
3. It hinders blood circulation.
2. Ammonia and water vapour are the only end
4. It causes carcinogenesis.
products in this decomposition which are released
Select the correct answer from codes given below:
into the atmosphere.
Codes :
Which of the statements given above is/are correct :
(a) 1 and 2 (b) 1 and 4
(a) 1 only (b) 2 only
(c) 1, 3 and 4 (d) 2, 3 and 4
(c) Both 1 and 2 (d) Neither 1 nor 2
U.P.P.C.S. (Pre) 2009
I.A.S. (Pre) 2015
Ans. (c)
Ans. (d)
Radioactive pollution causes hereditary changes in the all
Bio-toilets, which have been developed by the railways and
living organism. It hinders blood circulation and causes
the Defence Research Development Organization (DRDO)
carcinogenesis. It does not cause disbalance among different
has a colony of anaerobic bacteria that converts human waste
minerals in the soil. Hence (c) is the correct answer.
into water and a few gases. The gases are released into the
23. Recently, ‘oil zapper’ was in the news. What is it? atmosphere and the water is discharged after the chlorination
(a) It is an eco-friendly technology for the remediation on the tracks. Hence both of the statements are not correct.
of oily sludge and oil spills 26. Which of the following can be found as pollutants in
(b) It is the latest technology developed for undersea oil the drinking water in some parts of India?

F–212 General Studies Environment and Ecology


1. Arsenic phytoplankton larvae will be adversely affected.
2. Sorbitol 4. The cloud seeding and formation of clouds will be
3. Fluoride adversely affected.
4. Formaldehyde Which of the statement(s) given above is/are correct?
5. Uranium (a) 1, 2 and 3 (b) Only 2
Select the correct answer using codes given below : (c) 1 and 3 (d) 1, 2, 3 and 4
(a) 1 and 3 (b) 2, 4 and 5 I.A.S. (Pre) 2012
(c) 1, 3 and 5 (d) All of these Ans. (a)
I.A.S. (Pre) 2013
Ocean acidification is the ongoing decrease in the pH value
Ans. (c) of the earth’s oceans, caused by the intake of carbon dioxide
Water pollution is a major environmental issue in India. The from the atmosphere. The acidification of sea water would
largest source of water pollution in India is untreated sewage. adversely affect the growth and survival of calcareous
The groundwater contains arsenic, fluoride and uranium phytoplankton, the growth and survival of coral reefs and
consumed by nearly 85% of the population. A substantial survival of some animals that have phytoplankton larvae.
quantity of Uranium had been found in the tested sample Hence (a) is the correct answer.
of groundwater in Punjab. Hence, option (c) is the correct 30. Euro emission norms are emission standards and
answer.
these represent a package setting limits for emission
27. “Knock-Knee Syndrome” is caused by the pollution of from a vehicle. Which of the following gases is covered
(a) Mercury (b) Lead under this?
(c) Arsenic (d) Fluoride (a) Carbon monoxide (b) Hydrocarbons
U.P.P.C.S. (Mains) 2016 (c) Lead (d) All of the above
Ans. (d) U.P.P.C.S.(Pre) 2013
When fluoride level exceeds 0.5 ppm over a period of 5–10 Ans. (d)
years it may result in fluorosis. It has an affinity for calcium Euro emission standards define the acceptable limits for
and gets accumulated in bones resulting in pain in bones and exhaust emission of new vehicles sold in the European
joints and outward bending of legs from the knees known as Union. Emission of nitrogen oxides, hydrocarbon, non-
Knock-Knee Syndrome. methane hydrocarbons, carbon monoxide and particulate
28. Which pollution is responsible for 'Knock-Knee- matter are regulated for the most vehicle type. It is notable
Syndrome"? that European countries introduced 'Euro I' standard in 1992.
Significantly, Mashelkar Committee for National Autofuel
(a) Fluoride (b) Mercury
Policy suggested implementation of Euro standard to control
(c) Arsenic (d) Cadmium pollution. Importantly, BS IV standard was implemented in
M.P.P.C.S. (Pre) 2019 India starting April 1, 2017.
Ans. (a)
31. To meet Euro-II emission standards, what should be
Fluoride pollution is responsible for 'Knock-Knee Syndrome'. the sulphur content in the ultra-low sulphur diesel?
Fluoride has an affinity for calcium and gets accumulated in
(a) 0.05 per cent or less than this
bones resulting pain in bones and joints and outward bending
of legs from the knees. (b) 0.10 per cent
(c) 0.15 per cent
29. The acidification of oceans is increasing. Why is this
(d) 0.20 per cent
phenomenon a cause of concern?
U.P.P.C.S.(Pre) 2013
1. T h e g ro w t h a n d s u r v i v a l o f c a l c a re o u s
Ans. (a)
phytoplankton will be adversely affected.
2. The growth and survival of coral reefs will be To meet Euro-II emission standards, the sulphur content should
adversely affected. be 0.05 per cent or less than this in the ultralow sulphur diesel.
3. The survival of some animals that have 32. Euro Norms place limits on the amount of emission

Environment and Ecology General Studies F–213


of gas in automobiles. The gas is: U.P.P.C.S (Pre) 2020
(a) Carbon dioxide (b) Carbon monoxide Ans. (d)
(c) Nitrogen (d) Methane
National Clean Air Programme was launched by the Ministry
R.A.S./R.T.S.(Pre) 1999 of Environment, Forest and Climate Change in January
Ans. (b) 2019. Its aim is to tackle the air pollution problem across the
Euro Emission standard is pollution regulation applicable to country in a comprehensive manner with targets to achieve
all automobiles in Europe. Euro norms place a limit on the 20-30% reduction in Particulate Matter (PM 10 & PM 2.5)
amount of emission of oxides of nitrogen, carbon monoxide, concentrations by 2024 keeping 2017 as the base year.
all hydrocarbons and suspended particulate matter. 35. Carbon monoxide, emitted by vehicles which turn
33. In the cities of our country, which among the following to carbon dioxide by the catalyst converter, whose
atmospheric gases are normally considered in ceramic disc is controlled by?
calculating the value of Air Quality Index? (a) Silver (b) Gold
1. Carbon dioxide (c) Copper (d) Palladium
2. Carbon monoxide R.A.S./R.T.S.(Pre) 1994
3. Nitrogen dioxide Ans. (d)
4. Sulfur dioxide
A catalytic converter is an emission control device that
5. Methane
converts toxic gases and pollutants in the exhaust gas to less
Select the correct answer using codes given below: toxic pollutants by catalyzing a redox reaction. The catalyst
(a) 1, 2 and 3 only (b) 2, 3 and 4 only itself is most often a mix of precious metals. Palladium and
(c) 1, 4 and 5 only (d) 1, 2, 3, 4 and 5 Platinum are two such precious metals used.
I.A.S. (Pre) 2016
36. Consider the following statement :
Ans. (b)
Assertion (A) : Soil pollution is more dangerous as
Like other countries, India's Air Quality Index is based on compared to industrial pollution.
8 major pollutants–
Reason (R) : Fertilizer and pesticide enter into food
(a) < 10 micro meter size particulate matter (PM10) cycle.
(b) < 2.5 micro meter size particulate matter (PM 2.5) Choose the correct answer from codes given:
(c) Nitrogen dioxide (NO2) Codes :
(d) Carbon Mono oxide (CO) (a) Both (A) and (R) are true and (R) is the correct
explanation of (A)
(e) Ozone (O3)
(b) Both (A) and (R) are true but (R) is not the correct
(f) Sulphur dioxide (SO2)
explanation of (A)
(g) Ammonia (NH3)
(c) (A) is true but (R) is false
(h) Lead (Pb) (d) (A) is false but (R) is true
CO and O3 are measured on the average of 8 hours and U.P.P.S.C. (GIC) 2010
other pollutants are measured on the average of 24 hours, Ans. (a)
Measurement unit for NO2 and O3 is microgram per cubic
metre and for CO milligram per cubic meter. Soil pollution is more dangerous as compared to industrial
pollution as fertilizers and pesticides used on crops enter
Notably, the National Air Quality Index was issued on 17
into food cycle which affects human health. Hence, both (a)
October, 2014 by Environment and Forest Ministry. This
and (R) are correct and (R) is the correct explanation of (A).
index uses one -Number, One-colour, One-description
method. 37. Which megacity of India generates the largest solid
34. National Clean Air Programme has been launched by waste per capita annually?
the Central Government in the year : (a) Bangalore (b) Chennai
(a) 2018 (b) 2017 (c) Delhi (d) Mumbai
(c) 2020 (d) 2019 U.P.P.C.S (Pre) 2010

F–214 General Studies Environment and Ecology


Ans. (*) When soil condition is favourable to denitrification and in that
According to the latest data of CPCB (Central Polluation land huge amount of fertilizers are used then large amount
Control Board) Mumbai is the largest producer of solid waste. of nitrous oxide is usually produced from that agriculture
While during the 2009-10 Delhi was the largest producer of land. Hence statement 1 is correct. Ammonia is a common
solid waste.
by-product of animal waste in livestock farming. Hence
38. Brominated flame retardants are used in many second statement is also correct.
household products like mattresses and upholstery.
Ammonia (NH3) Nitrate (NO3) Nitric oxide (NO), Nitrous
Why is there some concern about their use?
oxide (N2O) falls under category of Reactive Nitrogen.
1. They are highly resistant to degradation in the
environment. Livestock and Poultry are one of the sources for emission of
2. They can accumulate in humans and animals. Reactive Nitrogen. Hence statement 3 is also correct.
Select the correct answer using codes given below. 40. What is the importance of crop rotation in the
(a) 1 only (b) 2 only farming?
(c) Both 1 and 2 (d) Neither 1 nor 2
(a) It increases the production
I.A.S. (Pre) 2014
Ans. (c) (b) Soil fertility is conserved
(c) Soil erosion is reduced
Brominated flame retardants are organobromine compounds
that have an inhibitory effect on combustion chemistry and (d) All the above
tend to reduce the flammability of products containing U.P.R.O./A.R.O. (Mains) 2016
them. Some brominated flame retardants were identified as Ans. (d)
persistent, bio-accumulative and toxic to both humans and the
environment and were suspected of causing neurobehavioral In the farming, crop rotation plays an important role to
effect and endocrine disruption. manage soil fertility, reduces erosion, improves soil’s health
39. Consider the following statements: and increases nutrients available for crops. It is also beneficial
1. Agricultural soils release nitrogen oxides into for improving crop yields and the work ability of the soil.
environment. 41. There is some concerns regarding the nanoparticles of
2. Cattle release ammonia into environment.
some chemical elements that are used by the industry
3. Poultry industry releases reactive nitrogen
in the manufacture of various products. Why?
compounds into environment.
Which of the statements given above is/are correct? 1. They can accumulate in the environment, and
(a) 1 and 3 only contaminate water and soil.
(b) 2 and 3 only 2. They can enter food chains.
(c) 2 only 3. They can trigger the production of free radicals.
(d) 1, 2 and 3
Select the correct answer using codes given below.
I.A.S. (Pre) 2019
Ans. (d) (a) 1 and 2 only
(b) 3 only
(c) 1 and 3 only
(d) 1, 2 and 3
I.A.S. (Pre) 2014
Ans. (d)
Nanoparticles pollute water, soil and air. After being received
by bacteria, the whole food chain can get affected. Nano-
particles of zin coxide and titanium dioxide produce free
radicals. Hence, statements (1), (2) and (3) are correct.

Environment and Ecology General Studies F–215


Water Conservation
Water Resources *Chlorine gas is used in the primary stage of purification of
water to deactivate the bacteria.
*Biotic resources are obtained from the living and organic
*Water purification is also done by ultraviolet radiation.
material such as forests and animals and the materials that can
Ultraviolet radiation is a type of electromagnetic radiation.
be obtained from them. Fossil fuels such as coal and petroleum
are also included in this category because they are formed from In the ultraviolet radiation system, water is made insect free
decayed organic matter. without the use of any chemical. There is no change in the taste,
*Pure water is not abiotic resource. It is found naturally on smell and colour of water by the use of UV. rays.
Earth. There are various techniques used for the conservation of
the water because drinking water found on the earth is available
in very less amount.
*World Water Day is an annual event celebrated on March
22nd. The day focuses attention on the importance of freshwater
and advocates for the sustainable management of freshwater
resources.
*The theme of World Water Day, 2022 is "Groundwater :
Making the invisible visible".
*The First World Water Day designated by United Nations was
celebrated in 1993. This decision was taken by United Nations
conservation of Environment and Development for fresh water.
*UN water programme has been continued since the year
2003. This programme focuses on the issues related to clean
water resources, cleanliness and water-related disasters.
*Besides this there are various important personalities also
who gave their contribution in water conservation and in
environmental protection. Rajendra Singh is well known as
'Waterman of India'. He runs an NGO (non-governmental
organization) named as 'Tarun Bharat Sangh' (TBS). He
won the Stockholm Water Prize in 2015 and Raman Magsaysay
Award in 2001.
*It is noteworthy that Arsenic pollutant is maximum in the
Water Pollution and Prevention drinking water of Kolkata. According to a study by Jadavpur
*Water pollution is a severe problem in the modern era. Various University, out of the 141 wards, almost 77 wards has a higher
techniques can be used for making water bacteria free. e.g. concentration of arsenic pollutants in the water. Water-related
(i) By temperature and other physical factors. necessities of the urban region can be completed by recycling
(ii) By ultraviolet rays and radioactive ions. of wastewater.
(iii) Through oxidation by chemicals e.g. compound of Ozone, *National Water Mission can play a vital role in the
iodine and chlorine. management of wastewater.
*To make water bacteria free chloramine (NH2Cl) and chlorine *It is notable that plan to connect the rivers is not included
dioxide (ClO2) is also used. Chlorine gas and bleaching powder under the National water mission.
are used to make drinking water free from bacteria. *It is notable that National Action Plan for climate change has

F–216 General Studies Environment and Ecology


been started since June 30, 2008. National water mission is a in the National Lake Conservation Plan (NLCP) are-
part of this action plan. This mission help will in integrated
Lake Related place and their State
water resource management. It helps well in water
Srinagar (Jammu and
conservation, equitable distribution of water and to reduce the Dal Lake
Kashmir)
wastage of water. National water mission will help in making
Channa Patna lake Hasan (Karnataka)
a strategy for the attainment of five major goals.
Bellandur Lake Bengaluru (Karnataka)
Sagar, Rani Talab, Shivpuri
Madhya Pradesh
Lake
Powai lake Maharashtra
Pushkar lake Ajmer (Rajasthan)
Lake Pichola Udaipur (Rajasthan)
Nakki lake Mount Abu (Rajasthan)
Ooty, Kodaikanal Tamil Nadu
Bhimtal, Khurpatal, Nainital,
Uttarakhand
Naukuchia Taal
Lakshmi Tal Jhansi
Mansi Ganga lake Govardhan (Uttar Pradesh)
*The Water (Prevention and Control of Pollution) Act Ramgarh Tal Gorakhpur (Uttar Pradesh)
was enacted in 1974 to provide for the prevention and control Veli Akkulam Thiruvanantpuram (Keral)
of water pollution and for maintaining and restoring the Twin Lakes Nagaland
wholesomeness of water in the country. The Act was amended
Bindusagar Lake Odisha
in 1988.
Lakshmi Narayan Bari Lake Tripura
*It is notable that the Water (Prevention and Control of
Rabindra Sarovar Kolkata (West Bengal)
Pollution) Act was enforced on 7th December, 1977. It
extended to whole of India including Union Territory of Jammu Mirik Lake Darjeeling (West Bengal)
and Kashmir from the date of 31 October, 2019. Moti Lake Bihar

Water Conservation *The objective of the scheme is to restore and conserve the
*Water conservation is not only a concern to humans only but urban and semi-urban lakes of the country degraded due to
plants also adopt a number of processes for the conservation wastewater discharge into the lakes and other reasons, through
of water. Leaves of plant found in the desert changes itself into an integrated ecosystem approach.
spines for the conservation of water. *The Sukhna lake in Chandigarh, Bhoj in Madhya Pradesh
*The water found in lakes and river are useful for human and Pichola Lake in Udaipur, Rajasthan has been declared a
culture and animals. So the conservation of these water bodies wetland covered under National Lake Conservation Plan.
is very important. In this context, a centrally sponsored scheme Chilika Lake in Odisha is not covered under NLCP list.
National Lake Conservation Plan (NCCP) was started by the
Ganga Conservation
Central Forest and Environment Ministry in June 2001.
*The Cabinet Committee on Economic Affairs (CCEA) in their *Ganga is the most important river system of India. Various
meeting held on 7th February, 2013 has approved the proposal steps have been taken from time to time to make Ganga
for merger of National Lake Conservation Plan and National pollution free.
Wetlands Conservation Programme into a new scheme named *On 4 November, 2008 the Central Government declared Ganga
National Plan for the Conservation of Aquatic Ecosystem as National River.
(NPCA). Till March 2021, list of some important lakes included *The 'Central Ganga Authority' was constituted in February,

Environment and Ecology General Studies F–217


1985 under the aegis of former Prime Minister Rajiv Gandhi. *There are various provisions under the program :
In September, 1995 its name was changed to "National River 1. Nirmal Dhara
Conservation Authority" (NRCA) with Prime Minister as 2. Ecological entity
its Chairman. 3. Geographic entity
*The Objective of NRCP (National River Conservation 4. Aviral Dhara
Plan) is to improve the water quality of rivers, through the *It is notable that the first phase of Ganga Action Plan was
implementation of pollution abatement works. started in the year 1985.
*The activities under NRCP include the following : *Yamuna Action Plan and 'Gomti Action Plan' were approved
(i) Interception and Diversion work to capture the raw in the second phase of Ganga Action Plan in year 1993.
sewage flowing into the river through open drains and *First phase of Yamuna Action Plan has been executed from
divert them for treatment. April, 1993 to the year 2003.
(ii) Sewage treatment plants for treating the diverted sewage. The second phase of Yamuna Action Plan was started in
(iii) Low-cost sanitation works to prevent open defecation on December, 2004. The third phase of Yamuna Action Plan has
riverbanks. been started on May 7, 2016 under the 'Namami Gange' scheme.
(iv) Electric crematorium and improved wood crematoria to
conserve the use of wood and help in ensuring proper
cremation of bodies brought to the ghats.
(v) Public awareness and public participation etc.
It is notable that all these activities have not attained the
expected success.
*National Ganga River Basin Authority (NGRBA) was
established by the Central Government of India, on 20 February,
2009 under Environment Protection Act, 1986. The Prime
Minister is its Chairman. The Chief Minister of the states
through which the Ganges flows are its members.
*It is financing, planning, implementing, monitoring and
co-ordinating authority of Ganges river.
*The objective of the Authority was to ensure effective
abatement of pollution and conservation of river Ganga by
the year 2020.
*It is remarkable that Union Ministry of Water Resources,
River Development and Ganga Rejuvenation on 22 July, 2016
*It is notable that 'Maili Se Nirmal Yamuna Revitalisation
constituted a committee to prepare guidelines for distillation
Project' has been enforced from the year 2017. It will play a
of river Ganga.
major role in cleaning of Yamuna.
*Madhav Chitale, an expert member of National Ganga River
*It is notable that Ganges Dolphin is one among the four
Basin Authority (NGRBA) has been appointed as a Chairman
freshwater dolphins in the world. They are found in Ganga and
of the Committee.
Brahmaputra and their adjoining rivers.
*Union Minister Arun Jaitley in the Budget of 2014-15 has
*Government of India declared it as the National aquatic
announced an integrated programme for the conservation of
animal in 2009.
Ganga named "Namami Gange" with a outlay of Rs. 2037
*They are mostly found in deep water and meeting point of
crore. He also declared allocation of Rs. 100 crore to set aside
rivers.
for ghat development and beautification of the river- front at
*According to IUCN approximately 1200-1400 dolphins exists
Kedarnath, Haridwar, Kanpur, Varanasi, Allahabad, Patna and
Delhi. presently.

F–218 General Studies Environment and Ecology


*According to IUCN Red List status of Gharial was in critically 5. In the Union Budget, 2014 the Integrated Ganga
endangered category and the crocodile was placed in the Conservation Mission has been called as :
vulnerable category. The scientific name of Ganges Dolphin (a) Clean Ganga
is Plastanista Gangetica. It is a mammal. (b) Sacred Ganga
(c) The Great Ganga
(d) Namami Gange
Basic Questions Uttarakhand P.C.S. (Pre) 2012
1. The 'World Water Day' is celebrated on: Ans. (d)
(a) 28 February (b) 22 March Then Union Finance Minister Arun Jaitley in Union budget
(c) 5 June (d) 11 July 2014-2015 announced Namami Gange Programme- an
U.P.P.C.S (Pre) 2011 integrated conservation mission, as Flagship Programme
Ans. (b) with budget outlay of 20,000 crores. The objective of
this programme was an effective abatement of pollution,
World Water Day is an annual event celebrated on March 22.
conservation and rejuvenation of National River Ganga. Main
The day focuses attention on the importance of fresh water
pillars of the Namami Gange Programme are
and advocates for the sustainable management of freshwater
resources. The theme of World Water Day 2019 was "leaving – Sewage Treatment Infrastructure
no one behind". The theme of World Water Day, 2022 is – River Front Development
Groundwater : making the invisible visible.
– River Surface Cleaning
2. Which one of the following is not a biotic resource?
– Bio-diversity
(a) Freshwater (b) Coal
(c) Petroleum (d) Fish – Afforestation
U.P.R.O./A.R.O. (Pre) 2016 – Public Awareness
Ans. (a) – Industrial Effluent Monitoring
Biotic resources are obtained from the living and organic – Ganga Gram
material such as forests and animals and the materials that 6. National Ganga River Basin Authority has been
can be obtained from them. Fossil fuels such as coal and constituted in –
petroleum are also included in this category because they
(a) October, 2008
are formed from decayed organic matter.
(b) February, 2009
3. In which of the following years was constituted the (c) October, 2009
‘Central Ganga Authority’ by Government of India? (d) March, 2010
(a) 1975 (b) 1982 U.P.P.C.S. (Mains) 2010
(c) 1985 (d) 1995 Ans. (b)
U.P.P.C.S. (Mains) 2014
Ans. (c) The Government of India has given Ganga the status of
National River and has constituted the National Ganga River
The 'Central Ganga Authority' was constituted in February, Basin Authority (NGRBA) on 20th February 2009 under
1985 under the aegis of former Prime Minister Rajiv Gandhi. section 3(3) of the Environment Protection Act, 1986.
In September, 1995 its name was changed to "National River
Conservation Authority." 7. In the year 2009, India set-up the following to clean
the Ganga :
4. When was the ‘Central Ganga Authority’ established
(a) National Ganga Commission
in India?
(b) Clean Ganga Authority
(a) 1998 (b) 1972
(c) National Ganga River Basin Authority
(c) 2001 (d) 1985
(d) Natinoal Mission for clean Ganga River
M.P.P.C.S. (Pre) 2016
(e) Ganga Seva Mission
Ans. (d)
Chhattisgarh P.C.S. (Pre) 2013
See the explanation of the above question. Ans. (c)

Environment and Ecology General Studies F–219


See the explanation of the above question. for the decline in the population of Ganges River
Dolphins?
8. Who among the following environmentalists is known 1. Construction of dams and barrages on rivers
by the name of ‘Jal Purush’? 2. Increase in the population of crocodiles in rivers
(a) Sunderlal Bahuguna 3. Getting trapped in fishing nets accidentally
(b) Rajendra Singh 4. Use of synthetic fertilizers and other agricultural
(c) B. Venkateshwarlu chemicals in crop-fields in the vicinity of rivers
(d) Salim Ali Select the correct answer using the code given below :
M.P.P.C.S. (Pre) 2016 (a) 1 and 2 only
Ans. (b) (b) 2 and 3 only
Rajendra Singh is a well-known water conservationist from (c) 1, 3 and 4 only
Alwar district, Rajasthan also known as ‘waterman of India’. (d) 1, 2, 3 and 4
He won the Stockholm Water Prize, an award as in 2015. I.A.S. (Pre) 2014
Previously he won the Ramon Magsaysay Award 2001 for Ans. (c)
his pioneering work in community-based efforts in water
The population of the Ganga river Dolphin has declined
harvesting and water management.
due to the construction of dams and water pollution caused
9. Which one of the following gases is used for the by pesticides, fertilizers and industrial effluents. The
purification of drinking water? threatened ecosystem is posing a danger to the existence
(a) Sulphur dioxide of freshwater dolphins. The Ganga river dolphins inhabits
(b) Chlorine Ganges–Brahmaputra–Meghna and Karnaphuli– Sangu river
(c) Fluorine systems of Nepal, India and Bangladesh. As per the reports,
these Dolphins often get trapped in fishing nets accidentally
(d) Carbon dioxide
which puts them in danger. Industrial actions and intensive
U.P.P.C.S. (Pre) 2013
agriculture along the coast of the river have introduced new
Ans. (b) threats to the dolphins. Use of pesticides and fertilisers have
Chlorine is used for the purification of drinking water. impacted the freshwater biodiversity. This dolphins is among
10. Which of the following leaf modifications occurs/occur the four 'obligate' freshwater dolphins in the world.
in desert areas to inhibit water loss?
1. Hard and waxy leaves High Level Questions
2. Tiny leaves or no leaves
3. Thorns instead of leaves 1. If the National Water Mission is properly and
Select the correct answer using the code given below : completely implemented, how will it impact the
(a) 1 and 2 country?
(b) Only 2 1. Part of the water needs of urban areas will be met
(c) 1 and 3 through recycling of waste-water.
(d) All of these 2. The water requirements of coastal cities with
I.A.S. (Pre) 2013 inadequate alternative sources of water will be met
Ans. (d) by adopting appropriate technologies that allow
the use of ocean water.
Due to the scarcity of water in the desert, in some plants,
leaves are modified to reduce water loss. Hard and waxy 3. All the rivers of Himalayan origin will be linked to
leaves help to minimise water loss through diffusion. Water the rivers of peninsular India.
loss is also prevented in the form of gummy substance in 4. The expenses incurred by farmers for digging bore-
leaves minimising the surface area to volume ratio. Thorns wells and for installing motors and pump-sets to
have a high surface area but they help in water conservation. draw groundwater will be completely reimbursed
They also help in condensing moisture from the air which by the Government.
then drips to the ground and is absorbed by the roots. Select the correct answer using codes given below :
11. Other than poaching, what are the possible reasons (a) Only 1

F–220 General Studies Environment and Ecology


(b) 1 and 2 (a) 1 and 2 only (b) 3 only
(c) 3 and 4 (c) 1 and 3 only (d) 1, 2 and 3
(d) 1, 2, 3 and 4 I.A.S. (Pre) 2018
I.A.S. (Pre) 2012 Ans. (b)
Ans. (b)
Soil water holding capacity is the amount of water that a
The main objective of the National Water Mission is
given soil can hold for crop use. Soil texture and organic
"conservation of water, minimizing wastage and ensuring
matter are the key components that determines soil water
its more equitable distribution both across and within
holding capacity. Organic matter influences the physical
States through integrated water resources development
conditions of soil in several ways. Plant residues that cover
and management". National Water Mission is a part of the
the soil surface, prevent the soil from sealing and crusting by
National Action Plan on Climate Change which was started
raindrop impact, thereby enhancing rainwater infiltration and
on 30 June, 2008. The five identified goals of the mission
reducing runoff organic matter improves the water holding
are (i) Comprehensive water database in public domain
capacity of the soil.
and assessment of impact of climate change on water
resource (ii) Promotion of citizen and state action for water Sulphur is one of the three nutrients that are cycled between
conservation augmentation and preservation (iii) Focused the soil, living matter and the atmosphere.
attention to vulnerable areas including over-exploited areas Salinization is a major problem associated with irrigation
(iv) Increasing water use efficiency by 20% because deposits of salts build up in the soil and can reach
(v) Promotion of basin level integrated water resource levels that are harmful to crops.
management. 4. Consider the following statements :
Hence (b) is the correct answer. 1. 36% of India’s districts are classified as
2. Which of the following is/are the possible consequence/s ‘‘overexploited’’ or ‘‘critical’’ by the Central
of heavy sand mining in riverbeds? Ground Water Authority (CGWA).
1. Decreased salinity in the river 2. CGWA was formed under the Environment
2. Pollution of groundwater (Protection) Act.
3. Lowering of the water-table 3. India has the largest area under groundwater
Select the correct answer using codes given below :
irrigation in the world.
(a) 1 only (b) 2 and 3 only
Which of the statements given below is/are correct?
(c) 1 and 3 only (d) 1, 2 and 3
(a) 1 only
I.A.S. (Pre) 2018
(b) 2 and 3 only
Ans. (b)
(c) 2 only
The removal of sand from the river bed increases the velocity (d) 1 and 3 only
of the flowing water with the distorted flow-regime eventually
I.A.S. (Pre.) 2020
eroding the river banks. Sands acts like a sponge which helps
Ans. (b)
in recharging the water table. Its progressive depletion in the
river is accompanied by declining water tables in the nearby CGWA (Central Ground Water Authority) was formed
areas. The sand acts as an efficient filter for various pollutants under the Environment (Protection) Act, 1986 to regulate
and thus maintains the quality of water in rivers and another and control development and management of ground water
aquatic ecosystem. resources in the country. India has the largest area under
ground water irrigation in the world followed by China and
3. With reference to agricultural soils, consider the
the USA. So, option (b) is correct answer.
following statements :
1. High content of organic matter in soil drastically 5. Water (Prevention and Control of Pollution) Cess was
reduces its water holding capacity. enforced in –
2. Soil does not play any role in the sulphur cycle. (a) 1973 (b) 1975
3. Irrigation over a period of time can contribute to (c) 1977 (d) 1979
the salinization of some agricultural lands. U.P.R.O/A.R.O. (Mains) 2013
Which of the statements given above is / are correct? Ans. (*)

Environment and Ecology General Studies F–221


The Water cess Act was enacted in 1977. But it was enforced 3. It quickens the sedimentation of solid particles,
removes turbidity and improves the clarity of
in 1978.
water Which of the statements given above is/are
6. In which year was the Water Pollution Prevention and correct?
Control Act enforced? (a) Only 1
(a) 1980 (b) 2 and 3
(b) 1974 (c) 1 and 3
(c) 1981 (d) 1, 2 and 3
(d) None of the above I.A.S. (Pre) 2012
M.P.P.C.S. (Pre) 2017 Ans. (a)
Ans. (b)
Ultra-violet water purification is the most effective method
The Water (Prevention and Control of Pollution) Act was for disinfecting bacteria from the water. Ultra-violet rays
enacted in 1974 to provide for the prevention and control penetrate harmful pathogens in water and destroy illness
of water pollution and for maintaining or restoring the causing micro-organisms. This is extremely efficient in
wholesomeness of water in the country. The Act was amended eliminating their ability to reproduce. UV systems destroy
in 1988. 99.99% of harmful micro-organism without adding chemicals
7. In which year Central Water Commission established or changing water's taste or odour. Hence, option (a) is the
"Irrigation Research and Management Organization"? correct answer.
(a) 1984 10. Which of the following is/are used as a disinfectant of
(b) 1948 water?
(c) 1964 (a) Ozone
(d) 2004 (b) Chlorine dioxide
(e) None of these (c) Chloramine
Chhattisgarh P.C.S. (Pre) 2016 (d) All of the above
Ans. (a) U.P.P.C.S. (Pre) 2016
In 1984, the Central Water Commission established "Irrigation Ans. (d)
Research and Management Organization". The answer to this Water purification is the process of removing undesirable
question in the answer sheet issued by Chhattisgarh Public chemicals, biological contaminants, suspended solids and
Service Commission was option (a) while the option (e) was gases from water. Water filtration can be done in many ways
considered as the correct answer in the revised answer sheet like- through temperature, ultraviolet rays and radioactive
whereas option (a) is the correct answer. ions, oxidation of chemicals using a compound of ozone,
8. The arsenic contamination of drinking water is highest Iodine and Chlorine. Cloramine (NH2Cl) and Chlorine
in– dioxide (ClO2) are also used for water purification.
(a) Chennai
11. Greenwash refers to
(b) Kanpur
(a) Promoting Jhum agriculture
(c) Kolkata
(b) Cleaning of river water
(d) Mumbai
(c) Removing greenery
U.P.P.C.S. (Pre) 2005
(d) Making false promises of protecting the environment
Ans. (c)
U.P. P.C.S. (Mains) 2017
The arsenic contamination of drinking water is highest in Ans. (d)
Kolkata, West Bengal. Greenwashing is the practice of making an unsubstantiated
9. What is the role of Ultra-violet (UV) radiation in the or misleading claim about the environmental benefit
of a product, service technology or company practice.
water purification systems?
Greenwashing can make a company appear to be more
1. It inactivates, kills the harmful microorganisms in
environment friendly than it really is.
water.
2. It removes all the undesirable odours from the 12. The Yamuna Action Plan was formally launched in
water. (a) 1991

F–222 General Studies Environment and Ecology


(b) 1992 (a) 1, 2 and 3
(c) 1993 (b) 2, 3 and 4
(d) 1994 (c) 1, 3 and 4
U.P.P.C.S. (Mains) 2016 (d) 1, 2 and 4
Ans. (c) U.P.P.C.S (Pre) 2002
Ans. (d)
Yamuna Action Plan (YAP) to clean the dirtiest river in
the country was formally launched in 1993. It is a bilateral National Lake Conservation Plan (NLCP) is a centrally
project between the Government of India and Japan. The YAP sponsored scheme exclusively aimed at restoring the water
has so far completed two phases as YAP-1 and YAP-II. The quality and ecology of the lakes in an urban and semi-urban
YAP-I covered Delhi, eight towns in Uttar Pradesh and six area of the country. The objective of the scheme is to restore
towns in Haryana. Under YAP II, the emphasis was on the and conserve the urban and semi-urban lakes of the country
22 km stretch of the Yamuna in Delhi. Notably, YAP III was degraded due to wastewater discharge into the lakes and
launched on 7 May, 2016 under Namami Gange Programme. other reasons, through an integrated ecosystem approach.
The Sukhna Lake in Chandigarh, Bhoj in Madhya Pradesh
13. Which of the following are the key features of ‘National
and Pichola Lake in Udaipur, Rajasthan have been declared
Ganga River Basin Authority (NGRBA)’?
a Wetland covered under National Lake Conservation Plan.
1. River basin is the unit of planning and management. Chilika lake in Odisha is not covered under NLCP list.
2. It spearheads the river conservation efforts at the
15. Which of the following has/have shrunk immensely/
national level.
dried up in the recent past due to human activities?
3. One of the Chief Ministers of the States through
which the Ganga flows becomes the Chairman of 1. Aral Sea
NGRBA on a rotational basis. 2. Black Sea
Select the correct answer using codes given below: 3. Lake Baikal
(a) 1 and 2 only Select the correct answer using the code given below :
(b) 2 and 3 only (a) 1 only
(c) 1 and 3 only (b) 2 and 3
(d) 1, 2 and 3 (c) 2 only
I.A.S. (Pre) 2016 (d) 1 and 3
Ans. (a) I.A.S. (Pre) 2018

The National Ganga River Basin Authority (NGRBA) Ans. (a)


is planning, financing, monitoring and coordinating The Aral Sea is a saline lake located in Central Asia that was
body. The objective of NGRBA is to ensure effective once the world's 4th largest salt lake. The Aral sea which was
abatement of pollution and conservation of the river Ganga once 26,300 miles square in size had decreased in size by
by adopting a river basin approach for comprehensive 2007 to only 10% of its original area.
planning and management. Hence statement 1 is correct.
The program of river cleaning was extended to other 16. Which one of the following lakes have been recently
major rivers of the country under two separate schemes. included under National Lake Conservation Project?
Hence statement 2 is correct. The Apex body of NGRBA (a) Bhimtal
is headed by the Prime Minister for Policy decisions. (b) Pulicat
14. The wetlands falling within urban areas which are (c) Ooty
being dealt with under the National Lake Conservation (d) Sambhar
Plan with the aim of Pollution control are – U.P.P.C.S (Pre) 2010
1. Bhoj - Madhya Pardesh Ans. (a)
2. Sukhana - Chandigarh National Lake Conservation Plan was launched in June,
3. Chilka - Orissa 2001 as a centrally sponsored scheme. This plan included
4. Pichola - Rajasthan 58 lakes of 14 states by December, 2009. The list included
Select the correct answer using codes given below: Bhimtal and Ooty. Since Ooty was a part of the project from
Codes : the begininng, it is Bhimtal which was included recently.

Environment and Ecology General Studies F–223


Miscellaneous
Agenda 21 and other Conventions the year 1935 by Ministry of Agriculture and Farmers Welfare
of India as a agricultural assistance and serve as a subsidiary
*Agenda 21 is legally non-binding action plan of the United
department to Farmers Welfare.
Nations with regard to sustainable development. *Its aim is
to conserve World's environment in 21st Century. *This plan 'Organizations and Institutions'
was prepared in Rio-de-Janeiro, Brazil in the year 1992
*'Southern Gangotri' is the first research centre of India in
during an event on UNCED (United Nations Conference
Antarctica. *It was established in year 1983-84.
on Environment and Development).*This is a 351 page
*In Antarctica India's second research centre 'Maitri' was
document in which there are 40 chapters being categorized
established in the year 1988-89.
into four groups.
*In Antarctica India's third research centre is 'Bharati'.
*Organization for the Prohibition of Chemical Weapons–
*'Bharati' was established in the year 2012.
(OPCW) is an implementing body for the Chemical Weapons
*'National Institute of Nutrition' is situated in Hyderabad.
Convention.* It came into force in the year 1997. * Currently
*'Wildlife Institute of India' is situated in Dehradun.
there are 193 countries in OPCW, which is determined for
*'National Institute of Ayurveda' is situated in Jaipur.
making the world free from chemical weapons. *It provides *'National Institute of Naturopathy' is situated in Pune.
assistance and protection to States (parties) against the threat of *'National Institute of Forest Management' is situated in
chemical weapons. Its Headquarters is situated in The Hague, Bhopal.
Netherlands. *Central Arid Zone Research Institute (CAZRI) is situated
*The organization was awarded the 2013 Nobel Peace Prize for in Jodhpur, Rajasthan.
its extensive efforts to eliminate chemical weapons. *'Indian Institute of Ecology and Environment' is situated
*It is notable that the AGMARK Act or Agricultural Produce in New Delhi.
Grading and Marking Act came into force in year 1937.
*Bureau of Indian Standards : BIS is a National Standards 'Various Awards'
Organization of India. *To make citizens aware of consciousness towards environment
*In India Industrial development, consumer protection and various awards are given from time to time.
formation of standards according to an organized process in *Indira Gandhi Paryavaran Puraskar is an environment
daily life started with the establishment of Indian Standards award instituted by the Ministry of Environment and Forest,
Institution: ISI in January, 1947. Government of India in the year 1987 in order to encourage
*Bureau of Indian Standards (BIS) was established by the BIS public participation in environment. It is awarded to individuals
Act of 1986. This act came into force in 1987. and organizations that have made a significant and measurable
*The aim of this organization is standardization, symbolization contribution in the field of environment protection and
of objects and to provide a quality certificate. improvement.
*ISI mark is the main symbol provided by Bureau of Indian *'Tyler Prize' is provided for exceptional contribution in the
Standards.* This is often labelled on every usable processed field of environment protection and ecology. This prize is also
food products with products of daily use like electric equipment, known as 'Nobel Prize for Environments'.
cement, iron pipes etc.
*In April, 2015, this award was provided to Indian
*'AGMARK' is a quality certificate symbol generated by
environmentalist Madhav Gadgil and Jane Lubchenco in
Directorate of Marketing and Inspection of India.
Los Angeles California (America).
*Directorate of Marketing and Inspection categorize and *In a year 2016 this award was provided to Partha Dasgupta
symbolizes food products. *This department was established in of Cambridge University.

F–224 General Studies Environment and Ecology


*In the year 2017 Tyler Award was given to Ecologist Professor
World Environment Day 5 June
Jose Sarukhan Kermez of Mexico.
World Forest Day 21 March
*In the year 2018 Tyler Award was provided to two American
Scientists James J. McCarthy and Paul Falkowski. The award World Habitat Day First Monday of October
was given to them to study the effects of climate change. World Ozone Day 16 September
*Michael Mann, distinguish professor of atmospheric science,
Earth Day 22 April
Penn State, has been awarded the 2019 Tyler Prize. Tyler Prize
World Toilet Day 19 November
2020 was awarded to Pavan Sukhdev and Gretchen C. Daily.
*'Rajiv Gandhi Environment Award' is awarded for work World Water Day 22 March
in the area of new technology and development by Ministry of World No Tobacco Day 31 May
Environment, Forest and Climate Change.
World Wildlife Day 3 March
*'Global 500 Award' was launched in 1987 by the United
Nations Environment Programme (UNEP) to recognize and *On 2nd October, 2014, Prime Minister Narendra Modi officially
honour environmental achievement. launched 'Swachh Bharat Abhiyaan' on the occasion of
*This award was a tribute to success on the front lines Gandhi Jayanti.*The aim of this mission was to complete the
dream of Mahatma Gandhi of 'Clean India (Swachh Bharat)
of environmental action and honoured individuals and
on his 150th Anniversary till 2019.
organizations that combat environmental issues despite
*In Sierra Leone last Saturday of every month is celebrated as
immense social, political and logistical obstacles.
National Cleanliness Day.
*In 2003, India's Dr. Bindeshwar Pathak (founder of Sulabh
*It is noteworthy that 5 June is observed every year as 'World
International) was awarded.
Environment Day'. The theme for 2022 is 'Only One Earth'.
*Later on at the place of this award 'Champions of the Earth' *International Earth Day is celebrated on 22nd April every
award was given from the year 2005. year. The theme for the Day of 2022 is Invest in Our Planet.
*'Champions of the Earth' Award achievers for the year 2013
include American Virbhadran Ramanathan of Indian origin.
Various Facts Regarding
*'Champions of the Earth' Award achievers for the year 2016
Environment
include Mumbai's Afroz Shah. *Sun temple of Konark was included as a World Heritage
Site in 1984 while Mahabodhi Temple complex at Bodh Gaya
'Days and Dates' Bihar in 2002.
*World No Tobacco Day is observed on 31st May every year Currently, there are 40 World Heritage Sites in India that are
by the United Nations. * Theme of World No Tobacco Day, recognized by the UNESCO. Out of these, 32 are cultural 7
2020 was 'Protecting Youth From Industry Manipulation are natural and 1 is mixed. The latest site to be inscribed is
and preventing them from Tobacco and Nicotine use'. Dholavira Gujarat in 2021. Maharashtra is having highest
Theme of World No Tobacco Day, 2022 is "Tobacco Threat to number of sites.
our Environment". *The Tyler Prize for Environment Achievement is the premier
*19th November is observed every year as World Toilet award for environmental science, environmental health and
Day. The resolution declaring the Day titled 'Sanitation for energy conferring, a great benefit upon mankind. For 2016,
All' (AIRES/67/291) was adopted on 24 July, 2013 and urged this prize was given to Gretcher Daily and Pavan Sukhdev.
UN member States and relevant stakeholders to encourage *Medha Patekar is related with Narmada Bachao Andolan.*Rally
behavioural change and the implementation of policies to for Valley programme is organized for rehabilitating people
increase access to sanitation among the poor, along with a call facing the problem of rehabilitation in Narmada Valley.* Baba
to end the practice of open-air defecation. *The theme of World Amte is related to the Wildlife Protection Act and Narmada
Toilet Day, 2021 was "Valuing Toilets". Bachao Andolan.
*On the recommendations of Madhya Pradesh WildLife Board
Important Days Dates the National Dinosaur Fossil Park is being established in the

Environment and Ecology General Studies F–225


Dhar District of Madhya Pradesh. The Park will be near Bagh roots, insects, fish, frog, etc.
Caves in Dhar Forest Division spreading over 89.74 hectares. *Primary productivity is the rate at which energy is converted
It is important to note that in the year 2006, more than 100 by photosynthetic and chemosynthetic autotrophs to organic
Dinosaur eggs were discovered in Dhar district. They were substances. The total amount of productivity in a region or
estimated to be 65 to 70 million years old. system is gross primary productivity.
*The National Mission for Green India (GIM) is one of *Primary productivity of temperate grassland is approx
the eight missions outlined in the National Action Plan on 500gm per cm2 per year. Primary productivity of temperate
Climate Change (NAPCC). It aims at protecting, restoring and forest is approx 1000 gm per cm2 per year.
enhancing India’s diminishing forest cover and responding to *Primary productivity of Tropical forests is approx 2000gm
climate change by a combination of adaptation and mitigation per cm2 per year and primary productivity of Tropical savanna
measures. is approx 700 gm per cm2 per year.
*The main aim of green accounting is a comprehensive *Marine National Park in Gulf of Kutch is situated on the
estimation of economic activities and their effects on the southern shore of the Gulf in the Jamnagar district of Gujarat.
environment.
It is the first national marine park of India with 42 islands.
*The United Nations General Assembly designated the
*Bhitarkanika is a location of rich, lush green vibrant eco-
1990-99 as the International Decade for Natural Disaster
system lying in the estuarine region of Brahmani- Baitarani
Reduction. The basic objective was to decrease the loss of
rivers in the north-eastern corner of Kendrapara district of
life, property destruction and social and economic disruption
Odisha.
caused by natural disasters such as earthquakes, tsunamis,
*Prosopis Juliflora is an exotic plant that wipes out most of
floods, landslides, volcanic eruptions, droughts and other
the native plants and animals that room around it once. It tends
disasters of natural origin. The United Nations designated
to reduce the biodiversity in the area in which it grows.
2021-2030 as United Nations Decade on Ecosystem
*Namdapha, Bandipur, Periyar and Lamjao Sanctuary are
Restoration.
respectively located in Arunachal Pradesh, Karnataka, Kerala
*National Center for Disaster Mangement (NCDM) is
and Manipur.
located in New Delhi. It was founded in 1995.
*SODAR (Sonic Detection and Ranging) is a meteorological
*The National Disaster Management Institute was
established under the Disaster Management Act, 2005. instrument used as a wind profiler to measure the scattering of
*Mega-Divers is a group of those countries where maximum sound waves by atmospheric turbulence.
species are found on earth, due to which these countries are *SODAR systems are used to measure wind speed at various
known as excessive bio-diverse. heights above the ground.
*American non-profit organization related to the environment, *In India, SODAR is established at Kaiga, Kalpakkam,
Conservation International in year 1998 identified 17 mega- Tarapur and Trombay.
divers nation in which countries like Australia, China, South- *Important days and themes:
Africa, America and India are included. Days Theme
*Loquat is an evergreen tree. In Japan, Loquat is excessively
Wetlands Action for People
produced. World Wetlands Day, 2022
and Nature
*Dr. Salim Ali was an Indian ornithologist and naturalist. He
World Environment Day,
is some times referred to as 'Birdman of India'. In his book Only One Earth
2022
"Handbook of the birds of India and Pakistan Vol. 5", he Groundwater, making the
wrote that common Myna stalking alongside the cattle seizes World Water Day, 2022
invisible, visible
the insects disturbed by their movement through grasses.
World Toilet Day, 2022 Valuing Toilets
*Painted stork is found in the southern Himalayas and feed
on small fishes near wetlands. International Earth Day, 2022 Invest in Our Planet
*Black-necked crane is found in Tibetan Plateau, Bhutan, International Forest Day, Forests and sustainable
Arunachal Pradesh and Ladakh. It is an omnivore and eat plants 2022 production and consumption

F–226 General Studies Environment and Ecology


(c) Patent protection
Basic Questions (d) Agricultural subsidies
U.P.R.O./A.R.O. (Pre) 2014
1. Which of the following is known as “Dakshin – Ans. (a)
Gangotri” ?
See the explanation of the above question.
(a) The place of origin of the river Kaveri
(b) The place where Periyar flows Northwards 5. "Momentum for Change: Climate Neutral Now" is an
(c) India’s first Antarctic Research Station initiative launched by:
(d) The rocket launching centre in Kerala (a) The Intergovernmental Panel on Climate Change
Uttrakhand U.D.A./L.D.A. (Mains) 2007 (b) The UNEP Secretariat
Ans. (c) (c) The UNFCCC Secretariat
Dakshin Gangotri is the first research base station of India (d) The World Meteorological Organization
situated in Antarctica. It is currently being used as a supply I.A.S. (Pre) 2018
base and transit camp. India's second research centre 'Maitree' Ans. (c)
was established in 1988-89. India's third research base The "Momentum for Change: Climate Neutral Now" is
'Bharati' was established in 2012. an initiative launched by the UNFCCC secretariat. It was
2. The name of India's third research station in launched as a part of larger efforts to showcase successful
Antarctica? climate action around the world.
(a) Bharati 6. Match List-I with List-II and select the correct answer
(b) Swagatam from codes given below :
(c) Hindustan List – I List – II
(d) Maitri A. Skin Cancer 1. Clorofluorocarbon
53rd to 55th B.P.S.C. (Pre) 2011 B. Noise Pollution 2. Ultraviolet Light
Ans. (a) C. Global Warming 3. Decibel
Bharati is an Antarctic research station commissioned by D. Ozone Hole 4. Carbon dioxide
India. It is India's third Antarctic research facility and one of Codes :
the two active research stations alongside Maitri. A B C D
(a) 1 2 3 4
3. Which one of the following is the correct definition of
(b) 2 3 4 1
“Agenda 21”?
(c) 2 3 1 4
(a) It is an action plan of U.N.O. for protecting human
rights. (d) 4 3 2 1
(b) It is a book of 21st chapters on nuclear disarmament. U.P. Lower Sub. (Pre) 2015
(c) It is an action plan for the conservation of the global Ans. (b)
environment in the 21st century. Below are the correct match between List-I and List-II :
(d) It is an agenda for the election of the President in the Skin Cancer - Ultraviolet Light
next meeting of SAARC. Noise Pollution - Decibel
U.P.P.C.S. (Pre) 2013 Global Warming - Carbon dioxide
Ans. (c) Ozone Hole - Chlorofluorocarbons

Agenda 21 is a legally non-binding, voluntarily implemented 7. In Uttar Pradesh, the first biotech park was established–
action plan of the United Nations with regard to sustainable (a) Agra (b) Allahabad
development. It is a product of Earth Summit (UN Conference (c) Lucknow (d) Varanasi
on Environment and Development) held in Rio-de-Janeiro, U.P.P.C.S (Mains) 2011
Brazil in 1992. Agenda 21 is a 350 page document divided U.P.P.C.S. (Mains) 2008
in to 40 chapters that have been grouped into 4 sections. U.P.P.C.S. (Mains) 2010
4. 'Agenda-21' is related to which field? U.P. U.D.A./L.D.A. (Mains) 2010
(a) Sustainable development U.P.P.C.S. (Mains) 2005
(b) Nuclear disarmament Ans. (c)

Environment and Ecology General Studies F–227


Biotechnology Park in Uttar Pradesh has been established in UPPCS (Pre) 2021
Lucknow. It was set up in 2002. Ans. (c)

8. Match List-I with List-II and select the correct answer To increase consumer awareness, the Government of India
from codes given below : launched the eco-labelling scheme known as 'Ecomark' in
List – I List – II 1991 for identification of environment-friendly products.
(Institute) (Place) Any product which is made, used or disposed of in a that
significantly reduces the harm it would otherwise cause
A. National Institute of Nutrition 1. Pune
the environment could considered Environment-Friendly
B. Wild Life Institute of India 2. Jaipur
Product.
C. National Institute of Ayurveda 3. Dehradun
D. National Institute of Naturopathy 4. Hyderabad 11. With Jaipur's 'Jantar Mantar' being granted World
Codes : Heritage site status by UNESCO, how many sites in
A B C D India till August 2010 have been granted this status?
(a) 1 2 3 4 (a) 29
(b) 2 3 4 1 (b) 28
(c) 3 4 1 2 (c) 27
(d) 4 3 2 1 (d) 26
U.P. Lower Sub. (Spl) (Pre) 2004 U.P. Lower Sub. (Pre) 2009
Ans. (d) Ans. (b)

The institutes with their correct location are given below: In 2010, by adding Jantar Mantar of Jaipur, a number of World
(Institute) (Place) Heritage Sites in India reached to 28. Currently, there are 40
National Institute of Nutrition Hyderabad World Heritage Sites in India.
Wild Life Institute of India Dehradun 12. The recently included building in the list of the world
National Institute of Ayurveda Jaipur heritage site list of UNESCO is –
National Institute of Naturopathy Pune (a) Buland Darwaza
9. Which among the following are India's permanent and (b) Char Minar
operational research stations in Antarctica? (c) Mahabodhi Temple
(a) Bharati and Arya (d) Sun Temple of Konark
(b) Bharati and Dakshin Gangotri U.P.P.C.S. (Mains) 2002
(c) Bharati and Maitri Ans. (c)
(d) Dakshin Gangotri and Maitri Sun temple of Konark was included as a World Heritage Site
UPPCS (Pre) 2021 in 1984 while Mahabodhi Temple complex at Bodh Gaya
Ans. (c) Bihar in 2002.

After a humble beginning, the Indian Antarctic programme 13. The "Kailash Sacred Landscape Conservation and
has now credited to have built three permanent research Development Initiative" (KSLCDI) is a collaboration
base stations in Antarctica-named Dakshin Gangotri, Maitri, among which three nations?
and Bharati. As of today, India has two operational research (a) India, Afghanistan, China
stations in Antarctica named Maitri and Bharati. (b) India, Nepal, Afghanistan
10. The symbol 'ECOMARC' related to which of the (c) India, Nepal, Pakistan
following? (d) India, China, Nepal
(a) Goods of best quality Uttarakhand P.C.S. (Pre) 2016
(b) Safe goods for environment Ans. (d)
(c) Exported goods The "Kailash Sacred Landscape Conservation and Development
(d) Imported goods

F–228 General Studies Environment and Ecology


Initiative" (KSLCDI) is a collaboration among India, China C. Central Arid Zone 3. Dehradun
and Nepal. Research Institute
D. Indian Agricultural 4. Jhansi
14. 'Vegetation is the true index of climate'. This statement Research Institute
is associated with Codes :
(a) Thornthwaite A B C D
(b) Koppen (a) 2 4 1 3
(c) Trewartha (b) 3 4 1 2
(d) Stamp (c) 1 3 2 4
U.P. P.C.S. (Pre) 2018 (d) 4 2 3 1
Ans. (b) U.P.P.C.S. (Pre) 2017
The statement 'Vegetation is the true index of climate' is Ans. (b)
associated with Koppen Climate Classification, a system The correct match is as follows -
first published in 1900 and revised in 1918 by Wladimir List-I List-II
Peter Koppen. (Institute) (City)
15. Tsunami is the result of – Forest Research Institute - Dehradun
(a) Shrinking of the Earth’s crust Indian Grassland and Fodder Research - Jhansi
(b) Cyclones Institute
(c) Submarine earthquakes Central Arid Zone Research Institute - Jodhpur
(d) Tides Indian Agricultural Research Institute - New Delhi
Jharkhand P.C.S. (Pre) 2013 18. The Indira Gandhi Paryavaran Puraskar is awarded
Ans. (c) every year for –
A tsunami is a series of large waves generated by an abrupt (a) Significant contribution to the field of environment
movement on the ocean floor that can result from an (b) Outstanding contribution in the field of afforestation
earthquake, an underwater landslide, a volcanic eruption or (c) Outstanding contribution in the field of waste land
very rarely a large meteorite strike. However, powerful development
undersea earthquakes are responsible for a most tsunami. (d) Exemplary work in the field of wildlife conservation
and research
16. Which one of the following is not correctly matched ?
U.P.P.C.S. (Spl) (Mains) 2004
(a) Centre for Ecological - Bengaluru
Ans. (a)
Science (CES)
(b) Wildlife Institute of India - Dehradun Indira Gandhi Paryavaran Puraskar is an environment
(c) Indian Institute of - Kolkata award instituted by the Ministry of Environment and
Forest Management Forest, Government of India in the year 1987 in order to
(d) G.B. Pant Institute of Himalayan - Almora encourage public participation in environment. It is awarded
Environment and Development to individuals and organizations that have made a significant
U.P.P.C.S.(Pre) 2012 and measurable contribution in the field of environment
Ans. (c) protection and improvement.
19. Which one of the following pairs is not matched
Indian Institute of Forest Management is situated in Bhopal
correctly?
not in Kolkata. Rest of the pairs are correctly matched.
(a) Appico Movement - P. Hegde
17. Match List-I with List-II and select the correct answer (b) Chipko Movement - S.L. Bahuguna
using codes given below the lists: (c) Sava Narmada Movement - Medha Patkar
List-I List-II (d) Silent Valley Movement - Baba Amte
(Institute) (City) U.P. Lower Sub. (Pre) 2009
A. Forest Research 1. Jodhpur Ans. (d)
Institute
B. Indian Grassland 2. New Delhi Baba Amte was related to wildlife conservation and Narmada
and Fodder Research Bachao Andolan and Dr. Salim Ali related with Silent Valley
Movement.
Institute

Environment and Ecology General Studies F–229


20. Medha Patkar is related to which movement? to provide adequate shelter for all.
(a) Chipko Movement 2. Its partners are either governments or local urban
(b) Narmada Bachao Andolan authorities only.
(c) Naxalite Movement 3. UN-Habitat contributes to the overall objective of
(d) Harm to agricultural land the United Nations system to reduce poverty and
M.P.P.C.S. (Pre) 1993 to promote access to safe drinking water and basic
Ans. (b) sanitation.
Select the correct answer using codes given below:
Medha Patkar was related with Narmada Bachao Andolan.
(a) 1, 2 and 3 (b) 1 and 3 only
21. What is the full form of NGT? (c) 2 and 3 only (d) 1 only
(a) National Green Tribunal I.A.S. (Pre) 2017
(b) National General Tribe Ans. (b)
(c) New General Tribunal
(d) National Green Tribe The United Nations Human Settlements Programme is the
United Nations agency for human settlements and sustainable
M.P. P.C.S. (Pre) 2018
urban development. It was established in 1978, at Nairobi
Ans. (a)
the capital of Kenya. This agency has been mandated by the
The full form of NGT is National Green Tribunal. The United Nations General Assembly to promote socially and
National Green Tribunal has been established on 18 environmentally sustainable towns and cities that provide
October, 2010 under the National Green Tribunal Act, equate shelter for all. Its partners include not only the
2010 for effective and expeditious disposal cases relating governments and the local municipal authority, but also the
to environmental protection and conservation of forests and personal and non-governmental partners. It contributes to the
other natural resources. overall objective of the United Nations to reduce poverty and
22. "Rally for Valley" programme in India was organized to promote access to safe drinking water and basic sanitation.
to highlight the problems of : 25. United Nations Conference on climate change, COP21
(a) Environment Degradation was held in
(b) Biodiversity (a) Moscow (b) Paris
(c) Resettlement (c) Berlin (d) Tokyo
(d) Harm to agricultural land U.P. P.C.S. (Pre) 2018
U.P.P.C.S. (Mains) 2003 Ans. (b)
Ans. (c)
The 2015 United Nations Climate Change Conference
Rally for Valley programme was related to the resettlement (COP21) was held in Paris. It was the 21st yearly session
of displaced persons of Narmada Valley. of Conference of Parties (COP) to the 1992 United Nations
23. The World Habitat Day is observed on: Framework Convention on Climate Change (UNFCC) and
the 11th session of the Meeting of the Parties to the 1997
(a) March 21
Kyoto Protocol. The COP 27 Summit will take place in
(b) March 23
Egypt's Sharm El Sheikh City.
(c) June 5
(d) October 5 26. The World Anti-Tobacco Day is Celebrated every year on–
U.P.P.C.S. (Pre) 2000 (a) 12th May (b) 13th May
Ans. (d) (c) 21 May
st
(d) 31st May
U.P.P.C.S. (Mains) 2008
World Habitat Day is observed every year on the first Monday
Ans. (d)
of October. It was officially designated by the United Nations
and first celebrated in 1986. World No Tobacco Day is observed around the world every
year on May 31. It is intended to encourage abstinence from
24. With reference to the role of UN-Habitat in the United
all forms of tobacco consumption around the globe.
Nations programme working towards a better urban
future, which of the statements is/are correct? 27. 19th November is observed as the
1. UN-Habitat has been mandated by the United (a) World Green Day
Nations General Assembly to promote socially (b) World Anti-Poverty Day
and environmentally sustainable towns and cities (c) World Toilet Day

F–230 General Studies Environment and Ecology


(d) World Clean Water Day The correct match of List-I with List-II is as follows -
U.P.P.C.S. (Pre) (Re. Exam) 2015 List-I List-II
Ans. (c) (World Days) (Date)
In 2013, the United Nations General Assembly officially World Earth Day 22 April
designated November, 19 as World Toilet Day. World Toilet World Cities Day 31 October
Day is coordinated by UN-Water in collaboration with World Population Day 11 July
Government and partners. The theme of World Toilet Day World Water Day 22 March
2021 was Valuing Toilets. 30. Prime Minister of India launched "Swachha Bharat
28. Match List - I with List - II and select the correct Mission" officially on –
answer using codes given below the lists : (a) Independence Day
(b) Republic Day
List - I List - II
(c) Gandhi Jayanti
(Important Day) (Date)
(d) Environment Day
A. World Environment Day 1. March 21
M.P.P.C.S. (Pre) 2014
B. World Forestry Day 2. June 5
Ans. (c)
C. World Habitat Day 3. September 16
D. World Ozone Day 4. October 4 Prime Minister Shri Narendra Modi launched the ambitious
5. October 10 'Swachh Bharat Abhiyan' (Clean India Mission) on 2nd
October, 2014. The Abhiyan was launched on the occassion
Codes :
of Mahatma Gandhi's 145th birth anniversary.
A B C D
(a) 2 1 4 5 31. United Nation's highest environmental honour -
(b) 1 2 4 3 'Champion of the Earth' award 2018 was conferred
(c) 1 2 3 4 on-
(d) 2 1 4 3 (a) Narendra Modi
(b) Medha Patkar
I.A.S. (Pre) 2001
(c) Uma Bharti
Ans. (d)
(d) Sonam Wangchuk
The correct match is given below: U.P. R.O./A.R.O. (Mains) 2017
World Environment Day - June 5 Ans. (a)
World Forestry Day - March 21
Prime Minister Narendra Modi was conferred with United
World Habitat Day - First Monday of October
Nation's 'Champions of the Earth' award 2018 by UN
World Ozone Day - September 16
Secretary-General Antonio Guterres at a ceremony in Delhi.
29. Match List-I and List-II and select the correct answer 32. In which district will the National Dinosaur Fossil Park
from codes given below : be set up ?
List-I List-II (a) Balaghat
(World Days) (Date) (b) Mandsaur
A. World Earth Day 1. 22 March (c) Dhar
B. World Cities Day 2. 11 July (d) Mandla
C. World Population Day 3. 31 October M.P.P.C.S. (Pre) 2010
D. World Water Day 4. 22 April Ans. (c)
Codes :
On the recommendations of Madhya Pradesh Wildlife Board,
A B C D the National Dinosaur Fossil Park is being established in the
(a) 4 3 2 1 Dhar District of Madhya Pradesh. The Park will be near Bagh
(b) 1 2 3 4 Caves in Dhar Forest Division spreading over 89.74 hectares.
(c) 3 4 1 2 It is important to note that in the year 2006, more than 100
(d) 2 1 4 3 Dinosaur eggs were discovered in Dhar district. They were
U.P. P.C.S. (Mains) 2017 estimated to be 65 to 70 million years old.
Ans. (a) 33. India's first National Centre for Marine Biodiversity

Environment and Ecology General Studies F–231


(NCMB) is located in which city? (c) Exporting our food products to other countries
(a) Bhavnagar (b) Jamnagar (d) Permitting foreign educational institutions to set up
(c) Mumbai (d) Puducherry their campuses in our country
U.P. P.C.S. (Pre) 2018 I.A.S. (Pre) 2017
Ans. (b) Ans. (a)
India's first National Centre for Marine Biodiversity (NCMB) The term 'Domestic Content Requirement (DCR) has been
is located in Jamnagar (Gujarat). The centre will have the goal provided for developing solar power production in the
of safeguarding the biodiversity of coastal areas. country. In January 2010, the National Solar Policy which
34. Central Arid Zone Research Institute (CAZRI) is was earlier named Jawaharlal Nehru National Solar Mission
situated at : was launched to establish India as a global leader in the field
(a) Ahmedabad (b) Bangalore of solar energy. According to this, solar utilities and batteries
(c) Hyderabad (d) Jodhpur manufactured in India will be used for solar projects. Under
U.P. U.D.A./L.D.A. (Spl) (Mains) 2008 the first phase of the mission, the Ministry of New and
U.P. U.D.A./L.D.A. (Spl) (Pre) 2010 Renewable Energy gives 30%, subsidy on decentralized solar
devices like - lighting equipment, water heating equipment
Ans. (d) or solar coolers.
Central Arid Zone Research Institute (CAZRI) is situated 39. In which of the scientific fields, Borlaug award is given:
at Jodhpur. It was finally upgraded to Central Arid Zone (a) Medicine (b) Space Research
Research Institute in 1959 under Indian Council of (c) Agriculture (d) Atomic Physics
U.P.P.C.S. (Pre) 2001
Agricultural Research, New Delhi.
Ans. (c)
35. Indian Institute of Ecology and Environment is
situated in which of the following cities ? The Borlaug Award is given for outstanding research and
(a) New Delhi (b) Thiruvananthapuram contribution in the field of agriculture.
(c) Kolkata (d) Jodhpur 40. Which of the following is the main component of natural
U.P.U.D.A./L.D.A. (Spl) (Mains) 2014 gas?
Ans. (a) (a) Ethane (b) Butane
The Indian Institute of Ecology and Environment, New Delhi (c) Carbon mono-oxide (d) Methane
U.P. P.C.S. (Pre) 2018
was established on the occasion of the World Environment
Ans. (d)
Day on 5th June, 1980.
36. The Indian Institute of Ecology and Environment is located at– Natural gas is a colourless, odourless and environment
(a) New Delhi (b) Mumbai friendly source of energy. It is a naturally occurring
(c) Kolkata (d) Thiruvananthapuram hydrocarbon gas mixture consisting primarily of Methane.
U.P.P.C.S. (Mains) 2016
41. Which one of the following is the national aquatic
Ans. (a)
animal of India?
See the explanation of the above question. (a) Salt-water crocodile
(b) Olive ridley trutle
37. AGMARK Act came into force in India in – (c) Gangetic dolphin
(a) 1937 (b) 1952 (d) Gharial
(c) 1957 (d) 1965 I.A.S. (Pre) 2015
U.P.P.C.S. (Mains) 2009 Ans. (c)
Ans. (a)
Gangetic River Dolphin is the national aquatic animal of
AGMARK Act or Agriculture Produce (Grading and India. This mammal is also said to represent the purity of
Marking) Act, 1937 is an Act to provide for the grading the holy Ganga as it can only survive in pure and fresh
and marking of agriculture and other produce. It came into water. It was notified as a national aquatic animal of India by
effect in 1937. Environment and Forest Ministry on 18 May, 2010.
38. The term 'Domestic Content Requirement' is 42. For weather science dispatch, which of the following
sometimes seen in the news with reference to : is used to fill balloons?
(a) Developing solar power production in our country (a) Oxygen (b) Carbon dioxide
(b) Granting licenses to foreign T.V. channels in our (c) Methane (d) Helium
country R.A.S./R.T.S. (Pre) 1997

F–232 General Studies Environment and Ecology


Ans. (d) Hurricane Katrina, Vilma and Rita stormed the USA in 2005
Helium is an inert gas which is filled in balloons used for while hurricane Sandy stormed coastal area of north-east and
weather science dispatch. east of the USA in 2012.
43. Why is a plant called Prosopis juliflora often mentioned 47. Match List-I with List-II and select the correct answer
in news? from codes given below the lists :
(a) Its extract is widely used in cosmetics List-I (State) List-II (National Park)
(b) It tends to reduce the biodiversity in the area in which A. Assam 1. Kaziranga
B. Gujarat 2. Kanha
it grows
C. Madhya Pradesh 3. Corbett
(c) Its extract is used in the systhesis of pesticides
D. Uttaranchal 4. Gir
(d) None of the above
Codes :
I.A.S. (Pre) 2018 A B C D
Ans. (b) (a) 3 2 4 1
Prosopis juliflora is an exotic plant that wipes out most of (b) 2 4 1 3
the native plants and animals that room around it once. It (c) 1 3 2 4
tends to reduce the biodiversity in the area in which it grows. (d) 1 4 2 3
U.P.P.C.S. (Mains) 2002
44. For the betterment of human life, which of the Ans. (d)
following steps is important?
(a) Afforestation The correctly matched list of states and their respective
(b) Ban on mining national parks is given below –
(c) Protection of Wildlife Assam - Kaziranga
(d) Reduce the use of natural resources Gujarat - Gir
I.A.S. (Pre) 2009 Madhya Pradesh - Kanha
Ans. (a) Uttarakhand - Corbett

Afforestation would increase greenery, reduce air pollution, 48. Tree useful for prevention of dust pollution –
aid to minimize global warming, increases oxygen content (a) Sita Ashok (b) Mahua
in the atmosphere and reduce soil erosion. With all (c) Poplar (d) Neem
these, afforestation is a major step for the betterment of U.P.P.C.S. (Pre) 2007
human life. Ans. (a)
45. If LANDSAT data for an area is received today, Among the given options Sita Ashok tree is the best safeguard
then the data for an adjacent area to its west will be against dust pollution.
available – 49. Seaweeds are the main and important source of the
(a) In the next orbit of the satellite on the same day following element :
(b) Next day at the same local time (a) Iodine (b) Chlorine
(c) After a certain number of days at the same local time (c) Bromine (d) None of the above
U.P. R.O./A.R.O. (Pre) 2017
(d) After the variable controllable time
Ans. (a)
Jharkhand P.C.S. (Pre) 2013
Ans. (c) Sea weeds are an important source of Iodine. Other important
sources of Iodine are seafood, dairy products etc.
If LANDSAT data for an area is received today, the data for
an adjacent area to it's west will be available after the certain 50. Which of the following is not correctly matched?
number of days at the same local time due to the revolution Animal Breed
of the satellite around the earth. (a) Buffalo - Bhadawari
(b) Cow - Sindhi
46. Which of the following hurricane affected the north-
(c) Goat - Jamunapari
eastern and eastern coastal area of USA in 2012? (d) Sheep - Tharparkar
(a) Katrina U.P.R.O./A.R.O. (Pre) 2021
(b) Vilma Ans. (*)
(c) Rita
(d) Sandy Bhadawari is a breed of Buffalo and found in Uttar Pradesh
U.P.R.O./A.R.O. (Mains) 2014 and Madhya Pradesh. Tharparkar is a breed of Cow and not
Ans. (d) Sheep and found in Gujarat and Rajasthan. As per UPPSC

Environment and Ecology General Studies F–233


official key option (a) is not correctly matched, while the building. The OPCW member states represent 98% of
correct answer should be option (d). the global population and landmass, as well as 98% of
the worldwide chemical industry. Hence (b) is the correct
51. Ringelmann scale is used to measure density of
answer.
(a) Smoke (b) Polluted water
(c) Fog (d) Noise 3. Which of the following best describes/describe the aim
U.P.P.C.S. (Pre) 2021 of ‘Green India Mission’ of the Government of India?
Ans. (c) 1. Incorporating environmental benefits and costs into
the Union and State Budgets thereby implementing
The Ringelmann scale is a scale for measuring the apparent the ‘green accounting’
density or opacity of Smoke. It was developed by a 2. Launching the second green revolution to enhance
French professor of agricultural engineering Maximilien agricultural output so as to ensure food security to
Ringelmann of La Station d'Essais de Machines in Paris in one and all in the future
1888. Ringelmann also discovered what is now called the 3. Restoring and enhancing forest cover and
responding to climate change by a combination of
Ringelmann effect.
adaptation and mitigation measures.
Select the correct answer using codes given below:
High-Level Questions (a) 1 only
(c) 3 only
(b) 2 and 3 only
(d) 1, 2 and 3
1. How many agreements are there in Agenda-21? I.A.S. (Pre) 2016
(a) 4 (b) 5 Ans. (c)
(c) 6 (d) 7
The National Mission for Green India (GIM) is one of
U.P.P.C.S. (Pre) 2014
the eight missions outlined in the National Action Plan on
Ans. (a)
Climate Change (NAPCC). It aims at protecting, restoring and
Agenda 21, is a non-binding volantarily implemented enhancing India’s diminishing forest cover and responding to
action plan of the United Nations with regard to sustainable climate change by a combination of adaptation and mitigation
development. There are 4 agreement in Agenda-2021. measures. It envisages a holistic view of greening and focuses
2. With reference to ‘Organization for the Prohibition of on multiple ecosystem services especially biodiversity, water
Chemical Weapons (OPCW)’, consider the following biomass, preserving mangroves, wetland, critical habitats etc.
statements: Hence (c) is the correct answer.
1. It is an organization of European Union in working 4. How is the National Green Tribunal (NGT) different
relation with NATO and WHO. from the Central Pollution Control Board (CPCB)?
2. It monitors the chemical industry to prevent new 1. The NGT has been established by an Act whereas
weapons from emerging. the CPCB has been created by executive order of
3. It provides assistance and protection to States the Government.
(Parties) against chemical weapons threats. 2. The NGT provides environmental justice and helps
Which of the statements given above is/are correct? reduce the burden of litigation in the higher courts
(a) 1 only (b) 2 and 3 only whereas the CPCB promotes cleanliness of streams
(c) 1 and 3 only (d) 1, 2 and 3 and wells, and aims to improve the quality of air
I.A.S. (Pre) 2016 in the country.
Ans. (b) Which of the statements given above is/are correct?
The mission of the Organization for the Prohibition of (a) 1 only (b) 2 only
Chemical Weapons (OPCW) is to implement the provisions (c) Both 1 and 2 (d) Neither 1 nor 2
of the Chemical Weapons Convention (CWC) to achieve the I.A.S. (Pre) 2018
OPCW’s vision of the world that is free of chemical weapons and Ans. (b)
the threat of their use. This program has following broad aims. The National Green Tribunal has been established on 18
1. To ensure a credible and transparent regime for verifying October, 2010 under the National Green Tribunal Act 2010,
the destruction of chemical weapons and to prevent their for effective and expeditious disposal of cases relating to the
re-emergence, while protecting the legitimate national environmental protection and conservation of forests and
security and proprietary interest. other natural resources including enforcement of any legal
2. To provide protection and operation in peaceful uses of rights relating to environment. The Central Pollution Control
chemistry Board (CPCB), a statutory organization was constituted in
3. To bring about universal membership of the OPCW by September 1974 under the Water (Prevention and Control
facilitating international cooperation and national capacity of Pollution) Act 1974. Principal functions of the CPCB are

F–234 General Studies Environment and Ecology


(i) to promote cleanliness of streams and wells in different and honour environmental achievement. This award was a
areas of the States by prevention, control and abatement of tribute to success on the front lines of environmental action
water pollution, and (ii) to improve the quality of air and to and honoured individuals and organizations that combat
prevent, control or abate air pollution in the country. Hence, environmental issues despite immense social, political and
statement 1 is incorrect and statement 2 is correct. logistical obstacles.
5. Prestigious 'Tyler Prize' is given in the field of – 8. The 'Global-500' Award is given for rare achievement
(a) Drama in which of the following fields?
(b) Woman Rights (a) Gene preservation
(c) Child Rights (b) Environment conservation
(d) Environment Protection (c) Population control
U.P.R.O./A.R.O. (Pre) 2004 (d) Pollution control
Ans. (d) U.P.P.C.S. (Pre) 2019
Ans. (b)
The Tyler Prize for Environment Achievement is the premier
award for environmental science, environmental health and See the explanation of above question.
energy conferring a great benefit upon mankind. In 2016,
9. 'R2 Code of Practices' constitutes a tool available for
this prize was given to Sir Partha Das Gupta, Professor,
promoting the adoption of
University of Cambridge. Tyler Prize 2018 was awarded to
(a) environmentally responsible practices in electronics
two biological oceanographers: Paul Falkowski and James
recycling industry
J. Mc Carthy. Tyler Prize 2020 is awarded to Pavan Sukhdev
and Gretchen C. Daily. (b) ecological management of 'Wetlands of International
Importance' under the Ramsar Convention
6. Rajiv Gandhi Environment Award is given for (c) sustainable practices in the cultivation of agricultural
outstanding contribution to : crops in degraded lands
(a) Afforestation and conservation of wastelands
(d) 'Environmental Impact Assessment' in the exploitation
(b) Neat technology and development
of natural resources
(c) Wildlife Conservation
I.A.S (Pre) 2021
(d) Hindi books on environment
Ans. (a)
U.P.P.C.S. (Mains) 2006
Ans. (b) R2 Code of Practices refers to set of activities that needs to
be done by an Electronics Company related to Recycling.
Rajiv Gandhi Environment Award is given to the industrial Once the company follows all the required steps under R2
units that make a significant contribution towards the Practices, the Company becomes R2 Certified.
development of new or the innovative modification of An electronics recycling company that is R2 certified in
existing technologies or adoptation and use of clean accordance with this Code of Practices will benefit by
technologies and practices that substantially reduce or prevent achieving higher profit margins and additional market share
environmental pollution. through improvements in its operating systems and processes,
and the status bestowed by certification.
7. ‘Global 500’ awards are given for achievements in:
(a) Population control Thus, environmentally responsible practices in the electronics
(b) Campaign against terrorism recycling industry are compiled under R2 Code of Practices.
(c) Protection of environment 10. International Decade for Natural Disaster Reduction
(d) Campaign against drugs (IDNDR) is :
U.P.P.C.S. (Mains) 2005 (a) 1991-2000
U.P.P.C.S. (Mains) 2004 (b) 1981-1990
U.P.P.C.S. (Pre) 2003 (c) 2001-2010
U.P. Lower Sub. (Pre) 2002 (d) 2011-2020
U.P.P.C.S.(Pre) 2001 M.P.P.C.S. (Pre) 2013
Ans. (c) Ans. (*)

The Global 500 Award was launched in 1987 by the United The United Nations General Assembly designated the 1990-
Nations Environment Programme (UNEP) to recognize 99 as the International Decade for Natural Disaster

Environment and Ecology General Studies F–235


Reduction. The basic objective was to decrease the loss of (NDMA), headed by the Prime Minister, and State Disaster
life, property destruction and social and economic disruption Management Authorities (SDMAs) headed by respective
caused by natural disasters such as earthquakes, tsunamis, Chief Ministers, to spearhead and implement a holistic
floods, landslides, volcanic eruptions, droughts and other and integrated approach to Disaster Management in India.
disasters of natural origin. NDMA, as the apex body, is mandated to lay down the
11. District Disaster Management Authority (DDMA) is policies, plans and guidelines for Disaster Management to
ensure timely and effective response to disasters.
headed by
(a) District Magistrate 15. Functions of Development of Disaster Management
(b) District Collector Knowledge-cum-Demonstration Center (SRIJAN) are
(c) Any of them as the case may be (a) Creating knowledge-cum-demonstration centres
(d) None of the above (b) Creating awareness
Jharkhand P.C.S. (Pre) 2016 (c) Providing local need-based information
Ans. (c) (d) All the above
Jharkhand P.C.S. (Pre) 2016
The District Disaster Management Authority (DDMA) is
Ans. (d)
headed by district collector or district magistrate. Thus district
Disaster Management Authority in Jharkhand is headed by The major function of the Development of Disaster
the District Magistrate or District Collector. So, option (c) Management-cum-Demonstration Centre (SRIJAN) is to
is the correct answer. spread awareness among the community and people towards
various types of potential disasters. SRIJAN has been
12. Where is the National Institute of Disaster Management
developed and designed for disasters like - drought, flood,
(NIDM) located ?
forest fire etc. These centres will work as a major centre for
(a) Nagpur (b) New Delhi
spreading local need-based information, communication and
(c) Hyderabad (d) Pune
technology equipment.
Jharkhand P.C.S. (Pre) 2016
Ans. (b) 16. Section ....... of the Disaster Management Act, 2005
empowers Governor of a State to establish a State
National Institute of Disaster Mangement (NIDM) is located Disaster Management Authority (SDMA).
in New Delhi. It was founded in 1995. The National Disaster
(a) Section 14(1)
Management Institute was established under the Disaster
(b) Section 15
Management Act, 2005.
(c) Section 16
13. The Disaster Management Act was made in (d) None of the above
(a) 2006 (b) 2003 Jharkhand P.C.S. (Pre) 2016
(c) 2005 (d) 2009 Ans. (a)
M.P.P.C.S. (Pre) 2020
Section 14(I) of the Disaster Management Act, 2005
Ans. (c)
empowers Governor of a State to establish State Disaster
The Disaster Management Act was passed by the government Management Authority (SDMA). In each state, after the
of India in 2005 for the ‘efficient management of disasters notification under Section 3(1) of this Act, the Governor shall
and other matters connected to it. However it came into force establish the State Disaster Management Authority.
in January 2006. Hence option(c) is correct answer.
17. Which of the following statements are true?
14. When was National Disaster Management Authority
1. Natural disasters cause maximum damage in
formed?
developing countries.
(a) 2006 (b) 2008
2. Bhopal gas tragedy was man-made.
(c) 2011 (d) 2005
3. India is a disaster-free country.
Jharkhand P.C.S. (Pre) 2021
4. Mangroves reduce the impact of cyclones.
Ans. (a)
Select the correct answer from codes given below:
NDMA was formed on September 27, 2006 through DM Codes :
Act, 2005. On 23 December 2005, the Government of India (a) 1, 2 and 3 (b) 2, 3 and 4
enacted the Disaster Management Act, which envisaged (c) 1, 2 and 4 (d) 1, 3 and 4
the creation of National Disaster Management Authority U.P. P.C.S. (Pre) 2018

F–236 General Studies Environment and Ecology


Ans. (c) Africa, Medagascar, India, Malaysia, Indonesia, Philippines,
India is a land prone to many kinds of disasters, both natural Papua New Guinea, China and Australia.
and man-made. Droughts, floods, earthquakes, cyclones, 21. Which of the following is called “grave yard of
avalanches etc are some of the many examples of disasters in Dinosaurs”?
India. Floods are the most common natural disaster in India. (a) China (b) Montana
The heavy southwest monsoon rain causes the Brahmaputra (c) Argentina (d) Brazil
and other rivers to extend their banks often flooding the
R.A.S./R.T.S.(Pre) 2010
surrounding area.
Ans. (b)
Note : In the Hindi version of the question, the meaning of the
Montana in the United States of America is called as the
option (c) as per the statement becomes, "India is a disaster-
graveyard of Dinosaurs.
prone country." Hence none of the given options become
wrong. Hence the Uttar Pradesh Public Service Commission 22. The ‘Eco Mark’ Scheme was launched in 1991 to
has removed the questions from evaluation. encourage consumers to buy products of less harmful
environmental impact. Which one of the following
18. Which of the following is post disaster measure?
consumer products is not notified under this Scheme?
(a) Zoning
(a) Soaps and Detergents
(b) Reconstruction
(b) Paper and Plastics
(c) Planning and Policies
(c) Cosmetics and Aerosols
(d) Early warning
(d) Drugs and Antibiotics
Jharkhand P.C.S. (Pre) 2021
U.P.P.C.S.(Pre) 2013
Ans. (b)
Ans. (d)
Disaster Management consists of five main phases:
Eco-Mark is an eco-labelling scheme which was constituted
prevention, preparedness, response, mitigation and recovery.
by the Government of India in 1991 for easy identification
Reconstruction, recovery and response are post disaster
of environment-friendly product. Drugs and antibiotics are
measures. While prevention, mitigation and preparedness
not in this list.
are pre disaster measures.
19. Which one of the following countries celebrate the 23. With reference to pre-packaged items in India, it is
National Cleanliness Day on last Saturday of every mandatory for the manufacturer to put which of the
month: following information on the main label, as per the
(a) France Food Safety and Standards (Packaging and Labelling)
(b) Italy Regulations, 2011?
(c) Sierra Leone 1. List of ingredients including additives
2. Nutrition information
(d) Singapore
3. Recommendations, if any, made by the medical
U.P.P.C.S. (Pre) 2003
profession about the possibility of any allergic
U.P.U.D.A./L.D.A. (Pre) 2002
reactions
Ans. (c)
4. Vegetarian/non-vegetarian
Sierra Leone celebrates the National cleanliness day on last Select the correct answer using the code given below:
Saturday of every month. (a) 1, 2 and 3 (b) 2, 3 and 4
20. Which one of the following has been recognized as a (c) 1, 2 and 4 (d) 1 and 4 only
Mega diverse country?
I.A.S. (Pre) 2016
(a) New Zealand (b) Australia
Ans. (c)
(c) Nepal (d) Austria
U.P.P.C.S. (Pre) (Re Exam.) 2015 In addition to the general labelling, every package of food
Ans. (b) shall carry the following information on the label, namely,
1. The name of food,
Mega diverse countries is a term used to refer to the world's
top bio-diversity rich countries. The identified Mega diverse 2. List of Ingredients,
countries are U.S.A., Mexico, Colombia, Ecuador, Peru, 3. Nutritional information,
Venezuela, Brazil, Democratic Republic of Congo, South 4. Energy value in Kcal,

Environment and Ecology General Studies F–237


5. The amount of protein, carbohydrate (specify the 27. Weather monitoring device SODAR is established at –
quantity of sugar), and fat in gram (g) or ml (a) Jaipur (b) Cuttak
6. The amount of any other nutrient for which a nutrition (c) Chandigarh (d) Kaiga and Kalpakkam
or health claim is made U.P.P.C.S. (Mains) 2004
7. Declaration regarding vegetarian or non-vegetarian Ans. (d)
8. Declaration regarding food additives etc. SODAR (Sonic Detection and Ranging) is a meteorological
instrument used as a wind profiler to measure the scattering
24. Which Indian scientist is recognised by ‘UNEP’ as
of sound waves by atmospheric turbulence. SODAR
Father of Economic Ecology ?
systems are used to measure wind speed at various heights
(a) B.P. Pal (b) Yash Pal
above the ground. In India, SODAR is established at Kaiga,
(c) M.S. Swaminathan (d) R. Mishra Kalpakkam, Tarapur and Trombey.
U.P.P.C.S. (Mains) 2007
28. Winterline, the natural phenomena is observed in
Ans. (c)
which part of the country?
The United Nations Environment Programme (UNEP) has (a) Shimla (b) Darjeeling
recognised Dr. M.S. Swaminathan as Father of Economic (c) Mussoorie (d) Nanital
Ecology. Uttarakhand Lower Sub. (Pre) 2010
25. With reference to the Bombay Natural History Society Ans. (c)
(BNHS), consider the following statements :
From mid October to January, when the sun sets, the western
1. It is an autonomous organization under the horizon turns a myriad combination of yellow, red, orange
Ministry of Environment and Forests. and mauve. This is known as winterline. The phenomenon
2. It strives to conserve nature through action-based is unique to Mussoorie and a part of Switzerland.
research, education and public awareness. 29. Glaciation of peninsular India occurred during which
3. It organizes and conducts nature trails and camps of the following ice-age?
for the general public. (a) Huronian ice age (b) Carboniferous ice age
Which of the statements given above is/ are correct? (c) Plestocin Ice age (d) Little Ice age
(a) 1 and 3 only (b) 2 only U.P.U.D.A./L.D.A. (Spl) (Mains) 2010
(c) 2 and 3 only (d) 1, 2 and 3 Ans. (c)
I.A.S. (Pre) 2014 Glaciation of peninsular India occured in Plestocin Ice age.
Ans. (c) 30. The Monoclimax Theory was propounded by -
Bombay Natural History Society (BNHS) India, a pan (a) F.E. Clements (b) C.C. Park
India wildlife research organization, has been promoting the (c) D.V. Ager (d) S.J. Gould
cause of nature conservation since 1883. Main activities of U.P.R.O./A.R.O. (Pre) 2021
BNHS are – Ans. (a)
Natural history collection Monoclimax was propounded by F.E. Clement. According
Research on various species and habitats to the mono-climax theory of succession (Clements, 1936),
Conservation of landscapes and seascapes every region has one climax community toward which all
Environmental Information System communities are developing. He believed that climate was
Outreach (Camp, exhibitions, lectures, other events) the determining factor for vegetation and the climax of any
Communications and Advocacy area was solely a function of its climate.
26. Which one of the following is the evergreen fruit 31. If you walk through the countryside, you are likely to
plant? see some birds stalking alongside the cattle to seize the
(a) Apple (b) Almond insects disturbed by their movement through grasses.
(c) Peach (d) Loquat Which of the following is/are such bird/birds?
1. Painted Stork 2. Common Myna
U.P.P.C.S. (Pre) 2008
3. Black-necked Crane
Ans. (d)
Select the correct answer using codes given below:
Loquat is an evergreen fruit plant, mostly found in Japan. (a) 1 and 2 (b) 2 only

F–238 General Studies Environment and Ecology


(c) 2 and 3 (d) 3 only U.P.P.C.S. (Spl) (Mains) 2008
I.A.S. (Pre) 2014 Ans. (c)
Ans. (b)
Bhitarkanika is the location of rich, lush green vibrant
Dr. Salim Ali was an Indian ornithologist and naturalist. eco-system lying in the estuarine region of Brahmani-
He is some times referred to as 'Birdman of India'. Baitarani rivers in the north-eastern corner of Kendrapara
In his book "Handbook of the birds of India and Pakistan district of Odisha.
Vol.5" he wrote that common Myna stalking alongside the
36. Match List-I with List-II and select the correct answer
cattle seizes the insects disturbed by their movement through
from codes given below the lists :
grasses. Painted stork is found in the southern Himalayas
and feed on small fishes near wetlands. Black-necked crane List-I List-II
is found in Tibetan Plateau, Bhutan, Arunachal Pradesh (Wild Life Sanctuary) (State)
and Ladakh. It is an omnivore and eats plants roots, insects, A. Namdapha 1. Karnataka
fish, frog etc. B. Bandipur 2. Arunachal Pradesh
C. Periyar 3. Manipur
32. Which of the following has the lowest mean net primary
productivity? D. Lamjao 4. Kerala
(a) Temperate forests (b) Tropical forests Codes :
(c) Temperate grasslands (d) Tropical savanna A B C D
U.P.P.C.S. (Re. Exam) (Pre) 2015 (a) 2 1 4 3
Ans. (c) (b) 4 3 2 1
(c) 4 2 1 3
Primary productivity is the rate at which energy is converted
(d) 2 4 3 1
by photosynthetic and chemosynthetic autotrophs to organic
U.P. Lower Sub. (Pre) 2015
substances. The total amount of productivity in a region or
Ans. (a)
system is gross primary productivity. Primary productivity
of temperate grassland is approx 500gm per cm2 per year. Listed below is the correctly matched list of wildlife
Primary productivity of temperate forest is approx 1000 gm sanctuaries with their states :
per cm2 per year. Primary productivity of Tropical forests is Wildlife Sanctuary States
approx 2000gm per cm2 per year and primary productivity of Namdapha - Arunachal Pradesh
Tropical savanna is approx 700 gm per cm2 per year. Bandipur - Karnataka
33. The Marine National Park of India is located in – Periyar - Kerala
(a) Kachchh (b) Sundarban Lamjao - Manipur
(c) Chilka Lake (d) Nicobar Islands 37. The five rhino range nations, who signed a declaration
U.P.P.C.S. (Mains) 2013 "The New Delhi Declaration on Asian Rhinos 2019"
Ans. (a) are India, Nepal, Malaysia, Indonesia and
Marine National Park in the Gulf of Kachchh is located in (a) Vietnam (b) Thailand
the Jamnagar district of Gujarat. (c) Myanmar (d) Bhutan
(e) None of the above/ more than one of the above
34. The Marine National Park is located in
65th B.P.S.C. (Pre) 2019
(a) Gulf of Kachchh (b) Sunderbans
Ans. (d)
(c) Chilka Lake (d) None of the above
U.P.P.C.S. (Mains) 2016 India and four rhino range nations (Nepal, Malaysia,
Ans. (a) Indonesia and Bhutan) have signed a declaration 'The New
Delhi Declaration on Asian Rhinos 2019' for the conservation
Marine National Park in Gulf of Kachchh is situated on the
and protection of the species at the recently held Second
southern shore of the Gulf in the Jamnagar district of Gujarat.
Asian Rhino Range countries meeting. It is noteworthy that
It is the first national marine park of India with 42 islands.
Second Asian Rhino Range countries meeting (from February
35. “Bhitarkanika” which has recently been included in 26 to 28 in 2019) was held at New Delhi and organized by
the list of `World Heritage Sites’ is situated in – the Ministry of Environment, Forest and Climate Change of
(a) Andhra Pradesh (b) Chhattisgarh Government of India in collaboration with the IUCN Asian
(c) Odisha (d) West Bengal Rhino Specialist Group, WWF - India and Aaranyak.

Environment and Ecology General Studies F–239


38. Which one of the following states has the maximum (a) Only1 (b) Only 2
area under teak forest in India? (c) Both 1 and 2 (d) Neither 1 nor 2
(a) Jharkhand U.P.P.C.S. (Pre) 2019
(b) Andhra Pradesh Ans. (c)
(c) Uttrakhand On the eve of Mahatma Gandhi's 150th birth anniversary,
(d) Madhya Pradesh India's largest 'Charkha' made of waste plastic was
M.P.P.C.S. (Pre) 2020 inaugurated in Noida.
Ans. (d) 42. How many Indian beaches have been recommended
Area-wise Madhya Pradesh has the largest forest cover in for the Blue Flag Certification recently?
the country followed by Arunachal Pradesh, Chhattisgarh, (a) Six (b) Seven
Odisha and Maharashtra. In terms of forest cover as (c) Eight (d) Nine
percentage of total geographical area, the top five States are (d) None of the above/More than one of the above
Mizoram (84.53%), Arunachal Pradesh (79.33%), Meghalaya 66th B.P.S.C. (Pre) 2020
(76.00%), Manipur (74.34%) and Nagaland (73.90%). As per Ans. (c)
IFSR 2021 ,Madhya Pradesh has the maximum area under Eight Indian beaches spread across five states and two union
teak forest in India. territories, have been awarded the ‘Blue Flag Certification’ by
39. Recently, there was a growing awareness in our country an eminent international jury, comprising members of UNEP
about the importance of Himalayan nettle (Girardinia (United Nations Environment Programme), UNWTO (United
diversifolia) because it is found to be a sustainable Nations World Tourism Organization), FEE (Foundation for
source of Environment Education) and IUCN (International Union for
(a) Anti-malarial drug (b) Biodiesel Conservation of Nature). The beaches that have been awarded
(c) Pulp for paper industry (d) Textile fibre the ‘Blue Flag’ are Shivrajpur (Dwarka-Gujarat), Ghoghla
I.A.S. (Pre) 2019 (Diu), Kararkod and Padubidri (Karnataka), Kappad (Kerala),
Ans. (d) Rushikonda (Andhra Pradesh), Radhanagar (Andaman &
Himalayan nettle (Girardinia diversifolia) is a fiber-yielding Nicobar Islands) and Golden beach (Puri-Odisha).
plant. In Himalayan region many people have made it an 43. Where was the Mitti Bachao (Save the Soil) Movement
important livelihood option because it is found to be a started in India?
sustainable source of textile fibre. (a) Thane, Maharashtra
40. World's most problematic aquatic weed is : (b) Mysore, Karnataka
(a) Azolla (c) Darbhanga, Bihar
(b) Eichornia (d) Hoshangabad, Madhya Pradesh
(c) Trapa U.P.P.C.S. (Pre) 2020
(d) Wolffia Ans. (d)
Jharkhand P.C.S. (Pre) 2021 The Mitti Bachao (Save the soil) Movement was started
Ans. (b) in Hoshangabad district in Madhya Pradesh against the
Among common aquatic weeds, Eichhornia crassipes construction of Tawa Hydropower Project in 1977.
(water hyacinth) remains the most widely distributed 44. World Ozone Day' is celebrated on
and vicious aquatic weed . Water hyacinth has become (a) 25th December (b) 21 April
the world’s most invasive weed due to its rapid spread, (c) 16th September (d) 30th January
ecological adaptability, and negative impacts it causes U.P.P.C.S. (Pre) 2021
on the environment, economic development, and human Ans. (c)
health . Water hyacinth derives its origin from the Amazonia
September 16 was designated by the United Nations General
basin in South America and holds the reputation of
Assembly as the International Day for the Preservation
being one of the most strenuous water weeds to manage.
of the Ozone Layer. This designation had been made on
41. Which of the following statements about India's largest December 19, 2000, in commemoration of the date, in 1987,
Charkha is/are true? on which nations signed the Montreal Protocol on Substances
1. It was inaugurated in Noida. that Deplete the Ozone Layer. The theme of World Ozone
2. It is made of used plastic waste. Day 2021 is 'Montreal Protocol Keeping us, our food, and
Select the correct answer from codes given below : vaccines cool."

F–240 General Studies Environment and Ecology

You might also like